Vous êtes sur la page 1sur 323

ADDITIONAL ASSIGNMENT

SECTION 1-S & 1-B


CONSTITUTIONAL LAW 1
Section 21
Bengzon v. Senate Blue Ribbon Committee (203 SCRA 767 [1991])
Arnault v. Nazareno (87 PHIL. 29 [1950])
Senate of the Phil. Vs. Ermita (488 SCRA 1; April 20, 2006)
Section 22
Senate of the Phil. Vs. Ermita (G.R. No. 169777, April 20 2006)
Section 24
Tolentino v. Secretary of Finance (235 SCRA 630 [1994])
Alvarez, et al. Vs. Guingona, et al. (252 SCRA 695 [1996])
Section 25
Garcia v. Mata (65 SCRA 517 [1975])
Demetria v. Alba (148 SCRA 208 [1987])
Philconsa v. Enriquez (235 SCRA 506 [1994])
Section 26
Philconsa v. Gimenez (15 SCRA 479 [1965])
Tio v. Videogram Regulatory Board (151 SCRA 208 [1987]
Philippine Judges Asso. V. Prado (227 SCRA 703 [1993])
Tolentino v. Secretary of Finance ( 235 SCRA 630 [1994])
Tobias v. Abalos (239 SCRA 106 [1994])
Section 27
Tolentino v. Secretary of Finance (235 SCRA 630 [1994)
Gonzales v. Macaraig (191 SCRA 452 [1990])
Bengzon v. Drilon (208 SCRA 133 [1992])
Philconsa v. Enriquez (235 SCRA 506 [1994])
Section 28
Kapatiran ng mgaNaglilingkodsaPamahalaan ng Pil. v. Tan (163 SCRA 371 [1988])
Province of Abra v. Judge Hernando (107 SCRA 104 [August 31, 1981])
Abra Valley College v. Aquino (162 SCRA 106 [1988])

Section 29
Pascual v. Sec. of Public Works (110 PHIL. 331 [1960])
Aglipay v. Ruiz (64 PHIL. 201 [1937])
Guingona v. Carague (196 SCRA 221 [1991])
Philconsa vs. Enriquez (235 SCRA 506 [1994)
Section 30
Diaz v. CA (238 SCRA 785 [1994])
Section 32
Subic Bay Metropolitan Authority v. COMELEC (262 SCRA 292 [September 26, 1996])
ATTY. ROCHELLE DAKANAY-GALANO

Republic of the Philippines


SUPREME COURT
Manila
EN BANC

G.R. No. 89914 November 20, 1991


JOSE F.S. BENGZON JR., ABELARDO TERMULO, JOSE MANTECON,
VICENTE MILLS JR., LEONARDO GAMBOA, KURT BACHMANN JR., JOSE
V.E. JIMENEZ, ERNESTO CALUYA, AGERICO UNGSON, SUSAN ROXAS,
ELVIE CASTILLO, and CYNTHIA SABIDO LIMJAP, petitioners,
vs.
THE SENATE BLUE RIBBON COMMITTEE AND ITS MEMBERS, represented
by and through the CHAIRMAN, HON. WIGBERTO TAADA, respondents,
JOSE S. SANDEJAS, intervenor.
Bengzon, Zarraga, Narciso, Cudala, Pecson & Bengson for petitioners.
Balgos & Perez for intervening petitioner.
Eddie Tamondong and Antonio T. Tagaro for respondents.

PADILLA, J.:p
This is a petition for prohibition with prayer for the issuance of a temporary
restraining order and/or injuective relief, to enjoin the respondent Senate Blue
Ribbon committee from requiring the petitioners to testify and produce evidence
at its inquiry into the alleged sale of the equity of Benjamin "Kokoy" Romualdez
to the Lopa Group in thirty-six (36) or thirty-nine (39) corporations.
On 30 July 1987, the Republic of the Philippines, represented by the Presidential
Commission on Good Government (PCGG), assisted by the Solicitor General,
filed with the Sandiganbayan Civil Case No. 0035 (PCGG Case No. 35) entitled
"Republic of the Philippines vs. Benjamin "Kokoy" Romualdez, et al.", for
reconveyance, reversion, accounting, restitution and damages.
The complaint was amended several times by impleading new defendants and/or
amplifying the allegations therein. Under the Second Amended Complaint, 1 the
herein petitioners were impleaded as party defendants.

The complaint insofar as pertinent to herein petitioners, as defendants, alleges


among others that:
14. Defendants Benjamin (Kokoy) Romualdez and Juliette Gomez
Romualdez, acting by themselves and/or in unlawful concert with
Defendants Ferdinand E. Marcos and Imelda R. Marcos, and taking
undue advantage of their relationship, influence and connection with
the latter Defendant spouses, engaged in devices, schemes and
strategems to unjuestly enrigh themselves at the expense of Plaintiff
and the Filipino people, among others:
(a) Obatained, with the active collaboration of
Defendants Sene J. Gabaldon, Mario D. Camacho,
Mamerto Nepomuceno, Carlos J. Valdez, Cesar C.
Zalamea and Francisco Tantuico, Atty. Jose Bengzon,
Jr. and his law partners, namely: Edilberto S. Narciso,
Jr., Jose Vicente E. Jimenez, Amando V. Faustino, Jr.,
and Leonardo C. Cruz; Jose S. Sandejas and his fellow
senior managers of FMMC/PNI Holdings groups of
companies such as Leonardo Gamboa, Vicente T. Mills,
Jr., Jose M. Mantecon, Abelardo S. Termulo, Rex C.
Drilon II and Kurt Bachmann, Jr., control of some of the
biggest business enterprises in the Philippines, such as
the Manila Corporation (MERALCO), Benguet

Consolidated and the Philippine Commercial


International Bank (PCI Bank) by employing devious
financial schemes and techniques calculated to require
the massive infusion and hemorrhage of government
funds with minimum or negligible "cashout" from
Defendant Benjamin Romualdez...
xxx xxx xxx
(m) manipulated, with the support, assistance and
collaboration of Philgurantee officials led by chairman
Cesar E.A. Virata and the Senior managers of
FMMC/PNI Holdings, Inc. led by Jose S. Sandejas, Jr.,
Jose M. Mantecom and Kurt S. Bachmann, Jr., among
others, the formation of Erectors Holdings, Inc. without
infusing additional capital solely for the purpose of
Erectors Incorporated with Philguarantee in the amount
of P527,387,440.71 with insufficient
securities/collaterals just to enable Erectors Inc, to
appear viable and to borrow more capitals, so much so
that its obligation with Philgurantee has reached a total
of more than P2 Billion as of June 30, 1987.
(n) at the onset of the present Administration and/or
within the week following the February 1986 People's
Revolution, in conspiracy with, supoort, assistance and
collaboration of the abovenamed lawyers of the
Bengzon Law Offices, or specifically Defendants Jose
F.S. Bengzon, Jr., V.E. Jimenez, Amando V. Faustino,
Jr., and Edilberto S. Narciso, Jr., manipulated,
shcemed, and/or executed a series of devices intended
to conceal and place, and/or for the purpose of
concealing and placing, beyond the inquiry and
jurisdiction of the Presidential Commission on Good
Government (PCGG) herein Defendant's individual and
collective funds, properties, and assets subject of and/or
suited int he instant Complaint.
(o) manuevered, with the technical know-how and
legalitic talents of the FMMC senior manager and some
of the Bengzon law partners, such as Attys. Jose F.S.
Bengzon, Jr., Edilberto S. Narciso, Jr., Amando V.
Faustino, Jose Vicente E. Jimenez and Leonardo C.

Cruz, the purported sale of defendant Benjamin


Romualdez's interests in the (i) Professional Managers,
(ii) A & E International Corporation (A & E), (iii) First
Manila Managerment Corporation (FMMC), (iv)
Philippine World Travel Inc. (PWTI) and its subsidiaries
consisting of 36 corporations in all, to PNI Holdings, Inc.
(wjose purported incorporations are all members of Atty.
Jose F.S. Bengzon's law firm) for only P5 million on
March 3, 1986 or three days after the creation of the
Presidential Commission on Good Government on
February 28, 1986, for the sole purpose of deceiving
and preempting the Government, particularly the
PCGG, and making it appear that defendant Benjamin
Romualdez had already divested himself of his
ownership of the same when in truth and in fact, his
interests are well intact and being protected by Atty.
Jose F.S. Bengzon, Jr. and some of his law partners,
together with the FMMC senior managers who still
control and run the affiars of said corporations, and in
order to entice the PCGG to approve the said fictitious
sale, the above-named defendants offered P20 million
as "donation" to the Government;
(p) misused, with the connivance, support and technical
assitance of the Bengzon law firm represented by Atty.
Jose F.S. Bengzon, Jr. as legal counsel, together with
defendants Cesar Zalamea, Antonio Ozaeta, Mario D.
Camacho amd Senen J. Gabaldon as members of the
Board of Directors of the Philippine Commercial
International bank (PCIB), the Meralco Pension Fund
(Fund, for short) in the amount of P25 million by cuasing
it to be invested in the PCIB and through the Bank's
TSG, assigned to PCI Development and PCI Equity at
50% each, the Fund's (a) 8,028.011 common shares in
the Bank and (b) "Deposit in Subscription" in the
amount of P4,929.972.50 but of the agreed
consideration of P28 million for the said assignment,
PCI Development and PCI Equity were able to pay only
P5,500.00 downpayment and the first amortization of
P3,937,500.00 thus prompting the Fund to rescind its
assignment, and the consequent reversion of the
assigned brought the total shareholding of the Fund to
11,470,555 voting shares or 36.8% of the voting stock

of the PCIB, and this development (which the


defendants themselves orchestrated or allowed to
happen) was used by them as an excuse for the
unlawful dismantling or cancellation of the Fund's 10
million shares for allegedly exceeding the 30-percent
ceiling prescribed by Section 12-B of the General
Banking Act, although they know for a fact that what the
law declares as unlawful and void ab initio are the
subscriptions in excess of the 30% ceiling "to the extent
of the excess over any of the ceilings prescribed ..." and
not the whole or entire stockholding which they allowed
to stay for six years (from June 30, 1980 to March 24,
1986);
(q) cleverly hid behind the veil of corporate entity,
through the use of the names and managerial expertise
of the FMMC senior manager and lawyers identified as
Jose B. Sandejas, Leonardo Gamboa, Vicente T. Mills,
Abelardo S, Termulo, Edilberto S. Narciso, Jr., Jose M.
Mantecon, Rex C. Drilon II, Kurt Bachmann, Jr. together
with the legal talents of corporate lawyers, such as
Attys. Jose F.S. Bengzon, Jr., Jose V.E. Jimenez,
Amando V. Faustino, Jr., and Leonardo C. Cruz, the illgotten wealth of Benjamin T. Romualdez including,
among others, the 6,229,177 shares in PCIB registered
in the names of Trans Middle East Phils. Equities, Inc.
and Edilberto S. Narciso, Jr. which they refused to
surrender to PCGG despite their disclosure as they tried
and continue to exert efforts in getting hold of the same
as well as the shares in Benguet registered in the
names of Palm Avenue Holdings and Palm Avenue
Realty Development Corp. purportedly to be applied as
payment for the claim of P70 million of a "merger
company of the First Manila Managerment Corp. group"
supposedly owned by them although the truth is that all
the said firms are still beneficially owned by defendants
Benjamin Romualdez.
xxx xxx xxx
On 28 September 1988, petitioner (as defendants) filed their respective
answers. 2 Meanwhile, from 2 to 6 August 1988, conflicting reports on the disposition by the PCGG of the "Romualdez corporations"
were carried in various metropolitan newspapers. Thus, one newspaper reported that the Romuladez firms had not been sequestered
because of the opposition of certain PCGG officials who "had worked prviously as lawyers of the Marcos crony firms." Another daily reported

otherwise, while others declared that on 3 March 1986, or shortly after the EDSA February 1986 revolution, the Romualdez companies" were
sold for P5 million, without PCGG approval, to a holding company controlled by Romualdez, and that Ricardo Lopa, the President's brotherin-law, had effectively taken over the firms, even pending negotiations for the purchase of the corporations, for the same price of P5 million
which was reportedly way below the fair value of their assets. 3

On 13 September 1988, the Senate Minority Floor Leader, Hon. Juan Ponce
Enrile delivered a speech "on a matter of personal privilege" before the Senate
on the alleged "take-over personal privilege" before the Senate on the alleged
"take-over of SOLOIL Incorporated, the flaship of the First Manila Management of
Companies (FMMC) by Ricardo Lopa" and called upon "the Senate to look into
the possible violation of the law in the case, particularly with regard to Republic
Act No. 3019, the Anti-Graft and Corrupt Practices Act." 4
On motion of Senator Orlando Mercado, the matter was referred by the Senate to
the Committee on Accountability of Public Officers (Blue Ribbon
Committee). 5 Thereafter, the Senate Blue Ribbon Committee started its investigation on the matter. Petitioners and Ricardo Lopa
were subpoenaed by the Committee to appear before it and testify on "what they know" regarding the "sale of thirty-six (36) corporations
belonging to Benjamin "Kokoy" Romualdez."

At the hearing held on 23 May 1989, Ricardo Lopa declined to testify on the
ground that his testimony may "unduly prejudice" the defendants in Civil Case
No. 0035 before the Sandiganbayan. Petitioner Jose F.S. Bengzon, Jr. likewise
refused to testify involing his constitutional right to due process, and averring that
the publicity generated by respondents Committee's inquiry could adversely
affect his rights as well as those of the other petitioners who are his codefendants in Civil Case No. 0035 before the Sandiganbayan.
The Senate Blue Ribbon Committee, thereupon, suspended its inquiry and
directed the petitioners to file their memorandum on the constitutional issues
raised, after which, it issued a resolution 6 dated 5 June 1989 rejecting the petitioner's plea to be excused from
testifying, and the Committee voted to pursue and continue its investigation of the matter. Senator Neptali Gonzales dissented. 7

Claiming that the Senate Blue Ribbon Committee is poised to subpoena them
and required their attendance and testimony in proceedings before the
Committee, in excess of its jurisdiction and legislative purpose, in clear and
blatant disregard of their constitutional rights, and to their grave and irreparable
damager, prejudice and injury, and that there is no appeal nor any other plain,
speedy and adequate remedy in the ordinary course of law, the petitioners filed
the present petition for prohibition with a prayer for temporary restraning order
and/or injunctive relief.
Meanwhile, one of the defendants in Civil Case No. 0035 before the
Sandiganbayan, Jose S. Sandejas, filed with the Court of motion for
intervention, 8 which the Court granted in the resolution 9 of 21 December 1989, and required the respondent Senate Blue Ribbon
Committee to comment on the petition in intervention. In compliance, therewith, respondent Senate Blue Ribbon Committee filed its
comment 10 thereon.

Before discussing the issues raised by petitioner and intervenor, we will first
tackle the jurisdictional question raised by the respondent Committee.
In its comment, respondent Committee claims that this court cannot properly
inquire into the motives of the lawmakers in conducting legislative investigations,
much less cna it enjoin the Congress or any its regular and special commitees
like what petitioners seek from making inquiries in aid of legislation, under the
doctrine of separation of powers, which obtaines in our present system of
government.
The contention is untenable. In Angara vs. Electoral Commission, 11 the Court held:
The separation of powers is a fundamental principle in our system of
government. It obtains not hrough express provision but by actual
division in our Constitution. Each department of the government has
exclusive cognizance of matters wihtin its jurisdiction, and is
supreme within its own sphere. But it does not follow from the fact
that the three powers are to be kept separate and distinct that the
Constitution intended them to be absolutely unrestrained and
independent of each other. The Constitution has provided for an
elaborate system of checks and balances to secure coordination in
the workings of the various departments of the government...
xxx xxx xxx
But in the main, the Constitution has blocked out with deft strokes
and in bold lines, allotment of power to the executive, the legislative
and the judicial departments of the government. The ovelapping and
interlacing of funcstions and duties between the several
deaprtments, however, sometimes makes it hard to say just where
the political excitement, the great landmarks of the Constitution are
apt to be forgotten or marred, if not entirely obliterated, in cases of
conflict, the judicial departments is the only constitutional organ
which can be called upon to determine the proper allocation of
powers between the several departments and among the integral or
constituent units thereof.
xxx xxx xxx
The Constitution is a definition of the powers of government. Who is
to determine the nature, scope and extent of such powers? The
Constitution itself has provided for the instrumentality of the judiciary
as the rational way. And when the judiciary mediates to allocate

constitutional boundaries; it does not assert any superiority over the


other departments; it does not inr eality nullify or invalidate an act of
the legislature, but only asserts the solemn and sacred obligation
assigned to it by tyhe Constitution to determine conflicting claims of
authority under the Constitution and to established for the parties in
an actual controversy the rights which that instrument secures and
guarantess to them. This is in thruth all that is involved in what is
termed "judicial supremacy" which properly is the power of judicial
review under the Constitution. Even the, this power of judicial review
is limited to actual cases and controversies to be exercised after full
opportunity of argument by the parties, and limited further to the
constitutional question raised or the very lis mota presented. Any
attempt at abstraction could only lead to dialectics and barren legal
questions and to sterile conclusions unrelated to actualities.
Narrowed as its function is in this manner, the judiciary does not
pass upon questions of wisdom, justice or expediency of legislation.
More thatn that, courts accord the presumption of constitutionality to
legislative enactments, not only because the legislature is presumed
to abide by the Constitution but also becuase the judiciary in the
determination of actual cases and controversies must reflect the
wisdom and justice of the people as expressed through their
representatives in the executive and legislative departments of the
government.
The "allocation of constituional boundaries" is a task that this Court must perfomr
under the Constitution. Moreowever, as held in a recent case, 12 "(t)he political question
doctrine neither interposes an obstacle to judicial determination of the rival claims. The jurisdiction to delimit constitutional boundaries has
been given to this Court. It cannot abdicate that obligation mandated by the 1987 Constitution, although said provision by no means does
away with kthe applicability of the principle in appropriate cases." 13

The Court is thus of the considered view that it has jurisdiction over the present
controversy for the purpose of determining the scope and extent of the power of
the Senate Blue Ribbon Committee to conduct inquiries into private affirs in
purported aid of legislation.
Coming to the specific issues raised in this case, petitioners contend that (1) the
Senate Blue Ribbon Committee's inquiry has no valid legislative purpose, i.e., it
is not done in aid of legislation; (2) the sale or disposition of hte Romualdez
corporations is a "purely private transaction" which is beyond the power of the
Senate Blue Ribbon Committee to inquire into; and (3) the inquiry violates their
right to due process.
The 1987 Constition expressly recognizes the power of both houses of Congress
to conduct inquiries in aid of legislation. 14 Thus, Section 21, Article VI thereof provides:

The Senate or the House of Representatives or any of its respective


committee may conduct inquiries in aid of legislation in accordance
with its duly published rules of procedure. The rights of persons
appearing in or affected by such inquiries shall be respected. 15
The power of both houses of Congress to conduct inquiries in aid of legislation is
not, therefore, absolute or unlimited. Its exercise is circumscribed by the aforequoted provision of the Constitution. Thus, as provided therein, the investigation
must be "in aid of legislation in accordance with its duly published rules of
procedure" and that "the rights of persons appearing in or affected by such
inquiries shall be respected." It follows then that the rights of persons under the
Bill of Rights must be respected, including the right to due process and the right
not to be compelled to testify against one's self.
The power to conduct formal inquiries or investigations in specifically provided for
in Sec. 1 of the Senate Rules of Procedure Governing Inquiries in Aid of
Legislation. Such inquiries may refer to the implementation or re-examination of
any law or in connection with any proposed legislation or the formulation of future
legislation. They may also extend to any and all matters vested by the
Constitution in Congress and/or in the Seante alone.
As held in Jean L. Arnault vs. Leon Nazareno, et al., 16 the inquiry, to be within the jurisdiction of the
legislative body making it, must be material or necessary to the exervise of a power in it vested by the Constitution, such as to legislate or to
expel a member.

Under Sec. 4 of the aforementioned Rules, the Senate may refer to any
committee or committees any speech or resolution filed by any Senator which in
tis judgment requires an appropriate inquiry in aid of legislation. In order
therefore to ascertain the character or nature of an inquiry, resort must be had to
the speech or resolution under which such an inquiry is proposed to be made.
A perusal of the speech of Senator Enrile reveals that he (Senator Enrile) made a
statement which was published in various newspapers on 2 September 1988
accusing Mr. Ricardo "Baby" Lopa of "having taken over the FMMC Group of
Companies." As a consequence thereof, Mr. Lopa wrote a letter to Senator Enrile
on 4 September 1988 categorically denying that he had "taken over " the FMMC
Group of Companies; that former PCGG Chairman Ramon Diaz himself
categorically stated in a telecast interview by Mr. Luis Beltran on Channel 7 on
31 August 1988 that there has been no takeover by him (Lopa); and that theses
repeated allegations of a "takeover" on his (Lopa's) part of FMMC are baseless
as they are malicious.
The Lopa reply prompted Senator Enrile, during the session of the Senate on 13
September 1988, to avail of the privilege hour, 17 so that he could repond to the said Lopa letter, and also

to vindicate his reputation as a Member of the Senate of the Philippines, considering the claim of Mr. Lopa that his (Enrile's) charges that he
(Lopa) had taken over the FMMC Group of Companies are "baseless" and "malicious." Thus, in his speech, 18 Senator Enrile said, among
others, as follows:

Mr. President, I rise this afternnon on a matter of personal privilege;


the privilege being that I received, Mr. President, a letter dated
September 4, 1988, signed by Mr. ricardo A. Lopa, a.k.a. or Baby
Lopa, wherein he denied categorically that he has taken over the
First Manila Management Group of Companies which includes
SOLOIL Incorporated.
xxx xxxx xxx
In answer to Mr. Lopa, I will quote pertinent portions from an Official
Memorandum to the Presidential Commission of Good Government
written and signed by former Governor, now Congressman Jose
Ramirez, in his capacity as head of the PCGG Task Force for
Region VIII. In his memorandum dated July 3, 1986, then Governor
Ramirez stated that when he and the members of his task force
sought to serve a sequestration order on the management of
SOLOIL in Tanauan, Leyte, management officials assured him that
relatives of the President of the Philippines were personally
discussing and representing SOLOIL so that the order of
sequestration would be lifted and that the new owner was Mr.
Ricardo A. Lopa.
I will quote the pertinent portions in the Ramire's memorandum.
The first paragraph of the memorandum reads as follows and I
quote, Mr. President:
"Our sequestration work of SOLOIL in Tanauan, Leyte
was not heeded by management because they said
another representation was being made to this
Commission for the ventual lifting of our
sequestrationorder. They even assured us that Mr.
Ricardo Lopa and Peping Cojunangco were personally
discussing and representing SOLOIL, so the order of
sequestration will finally be lifted. While we attempted to
carry on our order, management refused to cooperate
and vehemently turned down our request to make
available to us the records of the company. In fact it was
obviously clear that they will meet us with forcethe
moment we insist on doing normally our assigned task.

In view of the impending threat, and to avoid any


untoward incident we decided to temporarily suspend
our work until there is a more categorical stand of this
Commission in view of the seemingly influential
represetation being made by SOLOIL for us not to
continue our work."
Another pertinent portion of the same memorandum is paragraph
five, which reads as follows, and I quote Mr. President:
"The President, Mr. Gamboa, this is, I understand, the
President of SOLOIL, and the Plant Superintendent, Mr.
Jimenez including their chief counsel, Atty. Mandong
Mendiola are now saying that there have been
divestment, and that the new owner is now Mr. Ricardo
Lopa who according to them, is the brother-in-law of the
President. They even went further by telling us that
even Peping Cojuangco who we know is the brother of
her excellency is also interested in the ownership and
management of SOLOIL. When he demanded for
supporting papers which will indicate aforesaid
divestment, Messrs. Gamboa, Jimenez and Mendiola
refused vehemently to submit these papers to us,
instead they said it will be submitted directly to this
Commission. To our mind their continuous dropping of
names is not good for this Commission and even to the
President if our dersire is to achieve respectability and
stability of the government."
The contents of the memorandum of then Governor and now
Congressman Jose Ramirez were personally confirmed by him in a
news interview last September 7, 1988.
xxx xxxx xxx
Also relevant to this case, Mr. President, is a letter of Mr. Ricardo
Lopa himself in August 11, 1988 issue of the
newspaper Malaya headlined "On Alleged Takeover of Romualdez
Firms."
Mr. Lopa states in the last paragraph of the published letter and I
quote him:

12. As of this writing, the sales agreement is under


review by the PCGG solely to determine the appropriate
price. The sale of these companies and our prior rigtht
to requires them have never been at issue.
Perhaps I could not make it any clearer to Mr. Lopa that I was not
really making baseless and malicious statements.
Senator Enrile concluded his privilege speech in the following tenor:
Mr. President, it may be worthwhile for the Senate to look into the
possible violation of the law in the case particularly with regard to
Republic Act No. 3019, the Anti-Graft and Corrupt Practices Act,
Section 5 of which reads as follows and I quote:
Sec. 5. Prohibition on certain relatives. It shall be
unlawful for the spouse or for nay relative, by
consanguinity or affinity, within the third civil degree, of
the President of the Philippines, the Vice-President of
the Philippines, the President of the Senate, or the
Speaker of the House of Representatives, to intervene
directly or indirectly, in any business, transaction,
contract or application with the Government: Provided,
that this section shall not apply to any person who prior
to the assumption of office of any of the above officials
to whom he is related, has been already dealing with
the Government along the same line of business, nor to
any transaction, contract or application filed by him for
approval of which is not discretionary on the part of the
officials concerned but depends upon compliance with
requisites provided by law, nor to any act lawfully
performed in an official capacity or in the exercise of a
profession.
Mr. President, I have done duty to this Senate and to myself. I leave
it to this august Body to make its own conclusion.
Verily, the speech of Senator Enrile contained no suggestion of contemplated
legislation; he merely called upon the Senate to look into a possible violation of
Sec. 5 of RA No. 3019, otherwise known as "The Anti-Graft and Corrupt
Practices Act." I other words, the purpose of the inquiry to be conducted by
respondent Blue Ribbon commitee was to find out whether or not the relatives of
President Aquino, particularly Mr. ricardo Lopa, had violated the law in

connection with the alleged sale of the 36 or 39 corporations belonging to


Benjamin "Kokoy" Romualdez to the Lopaa Group. There appears to be,
therefore, no intended legislation involved.
The Court is also not impressed with the respondent Committee's argument that
the questioned inquiry is to be conducted pursuant to Senate Resolution No. 212.
The said resolution was introduced by Senator Jose D. Lina in view of the
representaions made by leaders of school youth, community groups and youth of
non-governmental organizations to the Senate Committee on Youth and Sports
Development, to look into the charges against the PCGG filed by three (3)
stockholders of Oriental petroleum, i.e., that it has adopted a "get-rich-quick
scheme" for its nominee-directors in a sequestered oil exploration firm.The
pertinent portion of Senate Resolution No. 212 reads as follows:
xxx xxx xxx
WHEREAS, recent developments have shown that no less than the
Solicitor-General has stated that the PCGG Chairman and at least
three Commissioners should resign and that the agency should rid
itself of "ineptness, incompetence and corruption" and that the
Sandiganbayan has reportedly ordered the PCGG to answer
charges filed by three stockholders of Oriental Petroleum that it has
adopted a "get-rich-quick scheme" for its nominee-directors in a
sequestered oil exploration firm;
WHEREAS, leaders of school youth, community groups and youth of
non-governmental organization had made representations to the
Senate Committee on Youth and Sports Development to look into
the charges against the PCGG since said agency is a symbol of the
changes expected by the people when the EDSA revolution took
place and that the ill-gotten wealth to be recovered will fund priority
projects which will benefit our people such as CARP, free education
in the elementary and secondary levels reforestration, and
employment generation for rural and urban workers;
WHEREAS, the government and the present leadeship must
demonstrate in their public and private lives integrity, honor and
efficient management of government services lest our youth become
disillusioned and lose hope and return to an Idelogy and form of
government which is repugnant to true freedom, democratic
participation and human rights: Now, therefore, be it.

Resolved by the Senate, That the activities of the Presidential


Commission on Good Government be investigated by the
appropriate Committee in connection with the implementation of
Section 26, Article XVIII of the Constitution. 19
Thus, the inquiry under Senate Resolution No. 212 is to look into the charges
against the PCGG filed by the three (3) stockholders of Oriental Petroleum in
connection with the implementation of Section 26, Article XVIII of the
Constitution.
It cannot, therefore, be said that the contemplated inquiry on the subject of the
privilege speech of Senator Juan Ponce Enrile, i.e., the alleged sale of the 36 (or
39) corporations belonging to Benjamin "Kokoy" Romualdez to the Lopa Group is
to be conducted pursuant to Senate Resolution No. 212 because, firstly, Senator
Enrile did not indict the PCGG, and, secondly, neither Mr. Ricardo Lopa nor the
herein petitioners are connected with the government but are private citizens.
It appeals, therefore, that the contemplated inquiry by respondent Committee is
not really "in aid of legislation" becuase it is not related to a purpose within the
jurisdiction of Congress, since the aim of the investigation is to find out whether
or not the ralatives of the President or Mr. Ricardo Lopa had violated Section 5
RA No. 3019, the "Anti-Graft and Corrupt Practices Act", a matter that appears
more within the province of the courts rather than of the legislature. Besides, the
Court may take judicial notice that Mr. Ricardo Lopa died during the pendency of
this case. In John T. Watkins vs. United States, 20 it was held held:
... The power of congress to conduct investigations in inherent in the
legislative process. That power is broad. it encompasses inquiries
concerning the administration of existing laws as well as proposed,
or possibly needed statutes. It includes surveys of defects in our
social,economic, or political system for the purpose of enabling
Congress to remedy them. It comprehends probes into departments
of the Federal Government to expose corruption, inefficiency or
waste. But broad asis this power of inquiry, it is not unlimited. There
is no general authority to expose the private affairs ofindividuals
without justification in terms of the functions of congress. This was
freely conceded by Solicitor General in his argument in this case.
Nor is the Congress a law enforcement or trial agency. These are
functions of the executive and judicial departments of government.
No inquiry is an end in itself; it must be related to and in furtherance
of a legitimate task of Congress. Investigations conducted soly for
the personal aggrandizement of the investigators or to "punish"
those investigated are indefensible. (emphasis supplied)

It can not be overlooked that when respondent Committee decide to conduct its
investigation of the petitioners, the complaint in Civil No. 0035 had already been
filed with the Sandiganbayan. A perusal of that complaint shows that one of its
principal causes of action against herein petitioners, as defendants therein, is the
alleged sale of the 36 (or 39) corporations belonging to Benjamin "Kokoy"
Romualdez. Since the issues in said complaint had long been joined by the filing
of petitioner's respective answers thereto, the issue sought to be investigated by
the respondent Commitee is one over which jurisdiction had been acquired by
the Sandiganbayan. In short, the issue had been pre-empted by that court. To
allow the respondent Committee to conduct its own investigation of an issue
already before the Sandiganbayan would not only pose the possibility of
conflicting judgments betweena legislative commitee and a judicial tribunal, but if
the Committee's judgment were to be reached before that of the Sandiganbayan,
the possibility of its influence being made to bear on the ultimate judgment of the
Sandiganbayan can not be discounted.
In fine, for the rspondent Committee to probe and inquire into the same
justiciable controversy already before the Sandiganbayan, would be an
encroachment into the exclusive domain of judicial jurisdiction that had much
earlier set in. In Baremblatt vs. United States, 21 it was held that:
Broad as it is, the power is not, howevern, without limitations. Since
congress may only investigate into those areas in which it may
potentially legislate or appropriate, it cannot inquire into matters
which are within the exclusive province of one of the other branches
of the government. Lacking the judicial power given to the Judiciary,
it cannot inquire into mattes that are exclusively the concern of the
Judiciary. Neither can it suplant the Executive in what exclusively
belongs to the Executive. ...
Now to another matter. It has been held that "a congressional committee's right
to inquire is 'subject to all relevant limitations placed by the Constitution on
governmental action,' including "'the relevant limitations of the Bill of Rights'." 22
In another case
... the mere semblance of legislative purpose would not justify an
inquiry in the face of the Bill of Rights. The critical element is the
exeistence of, and the weight to be ascribed to, the interest of the
Congress in demanding disclosures from an unwilling witness. We
cannot simply assume, however, that every congressional
investigation is justified by a public need that over-balances any
private rights affected. To do so would be to abdicate the

responsibility placed by the Constitution upon the judiciary to insure


that the Congress does not unjustifiably encroah upon an individual's
right to privacy nor abridge his liberty of speech, press, religion or
assembly. 23
One of the basic rights guaranteed by the Constitution to an individual is the right
against self-incrimination. 24 Thir right constured as the right to remain completely silent may be availed of by the
accused in a criminal case; but kit may be invoked by other witnesses only as questions are asked of them.

This distinction was enunciated by the Court in Romeo Chavez vs. The
Honorable Court of Appeals, et al. 25 thus
Petitioner, as accused, occupies a different tier of protection from an
ordinary witness. Whereas an ordinary witness may be compelled to
take the witness stand and claim the privilege as each question
requiring an incriminating answer is hot at him, an accused may
altother refuse to take the witness stand and refuse to answer any
all questions.
Moreover, this right of the accused is extended to respondents in administrative
investigations but only if they partake of the nature of a criminal proceeding or
analogous to a criminal proceeding. In Galman vs. Pamaran, 26 the Court reiterated the
doctrine in Cabal vs. Kapuanan (6 SCRA 1059) to illustrate the right of witnesses to invoke the right against self-incrimination not only in
criminal proceedings but also in all other types of suit

It was held that:


We did not therein state that since he is not an accused and the
case is not a criminal case, Cabal cannot refuse to take the witness
stand and testify, and that he can invoke his right against selfincrimination only when a question which tends to elicit an answer
that will incriminate him is propounded to him. Clearly then, it is not
the characeter of the suit involved but the nature of the proceedings
that controls. The privilege has consistenly been held to extend to all
proceedings sanctioned by law and to all cases in which punishment
is sought to be visited upon a witness, whether a party or not.
We do not here modify these doctrines. If we presently rule that petitioners may
not be compelled by the respondent Committee to appear, testify and produce
evidenc before it, it is only becuase we hold that the questioned inquiry is not in
aid of legislation and, if pursued, would be violative of the principle of separation
of powers between the legislative and the judicial departments of government,
ordained by the Constitution.

WHEREFORE, the petition is GRANTED. The Court holds that, under the facts,
including the circumtance that petitioners are presently impleaded as defendants
in a case before the Sandiganbayan, which involves issues intimately related to
the subject of contemplated inquiry before the respondet Committee, the
respondent Senate Blue Ribbon Committee is hereby enjoined from compelling
the petitioners and intervenor to testify before it and produce evidence at the said
inquiry.
SO ORDERED.
Fernan, C.J., Melencio-Herrera, Feliciano, Bidin, Grio-Aquino, Medialdea,
Regalado, Davide, Jr. and Romero, JJ., concur.

Bengzon v Senate Blue Ribbon Committee Digest


G.R. No. 89914 November 20, 1991
Padilla, J.:
Facts:
1. Petitioner was one of the defendants in a civil case filed by the government with the Sandiganbayan for
the alleged anomalous sale of Kokoy Romoaldez of several government corporations to the group of
Lopa, a brother-in-law of Pres. Aquino.
2.

By virtue of a privilege speech made by Sen. Enrile urging the Senate to look into the transactions, an
investigation was conducted by the Senate Blue Ribbon Committee. Petitioners and Ricardo Lopa were
subpoenaed by the Committee to appear before it and testify on "what they know" regarding the "sale
of thirty-six (36) corporations belonging to Benjamin "Kokoy" Romualdez."

3.

At the hearing, Lopa declined to testify on the ground that his testimony may "unduly prejudice" the
defendants in civil case before the Sandiganbayan.

4.

Petitioner filed for a TRO and/or injunctive relief claiming that the inquiry was beyond the jurisdiction of
the Senate. He contended that the Senate Blue Ribbon Committee acted in excess of its jurisdiction and
legislative purpose. One of the defendants in the case before the Sandiganbayan, Sandejas, filed with
the Court of motion for intervention. The Court granted it and required the respondent Senate Blue
Ribbon Committee to comment on the petition in intervention.
ISSUE: W/N the Blue Ribbon inquiry was in aid of legislation
NO.

1.

There appears to be no intended legislation involved. The purpose of the inquiry to be


conducted is not related to a purpose within the jurisdiction of Congress, it was conducted to find out
whether or not the relatives of President Aquino, particularly Mr. Lopa had violated RA 3019 in
connection with the alleged sale of the 36 or 39 corporations belonging to Benjamin "Kokoy" Romualdez
to the Lopa Group.

2.

The power of both houses of Congress to conduct inquiries in aid of legislation is not absolute or
unlimited. Its exercise is circumscribed by the Constitution. As provided therein, the investigation must
be "in aid of legislation in accordance with its duly published rules of procedure" and that "the rights of
persons appearing in or affected by such inquiries shall be respected." It follows then that the rights of
persons under the Bill of Rights must be respected, including the right to due process and the right not
to be compelled to testify against one's self.

3.

The civil case was already filed in the Sandiganbayan and for the Committee to probe and inquire into
the same justiciable controversy would be an encroachment into the exclusive domain of judicial
jurisdiction that had already earlier set in. The issue sought to be investigated has already been preempted by the Sandiganbayan. To allow the inquiry to continue would not only pose the possibility of
conflicting judgments between the legislative committee and a judicial tribunal.

4.

Finally, a congressional committees right to inquire is subject to all relevant limitations placed by the
Constitution on governmental action including the relevant limitations of the Bill of Rights. One of these
rights is the right of an individual to against self-incrimination. The right to remain silent is extended to
respondents in administrative investigations but only if it partakes of the nature of a criminal proceeding
or analogous to a criminal proceeding. Hence, the petitioners may not be compelled by respondent
Committee to appear, testify and produce evidence before it only because the inquiry is not in aid of
legislation and if pursued would be violative of the principle of separation of powers between the
legislative and the judicial departments of the government as ordained by the Constitution.

Republic of the Philippines


SUPREME COURT
Manila
EN BANC
G.R. No. L-3820

July 18, 1950

JEAN L. ARNAULT, petitioner,


vs.
LEON NAZARENO, Sergeant-at-arms, Philippine Senate, and EUSTAQUIO
BALAGTAS, Director of Prisons,respondents.
J.C. Orendain, Augusto Revilla, and Eduardo Arboleda for petitioner.
Office of the Solicitor General Felix Bautista Angelo, Lorenzo Sumulong, Lorenzo
Taada, and Vicente J. Francisco for respondents.
OZAETA, J.:

This is an original petition for habeas corpus to relieve the petitioner from his
confinement in the New Bilibid Prison to which he has been committed by virtue
of a resolution adopted by the Senate on May 15, 1950, which reads as follows:
Whereas, Jean L. Arnault refused to reveal the name of the person to
whom he gave the P440,000, as well as answer other pertinent questions
related to the said amount; Now, therefore, be it.
Resolved, that for his refusal to reveal the name of the person to whom he
gave the P440,000 Jean L. Arnault be committed to the custody of the
Sergeant-at-Arms and imprisoned in the New Bilibid Prison, Muntinlupa,
Rizal, until discharged by further order of the Senate or by the special
committee created by Senate Resolution No. 8, such discharge to be
ordered when he shall have purged the contempt by revealing to the
Senate or to the said special committee the name of the person to whom
he gave the P440,000, as well as answer other pertinent questions in
connection therewith.
The facts that gave rise to the adoption of said resolution, insofar as pertinent
here, may be briefly stated as follows:
In the latter part of October, 1949, the Philippine Government, through the Rural
Progress Administration, bought two estates known as Buenavista and
Tambobong for the sums of P4,500,000 and P500,000, respectively. Of the first
sum, P1,000,000 was paid to Ernest H. Burt, a nonresident American, thru his
attorney-in-fact in the Philippines, the Associated Estates, Inc., represented by
Jean L. Arnault, for alleged interest of the said Burt in the Buenavista Estate. The
second sum of P500,000 was all paid to the same Ernest H. Burt through his
other attorney-in-fact, the North Manila Development Co., Inc., also represented
by Jean L. Arnault, for the alleged interest of the said Burt in the Tambobong
Estate.
The original owner of the Buenavista Estate was the San Juan de Dios Hospital.
The Philippine Government held a 25-year lease contract on said estate, with an
option to purchase it for P3,000,000 within the same period of 25 years counted
from January 1, 1939. The occupation Republic of the Philippines purported to
exercise that option by tendering to the owner the sum of P3,000,000 and, upon
its rejection, by depositing it in court on June 21, 1944, together with the accrued
rentals amounting to P3224,000. Since 1939 the Government has remained in
possession of the estate.
On June 29, 1946, the San Juan de Dios Hospital sold the Buenavista Estate for
P5,000,000 to Ernest H. Burt, who made a down payment of P10,000 only and

agreed to pay P5000,000 within one year and the remainder in annual
installments of P500,000 each, with the stipulation that failure on his part to make
any of said payments would cause the forfeiture of his down payment of P10,000
and would entitle the Hospital to rescind to sale to him. Aside from the down
payment of P10,000, Burt has made no other payment on account of the
purchase price of said estate.
The original owner of the Tambobong Estate was the Philippine Trust Company.
On May 14, 1946, the Philippine Trust Company sold estate for the sum of
P1,200,000 to Ernest H. Burt, who paid P10,000 down and promise to pay
P90,000 within nine months and the balance of P1,100,000 in ten successive
installments of P110,000 each. The nine-month period within which to pay the
first installment of P90,000 expired on February 14, 1947, without Burt's having
paid the said or any other amount then or afterwards. On September 4, 1947, the
Philippine Trust Company sold, conveyed, and delivered the Tambobong Estate
to the Rural Progress Administration by an absolute deed of sale in consideration
of the sum of P750,000. On February 5, 1948, the Rural Progress Administration
made, under article 1504 of the Civil Code, a notarial demand upon Burt for the
resolution and cancellation of his contract of purchase with the Philippine Trust
Company due to his failure to pay the installment of P90,000 within the period of
nine months. Subsequently the Court of First Instance of Rizal ordered the
cancellation of Burt's certificate of title and the issuance of a new one in the
name of the Rural Progress Administration, from which order he appealed to the
Supreme Court.1
It was in the face of the antecedents sketched in the last three preceding
paragraphs that the Philippine Government, through the Secretary of Justice as
Chairman of the Board of Directors of the Rural Progress Administration and as
Chairman of the Board of Directors of the Philippine National Bank, from which
the money was borrowed, accomplished the purchase of the two estates in the
latter part of October, 1949, as stated at the outset.
On February 27, 1950, the Senate adopted its Resolution No. 8, which reads as
follows:
RESOLUTION CREATING A SPECIAL COMMITTEE TO INVESTIGATE
THE BUENAVISTA AND THE TAMBOBONG ESTATES DEAL.
WHEREAS, it is reported that the Philippine government, through the Rural
Progress Administration, has bought the Buenavista and the Tambobong
Estates for the aggregate sum of five million pesos;

WHEREAS, it is reported that under the decision of the Supreme Court


dated October 31, 1949, the Buenavista Estate could have been bought for
three million pesos by virtue of a contract entered into between the San
Juan de Dios Hospital and Philippine Government in 1939;
WHEREAS, it is even alleged that the Philippine Government did not have
to purchase the Buenavista Estate because the occupation government
had made tender of payment in the amount of three million pesos,
Japanese currency, which fact is believed sufficient to vest title of
Ownership in the Republic of the Philippines pursuant to decisions of the
Supreme Court sustaining the validity of payments made in Japanese
military notes during the occupation;
WHEREAS, it is reported that the Philippine Government did not have to
pay a single centavo for the Tambobong Estate as it was already
practically owned by virtue of a deed of sale from the Philippine Trust
Company dated September 3, 194, for seven hundred and fifty thousand
pesos, and by virtue of the recission of the contract through which Ernest
H. Burt had an interest in the estate; Now, therefore, be it.
RESOLVED, That a Special Committee, be, as it hereby is, created,
composed of five members to be appointed by the President of the Senate
to investigate the Buenavista and Tambobong Estate deals. It shall be the
duty of the said Committee to determine whether the said purchase was
honest, valid, and proper and whether the price involved in the deal was
fair and just, the parties responsible therefor, and any other facts the
Committee may deem proper in the premises. Said Committee shall have
the power to conduct public hearings; issue subpoena or subpoena duces
tecum to compel the attendance of witnesses or the production of
documents before it; and may require any official or employee of any
bureau, office, branch, subdivision, agency, or instrumentality of the
Government to assist or otherwise cooperate with the Special Committee
in the performance of its functions and duties. Said Committee shall submit
its report of findings and recommendations within two weeks from the
adoption of this Resolution.
The special committee created by the above resolution called and examined
various witnesses, among the most important of whom was the herein petitioner,
Jean L. Arnault. An intriguing question which the committee sought to resolve
was that involved in the apparent unnecessariness and irregularity of the
Government's paying to Burt the total sum of P1,500,000 for his alleged interest
of only P20,000 in the two estates, which he seemed to have forfeited anyway
long before October, 1949. The committee sought to determine who were

responsible for and who benefited from the transaction at the expense of the
Government.
Arnault testified that two checks payable to Burt aggregating P1,500,000 were
delivered to him on the afternoon of October 29, 1949; that on the same date he
opened a new account in the name of Ernest H. Burt with the Philippine National
Bank in which he deposited the two checks aggregating P1,500,000; and that on
the same occasion he draw on said account two checks; one for P500,000,
which he transferred to the account of the Associated Agencies, Inc., with the
Philippine National Bank, and another for P440,000 payable to cash, which he
himself cashed. It was the desire of the committee to determine the ultimate
recipient of this sum of P440,000 that gave rise to the present case.
At first the petitioner claimed before the Committee:
Mr. ARNAULT (reading from a note). Mr. Chairman, for questions involving
the disposition of funds, I take the position that the transactions were legal,
that no laws were being violated, and that all requisites had been complied
with. Here also I acted in a purely functional capacity of representative. I
beg to be excused from making answer which might later be used against
me. I have been assured that it is my constitutional right to refuse to
incriminate myself, and I am certain that the Honorable Members of this
Committee, who, I understand, are lawyers, will see the justness of my
position.
At as subsequent session of the committee (March 16) Senator De Vera, a
member of the committee, interrogated him as follows:
Senator DE VERA. Now these transactions, according to your own
typewritten statement, were legal?
Mr. ARNAULT. I believe so.
Senator DE VERA. And the disposition of that fund involved, according to
your own statement, did not violate any law?
Mr. ARNAULT. I believe so.
xxx

xxx

xxx

Senator DE VERA. So that if the funds were disposed of in such a manner


that no laws were violated, how is it that when you were asked by the

Committee to tell what steps you took to have this money delivered to Burt,
you refused to answer the questions, saying that it would incriminate you?
Mr. ARNAULT. Because it violates the rights of a citizen to privacy in his
dealings with other people.
xxx

xxx

xxx

Senator DE VERA. Are you afraid to state how the money was disposed of
because you would be incriminated, or you would be incriminating
somebody?
Mr. ARNAULT. I am not afraid; I simply stand on the privilege to dispose of
the money that has been paid to me as a result of a legal transaction
without having to account for any use of it.
But when in the same session the chairman of the committee, Senator
Sumulong, interrogated the petitioner, the latter testified as follows:
The CHAIRMAN. The other check of P440,000 which you also made on
October 29, 1949, is payable to cash; and upon cashing this P440,000 on
October 29, 1949, what did you do with that amount?
Mr. ARNAULT. I turned it over to a certain person.
The CHAIRMAN. The whole amount of P440,000?
Mr. ARNAULT. Yes.
The CHAIRMAN. Who was that certain person to whom you delivered
these P440,000 which you cashed on October 29, 1949?
Mr. ARNAULT. I don't remember the name; he was a representative of
Burt.
The CHAIRMAN. That representative of Burt to whom you delivered the
P440,000 was a Filipino?
Mr. ARNAULT. I don't know.
The CHAIRMAN. You do not remember the name of that representative of
Burt to whom you delivered this big amount of P440,000?
Mr. ARNAULT. I am not sure; I do not remember the name.

The CHAIRMAN. That certain person who represented Burt to whom you
delivered the big amount on October 29, 1949, gave you a receipt for the
amount?
Mr. ARNAULT. No.
The CHAIRMAN. Neither did you ask a receipt?
Mr. ARNAULT. I didn't ask.
The CHAIRMAN. And why did you give that certain person, representative
of Burt, this big amount of P440,000 which forms part of the P1- million
paid to Burt?
Mr. ARNAULT. Because I have instructions to that effect.
The CHAIRMAN. Who gave you the instruction?
Mr. ARNAULT. Burt.
The CHAIRMAN. Where is the instruction; was that in writing?
Mr. ARNAULT. No.
The CHAIRMAN. By cable?
Mr. ARNAULT. No.
The CHAIRMAN. In what form did you receive that instruction?
Mr. ARNAULT. Verbal instruction.
The CHAIRMAN. When did you receive this verbal instruction from Burt to
deliver these P440,000 to a certain person whose name you do not like to
reveal?
Mr. ARNAULT. I have instruction to comply with the request of the person.
The CHAIRMAN. Now, you said that instruction given to you by Burt was
verbal?
Mr. ARNAULT. Yes.
The CHAIRMAN. When was that instruction given to you by Burt?

Mr. ARNAULT. Long time ago.


The CHAIRMAN. In what year did Burt give you that verbal instruction;
when Burt was still here in the Philippines?
Mr. ARNAULT. Yes.
The CHAIRMAN. But at that time Burt already knew that he would receive
the money?
Mr. ARNAULT. No.
The CHAIRMAN. In what year was that when Burt while he was here in the
Philippines gave you the verbal instruction?
Mr. ARNAULT. In 1946.
The CHAIRMAN. And what has that certain person done for Burt to merit
receiving these P440,000?
Mr. ARNAULT. I absolutely do not know.
The CHAIRMAN. You do not know?
Mr. ARNAULT. I do not know.
The CHAIRMAN. Burt did not tell you when he gave you the verbal
instruction why that certain person should receive these P440,000?
Mr. ARNAULT. He did not tell me.
The CHAIRMAN. And Burt also authorized you to give this big amount to
that certain person without receipt?
Mr. ARNAULT. He told me that a certain person would represent him and
where could I meet him.
The CHAIRMAN. Did Burt know already that certain person as early as
1946?
Mr. ARNAULT. I presume much before that.

The CHAIRMAN. Did that certain person have any intervention in the
prosecution of the two cases involving the Buenavista and Tambobong
estates?
Mr. ARNAULT. Not that I know of.
The CHAIRMAN. Is that certain person related to any high government
official?
Mr. ARNAULT. No, I do not know.
The CHAIRMAN. Why can you not tell us the name of that certain person?
Mr. ARNAULT. Because I am not sure of his name; I cannot remember the
name.
The CHAIRMAN. When gave that certain person that P440,000 on
October 29, 1949, you knew already that person?
Mr. ARNAULT. Yes, I have seen him several times.
The CHAIRMAN. And the name of that certain person is a Filipino name?
Mr. ARNAULT. I would say Spanish name.
The CHAIRMAN. And how about his Christian name; is it also a Spanish
name?
Mr. ARNAULT. I am not sure; I think the initial is J.
The CHAIRMAN. Did he have a middle name?
Mr. ARNAULT. I never knew it.
The CHAIRMAN. And how about his family name which according to your
recollection is Spanish; can you remember the first letter with which that
family name begins?
Mr. ARNAULT. S, D or F.
The CHAIRMAN. And what was the last letter of the family name?
Mr. ARNAULT. I do not know.

The CHAIRMAN. Have you seen that person again after you have
delivered this P440,000?
Mr. ARNAULT. Yes.
The CHAIRMAN. Several times?
Mr. ARNAULT. Two or three times.
The CHAIRMAN. Here in Manila?
Mr. ARNAULT. Yes.
The CHAIRMAN. And in spite of the fact that you met that person two or
three times, you never were able to find out what was his name?
Mr. ARNAULT. If I knew, I would [have] taken it down. Mr. Peralta knows
my name; of course, we have not done business. Lots of people in Manila
know me, but they don't know my name, and I don't know them. They sa{ I
am "chiflado" because I don't know their names.
The CHAIRMAN. That certain person is a male or female?
Mr. ARNAULT. He is a male.
The CHAIRMAN. You are sure that he is a male at least?
Mr. ARNAULT. Let us say 38 or 40 years, more or less.
The CHAIRMAN. Can you give us, more or less, a description of that
certain person? What is his complexion: light, dark or light brown?
Mr. ARNAULT. He is like the gentleman there (pointing to Senator Cabili),
but smaller. He walks very straight, with military bearing.
The CHAIRMAN. Do you know the residence of that certain person to
whom you gave the P440,000?
Mr. ARNAULT. No.
The CHAIRMAN. During these frequent times that you met that certain
person, you never came to know his residence?
Mr. ARNAULT. No, because he was coming to the office.

The CHAIRMAN. How tall is that certain person?


Mr. ARNAULT. Between 5-2 and 5-6.
On May 15, 1950, the petitioner was haled before the bar of the Senate, which
approved and read to him the following resolution:
Be it resolved by the Senate of the Philippines in Session assembled:
That Jean L. Arnault, now at the bar of the Senate, be arraigned for
contempt consisting of contumacious acts committed by him during the
investigation conducted by the Special Committee created by Senate
Resolution No. 8 to probe the Tambobong and Buenavista estates deal of
October 21, 1949, and that the President of the Senate propounded to him
the following interrogatories:
1. What excuse have you for persistently refusing to reveal the name of the
person to whom you gave the P440,000 on October 29, 1949, a person
whose name it is impossible for you not to remember not only because of
the big amount of money you gave to him without receipt, but also by your
own statements you knew him as early as 1946 when General Ernest H.
Burt was still in the Philippines, you made two other deliveries of money to
him without receipt, and the last time you saw him was in December 1949?
Thereupon petitioner's attorney, Mr. Orendain, submitted for him a written answer
alleging that the questions were incriminatory in nature and begging leave to be
allowed to stand on his constitutional right not to be compelled to be a witness
against himself. Not satisfied with that written answer Senator Sumulong, over
the objection of counsel for the petitioner, propounded to the latter the following
question:
Sen. SUMULONG. During the investigation, when the Committee asked
you for the name of that person to whom you gave the P440,000, you said
that you can [could] not remember his name. That was the reason then for
refusing to reveal the name of the person. Now, in the answer that you
have just cited, you are refusing to reveal the name of that person to whom
you gave the P440,000 on the ground that your answer will be selfincriminating. Now, do I understand from you that you are abandoning your
former claim that you cannot remember the name of that person, and that
your reason now for your refusal to reveal the name of that person is that
your answer might be self-incriminating? In other words, the question is
this: What is your real reason for refusing to reveal the name of that person

to whom you gave the P440,000: that you do not remember his name or
that your answer would be self-incriminating?
xxx

xxx

xxx

Mr. ORENDAIN. Mr. President, we are begging for the rules of procedure
that the accused should not be required to testify unless he so desires.
The PRESIDENT. It is the duty of the respondent to answer the question.
The question is very clear. It does not incriminate him.
xxx

xxx

xxx

Mr. ARNAULT. I stand by every statement that I have made before the
Senate Committee on the first, second, and third hearings to which I was
made in my letter to this Senate of May 2, 1950, in which I gave all the
reasons that were in my powers to give, as requested. I cannot change
anything in those statements that I made because they represent the best
that I can do , to the best of my ability.
The PRESIDENT. You are not answering the question. The answer has
nothing to do with the question.
Sen. SUMULONG. I would like to remind you , Mr. Arnault, that the reason
that you gave during the investigation for not revealing the name of the
person to whom you gave the P440,000 is not the same reason that you
are now alleging because during the investigation you told us: "I do not
remember his name." But, now, you are now saying: "My answer might
incriminate me." What is your real position?
Mr. ARNAULT. I have just stated that I stand by my statements that I made
at the first, second, and third hearings. I said that I wanted to be excused
from answering the question. I beg to be excused from making any answer
that might be incriminating in nature. However, in this answer, if the detail
of not remembering the name of the person has not been included, it is an
oversight.
Sen. SUMULONG. Mr. Arnault, will you kindly answer a simple question:
Do you remember or not the name of the person to whom you gave the
P440,000?
Mr. ARNAULT. I do not remember .

Sen. SUMULONG. Now, if you do not remember the name of that person,
how can you say that your answer might be incriminating? If you do not
remember his name, you cannot answer the question; so how could your
answer be self-incriminating? What do you say to that?
Mr. ARNAULT. This is too complicated for me to explain. Please, I do not
see how to answer those questions. That is why I asked for a lawyer, so he
can help me. I have no means of knowing what the situation is about. I
have been in jail 13 days without communication with the outside. How
could I answer the question? I have no knowledge of legal procedure or
rule, of which I am completely ignorant.
xxx

xxx

xxx

Sen. SUMULONG. Mr. President, I ask that the question be answered.


The PRESIDENT. The witness is ordered to answer the question. It is very
clear. It does not incriminate the witness.
xxx

xxx

xxx

Mr. ARNAULT. I do not remember. I stand on my constitutional rights. I


beg to be excused from making further answer, please.
Sen. SUMULONG. In that mimeographed letter that you sent addressed to
the President of the Senate, dated May 2, 1950, you stated there that you
cannot reveal the name of the person to whom you gave the P440,000
because if he is a public official you might render yourself liable for
prosecution for bribery, and that if he is a private individual you might
render yourself liable for prosecution for slander. Why did you make those
statements when you cannot even tell us whether that person to whom you
gave the P440,000 is a public official or a private individual ? We are giving
you this chance to convince the Senate that all these allegations of yours
that your answers might incriminate you are given by you honestly or you
are just trying to make a pretext for not revealing the information desired
by the Senate.
The PRESIDENT. You are ordered to answer the question.
Mr. ARNAULT. I do not even understand the question. (The question is
restated and explained.)

Mr. ARNAULT. That letter of May 2, was prepared by a lawyer for me and
signed it. That is all I can say how I stand about this letter. I have no
knowledge myself enough to write such a letter, so I had to secure the help
of a lawyer to help me in my period of distress.
In that same session of the Senate before which the petitioner was called to
show cause why he should not be adjudged guilty of contempt of the Senate,
Senator Sumulong propounded to the petitioner questions tending to elicit
information from him as to the identity of the person to whom he delivered the
P440,000; but the petitioner refused to reveal it by saying that he did not
remember. The President of the Senate then propounded to him various
questions concerning his past activities dating as far back as when witness was
seven years of age and ending as recently as the post liberation period, all of
which questions the witness answered satisfactorily. In view thereof, the
President of the Senate also made an attempt to illicit the desired information
from the witness, as follows:
The PRESIDENT. Now I am convinced that you have a good memory.
Answer: Did you deliver the P440,000 as a gift, or of any consideration?
Mr. ARNAULT. I have said that I had instructions to deliver it to that
person, that is all.
The PRESIDENT. Was it the first time you saw that person?
Mr. ARNAULT. I saw him various times, I have already said.
The PRESIDENT. In spite of that, you do not have the least remembrance
of the name of that person?
Mr. ARNAULT. I cannot remember.
The PRESIDENT. How is it that you do not remember events that
happened a short time ago and, on the other hand, you remember events
that occurred during your childhood?
Mr. ARNAULT. I cannot explain.
The Senate then deliberated and adopted the resolution of May 15 hereinabove
quoted whereby the petitioner was committed to the custody of the Sergeant-atArms and imprisoned until "he shall have purged the contempt by revealing to the
Senate or to the aforesaid Special Committee the name of the person to whom

he gave the P440,000, as well as answer other pertinent questions in connection


therewith."
The Senate also adopted on the same date another resolution (No. 16) , to wit:
That the Special Committee created by Senate Resolution No. 8 be
empowered and directed to continue its investigation of the Tambobong
and Buenavista Estates deal of October 21, 1949, more particularly to
continue the examination of Jean L. Arnault regarding the name of the
person to whom he gave the P440,000 and other matters related
therewith.
The first session of the Second Congress was adjourned at midnight on May 18,
1950.
The case was argued twice before us. We have given its earnest and prolonged
consideration because it is the first of its kind to arise since the Constitution of
the Republic of the Philippines was adopted. For the first time this Court is called
upon to define the power of either House of Congress to punish a person not a
member for contempt; and we are fully conscious that our pronouncements here
will set an important precedent for the future guidance of all concerned.
Before discussing the specific issues raised by the parties, we deem it necessary
to lay down the general principles of law which form the background of those
issues.
Patterned after the American system, our Constitution vests the powers of the
Government in three independent but coordinate Departments Legislative,
Executive, and Judicial. The legislative power is vested in the Congress, which
consists of the Senate and the House of Representatives. (Section 1, Article VI.)
Each house may determine the rules of its proceedings, punish its Members for
disorderly behavior, and, with the concurrence of two-thirds of all its Members,
expel a Member. (Section 10, Article VI.) The judicial power is vested in the
Supreme Court and in such inferior courts as may be established by law.
(Section 1, Article VIII.) Like the Constitution of the United States, ours does not
contain an express provision empowering either of the two Houses of Congress
to punish nonmembers for contempt. It may also be noted that whereas in the
United States the legislative power is shared by and between the Congress of
the United States, on the one hand, and the respective legislatures of the
different States, on the other the powers not delegated to the United States by
the Constitution nor prohibited by it to States being reserved to the States,
respectively, or to the people in the Philippines, the legislative power is vested
in the Congress of the Philippines alone. It may therefore be said that the

Congress of the Philippines has a wider range of legislative field than the
Congress of the United States or any State Legislature. Our form of Government
being patterned after the American system the framers of our Constitution
having drawn largely from American institutions and practices we can, in this
case, properly draw also from American precedents in interpreting analogous
provisions of our Constitution, as we have done in other cases in the past.
Although there is no provision in the Constitution expressly investing either
House of Congress with power to make investigations and exact testimony to the
end that it may exercise its legislative functions as to be implied. In other words,
the power of inquiry with process to enforce it is an essential and
appropriate auxiliary to the legislative function. A legislative body cannot legislate
wisely or effectively in the absence of information respecting the conditions which
the legislation is intended to effect or change; and where the legislative body
does not itself possess the requisite information which is not infrequently true
recourse must be had to others who do possess it. Experience has shown that
mere requests for such information are often unavailing, and also that information
which is volunteered is not always accurate or complete; so some means of
compulsion is essential to obtain what is needed. (McGrain vs.Daugherty, 273
U.S., 135; 71 L. ed., 580; 50 A.L R., 1.) The fact that the Constitution expressly
gives to Congress the power to punish its Members for disorderly behavior, does
not by necessary implication exclude the power to punish for contempt any other
person. (Anderson vs. Dunn, 6, Wheaton, 204; 5 L. ed., 242.) But no person can
be punished for contumacy as a witness before either House, unless his
testimony is required in a matter into which that House has jurisdiction to inquire.
(Kilbourn vs. Thompson, 26 L. ed., 377.).
Since, as we have noted, the Congress of the Philippines has a wider range of
legislative field than either the Congress of the United States or a State
Legislature, we think it is correct to say that the field of inquiry into which it may
enter is also wider. It would be difficult to define any limits by which the subject
matter of its inquiry can be bounded. It is not necessary to do so in this case.
Suffice it to say that it must be coextensive with the range of the legislative
power.
In the present case the jurisdiction of the Senate, thru the Special Committee
created by it, to investigate the Buenavista and Tambobong Estates deal is not
challenged by the petitioner; and we entertain no doubt as to the Senate's
authority to do so and as to the validity of Resolution No. 8 hereinabove quoted.
The transaction involved a questionable and allegedly unnecessary and irregular
expenditure of no less than P5,000,000 of public funds, of which Congress is the
constitutional guardian. It also involved government agencies created by
Congress to regulate or even abolish. As a result of the yet uncompleted
investigation, the investigating committee has recommended and the Senate

approved three bills (1) prohibiting the Secretary of Justice or any other
department head from discharging functions and exercising powers other than
those attached to his own office, without ]previous congressional authorization;
(2) prohibiting brothers and near relatives of any President of the Philippines from
intervening directly or indirectly and in whatever capacity in transactions in which
the Government is a party, more particularly where the decision lies in the hands
of executive or administrative officers who are appointees of the President; and
(3) providing that purchases of the Rural Progress Administration of big landed
estates at a price of P100,000 or more, shall not become effective without
previous congressional confirmation.2
We shall now consider and pass upon each of the questions raised by the
petitioner in support of his contention that his commitment is unlawful.
First He contends that the Senate has no power to punish him for contempt for
refusing to reveal the name of the person to whom he gave the P440,000,
because such information is immaterial to, and will not serve, any intended or
purported legislation and his refusal to answer the question has not
embarrassed, obstructed, or impeded the legislative process. It is argued that
since the investigating committee has already rendered its report and has made
all its recommendations as to what legislative measures should be taken
pursuant to its findings, there is no necessity to force the petitioner to give the
information desired other than that mentioned in its report, to wit: "In justice to
Judge Quirino and to Secretary Nepomuceno, this atmosphere of suspicion that
now pervades the public mind must be dissipated, and it can only be done if
appropriate steps are taken by the Senate to compel Arnault to stop pretending
that he cannot remember the name of the person to whom he gave the P440,000
and answer the questions which will definitely establish the identity of that person
. . ." Senator Sumulong, Chairman of the Committee, who appeared and argued
the case for the respondents, denied that that was the only purpose of the
Senate in seeking the information from the witness. He said that the investigation
had not been completed, because, due to the contumacy of the witness, his
committee had not yet determined the parties responsible for the anomalous
transaction as required by Resolution No. 8; that, by Resolution No. 16, his
committee was empowered and directed to continue its investigation, more
particularly to continue its examination of the witness regarding the name of the
person to whom he gave the P440,000 and other matters related therewith; that
the bills recommended by his committee had not been approved by the House
and might not be approved pending the completion of the investigation; and that
those bills were not necessarily all the measures that Congress might deem it
necessary to pass after the investigation is finished.

Once an inquiry is admitted or established to be within the jurisdiction of a


legislative body to make, we think the investigating committee has the power to
require a witness to answer any question pertinent to that inquiry, subject of
course to his constitutional right against self-incrimination. The inquiry, to be
within the jurisdiction of the legislative body to make, must be material or
necessary to the exercise of a power in it vested by the Constitution, such as to
legislate, or to expel a Member; and every question which the investigator is
empowered to coerce a witness to answer must be material or pertinent to the
subject of the inquiry or investigation. So a witness may not be coerced to
answer a question that obviously has no relation to the subject of the inquiry. But
from this it does not follow that every question that may be propounded to a
witness must be material to any proposed or possible legislation. In other words,
the materiality of the question must be determined by its direct relation to any
proposed or possible legislation. The reason is, that the necessity or lack of
necessity for legislative action and the form and character of the action itself are
determined by the sum total of the information to be gathered as a result of the
investigation, and not by a fraction of such information elicited from a single
question.
In this connection, it is suggested by counsel for the respondents that the power
of the Court is limited to determining whether the legislative body has jurisdiction
to institute the inquiry or investigation; that once that jurisdiction is conceded, this
Court cannot control the exercise of that jurisdiction; and it is insinuated, that the
ruling of the Senate on the materiality of the question propounded to the witness
is not subject to review by this Court under the principle of the separation of
powers. We have to qualify this proposition. As was said by the Court of Appeals
of New York: "We are bound to presume that the action of the legislative body
was with a legitimate object if it is capable of being so construed, and we have no
right to assume that the contrary was intended." (People ex
rel. McDonald vs. Keeler, 99 N.Y., 463; 52 Am. Rep., 49; 2 N.E., 615, quoted
with approval by the Supreme Court of the United States in the said case of
McGrain vs. Daugherty, it is necessary deduction from the decision in Re
Chapman, 41 L. ed., 1154, that where the questions are not pertinent to the
matter under inquiry a witness rightfully may refuse to answer. So we are of the
opinion that where the alleged immateriality of the information sought by the
legislative body from a witness is relied upon to contest its jurisdiction, the court
is in duty bound to pass upon the contention. The fact that the legislative body
has jurisdiction or the power to make the inquiry would not preclude judicial
intervention to correct a clear abuse of discretion in the exercise of that power.
Applying the criterion laid down in the last two preceding paragraphs to the
resolution of the issue under consideration, we find that the question for the
refusal to answer which the petitioner was held in contempt by the Senate is

pertinent to the matter under inquiry. In fact, this is not and cannot be disputed.
Senate Resolution No. 8, the validity of which is not challenged by the petitioner,
requires the Special Committee, among other things, to determine the parties
responsible for the Buenavista and Tambobong estates deal, and it is obvious
that the name of the person to whom the witness gave the P440,000 involved in
said deal is pertinent to that determination it is in fact the very thing sought to
be determined. The contention is not that the question is impertinent to the
subject of the inquiry but that it has no relation or materiality to any proposed
legislation. We have already indicated that it is not necessary for the legislative
body to show that every question propounded to a witness is material to any
proposed or possible legislation; what is required is that is that it be pertinent to
the matter under inquiry.
It is said that the Senate has already approved the three bills recommended by
the Committee as a result of the uncompleted investigation and that there is no
need for it to know the name of the person to whom the witness gave the
P440,000. But aside from the fact that those bills have not yet been approved by
the lower house and by the President and that they may be withdrawn or
modified if after the inquiry is completed they should be found unnecessary or
inadequate, there is nothing to prevent the Congress from approving other
measures it may deem necessary after completing the investigation. We are not
called upon, nor is it within our province, to determine or imagine what those
measures may be. And our inability to do so is no reason for overruling the
question propounded by the Senate to the witness.
The case of Re Chapman , 166 U.S., 661; 41 L. ed., 1154, is in point here. The
inquiry there in question was conducted under a resolution of the Senate and
related to charges, published in the press, that senators were yielding to corrupt
influences in considering a tariff bill then before the Senate and were speculating
in stocks the value of which would be affected by pending amendments to the
bill. Chapman, a member of a firm of stock brokers dealing in the stock of the
American Sugar Refining Company, appeared before the committee in response
to a subpoena and asked, among others, the following questions:
Had the firm, during the month of March, 1894, bought or sold any stock or
securities, known as sugar stocks, for or in the interest, directly or
indirectly, of any United Senate senator?
Was the said firm at that time carrying any sugar stock for the benefit of, or
in the interest, directly or indirectly, of any United Senate senator?
He refused to answer the questions and was prosecuted under an Act of
Congress for contempt of the Senate. Upon being convicted and sent to jail he

petitioned the Supreme Court of the United States for a writ of habeas corpus.
One of the questions decided by the Supreme Court of the United States in that
case was whether the committee had the right to compel the witness to answer
said questions, and the Court held that the committee did have such right,
saying:
The questions were undoubtedly pertinent to the subject-matter of the
inquiry. The resolution directed the committee to inquire whether any
senator has been, or is, speculating in what are known as sugar stocks
during the consideration of the tariff bill now before the Senate." What the
Senate might or might not do upon the facts when ascertained, we cannot
say, nor are we called upon to inquire whether such ventures might be
defensible, as contended in argument, but is plain that negative answers
would have cleared that body of what the Senate regarded as offensive
imputations, while affirmative answers might have led to further action on
the part of the Senate within its constitutional powers. (Emphasis
supplied.)
It may be contended that the determination of the parties responsible for the deal
is incumbent upon the judicial rather than upon the legislative branch. But we
think there is no basis in fact or in law for such assumption. The petitioner has
not challenged the validity of Senate Resolution No. 8, and that resolution
expressly requires the committee to determine the parties responsible for the
deal. We are bound to presume that the Senate has acted in the due
performance of its constitutional function in instituting the inquiry, if the act is
capable of being so construed. On the other hand, there is no suggestion that the
judiciary has instituted an inquiry to determine the parties responsible for the
deal. Under the circumstances of the case, it appearing that the questioned
transaction was affected by the head of the Department of Justice himself, it is
not reasonable to expect that the Fiscal or the Court of First Instance of Manila
will take the initiative to investigate and prosecute the parties responsible for the
deal until and unless the Senate shall determined those parties are and shall
taken such measures as may be within its competence to take the redress the
wrong that may have been committed against the people as a result of the
transaction. As we have said, the transaction involved no less than P5,000,000 of
public funds. That certainly is a matter of a public concern which it is the duty of
the constitutional guardian of the treasury to investigate.
If the subject of investigation before the committee is within the range of
legitimate legislative inquiry and the proposed testimony of the witness called
relates to that subject, obedience, to its process may be enforced by the
committee by imprisonment. (Sullivan vs. Hill, 73 W. Va., 49; 79 S.E., 670; 40
Ann. Cas. [1916 B.], 1115.)

The decision in the case of Kilbourn vs. Thompson, 26 L. ed., 377, relied upon by
the petitioner, is not applicable here. In that case the inquiry instituted by the
House of Representatives of the United States related to a private real-estate
pool or partnership in the District of Columbia. Jay Cook and Company had had
an interest in the pool but become bankrupts, and their estate was in course of
administration in a federal bankruptcy court in Pennsylvania. The United States
was one of their creditors. The trustee in the bankruptcy proceeding had effected
a settlement of the bankrupts' interest in the pool, and of course his action was
subject to examination and approval or disapproval by the bankruptcy court.
Some of the creditors, including the United States, were dissatisfied with the
settlement. The resolution of the House directed the Committee "to inquire into
the nature and history of said real-estate pool and the character of said
settlement, with the amount of property involve, in which Jay Cooke and Co.
were interested, and the amount paid or to be paid in said settlement, with power
to send for persons and papers, and report to this House." The Supreme Court of
the United States, speaking thru Mr. Justice Miller, pointed out that the resolution
contained no suggestion of contemplated legislation; that the matter was one in
respect of which no valid legislation could be had; that the bankrupts' estate and
the trustee's settlement were still pending in the bankruptcy court; and that the
United States and other creditors were free to press their claims in that
proceeding. And on these grounds the court held that in undertaking the
investigation "the House of Representatives not only exceeded the limit of its own
authority, but assumed a power which could only be properly exercised by
another branch of the government, because the power was in its nature clearly
judicial." The principles announced and applied in that case are: that neither
House of Congress possesses a "general power of making inquiry into the
private affairs of the citizen"; that the power actually possessed is limited to
inquires relating to matters of which the particular House has jurisdiction, and in
respect of which it rightfully may take other action; that if the inquiry relates to a
matter wherein relief or redress could be had only by judicial proceeding, it is not
within the range of this power , but must be left to the court, conformably to the
constitutional separation of government powers.
That case differs from the present case in two important respects: (1) There the
court found that the subject of the inquiry, which related to a private real-estate
pool or partnership, was not within the jurisdiction of either House of Congress;
while here if it is not disputed that the subject of the inquiry, which relates to a
transaction involving a questionable expenditure by the Government of
P5,000,000 of public funds, is within the jurisdiction of the Senate, (2) There the
claim of the Government as a creditor of Jay Cooke and Company, which had
had an interest in the pool, was pending adjudication by the court; while here the
interposition of the judicial power on the subject of the inquiry cannot be
expected, as we have pointed out above, until after the Senate shall have

determined who the parties responsible are and shall have taken such measures
as may be within its competence to take to redress the wrong that may have
been committed against the people as a result of the transaction.
It is interesting to note that the decision in the case of Killbourn vs. Thompson
has evoked strong criticisms from legal scholars. (See Potts, Power of Legislative
Bodies to Punish for Contempt [1926], 74 U. Pa. L. Rev., 692-699; James L.
Land is, Constitutional Limitations on the Congressional Power of
Investigation [1926], 40 Harvard L. Rev., 153, 154, 214-220.) We quoted the
following from Professor Land is' criticism: "Mr. Justice Miller saw the case purely
as an attempt by the House to secure to the Government certain priority rights as
creditor of the bankrupt concern. To him it assumed the character of a lawsuit
between the Government and Jay Cooke and Co., with the Government, acting
through the House, attempting to override the orderliness of established
procedure and thereby prefer a creditors' bill not before the courts but before
Congress. That bankruptcy proceedings had already been instituted against Jay
Cooke and Co., in a federal court gave added impetus to such a conception. The
House was seeking to oust a court of prior acquired jurisdiction by an
extraordinary and unwarranted assumption of "judicial power"! The broader
aspect of the investigation had not been disclosed to the Court. That Jay Cooke
and Co.'s indebtedness and the particular funds in question were only part of the
great administrative problem connected with the use and disposition of public
monies, that the particular failure was of consequence mainly in relation to the
security demanded for all government deposits, that the facts connected with one
such default revealed the possibility of other and greater maladministration, such
considerations had not been put before the Court. Nor had it been acquainted
with the every-day nature of the particular investigation and the powers there
exerted by the House, powers whose exercise was customary and familiar in
legislative practice. Instead of assuming the character of an extraordinary judicial
proceeding, the inquiry, place in its proper background, should have been
regarded as a normal and customary part of the legislative process. Detailed
definiteness of legislative purpose was thus made the demand of the court in
Killbourn vs. Thompson. But investigators cannot foretell the results that may be
achieved. The power of Congress to exercise control over a real-estate pool is
not a matter for abstract speculation but one to be determined only after an
exhaustive examination of the problem. Relationship, and not their possibilities,
determine the extent of congressional power. Constitutionality depends upon
such disclosures. Their presence, whether determinative of legislative or judicial
power, cannot be relegated to guesswork. Neither Congress nor the Court can
predict, prior to the event, the result of the investigation."
The other case relied upon by the petitioner is Marshall vs. Gordon, 243 U.S.,
521; 61. ed., 881. The question there was whether the House of Representatives

exceeded its power in punishing, as for contempt of its authority, the District
Attorney of the Southern District of New York, who had written, published, and
sent to the chairman of one of its committees an ill-tempered and irritating letter
respecting the action and purposes of the committee in interfering with the
investigation by the grand jury of alleged illegal activities of a member of the
House of Representatives. Power to make inquires and obtain evidence by
compulsory process was not involved. The court recognized distinctly that the
House of Representatives had implied power to punish a person not a member
for contempt, but held that its action in this instance was without constitutional
justification. The decision was put on the ground that the letter, while offensive
and vexatious, was not calculated or likely to affect the House in any of its
proceedings or in the exercise of any of its functions. This brief statement of the
facts and the issues decided in that case is sufficient to show the inapplicability
thereof to the present case. There the contempt involved consisted in the district
attorney's writing to the chairman of the committee an offensive and vexatious
letter, while here the contempt involved consists in the refusal of the witness to
answer questions pertinent to the subject of an inquiry which the Senate has the
power and jurisdiction to make . But in that case, it was recognized that the
House of Representatives has implied power to punish a person not a member of
contempt. In that respect the case is applicable here in favor of the Senate's (and
not of the Petitioner's ) contention.
Second. It is next contended for the petitioner that the Senate lacks authority to
commit him for contempt for a term beyond its period of legislative session, which
ended on May 18, 1950. This contention is based on the opinion of Mr. Justice
Malcolm, concurred in by Justices Street and Villa-Real, in the case of Lopez vs.
De los Reyes (1930), 55 Phil., 170. In that case it appears that on October 23,
1929, Candido Lopez assaulted a member of the House of Representatives while
the latter was going to the hall of the House of Representatives to attend the
session which was then about to begin, as a result of which assault said
representative was unable to attend the sessions on that day and those of the
two days next following by reason of the threats which Candido Lopez made
against him. By the resolution of the House adopted November 6, 1929, Lopez
was declared guilty of contempt of the House of Representatives and ordered
punished by confinement in Bilibid Prison for a period of twenty-four hours. That
resolution was not complied with because the session of the House of
Representatives adjourned at midnight on November 8, 1929, and was reiterated
at the next session on September 16, 1930. Lopez was subsequently arrested,
whereupon he applied for the writ of habeas corpus in the Court of First Instance
of Manila, which denied the application. Upon appeal to the Supreme Court, six
justices voted to grant the writ: Justice Malcolm, Street, and Villa-real, on the
ground that the term of imprisonment meted out to the petitioner could not legally
be extended beyond the session of the body in which the contempt occurred; and

Justices Johns, Villamor, and Ostrand, on the ground that the Philippine
Legislature had no power to punish for contempt because it was a creature
merely of an Act of the Congress of the United States and not of a Constitution
adopted by the people. Chief Justice Avancea, Justice Johnson, and Justice
Romualdez wrote separate opinions, concurring with Justice Malcolm, Street,
and Villa-Real, that the Legislature had inherent power to punish for contempt but
dissenting from the opinion that the order of commitment could only be executed
during the particular session in which the act of contempt was committed.
Thus, on the question under consideration, the Court was equally divided and no
decisive pronouncement was made. The opinion of Mr. Justice Malcolm is based
mainly on the following passage in the case of Anderson vs.Dunn, supra:
And although the legislative power continues perpetual, the legislative
body ceases to exist on the moment of its adjournment or periodical
dissolution. It follows that imprisonment must terminate with that
adjournment.
as well as on the following quotation from Marshall vs. Gordon, supra:
And the essential nature of the power also makes clear the cogency and
application of the two limitations which were expressly pointed out in
Anderson vs. Dunn, supra, that is, that the power even when applied to
subjects which justified its exercise is limited to imprisonment and such
imprisonment may not be extended beyond the session of the body in
which the contempt occurred.
Interpreting the above quotations, Chief Justice Avancea held:
From this doctrine it follows, in my judgement, that the imposition of the
penalty is limited to the existence of the legislative body, which ceases to
function upon its final periodical dissolution. The doctrine refers to its
existence and not to any particular session thereof. This must be so,
inasmuch as the basis of the power to impose such penalty is the right
which the Legislature has to self-preservation, and which right is
enforceable during the existence of the legislative body. Many causes
might be conceived to constitute contempt to the Legislature, which would
continue to be a menace to its preservation during the existence of the
legislative body against which contempt was committed.
If the basis of the power of the legislature to punish for contempt exists
while the legislative body exercising it is in session, then that power and

the exercise thereof must perforce continue until the final adjournment and
the election of its successor.
Mr. Justice Johnson's more elaborate opinion, supported by quotations from
Cooley's Constitutional Limitations and from Jefferson's Manual, is to the same
effect. Mr. Justice Romualdez said: "In my opinion, where as in the case before
us, the members composing the legislative body against which the contempt was
committed have not yet completed their three-year term, the House may take
action against the petitioner herein."
We note that the quotations from Anderson vs. Dunn and Marshall vs. Gordon
relied upon by Justice Malcolm areobiter dicta. Anderson vs. Dunn was an action
of trespass against the Sergeant-at-Arms of the House of Representatives of the
United States for assault and battery and false imprisonment. The plaintiff had
been arrested for contempt of the House, brought before the bar of the House,
and reprimanded by the Speaker, and then discharged from custody. The
question as to the duration of the penalty was not involved in that case. The
question there was "whether the House of Representatives can take cognizance
of contempt committed against themselves, under any circumstances." The court
there held that the House of Representatives had the power to punish for
contempt, and affirmed the judgment of the lower court in favor of the defendant.
In Marshall vs. Gordon, the question presented was whether the House had the
power under the Constitution to deal with the conduct of the district attorney in
writing a vexatious letter as a contempt of its authority, and to inflict punishment
upon the writer for such contempt as a matter of legislative power. The court held
that the House had no such power because the writing of the letter did not
obstruct the performance of legislative duty and did not endanger the
preservation of the power of the House to carry out its legislative authority. Upon
that ground alone, and not because the House had adjourned, the court ordered
the discharge of the petitioner from custody.
The case where the question was squarely decided is
McGrain vs. Daugherty, supra. There it appears that the Senate had adopted a
resolution authorizing and directing a select committee of five senators to
investigate various charges of misfeasance and nonfeasance in the Department
of Justice after Attorney General Harry M. Daugherty became its supervising
head. In the course of the investigation the committee caused to be served on
Mally S. Daugherty, brother of Harry M. Daugherty and president of the Midland
National Bank of Washington Court House, Ohio, a subpoena commanding him
to appear before it for the purpose of giving testimony relating to the subject
under consideration. The witness failed to appear without offering any excuse for
his failure. The committee reported the matter to the Senate and the latter
adopted a resolution, "That the President of the Senate pro tempore issue his

warrant commanding the Sergeant-at-Arms or his deputy to take into custody the
body of the said M.S. Daugherty wherever found, and to bring the said M.S.
Daugherty before the bar of the Senate, then and there to answer such questions
pertinent to the matter under inquiry as the Senate may order the President of
the Senate pro tempore to propound; and to keep the said M.S. Daugherty in
custody to await the further order of the Senate." Upon being arrested, the
witness petitioned the federal court in Cincinnati for a writ of habeas corpus. The
federal court granted the writ and discharged the witness on the ground that the
Senate, in directing the investigation and in ordering the arrest, exceeded its
power under the Constitution. Upon appeal to the Supreme Court of the United
States, one of the contentions of the witness was that the case ha become moot
because the investigation was ordered and the committee was appointed during
the Sixty-eighth Congress, which expired on March 4, 1926. In overruling the
contention, the court said:
. . . The resolution ordering the investigation in terms limited the
committee's authority to the period of the Sixty-eighth Congress; but this
apparently was changed by a later and amendatory resolution authorizing
the committee to sit at such times and places as it might deem advisable
or necessary. It is said in Jefferson's Manual: "Neither House can continue
any portion of itself in any parliamentary function beyond the end of the
session without the consent of the other two branches. When done, it is by
a bill constituting them commissioners for the particular purpose." But the
context shows that the reference is to the two houses of Parliament when
adjourned by prorogation or dissolution by the King. The rule may be the
same with the House of Representatives whose members are all elected
for the period of a single Congress: but it cannot well be the same with the
Senate, which is a continuing body whose members are elected for a term
of six years and so divided into classes that the seats of one third only
become vacant at the end of each Congress, two thirds always continuing
into the next Congress, save as vacancies may occur through death or
resignation.
Mr. Hinds in his collection of precedents, says: "The Senate, as a
continuing body, may continue its committees through the recess following
the expiration of a Congress;" and, after quoting the above statement from
Jefferson's Manual, he says: "The Senate, however being a continuing
body, gives authority to its committees during the recess after the
expiration of a Congress." So far as we are advised the select committee
having this investigation in charge has neither made a final report nor been
discharged; nor has been continued by an affirmative order. Apparently its
activities have been suspended pending the decision of this case. But, be
this as it may, it is certain that the committee may be continued or revived

now by motion to that effect, and if, continued or revived, will have all its
original powers. This being so, and the Senate being a continuing body,
the case cannot be said to have become moot in the ordinary sense. The
situation is measurably like that in Southern P. Terminal Co. vs. Interstate
Commerce Commission, 219 U. S., 498, 514-516; 55 L. ed., 310, 315,
316; 31 Sup. Ct. Rep., 279, where it was held that a suit to enjoin the
enforcement of an order of the Interstate Commerce Commission did not
become moot through the expiration of the order where it was capable of
repetition by the Commission and was a matter of public interest. Our
judgment may yet be carried into effect and the investigation proceeded
with from the point at which it apparently was interrupted by reason of
the habeas corpus proceedings. In these circumstances we think a
judgment should be rendered as was done in the case cited.
What has been said requires that the final order in the District Court
discharging the witness from custody be reversed.
Like the Senate of the United States , the Senate of the Philippines is a
continuing body whose members are elected for a term of six years and so
divided that the seats of only one-third become vacant every two years, twothirds always continuing into the next Congress save as vacancies may occur
thru death or resignation. Members of the House of Representatives are all
elected for a term of four years; so that the term of every Congress is four years.
The Second Congress of the Philippines was constituted on December 30, 1949,
and will expire on December 30, 1953. The resolution of the Senate committing
the Petitioner was adopted during the first session of the Second Congress,
which began on the fourth Monday of January and ended in May 18, 1950.
Had said resolution of commitment been adopted by the House of
Representatives, we think it could be enforced until the final adjournment of the
last session of the Second Congress in 1953. We find no sound reason to limit
the power of the legislative body to punish for contempt to the end of every
session and not to the end of the last session terminating the existence of that
body. The very reason for the exercise of the power to punish for contempt is to
enable the legislative body to perform its constitutional function without
impediment or obstruction. Legislative functions may be and in practice are
performed during recess by duly constituted committees charged with the duty of
performing investigations or conducting hearing relative to any proposed
legislation. To deny to such committees the power of inquiry with process to
enforce it would be to defeat the very purpose for which that the power is
recognized in the legislative body as an essential and appropriate auxiliary to is
legislative function. It is but logical to say that the power of self-preservation is
coexistent with the life to be preserved.

But the resolution of commitment here in question was adopted by the Senate,
which is a continuing body and which does not cease exist upon the periodical
dissolution of the Congress or of the House of Representatives. There is no limit
as to time to the Senate's power to punish for contempt in cases where that
power may constitutionally be exerted as in the present case.
Mere reflection upon the situation at hand convinces us of the soundness of this
proposition. The Senate has ordered an investigation of the Buenavista and
Tambobong estates deal, which we have found it is within its competence to
make. That investigation has not been completed because of the refusal of the
petitioner as a witness to answer certain questions pertinent to the subject of the
inquiry. The Senate has empowered the committee to continue the investigation
during the recess. By refusing to answer the questions, the witness has
obstructed the performance by the Senate of its legislative function, and the
Senate has the power to remove the obstruction by compelling the witness to
answer the questions thru restraint of his liberty until he shall have answered
them. That power subsists as long as the Senate, which is a continuing body,
persists in performing the particular legislative function involved. To hold that it
may punish the witness for contempt only during the session in which
investigation was begun, would be to recognize the right of the Senate to perform
its function but at the same time to deny to it an essential and appropriate means
for its performance. Aside from this, if we should hold that the power to punish for
contempt terminates upon the adjournment of the session, the Senate would
have to resume the investigation at the next and succeeding sessions and repeat
the contempt proceedings against the witness until the investigation is
completed-an absurd, unnecessary, and vexatious procedure, which should be
avoided.
As against the foregoing conclusion it is argued for the petitioner that the power
may be abusively and oppressively exerted by the Senate which might keep the
witness in prison for life. But we must assume that the Senate will not be
disposed to exert the power beyond its proper bounds. And if, contrary to this
assumption, proper limitations are disregarded, the portals of this Court are
always open to those whose rights might thus be transgressed.
Third. Lastly, the petitioner invokes the privilege against self-incrimination. He
contends that he would incriminate himself if he should reveal the name of the
person to whom he gave the P440,000 if that person be a public official be
(witness) might be accused of bribery, and if that person be a private individual
the latter might accuse him of oral defamation.
The ground upon which the witness' claim is based is too shaky, in firm, and
slippery to afford him safety. At first he told the Committee that the transactions

were legal, that no laws were violated, and that all requisites had been replied
with; but at the time he begged to be excused from making answers "which might
later be used against me." A little later he explained that although the
transactions were legal he refused to answer questions concerning them
"because it violates the right of a citizen to privacy in his dealings with other
people . . . I simply stand on my privilege to dispose of the money that has been
paid to me as a result of a legal transaction without having to account for the use
of it." But after being apparently convinced by the Committee that his position
was untenable, the witness testified that, without securing any receipt, he turned
over the P440,000 to a certain person, a representative of Burt, in compliance
with Burt's verbal instruction made in 1946; that as far as he know, that certain
person had nothing to do with the negotiations for the settlement of the
Buenavista and Tambobong cases; that he had seen that person several times
before he gave him the P440,000 on October 29, 1949, and that since then he
had seen him again two or three times, the last time being in December, 1949, in
Manila; that the person was a male, 39 to 40 years of age, between 5 feet, 2
inches and 5 feet, 6 inches in height. Butt the witness would not reveal the name
of that person on these pretexts: " I don't remember the name; he was a
representative of Burt." "I am not sure; I don't remember the name."
We are satisfied that those answers of the witness to the important question,
what is the name of that person to whom you gave the P440,000? were
obviously false. His insistent claim before the bar of the Senate that if he should
reveal the name he would incriminate himself, necessarily implied that he knew
the name. Moreover, it is unbelievable that he gave the P440,000 to a person to
him unknown.
"Testimony which is obviously false or evasive is equivalent to a refusal to testify
and is punishable as contempt, assuming that a refusal to testify would be so
punishable." (12 Am. Jur., sec. 15, Contempt, pp. 399-400.) In the case of
Mason vs. U.S., 61 L. ed., 1198, it appears that Mason was called to testify
before a grand jury engaged in investigating a charge of gambling against six
other men. After stating that he was sitting at a table with said men when they
were arrested, he refused to answer two questions, claiming so to do might tend
to incriminate him: (1) "Was there a game of cards being played on this particular
evening at the table at which you are sitting?" (2) "Was there a game of cards
being played at another table at this time?" The foreman of the grand jury
reported the matter to the judge, who ruled "that each and all of said questions
are proper and that the answers thereto would not tend to incriminate the
witness." Mason was again called and refused to answer the first question
propounded to him, but, half yielding to frustration, he said in response to the
second question: "I don't know." In affirming the conviction for contempt, the
Supreme Court of the United States among other things said:

In the present case, the witness certainly were not relieved from answering
merely because they declared that so to do might incriminate them. The
wisdom of the rule in this regard is well illustrated by the enforced answer,
"I don't know ," given by Mason to the second question, after he had
refused to reply under a claim of constitutional privilege.
Since according to the witness himself the transaction was legal, and that he
gave the P440,000 to a representative of Burt in compliance with the latter's
verbal instruction, we find no basis upon which to sustain his claim that to reveal
the name of that person might incriminate him. There is no conflict of authorities
on the applicable rule, to wit:
Generally, the question whether testimony is privileged is for the
determination of the Court. At least, it is not enough for the witness to say
that the answer will incriminate him. as he is not the sole judge of his
liability. The danger of self-incrimination must appear reasonable and real
to the court, from all the circumstances, and from the whole case, as well
as from his general conception of the relations of the witness. Upon the
facts thus developed, it is the province of the court to determine whether a
direct answer to a question may criminate or not. . . . The fact that the
testimony of a witness may tend to show that he has violated the law is not
sufficient to entitle him to claim the protection of the constitutional provision
against self-incrimination, unless he is at the same time liable to
prosecution and punishment for such violation. The witness cannot assert
his privilege by reason of some fanciful excuse, for protection against an
imaginary danger, or to secure immunity to a third person. ( 3
Wharton's Criminal Evidence, 11th ed., secs. 1135,1136.)
It is the province of the trial judge to determine from all the facts and
circumstances of the case whether the witness is justified in refusing to
answer. (People vs. Gonzo, 23 N.E. [2d], 210 [Ill. App., 1939].) A witness is
not relieved from answering merely on his own declaration that an answer
might incriminate him, but rather it is for the trial judge to decide that
question. (Mason vs. U.S., 244 U. S., 362; 61 L. ed., 1193, 1200.)
As against witness's inconsistent and unjustified claim to a constitutional right, is
his clear duty as a citizen to give frank, sincere, and truthful testimony before a
competent authority. The state has the right to exact fulfillment of a citizen's
obligation, consistent of course with his right under the Constitution. The witness
in this case has been vociferous and militant in claiming constitutional rights and
privileges but patently recreant to his duties and obligations to the Government
which protects those rights under the law. When a specific right and a specific
obligation conflict with each other, and one is doubtful or uncertain while the

other is clear and imperative, the former must give way to the latter. The right to
life is one of the most sacred that the citizen may claim, and yet the state may
deprive him of it if he violates his corresponding obligation to respect the life of
others. As Mr. Justice Johnson said in Anderson vs. Dunn: "The wretch beneath
the gallows may repine at the fate which awaits him, and yet it is not certain that
the laws under which he suffers were made for the security." Paraphrasing and
applying that pronouncement here, the petitioner may not relish the restraint of
his liberty pending the fulfillment by him of his duty, but it is no less certain that
the laws under which his liberty is restrained were made for his welfare.
From all the foregoing, it follows that the petition must be denied, and it is so
ordered, with costs.
Paras, Pablo, Bengzon, Montemayor, and Reyes, JJ., concur.
Arnault v Nazareno digest
G.R. No. L-3820 July 18, 1950
Ozaeta, J.:
Topic: Legislative inquiry
Facts:
1. The controversy arose out of the Governments purchase of 2 estates.
Petitioner was the attorney in-fact of Ernest H. Burt in the negotiations for the
purchase of the Buenavista and Tambobong Estates by the Government of the
Philippines. The purchase was effected and the price paid for both estates was
P5,000,000. The Senate adopted Resolution No. 8 creating a Special Committee
to determine the validity of the purchase and whether the price paid was fair and
just. During the said Senate investigation, petitioner was asked to whom a part of
the purchase price, or P440,000, was delivered. Petitioner refused to answer this
question, hence the Committee cited him in contempt for contumacious acts and
ordered his commitment to the custody of the Sergeant at-arms of the Philippines
Senate and imprisoned in the new Bilibid Prison he reveals to the Senate or to
the Special Committee the name of the person who received the P440,000 and
to answer questions pertinent thereto.

2. It turned out that the Government did not have to pay a single centavo for the
Tambobong Estate as it was already practically owned by virtue of a deed of sale
from the Philippine Trust Company and by virtue of the recession of the contract
through which Ernest H. Burt had an interest in the estate. An intriguing question
which the committee sought to resolve was that involved in the apparent
irregularity of the Government's paying to Burt the total sum of P1,500,000 for his
alleged interest of only P20,000 in the two estates, which he seemed to have
forfeited anyway long before October, 1949. The committee sought to determine
who were responsible for and who benefited from the transaction at the expense
of the Government.
3. Arnault testified that two checks payable to Burt aggregating P1,500,000 were
delivered to him; and that on the same occasion he draw on said account two
checks; one for P500,000, which he transferred to the account of the Associated
Agencies, Inc., with PNB, and another for P440,000 payable to cash, which he
himself cashed.
4. Hence, this petition on following grounds:
a)

Petitioner contends that the Senate has no power to punish him for contempt for
refusing to reveal the name of the person to whom he gave the P440,000,
because such information is immaterial to, and will not serve, any intended or
purported legislation and his refusal to answer the question has not
embarrassed, obstructed, or impeded the legislative process.
b) Petitioner contended that the Senate lacks authority to commit him for
contempt for a term beyond its period of legislative session, which ended on May
18, 1950.
c) Also contended that he would incriminate himself if he should reveal the
name of the person

ISSUE: W/N either House of Congress has the power to punish a person
not a member for contempt

YES.
Once an inquiry is admitted or established to be within the jurisdiction of a
legislative body to make, the investigating committee has the power to require a
witness to answer any question pertinent to that inquiry, subject of course to his
constitutional right against self-incrimination. The inquiry, to be within the
jurisdiction of the legislative body to make, must be material or necessary to the
exercise of a power in it vested by the Constitution, such as to legislate, or to
expel a Member; and every question which the investigator is empowered to
coerce a witness to answer must be material or pertinent to the subject of the
inquiry or investigation. So a witness may not be coerced to answer a question
that obviously has no relation to the subject of the inquiry. Note that, the fact that
the legislative body has jurisdiction or the power to make the inquiry would not
preclude judicial intervention to correct a clear abuse of discretion in the exercise
of that power.
It is not necessary for the legislative body to show that every question
propounded to a witness is material to any proposed or possible legislation; what
is required is that is that it be pertinent to the matter under inquiry.
As to the self-incrimination issue, as against witness's inconsistent and
unjustified claim to a constitutional right, is his clear duty as a citizen to give
frank, sincere, and truthful testimony before a competent authority. The state has
the right to exact fulfillment of a citizen's obligation, consistent of course with his
right under the Constitution.
The resolution of commitment here in question was adopted by the Senate,
which is a continuing body and which does not cease exist upon the periodical
dissolution of the Congress or of the House of Representatives. There is no limit
as to time to the Senate's power to punish for contempt in cases where that
power may constitutionally be exerted as in the present case. That power
subsists as long as the Senate, which is a continuing body, persists in performing
the particular legislative function involved.

Republic of the Philippines


SUPREME COURT
Manila
EN BANC
G.R. No. 169777*

April 20, 2006

SENATE OF THE PHILIPPINES, represented by FRANKLIN M. DRILON, in


his capacity as Senate President, JUAN M. FLAVIER, in his capacity as
Senate President Pro Tempore, FRANCIS N. PANGILINAN, in his capacity
as Majority Leader, AQUILINO Q. PIMENTEL, JR., in his capacity as
Minority Leader, SENATORS RODOLFO G. BIAZON, "COMPANERA" PIA S.
CAYETANO, JINGGOY EJERCITO ESTRADA, LUISA "LOI" EJERCITO
ESTRADA, JUAN PONCE ENRILE, RICHARD J. GORDON, PANFILO M.
LACSON, ALFREDO S.LIM, M. A. MADRIGAL, SERGIO OSMENA III, RALPH
G. RECTO, and MAR ROXAS, Petitioners,
vs.
EDUARDO R. ERMITA, in his capacity as Executive Secretary and alter-ego
of President Gloria Macapagal-Arroyo, and anyone acting in his stead and
in behalf of the President of the Philippines, Respondents.
x-------------------------x
G.R. No. 169659

April 20, 2006

BAYAN MUNA represented by DR. REYNALDO LESACA, JR., Rep. SATUR


OCAMPO, Rep. CRISPIN BELTRAN, Rep. RAFAEL MARIANO, Rep. LIZA
MAZA, Rep. TEODORO CASINO, Rep. JOEL VIRADOR, COURAGE
represented by FERDINAND GAITE, and COUNSELS FOR THE DEFENSE
OF LIBERTIES (CODAL) represented by ATTY. REMEDIOS
BALBIN, Petitioners,
vs.
EDUARDO ERMITA, in his capacity as Executive Secretary and alter-ego of
President Gloria Macapagal-Arroyo, Respondent.
x-------------------------x
G.R. No. 169660

April 20, 2006

FRANCISCO I. CHAVEZ, Petitioner,


vs.
EDUARDO R. ERMITA, in his capacity as Executive Secretary, AVELINO J.

CRUZ, JR., in his capacity as Secretary of Defense, and GENEROSO S.


SENGA, in his capacity as AFP Chief of Staff, Respondents.
x-------------------------x
G.R. No. 169667

April 20, 2006

ALTERNATIVE LAW GROUPS, INC. (ALG), Petitioner,


vs.
HON. EDUARDO R. ERMITA, in his capacity as Executive
Secretary, Respondent.
x-------------------------x
G.R. No. 169834

April 20, 2006

PDP- LABAN, Petitioner,


vs.
EXECUTIVE SECRETARY EDUARDO R. ERMITA, Respondent.
x-------------------------x
G.R. No. 171246

April 20, 2006

JOSE ANSELMO I. CADIZ, FELICIANO M. BAUTISTA, ROMULO R. RIVERA,


JOSE AMOR AMORANDO, ALICIA A. RISOS-VIDAL, FILEMON C. ABELITA
III, MANUEL P. LEGASPI, J. B. JOVY C. BERNABE, BERNARD L. DAGCUTA,
ROGELIO V. GARCIA, and the INTEGRATED BAR FOR THE
PHILIPPINES,Petitioners,
vs.
HON. EXECUTIVE SECRETARY EDUARDO R. ERMITA, Respondent.
DECISION
CARPIO MORALES, J.:
A transparent government is one of the hallmarks of a truly republican state.
Even in the early history of republican thought, however, it has been recognized
that the head of government may keep certain information confidential in pursuit
of the public interest. Explaining the reason for vesting executive power in only
one magistrate, a distinguished delegate to the U.S. Constitutional Convention
said: "Decision, activity, secrecy, and dispatch will generally characterize the
proceedings of one man, in a much more eminent degree than the proceedings

of any greater number; and in proportion as the number is increased, these


qualities will be diminished."1
History has been witness, however, to the fact that the power to withhold
information lends itself to abuse, hence, the necessity to guard it zealously.
The present consolidated petitions for certiorari and prohibition proffer that the
President has abused such power by issuing Executive Order No. 464 (E.O. 464)
last September 28, 2005. They thus pray for its declaration as null and void for
being unconstitutional.
In resolving the controversy, this Court shall proceed with the recognition that the
issuance under review has come from a co-equal branch of government, which
thus entitles it to a strong presumption of constitutionality. Once the challenged
order is found to be indeed violative of the Constitution, it is duty-bound to
declare it so. For the Constitution, being the highest expression of the sovereign
will of the Filipino people, must prevail over any issuance of the government that
contravenes its mandates.
In the exercise of its legislative power, the Senate of the Philippines, through its
various Senate Committees, conducts inquiries or investigations in aid of
legislation which call for, inter alia, the attendance of officials and employees of
the executive department, bureaus, and offices including those employed in
Government Owned and Controlled Corporations, the Armed Forces of the
Philippines (AFP), and the Philippine National Police (PNP).
On September 21 to 23, 2005, the Committee of the Senate as a whole issued
invitations to various officials of the Executive Department for them to appear on
September 29, 2005 as resource speakers in a public hearing on the railway
project of the North Luzon Railways Corporation with the China National
Machinery and Equipment Group (hereinafter North Rail Project). The public
hearing was sparked by a privilege speech of Senator Juan Ponce Enrile urging
the Senate to investigate the alleged overpricing and other unlawful provisions of
the contract covering the North Rail Project.
The Senate Committee on National Defense and Security likewise issued
invitations2 dated September 22, 2005 to the following officials of the AFP: the
Commanding General of the Philippine Army, Lt. Gen. Hermogenes C. Esperon;
Inspector General of the AFP Vice Admiral Mateo M. Mayuga; Deputy Chief of
Staff for Intelligence of the AFP Rear Admiral Tirso R. Danga; Chief of the
Intelligence Service of the AFP Brig. Gen. Marlu Q. Quevedo; Assistant
Superintendent of the Philippine Military Academy (PMA) Brig. Gen. Francisco V.
Gudani; and Assistant Commandant, Corps of Cadets of the PMA, Col.

Alexander F. Balutan, for them to attend as resource persons in a public hearing


scheduled on September 28, 2005 on the following: (1) Privilege Speech of
Senator Aquilino Q. Pimentel Jr., delivered on June 6, 2005 entitled "Bunye has
Provided Smoking Gun or has Opened a Can of Worms that Show Massive
Electoral Fraud in the Presidential Election of May 2005"; (2) Privilege Speech of
Senator Jinggoy E. Estrada delivered on July 26, 2005 entitled "The Philippines
as the Wire-Tapping Capital of the World"; (3) Privilege Speech of Senator
Rodolfo Biazon delivered on August 1, 2005 entitled "Clear and Present Danger";
(4) Senate Resolution No. 285 filed by Senator Maria Ana Consuelo Madrigal
Resolution Directing the Committee on National Defense and Security to
Conduct an Inquiry, in Aid of Legislation, and in the National Interest, on the Role
of the Military in the So-called "Gloriagate Scandal"; and (5) Senate Resolution
No. 295 filed by Senator Biazon Resolution Directing the Committee on
National Defense and Security to Conduct an Inquiry, in Aid of Legislation, on the
Wire-Tapping of the President of the Philippines.
Also invited to the above-said hearing scheduled on September 28 2005 was the
AFP Chief of Staff, General Generoso S. Senga who, by letter3 dated September
27, 2005, requested for its postponement "due to a pressing operational situation
that demands [his utmost personal attention" while "some of the invited AFP
officers are currently attending to other urgent operational matters."
On September 28, 2005, Senate President Franklin M. Drilon received from
Executive Secretary Eduardo R. Ermita a letter4 dated September 27, 2005
"respectfully request[ing] for the postponement of the hearing [regarding the
NorthRail project] to which various officials of the Executive Department have
been invited" in order to "afford said officials ample time and opportunity to study
and prepare for the various issues so that they may better enlighten the Senate
Committee on its investigation."
Senate President Drilon, however, wrote5 Executive Secretary Ermita that the
Senators "are unable to accede to [his request]" as it "was sent belatedly" and
"[a]ll preparations and arrangements as well as notices to all resource persons
were completed [the previous] week."
Senate President Drilon likewise received on September 28, 2005 a letter6 from
the President of the North Luzon Railways Corporation Jose L. Cortes, Jr.
requesting that the hearing on the NorthRail project be postponed or cancelled
until a copy of the report of the UP Law Center on the contract agreements
relative to the project had been secured.
On September 28, 2005, the President issued E.O. 464, "Ensuring Observance
of the Principle of Separation of Powers, Adherence to the Rule on Executive

Privilege and Respect for the Rights of Public Officials Appearing in Legislative
Inquiries in Aid of Legislation Under the Constitution, and For Other
Purposes,"7 which, pursuant to Section 6 thereof, took effect immediately. The
salient provisions of the Order are as follows:
SECTION 1. Appearance by Heads of Departments Before Congress. In
accordance with Article VI, Section 22 of the Constitution and to implement the
Constitutional provisions on the separation of powers between co-equal
branches of the government, all heads of departments of the Executive Branch of
the government shall secure the consent of the President prior to appearing
before either House of Congress.
When the security of the State or the public interest so requires and the
President so states in writing, the appearance shall only be conducted in
executive session.
SECTION. 2. Nature, Scope and Coverage of Executive Privilege.
(a) Nature and Scope. - The rule of confidentiality based on executive privilege is
fundamental to the operation of government and rooted in the separation of
powers under the Constitution (Almonte vs. Vasquez, G.R. No. 95367, 23 May
1995). Further, Republic Act No. 6713 or the Code of Conduct and Ethical
Standards for Public Officials and Employees provides that Public Officials and
Employees shall not use or divulge confidential or classified information officially
known to them by reason of their office and not made available to the public to
prejudice the public interest.
Executive privilege covers all confidential or classified information between the
President and the public officers covered by this executive order, including:
Conversations and correspondence between the President and the public official
covered by this executive order (Almonte vs. Vasquez G.R. No. 95367, 23 May
1995; Chavez v. Public Estates Authority, G.R. No. 133250, 9 July 2002);
Military, diplomatic and other national security matters which in the interest of
national security should not be divulged (Almonte vs. Vasquez, G.R. No. 95367,
23 May 1995; Chavez v. Presidential Commission on Good Government, G.R.
No. 130716, 9 December 1998).
Information between inter-government agencies prior to the conclusion of treaties
and executive agreements (Chavez v. Presidential Commission on Good
Government, G.R. No. 130716, 9 December 1998);

Discussion in close-door Cabinet meetings (Chavez v. Presidential Commission


on Good Government, G.R. No. 130716, 9 December 1998);
Matters affecting national security and public order (Chavez v. Public Estates
Authority, G.R. No. 133250, 9 July 2002).
(b) Who are covered. The following are covered by this executive order:
Senior officials of executive departments who in the judgment of the department
heads are covered by the executive privilege;
Generals and flag officers of the Armed Forces of the Philippines and such other
officers who in the judgment of the Chief of Staff are covered by the executive
privilege;
Philippine National Police (PNP) officers with rank of chief superintendent or
higher and such other officers who in the judgment of the Chief of the PNP are
covered by the executive privilege;
Senior national security officials who in the judgment of the National Security
Adviser are covered by the executive privilege; and
Such other officers as may be determined by the President.
SECTION 3. Appearance of Other Public Officials Before Congress. All public
officials enumerated in Section 2 (b) hereof shall secure prior consent of the
President prior to appearing before either House of Congress to ensure the
observance of the principle of separation of powers, adherence to the rule on
executive privilege and respect for the rights of public officials appearing in
inquiries in aid of legislation. (Emphasis and underscoring supplied)
Also on September 28, 2005, Senate President Drilon received from Executive
Secretary Ermita a copy of E.O. 464, and another letter8 informing him "that
officials of the Executive Department invited to appear at the meeting [regarding
the NorthRail project] will not be able to attend the same without the consent of
the President, pursuant to [E.O. 464]" and that "said officials have not secured
the required consent from the President." On even date which was also the
scheduled date of the hearing on the alleged wiretapping, Gen. Senga sent a
letter9 to Senator Biazon, Chairperson of the Committee on National Defense and
Security, informing him "that per instruction of [President Arroyo], thru the
Secretary of National Defense, no officer of the [AFP] is authorized to appear
before any Senate or Congressional hearings without seeking a written approval
from the President" and "that no approval has been granted by the President to

any AFP officer to appear before the public hearing of the Senate Committee on
National Defense and Security scheduled [on] 28 September 2005."
Despite the communications received from Executive Secretary Ermita and Gen.
Senga, the investigation scheduled by the Committee on National Defense and
Security pushed through, with only Col. Balutan and Brig. Gen. Gudani among all
the AFP officials invited attending.
For defying President Arroyos order barring military personnel from testifying
before legislative inquiries without her approval, Brig. Gen. Gudani and Col.
Balutan were relieved from their military posts and were made to face court
martial proceedings.
As to the NorthRail project hearing scheduled on September 29, 2005, Executive
Secretary Ermita, citing E.O. 464, sent letter of regrets, in response to the
invitations sent to the following government officials: Light Railway Transit
Authority Administrator Melquiades Robles, Metro Rail Transit Authority
Administrator Roberto Lastimoso, Department of Justice (DOJ) Chief State
Counsel Ricardo V. Perez, then Presidential Legal Counsel Merceditas
Gutierrez, Department of Transportation and Communication (DOTC)
Undersecretary Guiling Mamonding, DOTC Secretary Leandro Mendoza,
Philippine National Railways General Manager Jose Serase II, Monetary Board
Member Juanita Amatong, Bases Conversion Development Authority
Chairperson Gen. Narciso Abaya and Secretary Romulo L. Neri.10 NorthRail
President Cortes sent personal regrets likewise citing E.O. 464.11
On October 3, 2005, three petitions, docketed as G.R. Nos. 169659, 169660, and
169667, for certiorari and prohibition, were filed before this Court challenging the
constitutionality of E.O. 464.
In G.R. No. 169659, petitioners party-list Bayan Muna, House of Representatives
Members Satur Ocampo, Crispin Beltran, Rafael Mariano, Liza Maza, Joel
Virador and Teodoro Casino, Courage, an organization of government
employees, and Counsels for the Defense of Liberties (CODAL), a group of
lawyers dedicated to the promotion of justice, democracy and peace, all claiming
to have standing to file the suit because of the transcendental importance of the
issues they posed, pray, in their petition that E.O. 464 be declared null and void
for being unconstitutional; that respondent Executive Secretary Ermita, in his
capacity as Executive Secretary and alter-ego of President Arroyo, be prohibited
from imposing, and threatening to impose sanctions on officials who appear
before Congress due to congressional summons. Additionally, petitioners claim
that E.O. 464 infringes on their rights and impedes them from fulfilling their
respective obligations. Thus, Bayan Muna alleges that E.O. 464 infringes on its

right as a political party entitled to participate in governance; Satur Ocampo, et


al. allege that E.O. 464 infringes on their rights and duties as members of
Congress to conduct investigation in aid of legislation and conduct oversight
functions in the implementation of laws; Courage alleges that the tenure of its
members in public office is predicated on, and threatened by, their submission to
the requirements of E.O. 464 should they be summoned by Congress; and
CODAL alleges that its members have a sworn duty to uphold the rule of law,
and their rights to information and to transparent governance are threatened by
the imposition of E.O. 464.
In G.R. No. 169660, petitioner Francisco I. Chavez, claiming that his
constitutional rights as a citizen, taxpayer and law practitioner, are affected by
the enforcement of E.O. 464, prays in his petition that E.O. 464 be declared null
and void for being unconstitutional.
In G.R. No. 169667, petitioner Alternative Law Groups, Inc.12 (ALG), alleging that
as a coalition of 17 legal resource non-governmental organizations engaged in
developmental lawyering and work with the poor and marginalized sectors in
different parts of the country, and as an organization of citizens of the Philippines
and a part of the general public, it has legal standing to institute the petition to
enforce its constitutional right to information on matters of public concern, a right
which was denied to the public by E.O. 464,13 prays, that said order be declared
null and void for being unconstitutional and that respondent Executive Secretary
Ermita be ordered to cease from implementing it.
On October 11, 2005, Petitioner Senate of the Philippines, alleging that it has a
vital interest in the resolution of the issue of the validity of E.O. 464 for it stands
to suffer imminent and material injury, as it has already sustained the same with
its continued enforcement since it directly interferes with and impedes the valid
exercise of the Senates powers and functions and conceals information of great
public interest and concern, filed its petition for certiorari and prohibition,
docketed as G.R. No. 169777 and prays that E.O. 464 be declared
unconstitutional.
On October 14, 2005, PDP-Laban, a registered political party with members duly
elected into the Philippine Senate and House of Representatives, filed a similar
petition for certiorari and prohibition, docketed as G.R. No. 169834, alleging that
it is affected by the challenged E.O. 464 because it hampers its legislative
agenda to be implemented through its members in Congress, particularly in the
conduct of inquiries in aid of legislation and transcendental issues need to be
resolved to avert a constitutional crisis between the executive and legislative
branches of the government.

Meanwhile, by letter14 dated February 6, 2006, Senator Biazon reiterated his


invitation to Gen. Senga for him and other military officers to attend the hearing
on the alleged wiretapping scheduled on February 10, 2005. Gen. Senga replied,
however, by letter15 dated February 8, 2006, that "[p]ursuant to Executive Order
No. 464, th[e] Headquarters requested for a clearance from the President to
allow [them] to appear before the public hearing" and that "they will attend once
[their] request is approved by the President." As none of those invited appeared,
the hearing on February 10, 2006 was cancelled.16
In another investigation conducted jointly by the Senate Committee on
Agriculture and Food and the Blue Ribbon Committee on the alleged
mismanagement and use of the fertilizer fund under the Ginintuang Masaganang
Ani program of the Department of Agriculture (DA), several Cabinet officials were
invited to the hearings scheduled on October 5 and 26, November 24 and
December 12, 2005 but most of them failed to attend, DA Undersecretary
Belinda Gonzales, DA Assistant Secretary Felix Jose Montes, Fertilizer and
Pesticide Authority Executive Director Norlito R. Gicana,17 and those from the
Department of Budget and Management18 having invoked E.O. 464.
In the budget hearings set by the Senate on February 8 and 13, 2006, Press
Secretary and Presidential Spokesperson Ignacio R. Bunye,19 DOJ Secretary
Raul M. Gonzalez20 and Department of Interior and Local Government
Undersecretary Marius P. Corpus21 communicated their inability to attend due to
lack of appropriate clearance from the President pursuant to E.O. 464. During the
February 13, 2005 budget hearing, however, Secretary Bunye was allowed to
attend by Executive Secretary Ermita.
On February 13, 2006, Jose Anselmo I. Cadiz and the incumbent members of
the Board of Governors of the Integrated Bar of the Philippines, as taxpayers,
and the Integrated Bar of the Philippines as the official organization of all
Philippine lawyers, all invoking their constitutional right to be informed on matters
of public interest, filed their petition for certiorari and prohibition, docketed as
G.R. No. 171246, and pray that E.O. 464 be declared null and void.
All the petitions pray for the issuance of a Temporary Restraining Order enjoining
respondents from implementing, enforcing, and observing E.O. 464.
In the oral arguments on the petitions conducted on February 21, 2006, the
following substantive issues were ventilated: (1) whether respondents committed
grave abuse of discretion in implementing E.O. 464 prior to its publication in the
Official Gazette or in a newspaper of general circulation; and (2) whether E.O.
464 violates the following provisions of the Constitution: Art. II, Sec. 28, Art. III,
Sec. 4, Art. III, Sec. 7, Art. IV. Sec. 1, Art. VI, Sec. 21, Art. VI, Sec. 22, Art. XI,

Sec. 1, and Art. XIII, Sec. 16. The procedural issue of whether there is an actual
case or controversy that calls for judicial review was not taken up; instead, the
parties were instructed to discuss it in their respective memoranda.
After the conclusion of the oral arguments, the parties were directed to submit
their respective memoranda, paying particular attention to the following
propositions: (1) that E.O. 464 is, on its face, unconstitutional; and (2) assuming
that it is not, it is unconstitutional as applied in four instances, namely: (a) the so
called Fertilizer scam; (b) the NorthRail investigation (c) the Wiretapping activity
of the ISAFP; and (d) the investigation on the Venable contract.22
Petitioners in G.R. No. 16966023 and G.R. No. 16977724 filed their memoranda on
March 7, 2006, while those in G.R. No. 16966725 and G.R. No. 16983426 filed
theirs the next day or on March 8, 2006. Petitioners in G.R. No. 171246 did not
file any memorandum.
Petitioners Bayan Muna et al. in G.R. No. 169659, after their motion for extension
to file memorandum27 was granted, subsequently filed a manifestation28 dated
March 14, 2006 that it would no longer file its memorandum in the interest of
having the issues resolved soonest, prompting this Court to issue a Resolution
reprimanding them.29
Petitioners submit that E.O. 464 violates the following constitutional provisions:
Art. VI, Sec. 2130
Art. VI, Sec. 2231
Art. VI, Sec. 132
Art. XI, Sec. 133
Art. III, Sec. 734
Art. III, Sec. 435
Art. XIII, Sec. 16 36
Art. II, Sec. 2837
Respondents Executive Secretary Ermita et al., on the other hand, pray in their
consolidated memorandum38 on March 13, 2006 for the dismissal of the petitions
for lack of merit.

The Court synthesizes the issues to be resolved as follows:


1. Whether E.O. 464 contravenes the power of inquiry vested in Congress;
2. Whether E.O. 464 violates the right of the people to information on
matters of public concern; and
3. Whether respondents have committed grave abuse of discretion when
they implemented E.O. 464 prior to its publication in a newspaper of
general circulation.
Essential requisites for judicial review
Before proceeding to resolve the issue of the constitutionality of E.O. 464,
ascertainment of whether the requisites for a valid exercise of the Courts power
of judicial review are present is in order.
Like almost all powers conferred by the Constitution, the power of judicial review
is subject to limitations, to wit: (1) there must be an actual case or controversy
calling for the exercise of judicial power; (2) the person challenging the act must
have standing to challenge the validity of the subject act or issuance; otherwise
stated, he must have a personal and substantial interest in the case such that he
has sustained, or will sustain, direct injury as a result of its enforcement; (3) the
question of constitutionality must be raised at the earliest opportunity; and (4) the
issue of constitutionality must be the very lis mota of the case.39
Except with respect to the requisites of standing and existence of an actual case
or controversy where the disagreement between the parties lies, discussion of
the rest of the requisites shall be omitted.
Standing
Respondents, through the Solicitor General, assert that the allegations in G.R.
Nos. 169659, 169660 and 169667 make it clear that they, adverting to the nonappearance of several officials of the executive department in the investigations
called by the different committees of the Senate, were brought to vindicate the
constitutional duty of the Senate or its different committees to conduct inquiry in
aid of legislation or in the exercise of its oversight functions. They maintain that
Representatives Ocampo et al. have not shown any specific prerogative, power,
and privilege of the House of Representatives which had been effectively
impaired by E.O. 464, there being no mention of any investigation called by the
House of Representatives or any of its committees which was aborted due to the
implementation of E.O. 464.

As for Bayan Munas alleged interest as a party-list representing the marginalized


and underrepresented, and that of the other petitioner groups and individuals
who profess to have standing as advocates and defenders of the Constitution,
respondents contend that such interest falls short of that required to confer
standing on them as parties "injured-in-fact."40
Respecting petitioner Chavez, respondents contend that Chavez may not claim
an interest as a taxpayer for the implementation of E.O. 464 does not involve the
exercise of taxing or spending power.41
With regard to the petition filed by the Senate, respondents argue that in the
absence of a personal or direct injury by reason of the issuance of E.O. 464, the
Senate and its individual members are not the proper parties to assail the
constitutionality of E.O. 464.
Invoking this Courts ruling in National Economic Protectionism Association v.
Ongpin42 and Valmonte v. Philippine Charity Sweepstakes Office,43 respondents
assert that to be considered a proper party, one must have a personal and
substantial interest in the case, such that he has sustained or will sustain direct
injury due to the enforcement of E.O. 464.44
That the Senate of the Philippines has a fundamental right essential not only for
intelligent public decision-making in a democratic system, but more especially for
sound legislation45 is not disputed. E.O. 464, however, allegedly stifles the ability
of the members of Congress to access information that is crucial to lawmaking.46 Verily, the Senate, including its individual members, has a substantial
and direct interest over the outcome of the controversy and is the proper party to
assail the constitutionality of E.O. 464. Indeed, legislators have standing to
maintain inviolate the prerogative, powers and privileges vested by the
Constitution in their office and are allowed to sue to question the validity of any
official action which they claim infringes their prerogatives as legislators.47
In the same vein, party-list representatives Satur Ocampo (Bayan Muna),
Teodoro Casino (Bayan Muna), Joel Virador (Bayan Muna), Crispin Beltran
(Anakpawis), Rafael Mariano (Anakpawis), and Liza Maza (Gabriela) are allowed
to sue to question the constitutionality of E.O. 464, the absence of any claim that
an investigation called by the House of Representatives or any of its committees
was aborted due to the implementation of E.O. 464 notwithstanding, it being
sufficient that a claim is made that E.O. 464 infringes on their constitutional rights
and duties as members of Congress to conduct investigation in aid of legislation
and conduct oversight functions in the implementation of laws.

The national political party, Bayan Muna, likewise meets the standing
requirement as it obtained three seats in the House of Representatives in the
2004 elections and is, therefore, entitled to participate in the legislative process
consonant with the declared policy underlying the party list system of affording
citizens belonging to marginalized and underrepresented sectors, organizations
and parties who lack well-defined political constituencies to contribute to the
formulation and enactment of legislation that will benefit the nation.48
As Bayan Muna and Representatives Ocampo et al. have the standing to file
their petitions, passing on the standing of their co-petitioners Courage and Codal
is rendered unnecessary.49
In filing their respective petitions, Chavez, the ALG which claims to be an
organization of citizens, and the incumbent members of the IBP Board of
Governors and the IBP in behalf of its lawyer members,50 invoke their
constitutional right to information on matters of public concern, asserting that the
right to information, curtailed and violated by E.O. 464, is essential to the
effective exercise of other constitutional rights51 and to the maintenance of the
balance of power among the three branches of the government through the
principle of checks and balances.52
It is well-settled that when suing as a citizen, the interest of the petitioner in
assailing the constitutionality of laws, presidential decrees, orders, and other
regulations, must be direct and personal. In Franciso v. House of
Representatives,53 this Court held that when the proceeding involves the
assertion of a public right, the mere fact that he is a citizen satisfies the
requirement of personal interest.
As for petitioner PDP-Laban, it asseverates that it is clothed with legal standing in
view of the transcendental issues raised in its petition which this Court needs to
resolve in order to avert a constitutional crisis. For it to be accorded standing on
the ground of transcendental importance, however, it must establish (1) the
character of the funds (that it is public) or other assets involved in the case, (2)
the presence of a clear case of disregard of a constitutional or statutory
prohibition by the public respondent agency or instrumentality of the government,
and (3) the lack of any party with a more direct and specific interest in raising the
questions being raised.54 The first and last determinants not being present as no
public funds or assets are involved and petitioners in G.R. Nos. 169777 and
169659 have direct and specific interests in the resolution of the controversy,
petitioner PDP-Laban is bereft of standing to file its petition. Its allegation that
E.O. 464 hampers its legislative agenda is vague and uncertain, and at best is
only a "generalized interest" which it shares with the rest of the political parties.
Concrete injury, whether actual or threatened, is that indispensable element of a

dispute which serves in part to cast it in a form traditionally capable of judicial


resolution.55 In fine, PDP-Labans alleged interest as a political party does not
suffice to clothe it with legal standing.
Actual Case or Controversy
Petitioners assert that an actual case exists, they citing the absence of the
executive officials invited by the Senate to its hearings after the issuance of E.O.
464, particularly those on the NorthRail project and the wiretapping controversy.
Respondents counter that there is no case or controversy, there being no
showing that President Arroyo has actually withheld her consent or prohibited the
appearance of the invited officials.56 These officials, they claim, merely
communicated to the Senate that they have not yet secured the consent of the
President, not that the President prohibited their attendance.57 Specifically with
regard to the AFP officers who did not attend the hearing on September 28,
2005, respondents claim that the instruction not to attend without the Presidents
consent was based on its role as Commander-in-Chief of the Armed Forces, not
on E.O. 464.
Respondents thus conclude that the petitions merely rest on an unfounded
apprehension that the President will abuse its power of preventing the
appearance of officials before Congress, and that such apprehension is not
sufficient for challenging the validity of E.O. 464.
The Court finds respondents assertion that the President has not withheld her
consent or prohibited the appearance of the officials concerned immaterial in
determining the existence of an actual case or controversy insofar as E.O. 464 is
concerned. For E.O. 464 does not require either a deliberate withholding of
consent or an express prohibition issuing from the President in order to bar
officials from appearing before Congress.
As the implementation of the challenged order has already resulted in the
absence of officials invited to the hearings of petitioner Senate of the Philippines,
it would make no sense to wait for any further event before considering the
present case ripe for adjudication. Indeed, it would be sheer abandonment of
duty if this Court would now refrain from passing on the constitutionality of E.O.
464.
Constitutionality of E.O. 464
E.O. 464, to the extent that it bars the appearance of executive officials before
Congress, deprives Congress of the information in the possession of these

officials. To resolve the question of whether such withholding of information


violates the Constitution, consideration of the general power of Congress to
obtain information, otherwise known as the power of inquiry, is in order.
The power of inquiry
The Congress power of inquiry is expressly recognized in Section 21 of Article VI
of the Constitution which reads:
SECTION 21. The Senate or the House of Representatives or any of its
respective committees may conduct inquiries in aid of legislation in accordance
with its duly published rules of procedure. The rights of persons appearing in or
affected by such inquiries shall be respected. (Underscoring supplied)
This provision is worded exactly as Section 8 of Article VIII of the 1973
Constitution except that, in the latter, it vests the power of inquiry in the
unicameral legislature established therein the Batasang Pambansa and its
committees.
The 1935 Constitution did not contain a similar provision. Nonetheless, in Arnault
v. Nazareno,58 a case decided in 1950 under that Constitution, the Court already
recognized that the power of inquiry is inherent in the power to legislate.
Arnault involved a Senate investigation of the reportedly anomalous purchase of
the Buenavista and Tambobong Estates by the Rural Progress Administration.
Arnault, who was considered a leading witness in the controversy, was called to
testify thereon by the Senate. On account of his refusal to answer the questions
of the senators on an important point, he was, by resolution of the Senate,
detained for contempt. Upholding the Senates power to punish Arnault for
contempt, this Court held:
Although there is no provision in the Constitution expressly investing either
House of Congress with power to make investigations and exact testimony to the
end that it may exercise its legislative functions advisedly and effectively, such
power is so far incidental to the legislative function as to be implied. In other
words, the power of inquiry with process to enforce it is an essential and
appropriate auxiliary to the legislative function. A legislative body cannot legislate
wisely or effectively in the absence of information respecting the conditions which
the legislation is intended to affect or change; and where the legislative body
does not itself possess the requisite information which is not infrequently true
recourse must be had to others who do possess it. Experience has shown that
mere requests for such information are often unavailing, and also that information
which is volunteered is not always accurate or complete; so some means of

compulsion is essential to obtain what is needed.59 . . . (Emphasis and


underscoring supplied)
That this power of inquiry is broad enough to cover officials of the executive
branch may be deduced from the same case. The power of inquiry, the Court
therein ruled, is co-extensive with the power to legislate.60 The matters which
may be a proper subject of legislation and those which may be a proper subject
of investigation are one. It follows that the operation of government, being a
legitimate subject for legislation, is a proper subject for investigation.
Thus, the Court found that the Senate investigation of the government
transaction involved in Arnault was a proper exercise of the power of inquiry.
Besides being related to the expenditure of public funds of which Congress is the
guardian, the transaction, the Court held, "also involved government agencies
created by Congress and officers whose positions it is within the power of
Congress to regulate or even abolish."
Since Congress has authority to inquire into the operations of the executive
branch, it would be incongruous to hold that the power of inquiry does not extend
to executive officials who are the most familiar with and informed on executive
operations.
As discussed in Arnault, the power of inquiry, "with process to enforce it," is
grounded on the necessity of information in the legislative process. If the
information possessed by executive officials on the operation of their offices is
necessary for wise legislation on that subject, by parity of reasoning, Congress
has the right to that information and the power to compel the disclosure thereof.
As evidenced by the American experience during the so-called "McCarthy era,"
however, the right of Congress to conduct inquiries in aid of legislation is, in
theory, no less susceptible to abuse than executive or judicial power. It may thus
be subjected to judicial review pursuant to the Courts certiorari powers under
Section 1, Article VIII of the Constitution.
For one, as noted in Bengzon v. Senate Blue Ribbon Committee,61 the inquiry
itself might not properly be in aid of legislation, and thus beyond the constitutional
power of Congress. Such inquiry could not usurp judicial functions.
Parenthetically, one possible way for Congress to avoid such a result as
occurred in Bengzon is to indicate in its invitations to the public officials
concerned, or to any person for that matter, the possible needed statute which
prompted the need for the inquiry. Given such statement in its invitations, along
with the usual indication of the subject of inquiry and the questions relative to and

in furtherance thereof, there would be less room for speculation on the part of the
person invited on whether the inquiry is in aid of legislation.
Section 21, Article VI likewise establishes crucial safeguards that proscribe the
legislative power of inquiry. The provision requires that the inquiry be done in
accordance with the Senate or Houses duly published rules of procedure,
necessarily implying the constitutional infirmity of an inquiry conducted without
duly published rules of procedure. Section 21 also mandates that the rights of
persons appearing in or affected by such inquiries be respected, an imposition
that obligates Congress to adhere to the guarantees in the Bill of Rights.
These abuses are, of course, remediable before the courts, upon the proper suit
filed by the persons affected, even if they belong to the executive branch.
Nonetheless, there may be exceptional circumstances, none appearing to obtain
at present, wherein a clear pattern of abuse of the legislative power of inquiry
might be established, resulting in palpable violations of the rights guaranteed to
members of the executive department under the Bill of Rights. In such instances,
depending on the particulars of each case, attempts by the Executive Branch to
forestall these abuses may be accorded judicial sanction.
Even where the inquiry is in aid of legislation, there are still recognized
exemptions to the power of inquiry, which exemptions fall under the rubric of
"executive privilege." Since this term figures prominently in the challenged order,
it being mentioned in its provisions, its preambular clauses,62 and in its very title,
a discussion of executive privilege is crucial for determining the constitutionality
of E.O. 464.
Executive privilege
The phrase "executive privilege" is not new in this jurisdiction. It has been used
even prior to the promulgation of the 1986 Constitution.63 Being of American
origin, it is best understood in light of how it has been defined and used in the
legal literature of the United States.
Schwartz defines executive privilege as "the power of the Government to
withhold information from the public, the courts, and the Congress."64 Similarly,
Rozell defines it as "the right of the President and high-level executive branch
officers to withhold information from Congress, the courts, and ultimately the
public."65
Executive privilege is, nonetheless, not a clear or unitary concept. 66 It has
encompassed claims of varying kinds.67Tribe, in fact, comments that while it is
customary to employ the phrase "executive privilege," it may be more accurate to

speak of executive privileges "since presidential refusals to furnish information


may be actuated by any of at least three distinct kinds of considerations, and
may be asserted, with differing degrees of success, in the context of either
judicial or legislative investigations."
One variety of the privilege, Tribe explains, is the state secrets privilege invoked
by U.S. Presidents, beginning with Washington, on the ground that the
information is of such nature that its disclosure would subvert crucial military or
diplomatic objectives. Another variety is the informers privilege, or the privilege
of the Government not to disclose the identity of persons who furnish information
of violations of law to officers charged with the enforcement of that law. Finally, a
generic privilege for internal deliberations has been said to attach to
intragovernmental documents reflecting advisory opinions, recommendations and
deliberations comprising part of a process by which governmental decisions and
policies are formulated. 68
Tribes comment is supported by the ruling in In re Sealed Case, thus:
Since the beginnings of our nation, executive officials have claimed a variety of
privileges to resist disclosure of information the confidentiality of which they felt
was crucial to fulfillment of the unique role and responsibilities of the executive
branch of our government. Courts ruled early that the executive had a right to
withhold documents that might reveal military or state secrets. The courts have
also granted the executive a right to withhold the identity of government
informers in some circumstances and a qualified right to withhold information
related to pending investigations. x x x"69 (Emphasis and underscoring supplied)
The entry in Blacks Law Dictionary on "executive privilege" is similarly instructive
regarding the scope of the doctrine.
This privilege, based on the constitutional doctrine of separation of powers,
exempts the executive from disclosure requirements applicable to the ordinary
citizen or organization where such exemption is necessary to the discharge of
highly important executive responsibilities involved in maintaining governmental
operations, and extends not only to military and diplomatic secrets but also to
documents integral to an appropriate exercise of the executive domestic
decisional and policy making functions, that is, those documents reflecting the
frank expression necessary in intra-governmental advisory and deliberative
communications.70 (Emphasis and underscoring supplied)
That a type of information is recognized as privileged does not, however,
necessarily mean that it would be considered privileged in all instances. For in
determining the validity of a claim of privilege, the question that must be asked is

not only whether the requested information falls within one of the traditional
privileges, but also whether that privilege should be honored in a given
procedural setting.71
The leading case on executive privilege in the United States is U.S. v.
Nixon, 72 decided in 1974. In issue in that case was the validity of President
Nixons claim of executive privilege against a subpoena issued by a district court
requiring the production of certain tapes and documents relating to the Watergate
investigations. The claim of privilege was based on the Presidents general
interest in the confidentiality of his conversations and correspondence. The U.S.
Court held that while there is no explicit reference to a privilege of confidentiality
in the U.S. Constitution, it is constitutionally based to the extent that it relates to
the effective discharge of a Presidents powers. The Court, nonetheless, rejected
the Presidents claim of privilege, ruling that the privilege must be balanced
against the public interest in the fair administration of criminal justice. Notably,
the Court was careful to clarify that it was not there addressing the issue of
claims of privilege in a civil litigation or against congressional demands for
information.
Cases in the U.S. which involve claims of executive privilege against Congress
are rare.73 Despite frequent assertion of the privilege to deny information to
Congress, beginning with President Washingtons refusal to turn over treaty
negotiation records to the House of Representatives, the U.S. Supreme Court
has never adjudicated the issue.74 However, the U.S. Court of Appeals for the
District of Columbia Circuit, in a case decided earlier in the same year as Nixon,
recognized the Presidents privilege over his conversations against a
congressional subpoena.75 Anticipating the balancing approach adopted by the
U.S. Supreme Court in Nixon, the Court of Appeals weighed the public interest
protected by the claim of privilege against the interest that would be served by
disclosure to the Committee. Ruling that the balance favored the President, the
Court declined to enforce the subpoena. 76
In this jurisdiction, the doctrine of executive privilege was recognized by this
Court in Almonte v. Vasquez.77Almonte used the term in reference to the same
privilege subject of Nixon. It quoted the following portion of the Nixon decision
which explains the basis for the privilege:
"The expectation of a President to the confidentiality of his conversations and
correspondences, like the claim of confidentiality of judicial deliberations, for
example, has all the values to which we accord deference for the privacy of all
citizens and, added to those values, is the necessity for protection of the public
interest in candid, objective, and even blunt or harsh opinions in Presidential
decision-making. A President and those who assist him must be free to explore

alternatives in the process of shaping policies and making decisions and to do so


in a way many would be unwilling to express except privately. These are the
considerations justifying a presumptive privilege for Presidential communications.
The privilege is fundamental to the operation of government and inextricably
rooted in the separation of powers under the Constitution x x x " (Emphasis and
underscoring supplied)
Almonte involved a subpoena duces tecum issued by the Ombudsman against
the therein petitioners. It did not involve, as expressly stated in the decision, the
right of the people to information.78 Nonetheless, the Court recognized that there
are certain types of information which the government may withhold from the
public, thus acknowledging, in substance if not in name, that executive privilege
may be claimed against citizens demands for information.
In Chavez v. PCGG,79 the Court held that this jurisdiction recognizes the common
law holding that there is a "governmental privilege against public disclosure with
respect to state secrets regarding military, diplomatic and other national security
matters."80 The same case held that closed-door Cabinet meetings are also a
recognized limitation on the right to information.
Similarly, in Chavez v. Public Estates Authority,81 the Court ruled that the right to
information does not extend to matters recognized as "privileged information
under the separation of powers,"82 by which the Court meant Presidential
conversations, correspondences, and discussions in closed-door Cabinet
meetings. It also held that information on military and diplomatic secrets and
those affecting national security, and information on investigations of crimes by
law enforcement agencies before the prosecution of the accused were exempted
from the right to information.
From the above discussion on the meaning and scope of executive privilege,
both in the United States and in this jurisdiction, a clear principle emerges.
Executive privilege, whether asserted against Congress, the courts, or the public,
is recognized only in relation to certain types of information of a sensitive
character. While executive privilege is a constitutional concept, a claim thereof
may be valid or not depending on the ground invoked to justify it and the context
in which it is made. Noticeably absent is any recognition that executive officials
are exempt from the duty to disclose information by the mere fact of being
executive officials. Indeed, the extraordinary character of the exemptions
indicates that the presumption inclines heavily against executive secrecy and in
favor of disclosure.
Validity of Section 1

Section 1 is similar to Section 3 in that both require the officials covered by them
to secure the consent of the President prior to appearing before Congress. There
are significant differences between the two provisions, however, which constrain
this Court to discuss the validity of these provisions separately.
Section 1 specifically applies to department heads. It does not, unlike Section 3,
require a prior determination by any official whether they are covered by E.O.
464. The President herself has, through the challenged order, made the
determination that they are. Further, unlike also Section 3, the coverage of
department heads under Section 1 is not made to depend on the department
heads possession of any information which might be covered by executive
privilege. In fact, in marked contrast to Section 3 vis--vis Section 2, there is no
reference to executive privilege at all. Rather, the required prior consent under
Section 1 is grounded on Article VI, Section 22 of the Constitution on what has
been referred to as the question hour.
SECTION 22. The heads of departments may upon their own initiative, with the
consent of the President, or upon the request of either House, as the rules of
each House shall provide, appear before and be heard by such House on any
matter pertaining to their departments. Written questions shall be submitted to
the President of the Senate or the Speaker of the House of Representatives at
least three days before their scheduled appearance. Interpellations shall not be
limited to written questions, but may cover matters related thereto. When the
security of the State or the public interest so requires and the President so states
in writing, the appearance shall be conducted in executive session.
Determining the validity of Section 1 thus requires an examination of the meaning
of Section 22 of Article VI. Section 22 which provides for the question hour must
be interpreted vis--vis Section 21 which provides for the power of either House
of Congress to "conduct inquiries in aid of legislation." As the following excerpt of
the deliberations of the Constitutional Commission shows, the framers were
aware that these two provisions involved distinct functions of Congress.
MR. MAAMBONG. x x x When we amended Section 20 [now Section 22 on the
Question Hour] yesterday, I noticed that members of the Cabinet cannot be
compelled anymore to appear before the House of Representatives or before the
Senate. I have a particular problem in this regard, Madam President, because in
our experience in the Regular Batasang Pambansa as the Gentleman himself
has experienced in the interim Batasang Pambansa one of the most competent
inputs that we can put in our committee deliberations, either in aid of legislation
or in congressional investigations, is the testimonies of Cabinet ministers. We
usually invite them, but if they do not come and it is a congressional
investigation, we usually issue subpoenas.

I want to be clarified on a statement made by Commissioner Suarez when he


said that the fact that the Cabinet ministers may refuse to come to the House of
Representatives or the Senate [when requested under Section 22] does not
mean that they need not come when they are invited or subpoenaed by the
committee of either House when it comes to inquiries in aid of legislation or
congressional investigation. According to Commissioner Suarez, that is allowed
and their presence can be had under Section 21. Does the gentleman confirm
this, Madam President?
MR. DAVIDE. We confirm that, Madam President, because Section 20 refers
only to what was originally the Question Hour, whereas, Section 21 would refer
specifically to inquiries in aid of legislation, under which anybody for that matter,
may be summoned and if he refuses, he can be held in contempt of the
House.83 (Emphasis and underscoring supplied)
A distinction was thus made between inquiries in aid of legislation and the
question hour. While attendance was meant to be discretionary in the question
hour, it was compulsory in inquiries in aid of legislation. The reference to
Commissioner Suarez bears noting, he being one of the proponents of the
amendment to make the appearance of department heads discretionary in the
question hour.
So clearly was this distinction conveyed to the members of the Commission that
the Committee on Style, precisely in recognition of this distinction, later moved
the provision on question hour from its original position as Section 20 in the
original draft down to Section 31, far from the provision on inquiries in aid of
legislation. This gave rise to the following exchange during the deliberations:
MR. GUINGONA. [speaking in his capacity as Chairman of the Committee on
Style] We now go, Mr. Presiding Officer, to the Article on Legislative and may I
request the chairperson of the Legislative Department, Commissioner Davide, to
give his reaction.
THE PRESIDING OFFICER (Mr. Jamir). Commissioner Davide is recognized.

|av vphi|.net

MR. DAVIDE. Thank you, Mr. Presiding Officer. I have only one reaction to the
Question Hour. I propose that instead of putting it as Section 31, it should follow
Legislative Inquiries.
THE PRESIDING OFFICER. What does the committee say?
MR. GUINGONA. I ask Commissioner Maambong to reply, Mr. Presiding Officer.

MR. MAAMBONG. Actually, we considered that previously when we sequenced


this but we reasoned that in Section 21, which is Legislative Inquiry, it is actually
a power of Congress in terms of its own lawmaking; whereas, a Question Hour is
not actually a power in terms of its own lawmaking power because in Legislative
Inquiry, it is in aid of legislation. And so we put Question Hour as Section 31. I
hope Commissioner Davide will consider this.
MR. DAVIDE. The Question Hour is closely related with the legislative power,
and it is precisely as a complement to or a supplement of the Legislative Inquiry.
The appearance of the members of Cabinet would be very, very essential not
only in the application of check and balance but also, in effect, in aid of
legislation.
MR. MAAMBONG. After conferring with the committee, we find merit in the
suggestion of Commissioner Davide. In other words, we are accepting that and
so this Section 31 would now become Section 22. Would it be, Commissioner
Davide?
MR. DAVIDE. Yes.84 (Emphasis and underscoring supplied)
Consistent with their statements earlier in the deliberations, Commissioners
Davide and Maambong proceeded from the same assumption that these
provisions pertained to two different functions of the legislature. Both
Commissioners understood that the power to conduct inquiries in aid of
legislation is different from the power to conduct inquiries during the question
hour. Commissioner Davides only concern was that the two provisions on these
distinct powers be placed closely together, they being complementary to each
other. Neither Commissioner considered them as identical functions of Congress.
The foregoing opinion was not the two Commissioners alone. From the abovequoted exchange, Commissioner Maambongs committee the Committee on
Style shared the view that the two provisions reflected distinct functions of
Congress. Commissioner Davide, on the other hand, was speaking in his
capacity as Chairman of the Committee on the Legislative Department. His views
may thus be presumed as representing that of his Committee.
In the context of a parliamentary system of government, the "question hour" has
a definite meaning. It is a period of confrontation initiated by Parliament to hold
the Prime Minister and the other ministers accountable for their acts and the
operation of the government,85 corresponding to what is known in Britain as the
question period. There was a specific provision for a question hour in the 1973
Constitution86 which made the appearance of ministers mandatory. The same
perfectly conformed to the parliamentary system established by that Constitution,

where the ministers are also members of the legislature and are directly
accountable to it.
An essential feature of the parliamentary system of government is the immediate
accountability of the Prime Minister and the Cabinet to the National Assembly.
They shall be responsible to the National Assembly for the program of
government and shall determine the guidelines of national policy. Unlike in the
presidential system where the tenure of office of all elected officials cannot be
terminated before their term expired, the Prime Minister and the Cabinet remain
in office only as long as they enjoy the confidence of the National Assembly. The
moment this confidence is lost the Prime Minister and the Cabinet may be
changed.87
The framers of the 1987 Constitution removed the mandatory nature of such
appearance during the question hour in the present Constitution so as to conform
more fully to a system of separation of powers.88 To that extent, the question
hour, as it is presently understood in this jurisdiction, departs from the question
period of the parliamentary system. That department heads may not be required
to appear in a question hour does not, however, mean that the legislature is
rendered powerless to elicit information from them in all circumstances. In fact, in
light of the absence of a mandatory question period, the need to enforce
Congress right to executive information in the performance of its legislative
function becomes more imperative. As Schwartz observes:
Indeed, if the separation of powers has anything to tell us on the subject under
discussion, it is that the Congress has the right to obtain information from any
source even from officials of departments and agencies in the executive
branch. In the United States there is, unlike the situation which prevails in a
parliamentary system such as that in Britain, a clear separation between the
legislative and executive branches. It is this very separation that makes the
congressional right to obtain information from the executive so essential, if the
functions of the Congress as the elected representatives of the people are
adequately to be carried out. The absence of close rapport between the
legislative and executive branches in this country, comparable to those which
exist under a parliamentary system, and the nonexistence in the Congress of an
institution such as the British question period have perforce made reliance by the
Congress upon its right to obtain information from the executive essential, if it is
intelligently to perform its legislative tasks. Unless the Congress possesses the
right to obtain executive information, its power of oversight of administration in a
system such as ours becomes a power devoid of most of its practical content,
since it depends for its effectiveness solely upon information parceled out ex
gratia by the executive.89 (Emphasis and underscoring supplied)

Sections 21 and 22, therefore, while closely related and complementary to each
other, should not be considered as pertaining to the same power of Congress.
One specifically relates to the power to conduct inquiries in aid of legislation, the
aim of which is to elicit information that may be used for legislation, while the
other pertains to the power to conduct a question hour, the objective of which is
to obtain information in pursuit of Congress oversight function.
When Congress merely seeks to be informed on how department heads are
implementing the statutes which it has issued, its right to such information is not
as imperative as that of the President to whom, as Chief Executive, such
department heads must give a report of their performance as a matter of duty. In
such instances, Section 22, in keeping with the separation of powers, states that
Congress may only request their appearance. Nonetheless, when the inquiry in
which Congress requires their appearance is "in aid of legislation" under Section
21, the appearance is mandatory for the same reasons stated in Arnault.90
In fine, the oversight function of Congress may be facilitated by compulsory
process only to the extent that it is performed in pursuit of legislation. This is
consistent with the intent discerned from the deliberations of the Constitutional
Commission.
Ultimately, the power of Congress to compel the appearance of executive
officials under Section 21 and the lack of it under Section 22 find their basis in
the principle of separation of powers. While the executive branch is a co-equal
branch of the legislature, it cannot frustrate the power of Congress to legislate by
refusing to comply with its demands for information.
When Congress exercises its power of inquiry, the only way for department
heads to exempt themselves therefrom is by a valid claim of privilege. They are
not exempt by the mere fact that they are department heads. Only one executive
official may be exempted from this power the President on whom executive
power is vested, hence, beyond the reach of Congress except through the power
of impeachment. It is based on her being the highest official of the executive
branch, and the due respect accorded to a co-equal branch of government which
is sanctioned by a long-standing custom.
By the same token, members of the Supreme Court are also exempt from this
power of inquiry. Unlike the Presidency, judicial power is vested in a collegial
body; hence, each member thereof is exempt on the basis not only of separation
of powers but also on the fiscal autonomy and the constitutional independence of
the judiciary. This point is not in dispute, as even counsel for the Senate, Sen.
Joker Arroyo, admitted it during the oral argument upon interpellation of the Chief
Justice.

Having established the proper interpretation of Section 22, Article VI of the


Constitution, the Court now proceeds to pass on the constitutionality of Section 1
of E.O. 464.
Section 1, in view of its specific reference to Section 22 of Article VI of the
Constitution and the absence of any reference to inquiries in aid of legislation,
must be construed as limited in its application to appearances of department
heads in the question hour contemplated in the provision of said Section 22 of
Article VI. The reading is dictated by the basic rule of construction that issuances
must be interpreted, as much as possible, in a way that will render it
constitutional.
The requirement then to secure presidential consent under Section 1, limited as it
is only to appearances in the question hour, is valid on its face. For under
Section 22, Article VI of the Constitution, the appearance of department heads in
the question hour is discretionary on their part.
Section 1 cannot, however, be applied to appearances of department heads in
inquiries in aid of legislation. Congress is not bound in such instances to respect
the refusal of the department head to appear in such inquiry, unless a valid claim
of privilege is subsequently made, either by the President herself or by the
Executive Secretary.
Validity of Sections 2 and 3
Section 3 of E.O. 464 requires all the public officials enumerated in Section 2(b)
to secure the consent of the President prior to appearing before either house of
Congress. The enumeration is broad. It covers all senior officials of executive
departments, all officers of the AFP and the PNP, and all senior national security
officials who, in the judgment of the heads of offices designated in the same
section (i.e. department heads, Chief of Staff of the AFP, Chief of the PNP, and
the National Security Adviser), are "covered by the executive privilege."
The enumeration also includes such other officers as may be determined by the
President. Given the title of Section 2 "Nature, Scope and Coverage of
Executive Privilege" , it is evident that under the rule of ejusdem generis, the
determination by the President under this provision is intended to be based on a
similar finding of coverage under executive privilege.
En passant, the Court notes that Section 2(b) of E.O. 464 virtually states that
executive privilege actually covers persons. Such is a misuse of the doctrine.
Executive privilege, as discussed above, is properly invoked in relation to specific
categories of information and not to categories of persons.

In light, however, of Sec 2(a) of E.O. 464 which deals with the nature, scope and
coverage of executive privilege, the reference to persons being "covered by the
executive privilege" may be read as an abbreviated way of saying that the person
is in possession of information which is, in the judgment of the head of office
concerned, privileged as defined in Section 2(a). The Court shall thus proceed on
the assumption that this is the intention of the challenged order.
Upon a determination by the designated head of office or by the President that
an official is "covered by the executive privilege," such official is subjected to the
requirement that he first secure the consent of the President prior to appearing
before Congress. This requirement effectively bars the appearance of the official
concerned unless the same is permitted by the President. The proviso allowing
the President to give its consent means nothing more than that the President
may reverse a prohibition which already exists by virtue of E.O. 464.
Thus, underlying this requirement of prior consent is the determination by a head
of office, authorized by the President under E.O. 464, or by the President herself,
that such official is in possession of information that is covered by executive
privilege. This determination then becomes the basis for the officials not showing
up in the legislative investigation.
In view thereof, whenever an official invokes E.O. 464 to justify his failure to be
present, such invocation must be construed as a declaration to Congress that the
President, or a head of office authorized by the President, has determined that
the requested information is privileged, and that the President has not reversed
such determination. Such declaration, however, even without mentioning the
term "executive privilege," amounts to an implied claim that the information is
being withheld by the executive branch, by authority of the President, on the
basis of executive privilege. Verily, there is an implied claim of privilege.
The letter dated September 28, 2005 of respondent Executive Secretary Ermita
to Senate President Drilon illustrates the implied nature of the claim of privilege
authorized by E.O. 464. It reads:
In connection with the inquiry to be conducted by the Committee of the Whole
regarding the Northrail Project of the North Luzon Railways Corporation on 29
September 2005 at 10:00 a.m., please be informed that officials of the Executive
Department invited to appear at the meeting will not be able to attend the same
without the consent of the President, pursuant to Executive Order No. 464 (s.
2005), entitled "Ensuring Observance Of The Principle Of Separation Of Powers,
Adherence To The Rule On Executive Privilege And Respect For The Rights Of
Public Officials Appearing In Legislative Inquiries In Aid Of Legislation Under The

Constitution, And For Other Purposes". Said officials have not secured the
required consent from the President. (Underscoring supplied)
The letter does not explicitly invoke executive privilege or that the matter on
which these officials are being requested to be resource persons falls under the
recognized grounds of the privilege to justify their absence. Nor does it expressly
state that in view of the lack of consent from the President under E.O. 464, they
cannot attend the hearing.
Significant premises in this letter, however, are left unstated, deliberately or not.
The letter assumes that the invited officials are covered by E.O. 464. As
explained earlier, however, to be covered by the order means that a
determination has been made, by the designated head of office or the President,
that the invited official possesses information that is covered by executive
privilege. Thus, although it is not stated in the letter that such determination has
been made, the same must be deemed implied. Respecting the statement that
the invited officials have not secured the consent of the President, it only means
that the President has not reversed the standing prohibition against their
appearance before Congress.
Inevitably, Executive Secretary Ermitas letter leads to the conclusion that the
executive branch, either through the President or the heads of offices authorized
under E.O. 464, has made a determination that the information required by the
Senate is privileged, and that, at the time of writing, there has been no contrary
pronouncement from the President. In fine, an implied claim of privilege has been
made by the executive.
While there is no Philippine case that directly addresses the issue of whether
executive privilege may be invoked against Congress, it is gathered from Chavez
v. PEA that certain information in the possession of the executive may validly be
claimed as privileged even against Congress. Thus, the case holds:
There is no claim by PEA that the information demanded by petitioner is
privileged information rooted in the separation of powers. The information does
not cover Presidential conversations, correspondences, or discussions during
closed-door Cabinet meetings which, like internal-deliberations of the Supreme
Court and other collegiate courts, or executive sessions of either house of
Congress, are recognized as confidential. This kind of information cannot be
pried open by a co-equal branch of government. A frank exchange of exploratory
ideas and assessments, free from the glare of publicity and pressure by
interested parties, is essential to protect the independence of decision-making of
those tasked to exercise Presidential, Legislative and Judicial power. This is not
the situation in the instant case.91 (Emphasis and underscoring supplied)

Section 3 of E.O. 464, therefore, cannot be dismissed outright as invalid by the


mere fact that it sanctions claims of executive privilege. This Court must look
further and assess the claim of privilege authorized by the Order to determine
whether it is valid.
While the validity of claims of privilege must be assessed on a case to case
basis, examining the ground invoked therefor and the particular circumstances
surrounding it, there is, in an implied claim of privilege, a defect that renders it
invalid per se. By its very nature, and as demonstrated by the letter of
respondent Executive Secretary quoted above, the implied claim authorized by
Section 3 of E.O. 464 is not accompanied by any specific allegation of the basis
thereof (e.g., whether the information demanded involves military or diplomatic
secrets, closed-door Cabinet meetings, etc.). While Section 2(a) enumerates the
types of information that are covered by the privilege under the challenged order,
Congress is left to speculate as to which among them is being referred to by the
executive. The enumeration is not even intended to be comprehensive, but a
mere statement of what is included in the phrase "confidential or classified
information between the President and the public officers covered by this
executive order."
Certainly, Congress has the right to know why the executive considers the
requested information privileged. It does not suffice to merely declare that the
President, or an authorized head of office, has determined that it is so, and that
the President has not overturned that determination. Such declaration leaves
Congress in the dark on how the requested information could be classified as
privileged. That the message is couched in terms that, on first impression, do not
seem like a claim of privilege only makes it more pernicious. It threatens to make
Congress doubly blind to the question of why the executive branch is not
providing it with the information that it has requested.
A claim of privilege, being a claim of exemption from an obligation to disclose
information, must, therefore, be clearly asserted. As U.S. v. Reynolds teaches:
The privilege belongs to the government and must be asserted by it; it can
neither be claimed nor waived by a private party. It is not to be lightly invoked.
There must be a formal claim of privilege, lodged by the head of the department
which has control over the matter, after actual personal consideration by that
officer. The court itself must determine whether the circumstances are
appropriate for the claim of privilege, and yet do so without forcing a disclosure of
the very thing the privilege is designed to protect.92 (Underscoring supplied)
Absent then a statement of the specific basis of a claim of executive privilege,
there is no way of determining whether it falls under one of the traditional

privileges, or whether, given the circumstances in which it is made, it should be


respected.93 These, in substance, were the same criteria in assessing the claim
of privilege asserted against the Ombudsman in Almonte v. Vasquez94 and, more
in point, against a committee of the Senate in Senate Select Committee on
Presidential Campaign Activities v. Nixon.95
A.O. Smith v. Federal Trade Commission is enlightening:
[T]he lack of specificity renders an assessment of the potential harm resulting
from disclosure impossible, thereby preventing the Court from balancing such
harm against plaintiffs needs to determine whether to override any claims of
privilege.96 (Underscoring supplied)
And so is U.S. v. Article of Drug:97
On the present state of the record, this Court is not called upon to perform this
balancing operation. In stating its objection to claimants interrogatories,
government asserts, and nothing more, that the disclosures sought by claimant
would inhibit the free expression of opinion that non-disclosure is designed to
protect. The government has not shown nor even alleged that those who
evaluated claimants product were involved in internal policymaking, generally, or
in this particular instance. Privilege cannot be set up by an unsupported claim.
The facts upon which the privilege is based must be established. To find these
interrogatories objectionable, this Court would have to assume that the
evaluation and classification of claimants products was a matter of internal policy
formulation, an assumption in which this Court is unwilling to indulge sua
sponte.98 (Emphasis and underscoring supplied)
Mobil Oil Corp. v. Department of Energy99 similarly emphasizes that "an agency
must provide precise and certain reasons for preserving the confidentiality of
requested information."
Black v. Sheraton Corp. of America100 amplifies, thus:
A formal and proper claim of executive privilege requires a specific designation
and description of the documents within its scope as well as precise and certain
reasons for preserving their confidentiality. Without this specificity, it is impossible
for a court to analyze the claim short of disclosure of the very thing sought to be
protected. As the affidavit now stands, the Court has little more than its sua
sponte speculation with which to weigh the applicability of the claim. An
improperly asserted claim of privilege is no claim of privilege. Therefore, despite
the fact that a claim was made by the proper executive as Reynolds requires, the
Court can not recognize the claim in the instant case because it is legally

insufficient to allow the Court to make a just and reasonable determination as to


its applicability. To recognize such a broad claim in which the Defendant has
given no precise or compelling reasons to shield these documents from outside
scrutiny, would make a farce of the whole procedure.101 (Emphasis and
underscoring supplied)
Due respect for a co-equal branch of government, moreover, demands no less
than a claim of privilege clearly stating the grounds therefor. Apropos is the
following ruling in McPhaul v. U.S:102
We think the Courts decision in United States v. Bryan, 339 U.S. 323, 70 S. Ct.
724, is highly relevant to these questions. For it is as true here as it was there,
that if (petitioner) had legitimate reasons for failing to produce the records of the
association, a decent respect for the House of Representatives, by whose
authority the subpoenas issued, would have required that (he) state (his) reasons
for noncompliance upon the return of the writ. Such a statement would have
given the Subcommittee an opportunity to avoid the blocking of its inquiry by
taking other appropriate steps to obtain the records. To deny the Committee the
opportunity to consider the objection or remedy is in itself a contempt of its
authority and an obstruction of its processes. His failure to make any such
statement was "a patent evasion of the duty of one summoned to produce papers
before a congressional committee[, and] cannot be condoned." (Emphasis and
underscoring supplied; citations omitted)
Upon the other hand, Congress must not require the executive to state the
reasons for the claim with such particularity as to compel disclosure of the
information which the privilege is meant to protect.103 A useful analogy in
determining the requisite degree of particularity would be the privilege against
self-incrimination. Thus, Hoffman v. U.S.104 declares:
The witness is not exonerated from answering merely because he declares that
in so doing he would incriminate himself his say-so does not of itself establish
the hazard of incrimination. It is for the court to say whether his silence is
justified, and to require him to answer if it clearly appears to the court that he is
mistaken. However, if the witness, upon interposing his claim, were required to
prove the hazard in the sense in which a claim is usually required to be
established in court, he would be compelled to surrender the very protection
which the privilege is designed to guarantee. To sustain the privilege, it need only
be evident from the implications of the question, in the setting in which it is
asked, that a responsive answer to the question or an explanation of why it
cannot be answered might be dangerous because injurious disclosure could
result." x x x (Emphasis and underscoring supplied)

The claim of privilege under Section 3 of E.O. 464 in relation to Section 2(b) is
thus invalid per se. It is not asserted. It is merely implied. Instead of providing
precise and certain reasons for the claim, it merely invokes E.O. 464, coupled
with an announcement that the President has not given her consent. It is woefully
insufficient for Congress to determine whether the withholding of information is
justified under the circumstances of each case. It severely frustrates the power of
inquiry of Congress.
In fine, Section 3 and Section 2(b) of E.O. 464 must be invalidated.
No infirmity, however, can be imputed to Section 2(a) as it merely provides
guidelines, binding only on the heads of office mentioned in Section 2(b), on what
is covered by executive privilege. It does not purport to be conclusive on the
other branches of government. It may thus be construed as a mere expression of
opinion by the President regarding the nature and scope of executive privilege.
Petitioners, however, assert as another ground for invalidating the challenged
order the alleged unlawful delegation of authority to the heads of offices in
Section 2(b). Petitioner Senate of the Philippines, in particular, cites the case of
the United States where, so it claims, only the President can assert executive
privilege to withhold information from Congress.
Section 2(b) in relation to Section 3 virtually provides that, once the head of office
determines that a certain information is privileged, such determination is
presumed to bear the Presidents authority and has the effect of prohibiting the
official from appearing before Congress, subject only to the express
pronouncement of the President that it is allowing the appearance of such official.
These provisions thus allow the President to authorize claims of privilege by
mere silence.
Such presumptive authorization, however, is contrary to the exceptional nature of
the privilege. Executive privilege, as already discussed, is recognized with
respect to information the confidential nature of which is crucial to the fulfillment
of the unique role and responsibilities of the executive branch,105 or in those
instances where exemption from disclosure is necessary to the discharge of
highly important executive responsibilities.106 The doctrine of executive privilege
is thus premised on the fact that certain informations must, as a matter of
necessity, be kept confidential in pursuit of the public interest. The privilege
being, by definition, an exemption from the obligation to disclose information, in
this case to Congress, the necessity must be of such high degree as to outweigh
the public interest in enforcing that obligation in a particular case.

In light of this highly exceptional nature of the privilege, the Court finds it
essential to limit to the President the power to invoke the privilege. She may of
course authorize the Executive Secretary to invoke the privilege on her behalf, in
which case the Executive Secretary must state that the authority is "By order of
the President," which means that he personally consulted with her. The privilege
being an extraordinary power, it must be wielded only by the highest official in the
executive hierarchy. In other words, the President may not authorize her
subordinates to exercise such power. There is even less reason to uphold such
authorization in the instant case where the authorization is not explicit but by
mere silence. Section 3, in relation to Section 2(b), is further invalid on this score.
It follows, therefore, that when an official is being summoned by Congress on a
matter which, in his own judgment, might be covered by executive privilege, he
must be afforded reasonable time to inform the President or the Executive
Secretary of the possible need for invoking the privilege. This is necessary in
order to provide the President or the Executive Secretary with fair opportunity to
consider whether the matter indeed calls for a claim of executive privilege. If,
after the lapse of that reasonable time, neither the President nor the Executive
Secretary invokes the privilege, Congress is no longer bound to respect the
failure of the official to appear before Congress and may then opt to avail of the
necessary legal means to compel his appearance.
The Court notes that one of the expressed purposes for requiring officials to
secure the consent of the President under Section 3 of E.O. 464 is to ensure
"respect for the rights of public officials appearing in inquiries in aid of
legislation." That such rights must indeed be respected by Congress is an echo
from Article VI Section 21 of the Constitution mandating that "[t]he rights of
persons appearing in or affected by such inquiries shall be respected."
In light of the above discussion of Section 3, it is clear that it is essentially an
authorization for implied claims of executive privilege, for which reason it must be
invalidated. That such authorization is partly motivated by the need to ensure
respect for such officials does not change the infirm nature of the authorization
itself.
Right to Information
E.O 464 is concerned only with the demands of Congress for the appearance of
executive officials in the hearings conducted by it, and not with the demands of
citizens for information pursuant to their right to information on matters of public
concern. Petitioners are not amiss in claiming, however, that what is involved in
the present controversy is not merely the legislative power of inquiry, but the right
of the people to information.

There are, it bears noting, clear distinctions between the right of Congress to
information which underlies the power of inquiry and the right of the people to
information on matters of public concern. For one, the demand of a citizen for the
production of documents pursuant to his right to information does not have the
same obligatory force as a subpoena duces tecum issued by Congress. Neither
does the right to information grant a citizen the power to exact testimony from
government officials. These powers belong only to Congress and not to an
individual citizen.
Thus, while Congress is composed of representatives elected by the people, it
does not follow, except in a highly qualified sense, that in every exercise of its
power of inquiry, the people are exercising their right to information.
To the extent that investigations in aid of legislation are generally conducted in
public, however, any executive issuance tending to unduly limit disclosures of
information in such investigations necessarily deprives the people of information
which, being presumed to be in aid of legislation, is presumed to be a matter of
public concern. The citizens are thereby denied access to information which they
can use in formulating their own opinions on the matter before Congress
opinions which they can then communicate to their representatives and other
government officials through the various legal means allowed by their freedom of
expression. Thus holds Valmonte v. Belmonte:
It is in the interest of the State that the channels for free political discussion be
maintained to the end that the government may perceive and be responsive to
the peoples will. Yet, this open dialogue can be effective only to the extent that
the citizenry is informed and thus able to formulate its will intelligently. Only when
the participants in the discussion are aware of the issues and have access to
information relating thereto can such bear fruit.107(Emphasis and underscoring
supplied)
The impairment of the right of the people to information as a consequence of
E.O. 464 is, therefore, in the sense explained above, just as direct as its violation
of the legislatures power of inquiry.
Implementation of E.O. 464 prior to its publication
While E.O. 464 applies only to officials of the executive branch, it does not follow
that the same is exempt from the need for publication. On the need for publishing
even those statutes that do not directly apply to people in general, Taada v.
Tuvera states:

The term "laws" should refer to all laws and not only to those of general
application, for strictly speaking all laws relate to the people in general albeit
there are some that do not apply to them directly. An example is a law granting
citizenship to a particular individual, like a relative of President Marcos who was
decreed instant naturalization. It surely cannot be said that such a law does not
affect the public although it unquestionably does not apply directly to all the
people. The subject of such law is a matter of public interest which any member
of the body politic may question in the political forums or, if he is a proper party,
even in courts of justice.108 (Emphasis and underscoring supplied)
Although the above statement was made in reference to statutes, logic dictates
that the challenged order must be covered by the publication requirement. As
explained above, E.O. 464 has a direct effect on the right of the people to
information on matters of public concern. It is, therefore, a matter of public
interest which members of the body politic may question before this Court. Due
process thus requires that the people should have been apprised of this issuance
before it was implemented.
Conclusion
Congress undoubtedly has a right to information from the executive branch
whenever it is sought in aid of legislation. If the executive branch withholds such
information on the ground that it is privileged, it must so assert it and state the
reason therefor and why it must be respected.
The infirm provisions of E.O. 464, however, allow the executive branch to evade
congressional requests for information without need of clearly asserting a right to
do so and/or proffering its reasons therefor. By the mere expedient of invoking
said provisions, the power of Congress to conduct inquiries in aid of legislation is
frustrated. That is impermissible. For
[w]hat republican theory did accomplishwas to reverse the old presumption in
favor of secrecy, based on the divine right of kings and nobles, and replace it
with a presumption in favor of publicity, based on the doctrine of popular
sovereignty. (Underscoring supplied)109
Resort to any means then by which officials of the executive branch could refuse
to divulge information cannot be presumed valid. Otherwise, we shall not have
merely nullified the power of our legislature to inquire into the operations of
government, but we shall have given up something of much greater value our
right as a people to take part in government.

WHEREFORE, the petitions are PARTLY GRANTED. Sections 2(b) and 3 of


Executive Order No. 464 (series of 2005), "Ensuring Observance of the Principle
of Separation of Powers, Adherence to the Rule on Executive
Privilege and Respect for the Rights of Public Officials Appearing in Legislative
Inquiries in Aid of Legislation Under the Constitution, and For Other Purposes,"
are declared VOID. Sections 1 and 2(a) are, however, VALID.
SO ORDERED.

Senate vs. Ermita G.R. No. 169777 April 20, 2006 Case
Digest
Nature of the Case: These are six consolidated cases docketed as following:
G.R. No. 169777 = Senate of the Philippines vs. Eduardo Ermita
G.R. No. 169659 = Bayan Muna vs. Eduardo Ermita
G.R. No. 169660 = Francisco Chavez vs. Eduardo Ermita
G.R. No. 169667 = Alternative Law Groups vs. Eduardo Ermita
G.R. No. 169834 = PDP Laban vs. Eduardo Ermita
G.R. No. 171246 = Jose Anselmo Cadiz v. Eduardo Ermita
Facts of the Case
These consolidated cases are petitions for certiorari stating that the President has
abused her power by issuing Executive Order No. 464 dated September 28, 2005.
Petitioners herein pray that such order be declared as null and void for being
unconstitutional.
In the exercise of its legislative power, the Senate of the Philippines through its various
Committees conducts inquiries and investigations in aid of legislation which call for
attendance of officials and employees of the executive department, bureaus, and offices
including those employed in the Government Owned and Controlled Corporations, the
Armed Forces of the Philippines (AFP) and the Philippine National Police (PNP).
On September 21 - 23, 2005 the Committee of the Senate issued invitations to various
officials of the Executive Department for them to appear on September 29, 2005 as
resource speakers in a public hearing on the railway project of North Luzon Railways
Corporation with the China National Machinery Group. Such railway project is called
the North Railway Project.

The Senate issued invitations on several AFP officials for them to attend as resource
persons in a public hearing scheduled on Sept. 28, 2005 for the privilege speech of the
following senators: Sen. Aquilino Pimentel, Jr., Sen. Jinggoy Estrada, Sen. Rodolfo
Biazon, Sen. Jamby Madrigal, Sen. Biazon. Also invited to the hearing was the AFP Chief
of Staff, General Generoso Senga who by letter dated September 28, 2005 requested for
its postponement due to a pressing operational situation that demands his utmost
attention.
On September 28, 2005 Senate Franklin Drilon received from Executive Secretary
Eduardo Ermita a letter respectfully requesting for the postponement of the hearing to
which various executive officials have been invited in order for said officials to study and
prepare for various issues so they can better enlighten the Senate Committee on its
investigation.
Senate Pres. Drilon however was unable to grant such request because it was sent
belatedly and all preparations are complete within that week. He also received a letter
from NorthRail project President Jose L. Cortes Jr. requesting that the hearing of the
NorthRail project be postponed or cancelled until a copy of the report of UP Law Center
on contract agreements related to the project had been secured.
On Sept. 28, 2005, the President issued Executive Order # 464 entitled Ensuring
Observance of Principle of Separation of Powers, Adherence to Rule on Executive
Privilege and Respect for Rights of Public Officials Appearing in Legislative Inquiries in
Aid of Legislation under the Constitution and for other Purposes", which states that all
heads of the executive branch shall secure the consent of the President prior to
appearing before either house of the Congress; public officials may not divulge
confidential classified information officially known to them by reason of their office and
not made available to the public to prejudice the public interest and that executive
matters shall only be conducted in executive session.
On September 28, Sen. Pres. Franklin Drilon received from Executive Secretary
Eduardo Ermita a copy of E.O. 464 and that executive officials invited were not able to
attend because they failed to secure the required consent from the President.
On October 3, 2005, three petitions docketed as G.R. No. 169659 (Bayan Muna vs.
Eduardo Ermita), 169660 (Francisco Chavez vs. Eduardo Ermita) and 169667
(Alternative Law Groups vs. Eduardo Ermita) were filed before the court also
challenging the constitutionality of E.O. 464.

In G.R. No. 169659 Bayan Muna v. Eduardo Ermita, partylist Bayan Muna and HR
members Satur Ocampo, Crispin Beltran, Rafael Mariano, Liza Maza, Joel Virador and
Teodoro Casino, COURAGE = organization of Govt employees and CODAL (Counsels
for Defense of Liberties) pray that E.O. 464 be declared unconstitutional and that
Executive Sec. Eduardo Ermita be prohibited from imposing sanctions on officials who
appear before Congress due to congressional summons. They also contend that E.O. 464
infringes on their rights and impedes them to fulfil their respective obligations.

In G.R. No. 169660 Francisco Chavez vs. Eduardo Ermita, Francisco Chavez claims that
his constitutional rights as a citizen and taxpayer and law practicioner are affected by
the enforcement of E.O. 464 thus he prays that such order be declared null and
unconstitutional.
In G.R. No. 169667 Alternative Law Groups vs. Eduardo Ermita, ALG claims that the
group has legal standing to institute the petition to enforce its constitutional right to
information
on
matters
of
public
concern.
On October 11, 2005, Senate of the Philippines alleging that it has a vital interest in the
resolution of the issue of validity in E.O. 464, claims that it prohibits the valid exercise
of the Senates powers and functions and conceals information of great public interest
and
concern.
On October 14, 2005, PDP- Laban a registered political party w/ members elected in the
Congress
filed
a
similar
petition.
Issue:

Whether respondents committed grave abuse of discretion in executing E.O.

464.
Whether E.O. 464 violates the following provisions of the Constitution: Art. II
Sec. 28, Art. III Sec. 4, Art. III Sec. 7, Art. VI Sec. I, Art. VI, Sec. 21, Art. VI Sec. 22,
Article XI sec. 1 and Art. XIII sec. 16.

Whether E.O. 464 contravenes the power of inquiry vested in the Congress.

Whether E.O. 464 violates the right of the people to information on matters of
public concern.

Court

Ruling:

E.O. 464 to the extent that it bars the appearance of executive officials before the
Congress, deprives the Congress of the information in the possession of these
officials. The power of inquiry, a power vested in the Congress, is expressly recognized
in Sec. 21 of Article VI because, according to the Court, a legislative body cannot
legislate wisely or effectively in the absence of information respecting the conditions
which the legislation intended to affect or change; thus, making it an essential and
appropriate
auxiliary
to
the
legislative
function.
However, even when the inquiry is in aid of legislation, there are still recognized
exemptions to the power of inquiry which exemptions fall under the rubric of executive
privilege.
The executive privilege, whether asserted against Congress, the courts, or the public; is
recognized only in relation to certain types of information of a sensitive character.
Executive privilege per se is not meant to cover up embarrassing information. It is a
relative concept, the validity of its assertion to a great extent depends upon the political
situation of the country.

Republic of the Philippines


SUPREME COURT
Manila
EN BANC

G.R. No. 115455 October 30, 1995


ARTURO M. TOLENTINO, petitioner,
vs.
THE SECRETARY OF FINANCE and THE COMMISSIONER OF INTERNAL
REVENUE, respondents.
G.R. No. 115525 October 30, 1995
JUAN T. DAVID, petitioner,
vs.
TEOFISTO T. GUINGONA, JR., as Executive Secretary; ROBERTO DE
OCAMPO, as Secretary of Finance; LIWAYWAY VINZONS-CHATO, as

Commissioner of Internal Revenue; and their AUTHORIZED AGENTS OR


REPRESENTATIVES, respondents.
G.R. No. 115543 October 30, 1995
RAUL S. ROCO and the INTEGRATED BAR OF THE
PHILIPPINES, petitioners,
vs.
THE SECRETARY OF THE DEPARTMENT OF FINANCE; THE
COMMISSIONERS OF THE BUREAU OF INTERNAL REVENUE AND
BUREAU OF CUSTOMS, respondents.
G.R. No. 115544 October 30, 1995
PHILIPPINE PRESS INSTITUTE, INC.; EGP PUBLISHING CO., INC.;
KAMAHALAN PUBLISHING CORPORATION; PHILIPPINE JOURNALISTS,
INC.; JOSE L. PAVIA; and OFELIA L. DIMALANTA, petitioners,
vs.
HON. LIWAYWAY V. CHATO, in her capacity as Commissioner of Internal
Revenue; HON. TEOFISTO T. GUINGONA, JR., in his capacity as Executive
Secretary; and HON. ROBERTO B. DE OCAMPO, in his capacity as
Secretary of Finance, respondents.
G.R. No. 115754 October 30, 1995
CHAMBER OF REAL ESTATE AND BUILDERS ASSOCIATIONS, INC.,
(CREBA), petitioner,
vs.
THE COMMISSIONER OF INTERNAL REVENUE, respondent.
G.R. No. 115781 October 30, 1995
KILOSBAYAN, INC., JOVITO R. SALONGA, CIRILO A. RIGOS, ERME
CAMBA, EMILIO C. CAPULONG, JR., JOSE T. APOLO, EPHRAIM TENDERO,
FERNANDO SANTIAGO, JOSE ABCEDE, CHRISTINE TAN, FELIPE L.
GOZON, RAFAEL G. FERNANDO, RAOUL V. VICTORINO, JOSE CUNANAN,
QUINTIN S. DOROMAL, MOVEMENT OF ATTORNEYS FOR
BROTHERHOOD, INTEGRITY AND NATIONALISM, INC. ("MABINI"),
FREEDOM FROM DEBT COALITION, INC., and PHILIPPINE BIBLE SOCIETY,
INC. and WIGBERTO TAADA,petitioners,
vs.
THE EXECUTIVE SECRETARY, THE SECRETARY OF FINANCE, THE

COMMISSIONER OF INTERNAL REVENUE and THE COMMISSIONER OF


CUSTOMS, respondents.
G.R. No. 115852 October 30, 1995
PHILIPPINE AIRLINES, INC., petitioner,
vs.
THE SECRETARY OF FINANCE and COMMISSIONER OF INTERNAL
REVENUE, respondents.
G.R. No. 115873 October 30, 1995
COOPERATIVE UNION OF THE PHILIPPINES, petitioner,
vs.
HON. LIWAYWAY V. CHATO, in her capacity as the Commissioner of
Internal Revenue, HON. TEOFISTO T. GUINGONA, JR., in his capacity as
Executive Secretary, and HON. ROBERTO B. DE OCAMPO, in his capacity
as Secretary of Finance, respondents.
G.R. No. 115931 October 30, 1995
PHILIPPINE EDUCATIONAL PUBLISHERS ASSOCIATION, INC. and
ASSOCIATION OF PHILIPPINE BOOK SELLERS, petitioners,
vs.
HON. ROBERTO B. DE OCAMPO, as the Secretary of Finance; HON.
LIWAYWAY V. CHATO, as the Commissioner of Internal Revenue; and
HON. GUILLERMO PARAYNO, JR., in his capacity as the Commissioner of
Customs, respondents.
RESOLUTION

MENDOZA, J.:
These are motions seeking reconsideration of our decision dismissing the
petitions filed in these cases for the declaration of unconstitutionality of R.A. No.
7716, otherwise known as the Expanded Value-Added Tax Law. The motions, of
which there are 10 in all, have been filed by the several petitioners in these
cases, with the exception of the Philippine Educational Publishers Association,
Inc. and the Association of Philippine Booksellers, petitioners in G.R. No.
115931.

The Solicitor General, representing the respondents, filed a consolidated


comment, to which the Philippine Airlines, Inc., petitioner in G.R. No. 115852,
and the Philippine Press Institute, Inc., petitioner in G.R. No. 115544, and Juan
T. David, petitioner in G.R. No. 115525, each filed a reply. In turn the Solicitor
General filed on June 1, 1995 a rejoinder to the PPI's reply.
On June 27, 1995 the matter was submitted for resolution.
I. Power of the Senate to propose amendments to revenue bills. Some of the
petitioners (Tolentino, Kilosbayan, Inc., Philippine Airlines (PAL), Roco, and
Chamber of Real Estate and Builders Association (CREBA)) reiterate previous
claims made by them that R.A. No. 7716 did not "originate exclusively" in the
House of Representatives as required by Art. VI, 24 of the Constitution.
Although they admit that H. No. 11197 was filed in the House of Representatives
where it passed three readings and that afterward it was sent to the Senate
where after first reading it was referred to the Senate Ways and Means
Committee, they complain that the Senate did not pass it on second and third
readings. Instead what the Senate did was to pass its own version (S. No. 1630)
which it approved on May 24, 1994. Petitioner Tolentino adds that what the
Senate committee should have done was to amend H. No. 11197 by striking out
the text of the bill and substituting it with the text of S. No. 1630. That way, it is
said, "the bill remains a House bill and the Senate version just becomes the text
(only the text) of the House bill."
The contention has no merit.
The enactment of S. No. 1630 is not the only instance in which the Senate
proposed an amendment to a House revenue bill by enacting its own version of a
revenue bill. On at least two occasions during the Eighth Congress, the Senate
passed its own version of revenue bills, which, in consolidation with House bills
earlier passed, became the enrolled bills. These were:
R.A. No. 7369 (AN ACT TO AMEND THE OMNIBUS INVESTMENTS CODE OF
1987 BY EXTENDING FROM FIVE (5) YEARS TO TEN YEARS THE PERIOD
FOR TAX AND DUTY EXEMPTION AND TAX CREDIT ON CAPITAL
EQUIPMENT) which was approved by the President on April 10, 1992. This Act
is actually a consolidation of H. No. 34254, which was approved by the House on
January 29, 1992, and S. No. 1920, which was approved by the Senate on
February 3, 1992.
R.A. No. 7549 (AN ACT GRANTING TAX EXEMPTIONS TO WHOEVER SHALL
GIVE REWARD TO ANY FILIPINO ATHLETE WINNING A MEDAL IN OLYMPIC
GAMES) which was approved by the President on May 22, 1992. This Act is a

consolidation of H. No. 22232, which was approved by the House of


Representatives on August 2, 1989, and S. No. 807, which was approved by the
Senate on October 21, 1991.
On the other hand, the Ninth Congress passed revenue laws which were also the
result of the consolidation of House and Senate bills. These are the following,
with indications of the dates on which the laws were approved by the President
and dates the separate bills of the two chambers of Congress were respectively
passed:
1. R.A. NO. 7642
AN ACT INCREASING THE PENALTIES FOR TAX EVASION,
AMENDING FOR THIS PURPOSE THE PERTINENT SECTIONS
OF THE NATIONAL INTERNAL REVENUE CODE (December 28,
1992).
House Bill No. 2165, October 5, 1992
Senate Bill No. 32, December 7, 1992
2. R.A. NO. 7643
AN ACT TO EMPOWER THE COMMISSIONER OF INTERNAL
REVENUE TO REQUIRE THE PAYMENT OF THE VALUE-ADDED
TAX EVERY MONTH AND TO ALLOW LOCAL GOVERNMENT
UNITS TO SHARE IN VAT REVENUE, AMENDING FOR THIS
PURPOSE CERTAIN SECTIONS OF THE NATIONAL INTERNAL
REVENUE CODE (December 28, 1992)
House Bill No. 1503, September 3, 1992
Senate Bill No. 968, December 7, 1992
3. R.A. NO. 7646
AN ACT AUTHORIZING THE COMMISSIONER OF INTERNAL
REVENUE TO PRESCRIBE THE PLACE FOR PAYMENT OF
INTERNAL REVENUE TAXES BY LARGE TAXPAYERS,
AMENDING FOR THIS PURPOSE CERTAIN PROVISIONS OF
THE NATIONAL INTERNAL REVENUE CODE, AS AMENDED
(February 24, 1993)
House Bill No. 1470, October 20, 1992

Senate Bill No. 35, November 19, 1992


4. R.A. NO. 7649
AN ACT REQUIRING THE GOVERNMENT OR ANY OF ITS
POLITICAL SUBDIVISIONS, INSTRUMENTALITIES OR
AGENCIES INCLUDING GOVERNMENT-OWNED OR
CONTROLLED CORPORATIONS (GOCCS) TO DEDUCT AND
WITHHOLD THE VALUE-ADDED TAX DUE AT THE RATE OF
THREE PERCENT (3%) ON GROSS PAYMENT FOR THE
PURCHASE OF GOODS AND SIX PERCENT (6%) ON GROSS
RECEIPTS FOR SERVICES RENDERED BY CONTRACTORS
(April 6, 1993)
House Bill No. 5260, January 26, 1993
Senate Bill No. 1141, March 30, 1993
5. R.A. NO. 7656
AN ACT REQUIRING GOVERNMENT-OWNED OR CONTROLLED
CORPORATIONS TO DECLARE DIVIDENDS UNDER CERTAIN
CONDITIONS TO THE NATIONAL GOVERNMENT, AND FOR
OTHER PURPOSES (November 9, 1993)
House Bill No. 11024, November 3, 1993
Senate Bill No. 1168, November 3, 1993
6. R.A. NO. 7660
AN ACT RATIONALIZING FURTHER THE STRUCTURE AND
ADMINISTRATION OF THE DOCUMENTARY STAMP TAX,
AMENDING FOR THE PURPOSE CERTAIN PROVISIONS OF THE
NATIONAL INTERNAL REVENUE CODE, AS AMENDED,
ALLOCATING FUNDS FOR SPECIFIC PROGRAMS, AND FOR
OTHER PURPOSES (December 23, 1993)
House Bill No. 7789, May 31, 1993
Senate Bill No. 1330, November 18, 1993
7. R.A. NO. 7717

AN ACT IMPOSING A TAX ON THE SALE, BARTER OR


EXCHANGE OF SHARES OF STOCK LISTED AND TRADED
THROUGH THE LOCAL STOCK EXCHANGE OR THROUGH
INITIAL PUBLIC OFFERING, AMENDING FOR THE PURPOSE
THE NATIONAL INTERNAL REVENUE CODE, AS AMENDED, BY
INSERTING A NEW SECTION AND REPEALING CERTAIN
SUBSECTIONS THEREOF (May 5, 1994)
House Bill No. 9187, November 3, 1993
Senate Bill No. 1127, March 23, 1994
Thus, the enactment of S. No. 1630 is not the only instance in which the Senate,
in the exercise of its power to propose amendments to bills required to originate
in the House, passed its own version of a House revenue measure. It is
noteworthy that, in the particular case of S. No. 1630, petitioners Tolentino and
Roco, as members of the Senate, voted to approve it on second and third
readings.
On the other hand, amendment by substitution, in the manner urged by petitioner
Tolentino, concerns a mere matter of form. Petitioner has not shown what
substantial difference it would make if, as the Senate actually did in this case, a
separate bill like S. No. 1630 is instead enacted as a substitute measure, "taking
into Consideration . . . H.B.11197."
Indeed, so far as pertinent, the Rules of the Senate only provide:
RULE XXIX
AMENDMENTS
xxx xxx xxx
68. Not more than one amendment to the original amendment shall
be considered.
No amendment by substitution shall be entertained unless the text
thereof is submitted in writing.
Any of said amendments may be withdrawn before a vote is taken
thereon.

69. No amendment which seeks the inclusion of a legislative


provision foreign to the subject matter of a bill (rider) shall be
entertained.
xxx xxx xxx
70-A. A bill or resolution shall not be amended by substituting it
with another which covers a subject distinct from that proposed in
the original bill or resolution. (emphasis added).
Nor is there merit in petitioners' contention that, with regard to revenue bills, the
Philippine Senate possesses less power than the U.S. Senate because of textual
differences between constitutional provisions giving them the power to propose
or concur with amendments.
Art. I, 7, cl. 1 of the U.S. Constitution reads:
All Bills for raising Revenue shall originate in the House of
Representatives; but the Senate may propose or concur with
amendments as on other Bills.
Art. VI, 24 of our Constitution reads:
All appropriation, revenue or tariff bills, bills authorizing increase of
the public debt, bills of local application, and private bills shall
originate exclusively in the House of Representatives, but the
Senate may propose or concur with amendments.
The addition of the word "exclusively" in the Philippine Constitution and the
decision to drop the phrase "as on other Bills" in the American version, according
to petitioners, shows the intention of the framers of our Constitution to restrict the
Senate's power to propose amendments to revenue bills. Petitioner Tolentino
contends that the word "exclusively" was inserted to modify "originate" and "the
words 'as in any other bills' (sic) were eliminated so as to show that these bills
were not to be like other bills but must be treated as a special kind."
The history of this provision does not support this contention. The
supposed indicia of constitutional intent are nothing but the relics of an
unsuccessful attempt to limit the power of the Senate. It will be recalled that the
1935 Constitution originally provided for a unicameral National Assembly. When
it was decided in 1939 to change to a bicameral legislature, it became necessary
to provide for the procedure for lawmaking by the Senate and the House of
Representatives. The work of proposing amendments to the Constitution was

done by the National Assembly, acting as a constituent assembly, some of


whose members, jealous of preserving the Assembly's lawmaking powers,
sought to curtail the powers of the proposed Senate. Accordingly they proposed
the following provision:
All bills appropriating public funds, revenue or tariff bills, bills of local
application, and private bills shall originate exclusively in the
Assembly, but the Senate may propose or concur with amendments.
In case of disapproval by the Senate of any such bills, the Assembly
may repass the same by a two-thirds vote of all its members, and
thereupon, the bill so repassed shall be deemed enacted and may
be submitted to the President for corresponding action. In the event
that the Senate should fail to finally act on any such bills, the
Assembly may, after thirty days from the opening of the next regular
session of the same legislative term, reapprove the same with a vote
of two-thirds of all the members of the Assembly. And upon such
reapproval, the bill shall be deemed enacted and may be submitted
to the President for corresponding action.
The special committee on the revision of laws of the Second National Assembly
vetoed the proposal. It deleted everything after the first sentence. As rewritten,
the proposal was approved by the National Assembly and embodied in
Resolution No. 38, as amended by Resolution No. 73. (J. ARUEGO, KNOW
YOUR CONSTITUTION 65-66 (1950)). The proposed amendment was submitted
to the people and ratified by them in the elections held on June 18, 1940.
This is the history of Art. VI, 18 (2) of the 1935 Constitution, from which Art. VI,
24 of the present Constitution was derived. It explains why the word
"exclusively" was added to the American text from which the framers of the
Philippine Constitution borrowed and why the phrase "as on other Bills" was not
copied. Considering the defeat of the proposal, the power of the Senate to
propose amendments must be understood to be full, plenary and complete "as
on other Bills." Thus, because revenue bills are required to originate exclusively
in the House of Representatives, the Senate cannot enact revenue measures of
its own without such bills. After a revenue bill is passed and sent over to it by the
House, however, the Senate certainly can pass its own version on the same
subject matter. This follows from the coequality of the two chambers of
Congress.
That this is also the understanding of book authors of the scope of the Senate's
power to concur is clear from the following commentaries:

The power of the Senate to propose or concur with amendments is


apparently without restriction. It would seem that by virtue of this
power, the Senate can practically re-write a bill required to come
from the House and leave only a trace of the original bill. For
example, a general revenue bill passed by the lower house of the
United States Congress contained provisions for the imposition of an
inheritance tax . This was changed by the Senate into a corporation
tax. The amending authority of the Senate was declared by the
United States Supreme Court to be sufficiently broad to enable it to
make the alteration. [Flint v. Stone Tracy Company, 220 U.S. 107,
55 L. ed. 389].
(L. TAADA AND F. CARREON, POLITICAL LAW OF THE
PHILIPPINES 247 (1961))
The above-mentioned bills are supposed to be initiated by the
House of Representatives because it is more numerous in
membership and therefore also more representative of the people.
Moreover, its members are presumed to be more familiar with the
needs of the country in regard to the enactment of the legislation
involved.
The Senate is, however, allowed much leeway in the exercise of its
power to propose or concur with amendments to the bills initiated by
the House of Representatives. Thus, in one case, a bill introduced in
the U.S. House of Representatives was changed by the Senate to
make a proposed inheritance tax a corporation tax. It is also
accepted practice for the Senate to introduce what is known as an
amendment by substitution, which may entirely replace the bill
initiated in the House of Representatives.
(I. CRUZ, PHILIPPINE POLITICAL LAW 144-145 (1993)).
In sum, while Art. VI, 24 provides that all appropriation, revenue or tariff bills,
bills authorizing increase of the public debt, bills of local application, and private
bills must "originate exclusively in the House of Representatives," it also adds,
"but the Senate may propose or concur with amendments." In the exercise of this
power, the Senate may propose an entirely new bill as a substitute measure. As
petitioner Tolentino states in a high school text, a committee to which a bill is
referred may do any of the following:
(1) to endorse the bill without changes; (2) to make changes in the
bill omitting or adding sections or altering its language; (3) to make

and endorse an entirely new bill as a substitute, in which case it will


be known as a committee bill; or (4) to make no report at all.
(A. TOLENTINO, THE GOVERNMENT OF THE PHILIPPINES 258
(1950))
To except from this procedure the amendment of bills which are required to
originate in the House by prescribing that the number of the House bill and its
other parts up to the enacting clause must be preserved although the text of the
Senate amendment may be incorporated in place of the original body of the bill is
to insist on a mere technicality. At any rate there is no rule prescribing this form.
S. No. 1630, as a substitute measure, is therefore as much an amendment of H.
No. 11197 as any which the Senate could have made.
II. S. No. 1630 a mere amendment of H. No. 11197. Petitioners' basic error is
that they assume that S. No. 1630 is an independent and distinct bill. Hence their
repeated references to its certification that it was passed by the Senate
"in substitution of S.B. No. 1129, taking into consideration P.S. Res. No. 734
and H.B. No. 11197," implying that there is something substantially different
between the reference to S. No. 1129 and the reference to H. No. 11197. From
this premise, they conclude that R.A. No. 7716 originated both in the House and
in the Senate and that it is the product of two "half-baked bills because neither H.
No. 11197 nor S. No. 1630 was passed by both houses of Congress."
In point of fact, in several instances the provisions of S. No. 1630, clearly appear
to be mere amendments of the corresponding provisions of H. No. 11197. The
very tabular comparison of the provisions of H. No. 11197 and S. No. 1630
attached as Supplement A to the basic petition of petitioner Tolentino, while
showing differences between the two bills, at the same time indicates that the
provisions of the Senate bill were precisely intended to be amendments to the
House bill.
Without H. No. 11197, the Senate could not have enacted S. No. 1630. Because
the Senate bill was a mere amendment of the House bill, H. No. 11197 in its
original form did not have to pass the Senate on second and three readings. It
was enough that after it was passed on first reading it was referred to the Senate
Committee on Ways and Means. Neither was it required that S. No. 1630 be
passed by the House of Representatives before the two bills could be referred to
the Conference Committee.
There is legislative precedent for what was done in the case of H. No. 11197 and
S. No. 1630. When the House bill and Senate bill, which became R.A. No. 1405
(Act prohibiting the disclosure of bank deposits), were referred to a conference

committee, the question was raised whether the two bills could be the subject of
such conference, considering that the bill from one house had not been passed
by the other and vice versa. As Congressman Duran put the question:
MR. DURAN. Therefore, I raise this question of order as to
procedure: If a House bill is passed by the House but not passed by
the Senate, and a Senate bill of a similar nature is passed in the
Senate but never passed in the House, can the two bills be the
subject of a conference, and can a law be enacted from these two
bills? I understand that the Senate bill in this particular instance does
not refer to investments in government securities, whereas the bill in
the House, which was introduced by the Speaker, covers two subject
matters: not only investigation of deposits in banks but also
investigation of investments in government securities. Now, since
the two bills differ in their subject matter, I believe that no law can be
enacted.
Ruling on the point of order raised, the chair (Speaker Jose B. Laurel, Jr.) said:
THE SPEAKER. The report of the conference committee is in order.
It is precisely in cases like this where a conference should be had. If
the House bill had been approved by the Senate, there would have
been no need of a conference; but precisely because the
Senate passed another bill on the same subject matter, the
conference committee had to be created, and we are now
considering the report of that committee.
(2 CONG. REC. NO. 13, July 27, 1955, pp. 3841-42 (emphasis
added))
III. The President's certification. The fallacy in thinking that H. No. 11197 and S.
No. 1630 are distinct and unrelated measures also accounts for the petitioners'
(Kilosbayan's and PAL's) contention that because the President separately
certified to the need for the immediate enactment of these measures, his
certification was ineffectual and void. The certification had to be made of the
version of the same revenue bill which at the moment was being considered.
Otherwise, to follow petitioners' theory, it would be necessary for the President to
certify as many bills as are presented in a house of Congress even though the
bills are merely versions of the bill he has already certified. It is enough that he
certifies the bill which, at the time he makes the certification, is under
consideration. Since on March 22, 1994 the Senate was considering S. No.
1630, it was that bill which had to be certified. For that matter on June 1, 1993
the President had earlier certified H. No. 9210 for immediate enactment because

it was the one which at that time was being considered by the House. This bill
was later substituted, together with other bills, by H. No. 11197.
As to what Presidential certification can accomplish, we have already explained
in the main decision that the phrase "except when the President certifies to the
necessity of its immediate enactment, etc." in Art. VI, 26 (2) qualifies not only
the requirement that "printed copies [of a bill] in its final form [must be] distributed
to the members three days before its passage" but also the requirement that
before a bill can become a law it must have passed "three readings on separate
days." There is not only textual support for such construction but historical basis
as well.
Art. VI, 21 (2) of the 1935 Constitution originally provided:
(2) No bill shall be passed by either House unless it shall have been
printed and copies thereof in its final form furnished its Members at
least three calendar days prior to its passage, except when the
President shall have certified to the necessity of its immediate
enactment. Upon the last reading of a bill, no amendment thereof
shall be allowed and the question upon its passage shall be taken
immediately thereafter, and the yeas and nays entered on the
Journal.
When the 1973 Constitution was adopted, it was provided in Art. VIII, 19 (2):
(2) No bill shall become a law unless it has passed three readings on
separate days, and printed copies thereof in its final form have been
distributed to the Members three days before its passage, except
when the Prime Minister certifies to the necessity of its immediate
enactment to meet a public calamity or emergency. Upon the last
reading of a bill, no amendment thereto shall be allowed, and the
vote thereon shall be taken immediately thereafter, and
the yeas and nays entered in the Journal.
This provision of the 1973 document, with slight modification, was adopted in Art.
VI, 26 (2) of the present Constitution, thus:
(2) No bill passed by either House shall become a law unless it has
passed three readings on separate days, and printed copies thereof
in its final form have been distributed to its Members three days
before its passage, except when the President certifies to the
necessity of its immediate enactment to meet a public calamity or
emergency. Upon the last reading of a bill, no amendment thereto

shall be allowed, and the vote thereon shall be taken immediately


thereafter, and the yeas and nays entered in the Journal.
The exception is based on the prudential consideration that if in all cases three
readings on separate days are required and a bill has to be printed in final form
before it can be passed, the need for a law may be rendered academic by the
occurrence of the very emergency or public calamity which it is meant to
address.
Petitioners further contend that a "growing budget deficit" is not an emergency,
especially in a country like the Philippines where budget deficit is a chronic
condition. Even if this were the case, an enormous budget deficit does not make
the need for R.A. No. 7716 any less urgent or the situation calling for its
enactment any less an emergency.
Apparently, the members of the Senate (including some of the petitioners in
these cases) believed that there was an urgent need for consideration of S. No.
1630, because they responded to the call of the President by voting on the bill on
second and third readings on the same day. While the judicial department is not
bound by the Senate's acceptance of the President's certification, the respect
due coequal departments of the government in matters committed to them by the
Constitution and the absence of a clear showing of grave abuse of discretion
caution a stay of the judicial hand.
At any rate, we are satisfied that S. No. 1630 received thorough consideration in
the Senate where it was discussed for six days. Only its distribution in advance in
its final printed form was actually dispensed with by holding the voting on second
and third readings on the same day (March 24, 1994). Otherwise, sufficient time
between the submission of the bill on February 8, 1994 on second reading and
its approval on March 24, 1994 elapsed before it was finally voted on by the
Senate on third reading.
The purpose for which three readings on separate days is required is said to be
two-fold: (1) to inform the members of Congress of what they must vote on and
(2) to give them notice that a measure is progressing through the enacting
process, thus enabling them and others interested in the measure to prepare
their positions with reference to it. (1 J. G. SUTHERLAND, STATUTES AND
STATUTORY CONSTRUCTION 10.04, p. 282 (1972)). These purposes were
substantially achieved in the case of R.A. No. 7716.
IV. Power of Conference Committee. It is contended (principally by Kilosbayan,
Inc. and the Movement of Attorneys for Brotherhood, Integrity and Nationalism,
Inc. (MABINI)) that in violation of the constitutional policy of full public disclosure

and the people's right to know (Art. II, 28 and Art. III, 7) the Conference
Committee met for two days in executive session with only the conferees
present.
As pointed out in our main decision, even in the United States it was customary
to hold such sessions with only the conferees and their staffs in attendance and it
was only in 1975 when a new rule was adopted requiring open sessions. Unlike
its American counterpart, the Philippine Congress has not adopted a rule
prescribing open hearings for conference committees.
It is nevertheless claimed that in the United States, before the adoption of the
rule in 1975, at least staff members were present. These were staff members of
the Senators and Congressmen, however, who may be presumed to be their
confidential men, not stenographers as in this case who on the last two days of
the conference were excluded. There is no showing that the conferees
themselves did not take notes of their proceedings so as to give petitioner
Kilosbayan basis for claiming that even in secret diplomatic negotiations involving
state interests, conferees keep notes of their meetings. Above all, the public's
right to know was fully served because the Conference Committee in this case
submitted a report showing the changes made on the differing versions of the
House and the Senate.
Petitioners cite the rules of both houses which provide that conference committee
reports must contain "a detailed, sufficiently explicit statement of the changes in
or other amendments." These changes are shown in the bill attached to the
Conference Committee Report. The members of both houses could thus
ascertain what changes had been made in the original bills without the need of a
statement detailing the changes.
The same question now presented was raised when the bill which became R.A.
No. 1400 (Land Reform Act of 1955) was reported by the Conference
Committee. Congressman Bengzon raised a point of order. He said:
MR. BENGZON. My point of order is that it is out of order to consider
the report of the conference committee regarding House Bill No.
2557 by reason of the provision of Section 11, Article XII, of the
Rules of this House which provides specifically that the conference
report must be accompanied by a detailed statement of the effects of
the amendment on the bill of the House. This conference committee
report is not accompanied by that detailed statement, Mr. Speaker.
Therefore it is out of order to consider it.
Petitioner Tolentino, then the Majority Floor Leader, answered:

MR. TOLENTINO. Mr. Speaker, I should just like to say a few words
in connection with the point of order raised by the gentleman from
Pangasinan.
There is no question about the provision of the Rule cited by the
gentleman from Pangasinan, but this provision applies to those
cases where only portions of the bill have been amended. In this
case before us an entire bill is presented; therefore, it can be easily
seen from the reading of the bill what the provisions are.
Besides, this procedure has been an established practice.
After some interruption, he continued:
MR. TOLENTINO. As I was saying, Mr. Speaker, we have to look
into the reason for the provisions of the Rules, and the reason for
the requirement in the provision cited by the gentleman from
Pangasinan is when there are only certain words or phrases inserted
in or deleted from the provisions of the bill included in the
conference report, and we cannot understand what those words and
phrases mean and their relation to the bill. In that case, it is
necessary to make a detailed statement on how those words and
phrases will affect the bill as a whole; but when the entire bill itself is
copied verbatim in the conference report, that is not necessary. So
when the reason for the Rule does not exist, the Rule does not exist.
(2 CONG. REC. NO. 2, p. 4056. (emphasis added))
Congressman Tolentino was sustained by the chair. The record shows that when
the ruling was appealed, it was upheld by viva voce and when a division of the
House was called, it was sustained by a vote of 48 to 5. (Id.,
p. 4058)
Nor is there any doubt about the power of a conference committee to insert new
provisions as long as these are germane to the subject of the conference. As this
Court held in Philippine Judges Association v. Prado, 227 SCRA 703 (1993), in
an opinion written by then Justice Cruz, the jurisdiction of the conference
committee is not limited to resolving differences between the Senate and the
House. It may propose an entirely new provision. What is important is that its
report is subsequently approved by the respective houses of Congress. This
Court ruled that it would not entertain allegations that, because new provisions
had been added by the conference committee, there was thereby a violation of
the constitutional injunction that "upon the last reading of a bill, no amendment
thereto shall be allowed."

Applying these principles, we shall decline to look into the


petitioners' charges that an amendment was made upon the last
reading of the bill that eventually became R.A. No. 7354 and
that copies thereof in its final form were not distributed among the
members of each House. Both the enrolled bill and the legislative
journals certify that the measure was duly enacted i.e., in
accordance with Article VI, Sec. 26 (2) of the Constitution. We are
bound by such official assurances from a coordinate department of
the government, to which we owe, at the very least, a becoming
courtesy.
(Id. at 710. (emphasis added))
It is interesting to note the following description of conference committees in the
Philippines in a 1979 study:
Conference committees may be of two types: free or instructed.
These committees may be given instructions by their parent bodies
or they may be left without instructions. Normally the conference
committees are without instructions, and this is why they are often
critically referred to as "the little legislatures." Once bills have been
sent to them, the conferees have almost unlimited authority to
change the clauses of the bills and in fact sometimes introduce new
measures that were not in the original legislation. No minutes are
kept, and members' activities on conference committees are difficult
to determine. One congressman known for his idealism put it this
way: "I killed a bill on export incentives for my interest group [copra]
in the conference committee but I could not have done so anywhere
else." The conference committee submits a report to both houses,
and usually it is accepted. If the report is not accepted, then the
committee is discharged and new members are appointed.
(R. Jackson, Committees in the Philippine Congress, in
COMMITTEES AND LEGISLATURES: A COMPARATIVE
ANALYSIS 163 (J. D. LEES AND M. SHAW, eds.)).
In citing this study, we pass no judgment on the methods of conference
committees. We cite it only to say that conference committees here are no
different from their counterparts in the United States whose vast powers we
noted in Philippine Judges Association v. Prado, supra. At all events, under Art.
VI, 16(3) each house has the power "to determine the rules of its proceedings,"
including those of its committees. Any meaningful change in the method and

procedures of Congress or its committees must therefore be sought in that body


itself.
V. The titles of S. No. 1630 and H. No. 11197. PAL maintains that R.A. No. 7716
violates Art. VI, 26 (1) of the Constitution which provides that "Every bill passed
by Congress shall embrace only one subject which shall be expressed in the title
thereof." PAL contends that the amendment of its franchise by the withdrawal of
its exemption from the VAT is not expressed in the title of the law.
Pursuant to 13 of P.D. No. 1590, PAL pays a franchise tax of 2% on its gross
revenue "in lieu of all other taxes, duties, royalties, registration, license and other
fees and charges of any kind, nature, or description, imposed, levied,
established, assessed or collected by any municipal, city, provincial or national
authority or government agency, now or in the future."
PAL was exempted from the payment of the VAT along with other entities by
103 of the National Internal Revenue Code, which provides as follows:
103. Exempt transactions. The following shall be exempt from
the value-added tax:
xxx xxx xxx
(q) Transactions which are exempt under special laws or
international agreements to which the Philippines is a signatory.
R.A. No. 7716 seeks to withdraw certain exemptions, including that granted to
PAL, by amending 103, as follows:
103. Exempt transactions. The following shall be exempt from
the value-added tax:
xxx xxx xxx
(q) Transactions which are exempt under special laws, except those
granted under Presidential Decree Nos. 66, 529, 972, 1491, 1590. . .
.
The amendment of 103 is expressed in the title of R.A. No. 7716 which reads:
AN ACT RESTRUCTURING THE VALUE-ADDED TAX (VAT)
SYSTEM, WIDENING ITS TAX BASE AND ENHANCING ITS
ADMINISTRATION, AND FOR THESE PURPOSES AMENDING
AND REPEALING THE RELEVANT PROVISIONS OF THE

NATIONAL INTERNAL REVENUE CODE, AS AMENDED, AND


FOR OTHER PURPOSES.
By stating that R.A. No. 7716 seeks to "[RESTRUCTURE] THE VALUE-ADDED
TAX (VAT) SYSTEM [BY] WIDENING ITS TAX BASE AND ENHANCING ITS
ADMINISTRATION, AND FOR THESE PURPOSES AMENDING AND
REPEALING THE RELEVANT PROVISIONS OF THE NATIONAL INTERNAL
REVENUE CODE, AS AMENDED AND FOR OTHER PURPOSES," Congress
thereby clearly expresses its intention to amend any provision of the NIRC which
stands in the way of accomplishing the purpose of the law.
PAL asserts that the amendment of its franchise must be reflected in the title of
the law by specific reference to P.D. No. 1590. It is unnecessary to do this in
order to comply with the constitutional requirement, since it is already stated in
the title that the law seeks to amend the pertinent provisions of the NIRC, among
which is 103(q), in order to widen the base of the VAT. Actually, it is the bill
which becomes a law that is required to express in its title the subject of
legislation. The titles of H. No. 11197 and S. No. 1630 in fact specifically referred
to 103 of the NIRC as among the provisions sought to be amended. We are
satisfied that sufficient notice had been given of the pendency of these bills in
Congress before they were enacted into what is now R.A.
No. 7716.
In Philippine Judges Association v. Prado, supra, a similar argument as that now
made by PAL was rejected. R.A. No. 7354 is entitled AN ACT CREATING THE
PHILIPPINE POSTAL CORPORATION, DEFINING ITS POWERS, FUNCTIONS
AND RESPONSIBILITIES, PROVIDING FOR REGULATION OF THE
INDUSTRY AND FOR OTHER PURPOSES CONNECTED THEREWITH. It
contained a provision repealing all franking privileges. It was contended that the
withdrawal of franking privileges was not expressed in the title of the law. In
holding that there was sufficient description of the subject of the law in its title,
including the repeal of franking privileges, this Court held:
To require every end and means necessary for the accomplishment
of the general objectives of the statute to be expressed in its title
would not only be unreasonable but would actually render legislation
impossible. [Cooley, Constitutional Limitations, 8th Ed., p. 297] As
has been correctly explained:
The details of a legislative act need not be specifically
stated in its title, but matter germane to the subject as
expressed in the title, and adopted to the
accomplishment of the object in view, may properly be

included in the act. Thus, it is proper to create in the


same act the machinery by which the act is to be
enforced, to prescribe the penalties for its infraction, and
to remove obstacles in the way of its execution. If such
matters are properly connected with the subject as
expressed in the title, it is unnecessary that they should
also have special mention in the title. (Southern Pac.
Co. v. Bartine, 170 Fed. 725)
(227 SCRA at 707-708)
VI. Claims of press freedom and religious liberty. We have held that, as a general
proposition, the press is not exempt from the taxing power of the State and that
what the constitutional guarantee of free press prohibits are laws which single out
the press or target a group belonging to the press for special treatment or which
in any way discriminate against the press on the basis of the content of the
publication, and R.A. No. 7716 is none of these.
Now it is contended by the PPI that by removing the exemption of the press from
the VAT while maintaining those granted to others, the law discriminates against
the press. At any rate, it is averred, "even nondiscriminatory taxation of
constitutionally guaranteed freedom is unconstitutional."
With respect to the first contention, it would suffice to say that since the law
granted the press a privilege, the law could take back the privilege anytime
without offense to the Constitution. The reason is simple: by granting
exemptions, the State does not forever waive the exercise of its sovereign
prerogative.
Indeed, in withdrawing the exemption, the law merely subjects the press to the
same tax burden to which other businesses have long ago been subject. It is
thus different from the tax involved in the cases invoked by the PPI. The license
tax in Grosjean v. American Press Co., 297 U.S. 233, 80 L. Ed. 660 (1936) was
found to be discriminatory because it was laid on the gross advertising receipts
only of newspapers whose weekly circulation was over 20,000, with the result
that the tax applied only to 13 out of 124 publishers in Louisiana. These large
papers were critical of Senator Huey Long who controlled the state legislature
which enacted the license tax. The censorial motivation for the law was thus
evident.
On the other hand, in Minneapolis Star & Tribune Co. v. Minnesota Comm'r of
Revenue, 460 U.S. 575, 75 L. Ed. 2d 295 (1983), the tax was found to be
discriminatory because although it could have been made liable for the sales tax

or, in lieu thereof, for the use tax on the privilege of using, storing or consuming
tangible goods, the press was not. Instead, the press was exempted from both
taxes. It was, however, later made to pay a special use tax on the cost of paper
and ink which made these items "the only items subject to the use tax that were
component of goods to be sold at retail." The U.S. Supreme Court held that the
differential treatment of the press "suggests that the goal of regulation is not
related to suppression of expression, and such goal is presumptively
unconstitutional." It would therefore appear that even a law that favors the press
is constitutionally suspect. (See the dissent of Rehnquist, J. in that case)
Nor is it true that only two exemptions previously granted by E.O. No. 273 are
withdrawn "absolutely and unqualifiedly" by R.A. No. 7716. Other exemptions
from the VAT, such as those previously granted to PAL, petroleum
concessionaires, enterprises registered with the Export Processing Zone
Authority, and many more are likewise totally withdrawn, in addition to
exemptions which are partially withdrawn, in an effort to broaden the base of the
tax.
The PPI says that the discriminatory treatment of the press is highlighted by the
fact that transactions, which are profit oriented, continue to enjoy exemption
under R.A. No. 7716. An enumeration of some of these transactions will suffice
to show that by and large this is not so and that the exemptions are granted for a
purpose. As the Solicitor General says, such exemptions are granted, in some
cases, to encourage agricultural production and, in other cases, for the personal
benefit of the end-user rather than for profit. The exempt transactions are:
(a) Goods for consumption or use which are in their original state
(agricultural, marine and forest products, cotton seeds in their
original state, fertilizers, seeds, seedlings, fingerlings, fish, prawn
livestock and poultry feeds) and goods or services to enhance
agriculture (milling of palay, corn, sugar cane and raw sugar,
livestock, poultry feeds, fertilizer, ingredients used for the
manufacture of feeds).
(b) Goods used for personal consumption or use (household and
personal effects of citizens returning to the Philippines) or for
professional use, like professional instruments and implements, by
persons coming to the Philippines to settle here.
(c) Goods subject to excise tax such as petroleum products or to be
used for manufacture of petroleum products subject to excise tax
and services subject to percentage tax.

(d) Educational services, medical, dental, hospital and veterinary


services, and services rendered under employer-employee
relationship.
(e) Works of art and similar creations sold by the artist himself.
(f) Transactions exempted under special laws, or international
agreements.
(g) Export-sales by persons not VAT-registered.
(h) Goods or services with gross annual sale or receipt not
exceeding P500,000.00.
(Respondents' Consolidated Comment on the Motions for
Reconsideration, pp. 58-60)
The PPI asserts that it does not really matter that the law does not discriminate
against the press because "even nondiscriminatory taxation on constitutionally
guaranteed freedom is unconstitutional." PPI cites in support of this assertion the
following statement in Murdock v. Pennsylvania, 319 U.S. 105, 87 L. Ed. 1292
(1943):
The fact that the ordinance is "nondiscriminatory" is immaterial. The
protection afforded by the First Amendment is not so restricted. A
license tax certainly does not acquire constitutional validity because
it classifies the privileges protected by the First Amendment along
with the wares and merchandise of hucksters and peddlers and
treats them all alike. Such equality in treatment does not save the
ordinance. Freedom of press, freedom of speech, freedom of religion
are in preferred position.
The Court was speaking in that case of a license tax, which, unlike an ordinary
tax, is mainly for regulation. Its imposition on the press is unconstitutional
because it lays a prior restraint on the exercise of its right. Hence, although its
application to others, such those selling goods, is valid, its application to the
press or to religious groups, such as the Jehovah's Witnesses, in connection with
the latter's sale of religious books and pamphlets, is unconstitutional. As the U.S.
Supreme Court put it, "it is one thing to impose a tax on income or property of a
preacher. It is quite another thing to exact a tax on him for delivering a sermon."
A similar ruling was made by this Court in American Bible Society v. City of
Manila, 101 Phil. 386 (1957) which invalidated a city ordinance requiring a

business license fee on those engaged in the sale of general merchandise. It


was held that the tax could not be imposed on the sale of bibles by the American
Bible Society without restraining the free exercise of its right to propagate.
The VAT is, however, different. It is not a license tax. It is not a tax on the
exercise of a privilege, much less a constitutional right. It is imposed on the sale,
barter, lease or exchange of goods or properties or the sale or exchange of
services and the lease of properties purely for revenue purposes. To subject the
press to its payment is not to burden the exercise of its right any more than to
make the press pay income tax or subject it to general regulation is not to violate
its freedom under the Constitution.
Additionally, the Philippine Bible Society, Inc. claims that although it sells bibles,
the proceeds derived from the sales are used to subsidize the cost of printing
copies which are given free to those who cannot afford to pay so that to tax the
sales would be to increase the price, while reducing the volume of sale. Granting
that to be the case, the resulting burden on the exercise of religious freedom is
so incidental as to make it difficult to differentiate it from any other economic
imposition that might make the right to disseminate religious doctrines costly.
Otherwise, to follow the petitioner's argument, to increase the tax on the sale of
vestments would be to lay an impermissible burden on the right of the preacher
to make a sermon.
On the other hand the registration fee of P1,000.00 imposed by 107 of the
NIRC, as amended by 7 of R.A. No. 7716, although fixed in amount, is really
just to pay for the expenses of registration and enforcement of provisions such as
those relating to accounting in 108 of the NIRC. That the PBS distributes free
bibles and therefore is not liable to pay the VAT does not excuse it from the
payment of this fee because it also sells some copies. At any rate whether the
PBS is liable for the VAT must be decided in concrete cases, in the event it is
assessed this tax by the Commissioner of Internal Revenue.
VII. Alleged violations of the due process, equal protection and contract clauses
and the rule on taxation. CREBA asserts that R.A. No. 7716 (1) impairs the
obligations of contracts, (2) classifies transactions as covered or exempt without
reasonable basis and (3) violates the rule that taxes should be uniform and
equitable and that Congress shall "evolve a progressive system of taxation."
With respect to the first contention, it is claimed that the application of the tax to
existing contracts of the sale of real property by installment or on deferred
payment basis would result in substantial increases in the monthly amortizations
to be paid because of the 10% VAT. The additional amount, it is pointed out, is

something that the buyer did not anticipate at the time he entered into the
contract.
The short answer to this is the one given by this Court in an early case:
"Authorities from numerous sources are cited by the plaintiffs, but none of them
show that a lawful tax on a new subject, or an increased tax on an old one,
interferes with a contract or impairs its obligation, within the meaning of the
Constitution. Even though such taxation may affect particular contracts, as it may
increase the debt of one person and lessen the security of another, or may
impose additional burdens upon one class and release the burdens of another,
still the tax must be paid unless prohibited by the Constitution, nor can it be said
that it impairs the obligation of any existing contract in its true legal sense." (La
Insular v. Machuca Go-Tauco and Nubla Co-Siong, 39 Phil. 567, 574 (1919)).
Indeed not only existing laws but also "the reservation of the essential attributes
of sovereignty, is . . . read into contracts as a postulate of the legal order."
(Philippine-American Life Ins. Co. v. Auditor General, 22 SCRA 135, 147 (1968))
Contracts must be understood as having been made in reference to the possible
exercise of the rightful authority of the government and no obligation of contract
can extend to the defeat of that authority. (Norman v. Baltimore and Ohio R.R.,
79 L. Ed. 885 (1935)).
It is next pointed out that while 4 of R.A. No. 7716 exempts such transactions as
the sale of agricultural products, food items, petroleum, and medical and
veterinary services, it grants no exemption on the sale of real property which is
equally essential. The sale of real property for socialized and low-cost housing is
exempted from the tax, but CREBA claims that real estate transactions of "the
less poor," i.e., the middle class, who are equally homeless, should likewise be
exempted.
The sale of food items, petroleum, medical and veterinary services, etc., which
are essential goods and services was already exempt under 103, pars. (b) (d)
(1) of the NIRC before the enactment of R.A. No. 7716. Petitioner is in error in
claiming that R.A. No. 7716 granted exemption to these transactions, while
subjecting those of petitioner to the payment of the VAT. Moreover, there is a
difference between the "homeless poor" and the "homeless less poor" in the
example given by petitioner, because the second group or middle class can
afford to rent houses in the meantime that they cannot yet buy their own homes.
The two social classes are thus differently situated in life. "It is inherent in the
power to tax that the State be free to select the subjects of taxation, and it has
been repeatedly held that 'inequalities which result from a singling out of one
particular class for taxation, or exemption infringe no constitutional limitation.'"
(Lutz v. Araneta, 98 Phil. 148, 153 (1955). Accord, City of Baguio v. De Leon,
134 Phil. 912 (1968); Sison, Jr. v. Ancheta, 130 SCRA 654, 663 (1984);

Kapatiran ng mga Naglilingkod sa Pamahalaan ng Pilipinas, Inc. v. Tan, 163


SCRA 371 (1988)).
Finally, it is contended, for the reasons already noted, that R.A. No. 7716 also
violates Art. VI, 28(1) which provides that "The rule of taxation shall be uniform
and equitable. The Congress shall evolve a progressive system of taxation."
Equality and uniformity of taxation means that all taxable articles or kinds of
property of the same class be taxed at the same rate. The taxing power has the
authority to make reasonable and natural classifications for purposes of taxation.
To satisfy this requirement it is enough that the statute or ordinance applies
equally to all persons, forms and corporations placed in similar situation. (City of
Baguio v. De Leon, supra; Sison, Jr. v. Ancheta, supra)
Indeed, the VAT was already provided in E.O. No. 273 long before R.A. No. 7716
was enacted. R.A. No. 7716 merely expands the base of the tax. The validity of
the original VAT Law was questioned in Kapatiran ng Naglilingkod sa
Pamahalaan ng Pilipinas, Inc. v. Tan, 163 SCRA 383 (1988) on grounds similar
to those made in these cases, namely, that the law was "oppressive,
discriminatory, unjust and regressive in violation of Art. VI, 28(1) of the
Constitution." (At 382) Rejecting the challenge to the law, this Court held:
As the Court sees it, EO 273 satisfies all the requirements of a valid
tax. It is uniform. . . .
The sales tax adopted in EO 273 is applied similarly on all goods
and services sold to the public, which are not exempt, at the
constant rate of 0% or 10%.
The disputed sales tax is also equitable. It is imposed only on sales
of goods or services by persons engaged in business with an
aggregate gross annual sales exceeding P200,000.00. Small corner
sari-sari stores are consequently exempt from its application.
Likewise exempt from the tax are sales of farm and marine products,
so that the costs of basic food and other necessities, spared as they
are from the incidence of the VAT, are expected to be relatively
lower and within the reach of the general public.
(At 382-383)
The CREBA claims that the VAT is regressive. A similar claim is made by the
Cooperative Union of the Philippines, Inc. (CUP), while petitioner Juan T. David
argues that the law contravenes the mandate of Congress to provide for a

progressive system of taxation because the law imposes a flat rate of 10% and
thus places the tax burden on all taxpayers without regard to their ability to pay.
The Constitution does not really prohibit the imposition of indirect taxes which,
like the VAT, are regressive. What it simply provides is that Congress shall
"evolve a progressive system of taxation." The constitutional provision has been
interpreted to mean simply that "direct taxes are . . . to be preferred [and] as
much as possible, indirect taxes should be minimized." (E. FERNANDO, THE
CONSTITUTION OF THE PHILIPPINES 221 (Second ed. (1977)). Indeed, the
mandate to Congress is not to prescribe, but to evolve, a progressive tax system.
Otherwise, sales taxes, which perhaps are the oldest form of indirect taxes,
would have been prohibited with the proclamation of Art. VIII, 17(1) of the 1973
Constitution from which the present Art. VI, 28(1) was taken. Sales taxes are
also regressive.
Resort to indirect taxes should be minimized but not avoided entirely because it
is difficult, if not impossible, to avoid them by imposing such taxes according to
the taxpayers' ability to pay. In the case of the VAT, the law minimizes the
regressive effects of this imposition by providing for zero rating of certain
transactions (R.A. No. 7716, 3, amending 102 (b) of the NIRC), while
granting exemptions to other transactions. (R.A. No. 7716, 4, amending 103 of
the NIRC).
Thus, the following transactions involving basic and essential goods and services
are exempted from the VAT:
(a) Goods for consumption or use which are in their original state
(agricultural, marine and forest products, cotton seeds in their
original state, fertilizers, seeds, seedlings, fingerlings, fish, prawn
livestock and poultry feeds) and goods or services to enhance
agriculture (milling of palay, corn sugar cane and raw sugar,
livestock, poultry feeds, fertilizer, ingredients used for the
manufacture of feeds).
(b) Goods used for personal consumption or use (household and
personal effects of citizens returning to the Philippines) and or
professional use, like professional instruments and implements, by
persons coming to the Philippines to settle here.
(c) Goods subject to excise tax such as petroleum products or to be
used for manufacture of petroleum products subject to excise tax
and services subject to percentage tax.

(d) Educational services, medical, dental, hospital and veterinary


services, and services rendered under employer-employee
relationship.
(e) Works of art and similar creations sold by the artist himself.
(f) Transactions exempted under special laws, or international
agreements.
(g) Export-sales by persons not VAT-registered.
(h) Goods or services with gross annual sale or receipt not
exceeding P500,000.00.
(Respondents' Consolidated Comment on the Motions for
Reconsideration, pp. 58-60)
On the other hand, the transactions which are subject to the VAT are those which
involve goods and services which are used or availed of mainly by higher income
groups. These include real properties held primarily for sale to customers or for
lease in the ordinary course of trade or business, the right or privilege to use
patent, copyright, and other similar property or right, the right or privilege to use
industrial, commercial or scientific equipment, motion picture films, tapes and
discs, radio, television, satellite transmission and cable television time, hotels,
restaurants and similar places, securities, lending investments, taxicabs, utility
cars for rent, tourist buses, and other common carriers, services of franchise
grantees of telephone and telegraph.
The problem with CREBA's petition is that it presents broad claims of
constitutional violations by tendering issues not at retail but at wholesale and in
the abstract. There is no fully developed record which can impart to adjudication
the impact of actuality. There is no factual foundation to show in the concrete the
application of the law to actual contracts and exemplify its effect on property
rights. For the fact is that petitioner's members have not even been assessed the
VAT. Petitioner's case is not made concrete by a series of hypothetical questions
asked which are no different from those dealt with in advisory opinions.
The difficulty confronting petitioner is thus apparent. He alleges
arbitrariness. A mere allegation, as here, does not suffice. There
must be a factual foundation of such unconstitutional taint.
Considering that petitioner here would condemn such a provision as
void on its face, he has not made out a case. This is merely to
adhere to the authoritative doctrine that where the due process and

equal protection clauses are invoked, considering that they are not
fixed rules but rather broad standards, there is a need for proof of
such persuasive character as would lead to such a conclusion.
Absent such a showing, the presumption of validity must prevail.
(Sison, Jr. v. Ancheta, 130 SCRA at 661)
Adjudication of these broad claims must await the development of a concrete
case. It may be that postponement of adjudication would result in a multiplicity of
suits. This need not be the case, however. Enforcement of the law may give rise
to such a case. A test case, provided it is an actual case and not an abstract or
hypothetical one, may thus be presented.
Nor is hardship to taxpayers alone an adequate justification for adjudicating
abstract issues. Otherwise, adjudication would be no different from the giving of
advisory opinion that does not really settle legal issues.
We are told that it is our duty under Art. VIII, 1, 2 to decide whenever a claim is
made that "there has been a grave abuse of discretion amounting to lack or
excess of jurisdiction on the part of any branch or instrumentality of the
government." This duty can only arise if an actual case or controversy is before
us. Under Art . VIII, 5 our jurisdiction is defined in terms of "cases" and all that
Art. VIII, 1, 2 can plausibly mean is that in the exercise of that jurisdiction we
have the judicial power to determine questions of grave abuse of discretion by
any branch or instrumentality of the government.
Put in another way, what is granted in Art. VIII, 1, 2 is "judicial power," which is
"the power of a court to hear and decide cases pending between parties who
have the right to sue and be sued in the courts of law and equity" (Lamb v.
Phipps, 22 Phil. 456, 559 (1912)), as distinguished from legislative and executive
power. This power cannot be directly appropriated until it is apportioned among
several courts either by the Constitution, as in the case of Art. VIII, 5, or by
statute, as in the case of the Judiciary Act of 1948 (R.A. No. 296) and the
Judiciary Reorganization Act of 1980 (B.P. Blg. 129). The power thus
apportioned constitutes the court's "jurisdiction," defined as "the power conferred
by law upon a court or judge to take cognizance of a case, to the exclusion of all
others." (United States v. Arceo, 6 Phil. 29 (1906)) Without an actual case
coming within its jurisdiction, this Court cannot inquire into any allegation of grave
abuse of discretion by the other departments of the government.
VIII. Alleged violation of policy towards cooperatives. On the other hand, the
Cooperative Union of the Philippines (CUP), after briefly surveying the course of
legislation, argues that it was to adopt a definite policy of granting tax exemption

to cooperatives that the present Constitution embodies provisions on


cooperatives. To subject cooperatives to the VAT would therefore be to infringe a
constitutional policy. Petitioner claims that in 1973, P.D. No. 175 was
promulgated exempting cooperatives from the payment of income taxes and
sales taxes but in 1984, because of the crisis which menaced the national
economy, this exemption was withdrawn by P.D. No. 1955; that in 1986, P.D. No.
2008 again granted cooperatives exemption from income and sales taxes until
December 31, 1991, but, in the same year, E.O. No. 93 revoked the exemption;
and that finally in 1987 the framers of the Constitution "repudiated the previous
actions of the government adverse to the interests of the cooperatives, that
is, the repeated revocation of the tax exemption to cooperatives and instead
upheld the policy of strengthening the cooperatives by way of the grant of tax
exemptions," by providing the following in Art. XII:
1. The goals of the national economy are a more equitable
distribution of opportunities, income, and wealth; a sustained
increase in the amount of goods and services produced by the
nation for the benefit of the people; and an expanding productivity as
the key to raising the quality of life for all, especially the
underprivileged.
The State shall promote industrialization and full employment based
on sound agricultural development and agrarian reform, through
industries that make full and efficient use of human and natural
resources, and which are competitive in both domestic and foreign
markets. However, the State shall protect Filipino enterprises against
unfair foreign competition and trade practices.
In the pursuit of these goals, all sectors of the economy and all
regions of the country shall be given optimum opportunity to
develop. Private enterprises, including corporations, cooperatives,
and similar collective organizations, shall be encouraged to broaden
the base of their ownership.
15. The Congress shall create an agency to promote the viability
and growth of cooperatives as instruments for social justice and
economic development.
Petitioner's contention has no merit. In the first place, it is not true that P.D. No.
1955 singled out cooperatives by withdrawing their exemption from income and
sales taxes under P.D. No. 175, 5. What P.D. No. 1955, 1 did was to
withdraw the exemptions and preferential treatments theretofore granted to
private business enterprises in general, in view of the economic crisis which then

beset the nation. It is true that after P.D. No. 2008, 2 had restored the tax
exemptions of cooperatives in 1986, the exemption was again repealed by E.O.
No. 93, 1, but then again cooperatives were not the only ones whose
exemptions were withdrawn. The withdrawal of tax incentives applied to all,
including government and private entities. In the second place, the Constitution
does not really require that cooperatives be granted tax exemptions in order to
promote their growth and viability. Hence, there is no basis for petitioner's
assertion that the government's policy toward cooperatives had been one of
vacillation, as far as the grant of tax privileges was concerned, and that it was to
put an end to this indecision that the constitutional provisions cited were adopted.
Perhaps as a matter of policy cooperatives should be granted tax exemptions,
but that is left to the discretion of Congress. If Congress does not grant
exemption and there is no discrimination to cooperatives, no violation of any
constitutional policy can be charged.
Indeed, petitioner's theory amounts to saying that under the Constitution
cooperatives are exempt from taxation. Such theory is contrary to the
Constitution under which only the following are exempt from taxation: charitable
institutions, churches and parsonages, by reason of Art. VI, 28 (3), and nonstock, non-profit educational institutions by reason of Art. XIV, 4 (3).
CUP's further ground for seeking the invalidation of R.A. No. 7716 is that it
denies cooperatives the equal protection of the law because electric cooperatives
are exempted from the VAT. The classification between electric and other
cooperatives (farmers cooperatives, producers cooperatives, marketing
cooperatives, etc.) apparently rests on a congressional determination that there
is greater need to provide cheaper electric power to as many people as possible,
especially those living in the rural areas, than there is to provide them with other
necessities in life. We cannot say that such classification is unreasonable.
We have carefully read the various arguments raised against the constitutional
validity of R.A. No. 7716. We have in fact taken the extraordinary step of
enjoining its enforcement pending resolution of these cases. We have now come
to the conclusion that the law suffers from none of the infirmities attributed to it by
petitioners and that its enactment by the other branches of the government does
not constitute a grave abuse of discretion. Any question as to its necessity,
desirability or expediency must be addressed to Congress as the body which is
electorally responsible, remembering that, as Justice Holmes has said,
"legislators are the ultimate guardians of the liberties and welfare of the people in
quite as great a degree as are the courts." (Missouri, Kansas & Texas Ry. Co. v.
May, 194 U.S. 267, 270, 48 L. Ed. 971, 973 (1904)). It is not right, as petitioner in
G.R. No. 115543 does in arguing that we should enforce the public accountability
of legislators, that those who took part in passing the law in question by voting for

it in Congress should later thrust to the courts the burden of reviewing measures
in the flush of enactment. This Court does not sit as a third branch of the
legislature, much less exercise a veto power over legislation.
WHEREFORE, the motions for reconsideration are denied with finality and the
temporary restraining order previously issued is hereby lifted.
SO ORDERED.
G.R. No. 115455
235 SCRA 630 (1994)
FACTS
RA 7716, otherwise known as the Expanded Value-Added Tax Law, is an act that seeks to
widen the tax base of the existing VAT system and enhance its administration by amending
the National Internal Revenue Code. There are various suits questioning and challenging
the constitutionality of RA 7716 on various grounds.
Tolentino contends that RA 7716 did not originate exclusively from the House of
Representatives but is a mere consolidation of HB. No. 11197 and SB. No. 1630 and it did
not pass three readings on separate days on the Senate thus violating Article VI, Sections
24 and 26(2) of the Constitution, respectively.
Art. VI, Section 24: All appropriation, revenue or tariff bills, bills authorizing increase of the
public debt, bills of local application, and private bills shall originate exclusively in the
House of Representatives, but the Senate may propose or concur with amendments.
Art. VI, Section 26(2): No bill passed by either House shall become a law unless it has
passed three readings on separate days, and printed copies thereof in its final form have
been distributed to its Members three days before its passage, except when the President
certifies to the necessity of its immediate enactment to meet a public calamity or
emergency. Upon the last reading of a bill, no amendment thereto shall be allowed, and
the vote thereon shall be taken immediately thereafter, and the yeas and nays entered in
the Journal.
ISSUE
Whether or not RA 7716 violated Art. VI, Section 24 and Art. VI, Section 26(2) of the
Constitution.

HELD
No. The phrase originate exclusively refers to the revenue bill and not to the revenue
law. It is sufficient that the House of Representatives initiated the passage of the bill which
may undergo extensive changes in the Senate.
SB. No. 1630, having been certified as urgent by the President need not meet the
requirement not only of printing but also of reading the bill on separate days.
[Syllabus]

EN BANC

[G.R. No. 118303. January 31, 1996]

SENATOR HEHERSON T. ALVAREZ, SENATOR JOSE D. LINA, JR.,


MR. NICASIO B. BAUTISTA, MR. JESUS P. GONZAGA, MR.
SOLOMON D. MAYLEM, LEONORA C. MEDINA, CASIANO S.
ALIPON, petitioners, vs. HON. TEOFISTO T. GUINGONA, JR., in
his capacity as Executive Secretary, HON. RAFAEL ALUNAN, in
his capacity as Secretary of Local Government, HON.
SALVADOR ENRIQUEZ, in his capacity as Secretary of Budget,
THE COMMISSION ON AUDIT, HON. JOSE MIRANDA, in his
capacity as Municipal Mayor of Santiago and HON. CHARITO
MANUBAY, HON. VICTORINO MIRANDA, JR., HON. ARTEMIO
ALVAREZ, HON. DANILO VERGARA, HON. PETER DE JESUS,
HON. NELIA NATIVIDAD, HON. CELSO CALEON and HON. ABEL
MUSNGI, in their capacity as SANGGUNIANG BAYAN
MEMBERS, MR. RODRIGO L. SANTOS, in his capacity as
Municipal Treasurer, and ATTY. ALFREDO S. DIRIGE, in his
capacity as Municipal Administrator, respondents.
DECISION
HERMOSISIMA, JR., J.:

Of main concern to the petitioners is whether Republic Act No. 7720, just
recently passed by Congress and signed by the President into law, is
constitutionally infirm.

Indeed, in this Petition for Prohibition with prayer for Temporary


Restraining Order and Preliminary Prohibitory Injunction, petitioners assail the
validity of Republic Act No. 7720, entitled, An Act Converting the Municipality
of Santiago, Isabela into an Independent Component City to be known as the
City of Santiago, mainly because the Act allegedly did not originate
exclusively in the House of Representatives as mandated by Section 24,
Article VI of the 1987 Constitution.
Also, petitioners claim that the Municipality of Santiago has not met the
minimum average annual income required under Section 450 of the Local
Government Code of 1991 in order to be converted into a component city.
Undisputed is the following chronicle of the metamorphosis of House Bill
No. 8817 into Republic Act No. 7720:
On April 18, 1993, HB No. 8817, entitled An Act Converting
the Municipality of Santiago into an Independent Component City to be known
as the City of Santiago, was filed in the House of Representatives with
Representative Antonio Abaya as principal author. Other sponsors included
Representatives Ciriaco Alfelor, Rodolfo Albano, Santiago Respicio and
Faustino Dy. The bill was referred to the House Committee on Local
Government and the House Committee on Appropriations on May 5, 1993.
On May 19, 1993, June 1, 1993, November 28, 1993, and December 1,
1993, public hearings on HB No. 8817 were conducted by the House
Committee on Local Government. The committee submitted to the House a
favorable report, with amendments, on December 9, 1993.
On December 13, 1993, HB No. 8817 was passed by the House of
Representatives on Second Reading and was approved on Third Reading
on December 17, 1993. On January 28, 1994, HB No. 8817 was transmitted
to the Senate.
Meanwhile, a counterpart of HB No. 8817, Senate Bill No. 1243, entitled,
An
Act
Converting
the
Municipality
of Santiago into
an
Independent] Component City to be Known as the City of Santiago, was filed
in the Senate. It was introduced by Senator Vicente Sotto III, as principal
sponsor, on May 19, 1993. This was just after the House of Representatives
had conducted its first public hearing on HB No. 8817.
On February 23, 1994, or a little less than a month after HB No. 8817 was
transmitted to the Senate, the Senate Committee on Local Government
conducted public hearings on SB No. 1243. On March 1, 1994, the said
committee submitted Committee Report No. 378 on HB No. 8817, with the
recommendation that it be approved without amendment, taking into

consideration the reality that H.B. No. 8817 was on all fours with SB No. 1243.
Senator Heherson T. Alvarez, one of the herein petitioners, indicated his
approval thereto by signing said report as member of the Committee on Local
Government.
On March 3, 1994, Committee Report No. 378 was passed by the Senate
on Second Reading and was approved on Third Reading on March 14, 1994.
On March 22, 1994, the House of Representatives, upon being apprised of
the action of the Senate, approved the amendments proposed by the Senate.
The enrolled bill, submitted to the President on April 12, 1994, was signed
by the Chief Executive on May 5, 1994 as Republic Act No. 7720. When a
plebiscite on the Act was held onJuly 13, 1994, a great majority of the
registered voters of Santiago voted in favor of the conversion of Santiago into
a city.
The question as to the validity of Republic Act No. 7720 hinges on the
following twin issues: (I) Whether or not the Internal Revenue Allotments
(IRAs) are to be included in the computation of the average annual income of
a municipality for purposes of its conversion into an independent component
city, and (II) Whether or not, considering that the Senate passed SB No. 1243,
its own version of HB No. 8817, Republic Act No. 7720 can be said to have
originated in the House of Representatives.
I
The annual income of a local
government unit includes the IRAs
----------------------------------------------------------Petitioners claim that Santiago could not qualify into a component city
because its average annual income for the last two (2) consecutive years
based on 1991 constant prices falls below the required annual income of
Twenty Million Pesos (P20,000,000.00) for its conversion into a city,
petitioners having computed Santiagos average annual income in the
following manner:
Total income (at 1991 constant prices) for 1991

P20,379,057.07

Total income (at 1991 constant prices) for 1992

P21,570,106.87

Total income for 1991 and 1992

P41,949,163.94

Minus:
IRAs for 1991 and 1992
Total income for 1991 and 1992
Average Annual Income

P15,730,043.00
P26,219,120.94
P13,109,960.47

By dividing the total income of Santiago for calendar years 1991 and 1992,
after deducting the IRAs, the average annual income arrived at would only be
P13,109,560.47 based on the 1991 constant prices. Thus, petitioners claim
that Santiagos income is far below the aforesaid Twenty Million Pesos
average annual income requirement.
The certification issued by the Bureau of Local Government Finance of the
Department of Finance, which indicates Santiagos average annual income to
be P20,974,581.97, is allegedly not accurate as the Internal Revenue
Allotments were not excluded from the computation. Petitioners asseverate
that the IRAs are not actually income but transfers and! or budgetary aid from
the national government and that they fluctuate, increase or decrease,
depending on factors like population, land and equal sharing.
In this regard, we hold that petitioners asseverations are untenable
because Internal Revenue Allotments form part of the income of Local
Government Units.
It is true that for a municipality to be converted into a component city, it
must, among others, have an average annual income of at least Twenty
Million Pesos for the last two (2) consecutive years based on 1991 constant
prices.1 Such income must be duly certified by the Department of Finance.2
Resolution
of
the
controversy
regarding
compliance
by
the Municipality of Santiago with the aforecited income requirement hinges on
a correlative and contextual explication of the meaning of internal revenue
allotments (IRAs) vis-a-vis the notion of income of a local government unit and
the principles of local autonomy and decentralization underlying the
institutionalization and intensified empowerment of the local government
system.
A Local Government Unit is a political subdivision of the State which is
constituted by law and possessed of substantial control over its own
affairs.3 Remaining to be an intra sovereign subdivision of one sovereign
nation, but not intended, however, to be an imperium in imperio,4 the local
government unit is autonomous in the sense that it is given more powers,

authority, responsibilities and resources.5 Power which used to be highly


centralized in Manila, is thereby deconcentrated, enabling especially the
peripheral local government units to develop not only at their own pace and
discretion but also with their oWn resources and assets.6
The practical side to development through a decentralized local
government system certainly concerns the matter of financial resources. With
its broadened powers and increased responsibilities, a local government unit
must now operate on a much wider scale. More extensive operations, in turn,
entail more expenses. Understandably, the vesting of duty, responsibility and
accountability in every local government unit is accompanied with a provision
for reasonably adequate resources to discharge its powers and effectively
carry out its functions.7 Availment of such resources is effectuated through the
vesting in every local government unit of (1) the right to create and broaden its
own source of revenue; (2) the right to be allocated a just share in national
taxes, such share being in the form of internal revenue allotments (IRAs); and
(3) the right to be given its equitable share in the proceeds of the utilization
and development of the national wealth, if any, within its territorial
boundaries.8.
The funds generated from local taxes, IRAs and national wealth utilization
proceeds accrue to the general fund of the local government and are used to
finance its operations subject to specified modes of spending the same as
provided for in the Local Government Code and its implementing rules and
regulations. For instance, not less than twenty percent (20%) of the IRAs must
be set aside for local development projects.9 As such, for purposes of budget
preparation, which budget should reflect the estimates of the income of the
local government unit, among others, the IRAs and the share in the national
wealth utilization proceeds are considered items of income. This is as it
should be, since income is defined in the Local Government Code to be all
revenues and receipts collected or received forming the gross accretions of
funds of the local government unit.10
The IRAs are items of income because they form part of the gross
accretion of the funds of the local government unit. The IRAs regularly and
automatically accrue to the local treasury without need of any further action on
the part of the local government unit.11 They thus constitute income which the
local government can invariably rely upon as the source of much needed
funds.
For purposes of converting the Municipality of Santiago into a city, the
Department of Finance certified, among others, that the municipality had an
average annual income of at least Twenty Million Pesos for the last two (2)

consecutive years based on 1991 constant prices. This, the Department of


Finance did after including the IRAs in its computation of said average annual
income.
Furthermore, Section 450 (c) of the Local Government Code provides that
the average annual income shall include the income accruing to the general
fund, exclusive of special funds, transfers, and non-recurring income. To
reiterate, IRAs are a regular, recurring item of income; nil is there a basis, too,
to classify the same as a special fund or transfer, since IRAs have a technical
definition and meaning all its own as used in the Local Government Code that
unequivocally makes it distinct from special funds or transfers referred to
when the Code speaks of funding support from the national government, its
instrumentalities and government-owned-or-controlled corporations.12
Thus, Department of Finance Order No. 359313 correctly encapsulizes the
full import of the above disquisition when it defined ANNUAL INCOME to be
revenues and receipts realized by provinces, cities and municipalities from
regular sources of the Local General Fund including the internal revenue
allotment and other shares provided for in Sections 284, 290 and 291 of the
Code, but exclusive of non-recurring receipts, such as other national aids,
grants, financial assistance, loan proceeds, sales of fixed assets, and similar
others (Italics ours).14 Such order, constituting executive or contemporaneous
construction of a statute by an administrative agency charged with the task of
interpreting and applying the same, is entitled to full respect and should be
accorded great weight by the courts, unless such construction is clearly
shown to be in sharp conflict with the Constitution, the governing statute, or
other laws.15
II
In the enactment of RA No. 7720,
there was compliance with Section 24,
Article VI of the 1987 Constitution
----------------------------------------------------------Although a bill of local application like HB No. 8817 should, by
constitutional prescription,16 originate exclusively in the House of
Representatives, the claim of petitioners that Republic Act No. 7720 did not
originate exclusively in the House of Representatives because a bill of the
same import, SB No. 1243, was passed in the Senate, is untenable because it
cannot be denied that HB No. 8817 was filed in the House of Representatives

first before SB No. 1243 was filed in the Senate. Petitioners themselves
cannot disavow their own admission that HB No. 8817 was filed on April 18,
1993 while SB No. 1243 was filed on May 19, 1993. The filing of HB No. 8817
was thus precursive not only of the said Act in question but also of SB No.
1243. Thus, HB No. 8817, was the bill that initiated the legislative process that
culminated in the enactment of Republic Act No. 7720. No violation of Section
24, Article VI, of the 1987 Constitution is perceptible under the circumstances
attending the instant controversy.
Furthermore, petitioners themselves acknowledge that HB No. 8817 was
already approved on Third Reading and duly transmitted to the Senate when
the Senate Committee on Local Government conducted its public hearing on
HB No. 8817. HB No. 8817 was approved on the Third Reading on December
17, 1993 and transmitted to the Senate on January 28, 1994; a little less than
a month thereafter, or on February 23, 1994, the Senate Committee on Local
Government conducted public hearings on SB No. 1243. Clearly, the Senate
held in abeyance any action on SB No. 1243 until it received HB No. 8817,
already approved on the Third Reading, from the House of Representatives.
The filing in the Senate of a substitute bill in anticipation of its receipt of the bill
from the House, does not contravene the constitutional requirement that a bill
of local application should originate in the House of Representatives, for as
long as the Senate does not act thereupon until it receives the House bill.
We have already addressed this issue in the case of
Tolentino vs. Secretary of Finance.17 There, on the matter of the Expanded
Value Added Tax (EVAT) Law, which, as a revenue bill, is nonetheless
constitutionally required to originate exclusively in the House of
Representatives, we explained:
x x x To begin with, it is not the law-but the revenue bill-which is required by the
Constitution to originate exclusively in the House of Representatives. It is important
to emphasize this, because a bill originating in the House may undergo such extensive
changes in the Senate that the result may be a rewriting of the whole. x x x as a result
of the Senate action, a distinct bill may be produced. To insist that a revenue statuteand not only the bill which initiated the legislative process culminating in the
enactment of the law-must substantially be the same as the House bill would be to
deny the Senates power not only to concur with amendments but also to propose
amendments. It would be to violate the coequality of legislative power of the two
houses of Congress and in fact make the House superior to the Senate.
xxx

xxx

xxx

It is insisted, however, that S. No. 1630 was passed not in substitution of H. No.
11197 but of another Senate bill (S. No. 1129) earlier filed and that what the Senate
did was merely to take [H. No. 11197] into consideration in enacting S. No. 1630.
There is really no difference between the Senate preserving H. No. 11197 up to the
enacting clause and then writing its own version following the enacting clause (which,
it would seem petitioners admit is an amendment by substitution), and, on the other
hand, separately presenting a bill of its own on the same subject matter. In either case
the result are two bills on the same subject.
Indeed, what the Constitution simply means is that the initiative for filing revenue,
tariff, or tax bills, bills authorizing an increase of the public debt, private bills and
bills of local application must come from the House of Representatives on the theory
that, elected as they are from the districts, the members of the House can be expected
to be more sensitive to the local needs and problems. On the other hand, the senators,
who are elected at large, are expected to approach the same problems from the
national perspective. Both views are thereby made to bear on the enactment of such
laws.
Nor does the Constitution prohibit the filing in the Senate of a substitute bill in
anticipation of its receipt of the bill from the House, so long as action by the Senate as
a body is withheld pending receipt of the House bill. x x x18
III
Every law, including RA No. 7720,
has in its favor the presumption
of constitutionality
-------------------------------------------------------------------It is a well-entrenched jurisprudential rule that on the side of every law lies
the presumption of constitutionality.19 Consequently, for RA No. 7720 to be
nullified, it must be shown that there is a clear and unequivocal breach of the
Constitution, not merely a doubtful and equivocal one; in other words, the
grounds for nullity must be clear and beyond reasonable doubt.20Those who
petition this court to declare a law to be unconstitutional must clearly and fully
establish the basis that will justify such a declaration; otherwise, their petition
must fail. Taking into consideration the justification of our stand on the
immediately preceding ground raised by petitioners to challenge the
constitutionality of RA No. 7720, the Court stands on the holding that
petitioners have failed to overcome the presumption. The dismissal of this
petition is, therefore, inevitable.

WHEREFORE, the instant petition is DISMISSED for lack of merit with


costs against petitioners.
SO ORDERED.
Municipal Corporation LGU Requirement Income Inclusion of IRAs
In April 1993, HB 8817 (An Act Converting the Municipality of Santiago into an
Independent Component City to be known as the City of Santiago) was passed in
the HOR.
In May 1993, a Senate bill (SB 1243) of similar title and content with that of HB
8817 was introduced in the Senate.
In January 1994, the HB 8817 was transmitted to the Senate. In February 1994,
the Senate conducted a public hearing on SB 1243. In March 1994, the Senate
Committee on Local Government rolled out its recommendation for approval of
HB 8817 as it was totally the same with SB 1243. Eventually, HB 8817 became a
law (RA 7720).
Now Alvarez et al are assailing the constitutionality of the said law on the ground
that the bill creating the law did not originate from the lower house and that the
Santiago was not able to comply with the income of at least P20M per annum in
order for it to be a city. That in the computation of the reported average income of
P20,974,581.97 included the IRA which should not be.
ISSUES:
1. Whether or not RA 7720 is invalid for not being originally from the HOR.
2. Whether or not the IRA should be included in the computation of an LGUs
income.
HELD: 1. NO. The house bill was filed first before the senate bill as the record
shows. Further, the Senate held in abeyance any hearing on the said SB while
the HB was on its 1 , 2 and 3 reading in the HOR. The Senate only conducted
its 1 hearing on the said SB one month after the HB was transmitted to the
Senate (in anticipation of the said HB as well).
2. YES. The IRA should be added in the computation of an LGUs average
annual income as was done in the case at bar. The IRAs are items of income
because they form part of the gross accretion of the funds of the local
government unit. The IRAs regularly and automatically accrue to the local
treasury without need of any further action on the part of the local government
unit. They thus constitute income which the local government can invariably rely
upon as the source of much needed funds.
To reiterate, IRAs are a regular, recurring item of income; nil is there a basis, too,
to classify the same as a special fund or transfer, since IRAs have a technical
st

st

nd

rd

definition and meaning all its own as used in the Local Government Code that
unequivocally makes it distinct from special funds or transfers referred to when
the Code speaks of funding support from the national government, its
instrumentalities and government-owned-or-controlled corporations.

Republic of the Philippines


SUPREME COURT
Manila
EN BANC

G.R. No. L-33713 July 30, 1975


EUSEBIO B. GARCIA, petitioner-appellant,
vs.
HON. ERNESTO S. MATA, Secretary of National Defense, and GENERAL
MANUEL T. YAN, Chief of Staff, Armed Forces of the
Philippines, respondents-appellees.
Emilio Purugganan for petitioner-appellant.
Office of the Solicitor General Estelito P. Mendoza, Assistant Solicitor General
Rosalio A. de Leon and Solicitor Eulogio Raquel-Santos for respondentsappellees.

CASTRO, J.:
This is a petition for certiorari to review the decision of the Court of First Instance
of Quezon City, Branch IX, in civil case Q-13466, entitled "Eusebio B. Garcia,
petitioner, versus Hon. Ernesto Mata (Juan Ponce Enrile), et al., respondents,"
declaring paragraph 11 of the "Special Provisions for the Armed Forces of the
Philippines" of Republic Act No. 1600 1 unconstitutional and therefore invalid and inoperative.
We affirm the judgment a quo.
The facts material to this case are embodied in the following stipulation submitted
jointly by both parties to the lower court:

Petitioner was a reserve officer on active duty with the Armed Forces
of the Philippines until his reversion to inactive status on 15
November 1960, pursuant to the provisions of Republic Act No.
2332. At the time of reversion, Petitioner held the rank of Captain
with a monthly emolument of P478.00, comprising his base and
longevity pay, quarters and subsistence allowances;
On June 18, 1955, the date when Republic Act No. 1382 took effect,
petitioner had a total of 9 years, 4 months and 12 days of
accumulated active commissioned service in the Armed Forces of
the Philippines;
On July 11, 1956, the date when Republic Act 1600 took effect,
petitioner had an accumulated active commissioned service of 10
years, 5 months and 5 days in the Armed Forces of the Philippines;
Petitioner's reversion to inactive status on 15 November 1960 was
pursuant to the provisions of Republic Act 2334, and such reversion
was neither for cause, at his own request, nor after court-martial
proceedings;
From 15 November 1960 up to the present, petitioner has been on
inactive status and as such, he has neither received any
emoluments from the Armed Forces of the Philippines, nor was he
ever employed in the Government in any capacity;
As a consequence of his reversion to inactive status, petitioner filed
the necessary petitions with the offices of the AFP Chief of Staff, the
Secretary of National Defense, and the President, respectively, but
received reply only from the Chief of Staff through the AFP Adjutant
General.
On September 17, 1969 the petitioner brought an action for "Mandamus and
Recovery of a Sum of Money" in the court a quo to compel the respondents
Secretary of National Defense and Chief of Staff of the Armed Forces of the
Philippines 2 to reinstate him in the active commissioned service of the Armed Forces of the
Philippines, to readjust his rank, and to pay all the emoluments and allowances due to him from the time
of his reversion to inactive status. On December 2, 1970 the trial court dismissed the petition. The court
ruled that paragraph 11 of the "Special Provisions for the Armed Forces of the Philippines" in Republic
Act 1600 is "invalid, unconstitutional and inoperative."

The petitioner had a total of 9 years, 4 months and 12 days of accumulated


active commissioned service in the AFP when Republic Act 1382 took effect on
June 18, 1955. Section I of this law provided:

Reserve officers with at least ten years of active accumulated


commissioned service who are still on active duty at the time of the
approval of this Act shall not be reverted into inactive status except
for cause after proper court-martial proceedings or upon their own
request: Provided, That for purposes of computing the length of
service, six months or more of active service shall be considered
one year. (emphasis supplied)
The petitioner's accumulated active commissioned service was thus short of the
minimum service requirement prescribed in the aforequoted provision of R.A.
1382.
On July 11, 1956, 3 while the petitioner was yet in the active service, Republic Act 1600 was enacted
into law. Paragraph 11 of the SPECIAL PROVISIONS FOR THE ARMED FORCES OF THE
PHILIPPINES (on page 892 of the Act) provided as follows:

11. After the approval of this Act, and when there is no emergency,
no reserve officer of the Armed Forces of the Philippines may be
called to a tour of active duty for more than two years during any
period of five consecutive years: PROVIDED, That hereafter reserve
officers of the Armed Forces of the Philippines on active duty for
more than two years on the date of the approval of this Act except
those whose military and educational training, experience and
qualifications are deemed essential to the needs of the service, shall
be reverted to inactive status within one year from the approval of
this Act: PROVIDED, FURTHER, That reserve officers with at least
ten years of active accumulated commissioned service who are still
on active duty at the time of the approval of this Act shall not be
reverted to inactive status except for cause after proper court-martial
proceedings or upon their request; PROVIDED, FURTHER, That
any such reserve officer reverted to inactive status who has at least
five of active commissioned service shall be entitled to a gratuity
equivalent to one month's authorized base and longevity pay in the
rank held at the time of such reversion for every year of active
commissioned service; PROVIDED, FURTHER, That any reserve
officer who receives a gratuity under the provisions of this Act shall
not except during a National emergency or mobilization, be called to
a tour of active duty within five years from the date of reversion:
PROVIDED, FURTHER, That the Secretary of National Defense is
authorized to extend the tour of active duty of reserve officers who
are qualified military pilots and doctors; PROVIDED, FURTHER,
That any savings in the appropriations authorized in this Act for the
Department of National Defense notwithstanding any provision of
this Act to the contrary and any unexpended balance of certification

to accounts payable since 1 July 1949 regardless of purpose of the


appropriation shall be made available for the purpose of this
paragraph: AND PROVIDED, FINALLY, That the Secretary of
National Defense shall render a quarterly report to Congress as to
the implementation of the provisions of this paragraph. ( pp. 892893, RA 1600) (emphasis supplied)
The petitioner consequently argues that his reversion to inactive status on
November 15, 1960 was in violation of the abovequoted provision which prohibits
the reversion to inactive status of reserve officers on active duty with at least ten
years of accumulated active commissioned service.
On the other hand, the respondents contend that the said provision has no
relevance or pertinence whatsoever to the budget in question or to any
appropriation item contained therein, and is therefore proscribed by Art. VI, Sec.
19, par. 2 4 of the 1935 Constitution of the Philippines, which reads:
No provision or enactment shall be embraced in the general
appropriation bill unless it relates specifically to some particular
appropriation therein; and any such provision or enactment shall be
limited in its operation to such appropriation.
A perusal of the challenged provision of R.A. 1600 fails to disclose its relevance
or relation to any appropriation item therein, or to the Appropriation Act as a
whole. From the very first clause of paragraph 11 itself, which reads,
After the approval of this Act, and when there is no emergency, no
reserve officer of the Armed Forces of the Philippines may be called
to a tour of active duty for more than two years during any period of
five consecutive years:
the incongruity and irrelevancy are already evident. While R.A. 1600
appropriated money for the operation of the Government for the fiscal year 19561957, the said paragraph 11 refers to the fundamental government policy matters
of the calling to active duty and the reversion to inactive status of reserve officers
in the AFP. The incongruity and irrelevancy continue throughout the entire
paragraph.
In the language of the respondents-appellees, "it was indeed a non-appropriation
item inserted in an appropriation measure in violation of the constitutional
inhibition against "riders" to the general appropriation act." It was indeed a new
and completely unrelated provision attached to the Appropriation Act.

The paragraph in question also violated Art. VI, Sec. 21, par. 1 5 of the 1935
Constitution of the Philippines which provided that "No bill which may be enacted into law shall embrace
more than one subject which shall be expressed in the title of the bill." This constitutional requirement
nullified and rendered inoperative any provision contained in the body of an act that was not fairly
included in the subject expressed in the title or was not germane to or properly connected with that
subject.

In determining whether a provision contained in an act is embraced in the subject


and is properly connected therewith, the subject to be considered is the one
expressed in the title of the act, and every fair intendment and reasonable doubt
should be indulged in favor of the validity of the legislative enactment. But when
an act contains provisions which are clearly not embraced in the subject of the
act, as expressed in the title, such provisions are inoperative and without effect.
We are mindful that the title of an act is not required to be an index to the body of
the act. Thus, in Sumulong vs. Comelec, 73 Phil. 288, 291, this Court held that it
is "a sufficient compliance with such requirement if the title expresses the general
subject and all the provisions of the statute are germane to that general subject."
The constitutional provision was intended to preclude the insertion of riders in
legislation, a rider being a provision not germane to the subject-matter of the bill. 6
The subject of R.A. 1600, as expressed in its title, is restricted to "appropriating
funds for the operation of the government." Any provision contained in the body
of the act that is fairly included in this restricted subject or any matter properly
connected therewith is valid and operative. But, if a provision in the body of the
act is not fairly included in this restricted subject, like the provision relating to the
policy matters of calling to active duty and reversion to inactive duty of reserve
officers of the AFP, such provision is inoperative and of no effect.
To quote the respondents-appellees on this point:
It is obvious that the statutory provision in question refers to security
of reserve officers from reversion to inactive status, whereas the
subject or title of the statute from which it derives its existence refers
to appropriations. Verily, it runs contrary to or is repugnant to the
above-quoted injunctive provision of the Constitution. Where a
conflict arises between a statute and the Constitution, the latter
prevails. It should be emphasized that a Constitution is superior to a
statute and is precisely called the "supreme law of the land" because
it is the fundamental or organic law which states the general
principles and builds the substantial foundation and general
framework of law and government, and for that reason a statute
contrary to or in violation of the Constitution is null and void (Talabon
vs. Iloilo Provincial Warden, 78 Phil. 599). If a law, therefore,
1w ph1.t

happens to infringe upon or violate the fundamental law, courts of


justice may step in to nullify its effectiveness (Mabanag vs. Lopez
Vito, 78 Phil. 1).
Upon the foregoing dissertation, we declare Paragraph 11 of the SPECIAL
PROVISIONS FOR THE ARMED FORCES OF THE PHILIPPINES as
unconstitutional, invalid and inoperative. Being unconstitutional, it confers no
right and affords no protection. In legal contemplation it is as though it has never
been passed. 7
Verily, not having shown a clear legal right to the position to which he desires to
be restored, the petitioner cannot compel the respondents to reinstate and/or call
him to active duty, promote or readjust his rank, much less pay him back
emoluments and allowances.
ACCORDINGLY, the instant petition is denied, and the decision of the lower
court dismissing the complaint is hereby affirmed. No pronouncement as to
costs.

Facts: Garcia was a reserve officer on active duty who


was reversed to inactive status. He filed an action for
mandamus to compel the DND and AFP to reinstate him
to active service and readjust his rank and pay
emoluments.
Garcia claims that his reversion to inactive status is
violation of RA 1600 which prohibits the reversion of
officers with at least 10 years of service.
On the other hand, the AFP and DND contend that the
said provision of RA 1600 has no relevance or pertinence
to the budget in question or to any appropriation item
therein. (RA 1600 was an appropriation law for 1956-57).
Issue: Whether RA 1600 is valid? Does it contain rider in
an appropriation bill?

Held: The incongruity and irrelevancy are already evident.


Section 11 of RA 1600 fails to disclose the relevance to
any appropriation item. RA 1600 is an appropriation law
for the operation of government while Section 11 refers to
a fundamental governmental policy of calling to active duty
and the reversion of inactive statute of reserve officers in
the AFP.
Hence it was A NON-APPROPRIATION ITEM INSERTED
IN AN APPROPRIATION MEASURE, in violation of the
constitutional prohibition against RIDERS to the general
appropriation act. It was indeed a new and completely
unrelated provision attached to the GAA.
It also violates the rule on one-bill, one subject. The
subject to be considered must be expressed in the title of
the act. When an act contains provisions which are clearly
not embraced in the subject of the act, as expressed in the
title, such provisions are void, inoperative and without
effect.
SECTION 11 is unconstitutional. Garcia cannot compel
the AFP to reinstate him.
Republic of the Philippines
SUPREME COURT
Manila
EN BANC
G.R. No. 71977 February 27, 1987

DEMETRIO G. DEMETRIA, M.P., AUGUSTO S. SANCHEZ, M.P., ORLANDO


S. MERCADO, M.P., HONORATO Y. AQUINO, M.P., ZAFIRO L. RESPICIO,
M.P., DOUGLAS R. CAGAS, M.P., OSCAR F. SANTOS, M.P., ALBERTO G.
ROMULO, M.P., CIRIACO R. ALFELOR, M.P., ISIDORO E. REAL, M.P.,
EMIGDIO L. LINGAD, M.P., ROLANDO C. MARCIAL, M.P., PEDRO M.
MARCELLANA, M.P., VICTOR S. ZIGA, M.P., and ROGELIO V. GARCIA.
M.P., petitioners,
vs.
HON. MANUEL ALBA in his capacity as the MINISTER OF THE BUDGET
and VICTOR MACALINGCAG in his capacity as the TREASURER OF THE
PHILIPPINES, respondents.

FERNAN, J.:
Assailed in this petition for prohibition with prayer for a writ of preliminary
injunction is the constitutionality of the first paragraph of Section 44 of
Presidential Decree No. 1177, otherwise known as the "Budget Reform Decree
of 1977."
Petitioners, who filed the instant petition as concerned citizens of this country, as
members of the National Assembly/Batasan Pambansa representing their
millions of constituents, as parties with general interest common to all the people
of the Philippines, and as taxpayers whose vital interests may be affected by the
outcome of the reliefs prayed for" 1 listed the grounds relied upon in this petition as follows:
A. SECTION 44 OF THE 'BUDGET REFORM DECREE OF 1977'
INFRINGES UPON THE FUNDAMENTAL LAW BY AUTHORIZING
THE ILLEGAL TRANSFER OF PUBLIC MONEYS.
B. SECTION 44 OF PRESIDENTIAL DECREE NO. 1177 IS
REPUGNANT TO THE CONSTITUTION AS IT FAILS TO SPECIFY
THE OBJECTIVES AND PURPOSES FOR WHICH THE
PROPOSED TRANSFER OF FUNDS ARE TO BE MADE.
C. SECTION 44 OF PRESIDENTIAL DECREE NO. 1177 ALLOWS
THE PRESIDENT TO OVERRIDE THE SAFEGUARDS, FORM
AND PROCEDURE PRESCRIBED BY THE CONSTITUTION IN
APPROVING APPROPRIATIONS.

D. SECTION 44 OF THE SAME DECREE AMOUNTS TO AN


UNDUE DELEGATION OF LEGISLATIVE POWERS TO THE
EXECUTIVE.
E. THE THREATENED AND CONTINUING TRANSFER OF FUNDS
BY THE PRESIDENT AND THE IMPLEMENTATION THEREOF BY
THE BUDGET MINISTER AND THE TREASURER OF THE
PHILIPPINES ARE WITHOUT OR IN EXCESS OF THEIR
AUTHORITY AND JURISDICTION. 2
Commenting on the petition in compliance with the Court resolution dated
September 19, 1985, the Solicitor General, for the public respondents,
questioned the legal standing of petitioners, who were allegedly merely begging
an advisory opinion from the Court, there being no justiciable controversy fit for
resolution or determination. He further contended that the provision under
consideration was enacted pursuant to Section 16[5], Article VIII of the 1973
Constitution; and that at any rate, prohibition will not lie from one branch of the
government to a coordinate branch to enjoin the performance of duties within the
latter's sphere of responsibility.
On February 27, 1986, the Court required the petitioners to file a Reply to the
Comment. This, they did, stating, among others, that as a result of the change in
the administration, there is a need to hold the resolution of the present case in
abeyance "until developments arise to enable the parties to concretize their
respective stands." 3
Thereafter, We required public respondents to file a rejoinder. The Solicitor
General filed a rejoinder with a motion to dismiss, setting forth as grounds
therefor the abrogation of Section 16[5], Article VIII of the 1973 Constitution by
the Freedom Constitution of March 25, 1986, which has allegedly rendered the
instant petition moot and academic. He likewise cited the "seven pillars"
enunciated by Justice Brandeis in Ashwander v. TVA, 297 U.S. 288 (1936) 4 as
basis for the petition's dismissal.

In the case of Evelio B. Javier v. The Commission on Elections and Arturo F.


Pacificador, G.R. Nos. 68379-81, September 22, 1986, We stated that:
The abolition of the Batasang Pambansa and the disappearance of
the office in dispute between the petitioner and the private
respondents both of whom have gone their separate ways
could be a convenient justification for dismissing the case. But there
are larger issues involved that must be resolved now, once and for
all, not only to dispel the legal ambiguities here raised. The more

important purpose is to manifest in the clearest possible terms that


this Court will not disregard and in effect condone wrong on the
simplistic and tolerant pretext that the case has become moot and
academic.
The Supreme Court is not only the highest arbiter of legal questions
but also the conscience of the government. The citizen comes to us
in quest of law but we must also give him justice. The two are not
always the same. There are times when we cannot grant the latter
because the issue has been settled and decision is no longer
possible according to the law. But there are also times when
although the dispute has disappeared, as in this case, it
nevertheless cries out to be resolved. Justice demands that we act
then, not only for the vindication of the outraged right, though gone,
but also for the guidance of and as a restraint upon the future.
It is in the discharge of our role in society, as above-quoted, as well as to avoid
great disservice to national interest that We take cognizance of this petition and
thus deny public respondents' motion to dismiss. Likewise noteworthy is the fact
that the new Constitution, ratified by the Filipino people in the plebiscite held on
February 2, 1987, carries verbatim section 16[5], Article VIII of the 1973
Constitution under Section 24[5], Article VI. And while Congress has not officially
reconvened, We see no cogent reason for further delaying the resolution of the
case at bar.
The exception taken to petitioners' legal standing deserves scant consideration.
The case of Pascual v. Secretary of Public Works, et al., 110 Phil. 331, is
authority in support of petitioners' locus standi. Thus:
Again, it is well-settled that the validity of a statute may be contested
only by one who will sustain a direct injury in consequence of its
enforcement. Yet, there are many decisions nullifying at the instance
of taxpayers, laws providing for the disbursement of public funds,
upon the theory that the expenditure of public funds by an officer of
the state for the purpose of administering an unconstitutional
actconstitutes a misapplication of such funds which may be enjoined
at the request of a taxpayer. Although there are some decisions to
the contrary, the prevailing view in the United States is stated in the
American Jurisprudence as follows:
In the determination of the degree of interest essential
to give the requisite standing to attack the
constitutionality of a statute, the general rule is that not

only persons individually affected, but also taxpayers


have sufficient interest in preventing the illegal
expenditures of moneys raised by taxation and may
therefore question the constitutionality of statutes
requiring expenditure of public moneys. [ 11 Am. Jur.
761, Emphasis supplied. ]
Moreover, in Tan v. Macapagal, 43 SCRA 677 and Sanidad v. Comelec, 73
SCRA 333, We said that as regards taxpayers' suits, this Court enjoys that open
discretion to entertain the same or not.
The conflict between paragraph 1 of Section 44 of Presidential Decree No. 1177
and Section 16[5], Article VIII of the 1973 Constitution is readily perceivable from
a mere cursory reading thereof. Said paragraph 1 of Section 44 provides:
The President shall have the authority to transfer any fund,
appropriated for the different departments, bureaus, offices and
agencies of the Executive Department, which are included in the
General Appropriations Act, to any program, project or activity of any
department, bureau, or office included in the General Appropriations
Act or approved after its enactment.
On the other hand, the constitutional provision under consideration reads as
follows:
Sec. 16[5]. No law shall be passed authorizing any transfer of
appropriations, however, the President, the Prime Minister, the
Speaker, the Chief Justice of the Supreme Court, and the heads of
constitutional commis ions may by law be authorized to augment any
item in the general appropriations law for their respective offices
from savings in other items of their respective appropriations.
The prohibition to transfer an appropriation for one item to another was explicit
and categorical under the 1973 Constitution. However, to afford the heads of the
different branches of the government and those of the constitutional commissions
considerable flexibility in the use of public funds and resources, the constitution
allowed the enactment of a law authorizing the transfer of funds for the purpose
of augmenting an item from savings in another item in the appropriation of the
government branch or constitutional body concerned. The leeway granted was
thus limited. The purpose and conditions for which funds may be transferred
were specified, i.e. transfer may be allowed for the purpose of augmenting an
item and such transfer may be made only if there are savings from another item
in the appropriation of the government branch or constitutional body.

Paragraph 1 of Section 44 of P.D. No. 1177 unduly over extends the privilege
granted under said Section 16[5]. It empowers the President to indiscriminately
transfer funds from one department, bureau, office or agency of the Executive
Department to any program, project or activity of any department, bureau or
office included in the General Appropriations Act or approved after its enactment,
without regard as to whether or not the funds to be transferred are actually
savings in the item from which the same are to be taken, or whether or not the
transfer is for the purpose of augmenting the item to which said transfer is to be
made. It does not only completely disregard the standards set in the fundamental
law, thereby amounting to an undue delegation of legislative powers, but likewise
goes beyond the tenor thereof. Indeed, such constitutional infirmities render the
provision in question null and void.
"For the love of money is the root of all evil: ..." and money belonging to no one in
particular, i.e. public funds, provide an even greater temptation for
misappropriation and embezzlement. This, evidently, was foremost in the minds
of the framers of the constitution in meticulously prescribing the rules regarding
the appropriation and disposition of public funds as embodied in Sections 16 and
18 of Article VIII of the 1973 Constitution. Hence, the conditions on the release of
money from the treasury [Sec. 18(1)]; the restrictions on the use of public funds
for public purpose [Sec. 18(2)]; the prohibition to transfer an appropriation for an
item to another [See. 16(5) and the requirement of specifications [Sec. 16(2)],
among others, were all safeguards designed to forestall abuses in the
expenditure of public funds. Paragraph 1 of Section 44 puts all these safeguards
to naught. For, as correctly observed by petitioners, in view of the unlimited
authority bestowed upon the President, "... Pres. Decree No. 1177 opens the
floodgates for the enactment of unfunded appropriations, results in uncontrolled
executive expenditures, diffuses accountability for budgetary performance and
entrenches the pork barrel system as the ruling party may well expand [sic]
public money not on the basis of development priorities but on political and
personal expediency."5 The contention of public respondents that paragraph 1 of Section 44 of
P.D. 1177 was enacted pursuant to Section 16(5) of Article VIII of the 1973 Constitution must perforce fall
flat on its face.

Another theory advanced by public respondents is that prohibition will not lie from
one branch of the government against a coordinate branch to enjoin the
performance of duties within the latter's sphere of responsibility.
Thomas M. Cooley in his "A Treatise on the Constitutional Limitations," Vol. 1,
Eight Edition, Little, Brown and Company, Boston, explained:
... The legislative and judicial are coordinate departments of the
government, of equal dignity; each is alike supreme in the exercise
of its proper functions, and cannot directly or indirectly, while acting

within the limits of its authority, be subjected to the control or


supervision of the other, without an unwarrantable assumption by
that other of power which, by the Constitution, is not conferred upon
it. The Constitution apportions the powers of government, but it does
not make any one of the three departments subordinate to another,
when exercising the trust committed to it. The courts may declare
legislative enactments unconstitutional and void in some cases, but
not because the judicial power is superior in degree or dignity to the
legislative. Being required to declare what the law is in the cases
which come before them, they must enforce the Constitution, as the
paramount law, whenever a legislative enactment comes in conflict
with it. But the courts sit, not to review or revise the legislative action,
but to enforce the legislative will, and it is only where they find that
the legislature has failed to keep within its constitutional limits, that
they are at liberty to disregard its action; and in doing so, they only
do what every private citizen may do in respect to the mandates of
the courts when the judges assumed to act and to render judgments
or decrees without jurisdiction. "In exercising this high authority, the
judges claim no judicial supremacy; they are only the administrators
of the public will. If an act of the legislature is held void, it is not
because the judges have any control over the legislative power, but
because the act is forbidden by the Constitution, and because the
will of the people, which is therein declared, is paramount to that of
their representatives expressed in any law." [Lindsay v.
Commissioners, & c., 2 Bay, 38, 61; People v. Rucker, 5 Col. 5;
Russ v. Com., 210 Pa. St. 544; 60 Atl. 169, 1 L.R.A. [N.S.] 409, 105
Am. St. Rep. 825] (pp. 332-334).
Indeed, where the legislature or the executive branch is acting within the limits of
its authority, the judiciary cannot and ought not to interfere with the former. But
where the legislature or the executive acts beyond the scope of its constitutional
powers, it becomes the duty of the judiciary to declare what the other branches of
the government had assumed to do as void. This is the essence of judicial power
conferred by the Constitution "in one Supreme Court and in such lower courts as
may be established by law" [Art. VIII, Section 1 of the 1935 Constitution; Art. X,
Section 1 of the 1973 Constitution and which was adopted as part of the
Freedom Constitution, and Art. VIII, Section 1 of the 1987 Constitution] and
which power this Court has exercised in many instances. *
Public respondents are being enjoined from acting under a provision of law which
We have earlier mentioned to be constitutionally infirm. The general principle
relied upon cannot therefore accord them the protection sought as they are not
acting within their "sphere of responsibility" but without it.

The nation has not recovered from the shock, and worst, the economic
destitution brought about by the plundering of the Treasury by the deposed
dictator and his cohorts. A provision which allows even the slightest possibility of
a repetition of this sad experience cannot remain written in our statute books.
WHEREFORE, the instant petition is granted. Paragraph 1 of Section 44 of
Presidential Decree No. 1177 is hereby declared null and void for being
unconstitutional.
SO ORDER RED.
Facts:
Petitioners assail the constitutionality of first paragraph of Sec 44 of PD 1177
(Budget Reform Decree of 1977)as concerned citizens, members of the National
Assembly, parties with general interest common to all people of the Philippines,
and as taxpayerson the primary grounds that Section 44 infringes upon the
fundamental law by authorizing illegal transfer of public moneys, amounting to
undue delegation of legislative powers and allowing the President to override the
safeguards prescribed for approving appropriations.
The Solicitor General, for the public respondents, questioned the legal standing
of the petitioners and held that one branch of the government cannot be enjoined
by another, coordinate branch in its performance of duties within its sphere of
responsibility. It also alleged that the petition has become moot and academic
after the abrogation of Sec 16(5), Article VIII of the 1973 Constitution by the
Freedom Constitution (which was where the provision under consideration was
enacted in pursuant thereof), which states that No law shall be passed
authorizing any transfer of appropriations, however, the Presidentmay by law
be authorized to augment any item in the general appropriations law for their
respective offices from savings in other items of their respective appropriations.
Issue:
1. W/N PD 1177 is constitutional
2. W/N the Supreme Court can act upon the assailed executive act

Held:
1. No. Sec 44 of PD 1177 unduly overextends the privilege granted under Sec16(5)
by empowering the President to indiscriminately transfer funds from one
department of the Executive Department to any program of any department
included in the General Appropriations Act, without any regard as to whether or
not the funds to be transferred are actually savings in the item. It not only
disregards the standards set in the fundamental law, thereby amounting to an
undue delegation of legislative powers, but likewise goes beyond the tenor
thereof.
Par. 1 of Sec. 44 puts all safeguards to forestall abuses in the expenditure of
public funds to naught. Such constitutional infirmities render the provision in
question null and void.
2. Yes. Where the legislature or executive acts beyond the scope of its
constitutional powers, it becomes the duty of the judiciary to declare what the
other branches of the government has assumed to do as void, as part of its
constitutionally conferred judicial power. This is not to say that the judicial power
is superior in degree or dignity. In exercising this high authority, the judges claim
no judicial supremacy; they are only the administrators of the public will.
Petition granted. Par. 1, Sec. 44 OF PD 1177 null and void.

Republic of the Philippines


SUPREME COURT
Manila
EN BANC

G.R. No. 113105 August 19, 1994


PHILIPPINE CONSTITUTION ASSOCIATION, EXEQUIEL B. GARCIA and A.
GONZALES, petitioners,
vs.
HON. SALVADOR ENRIQUEZ, as Secretary of Budget and Management;
HON. VICENTE T. TAN, as National Treasurer and COMMISSION ON
AUDIT, respondents.

G.R. No. 113174 August 19, 1994


RAUL S. ROCO, as Member of the Philippine Senate, NEPTALI A.
GONZALES, Chairman of the Committee on Finance of the Philippine
Senate, and EDGARDO J. ANGARA, as President and Chief Executive of
the Philippine Senate, all of whom also sue as taxpayers, in their own
behalf and in representation of Senators HEHERSON ALVAREZ, AGAPITO
A. AQUINO, RODOLFO G. BIAZON, JOSE D. LINA, JR., ERNESTO F.
HERRERA, BLAS F. OPLE, JOHN H. OSMENA, GLORIA MACAPAGALARROYO, VICENTE C. SOTTO III, ARTURO M. TOLENTINO, FRANCISCO S.
TATAD, WIGBERTO E. TAADA and FREDDIE N. WEBB,petitioners,
vs.
THE EXECUTIVE SECRETARY, THE DEPARTMENT OF BUDGET AND
MANAGEMENT, and THE NATIONAL TREASURER, THE COMMISSION ON
AUDIT, impleaded herein as an unwilling
co-petitioner, respondents.
G.R. No. 113766 August 19, 1994
WIGBERTO E. TAADA and ALBERTO G. ROMULO, as Members of the
Senate and as taxpayers, and FREEDOM FROM DEBT
COALITION, petitioners,
vs.
HON. TEOFISTO T. GUINGONA, JR. in his capacity as Executive Secretary,
HON. SALVADOR ENRIQUEZ, JR., in his capacity as Secretary of the
Department of Budget and Management, HON. CARIDAD VALDEHUESA, in
her capacity as National Treasurer, and THE COMMISSION ON
AUDIT, respondents.
G.R. No. 113888 August 19, 1994
WIGBERTO E. TAADA and ALBERTO G. ROMULO, as Members of the
Senate and as taxpayers, petitioners,
vs.
HON. TEOFISTO T. GUINGONA, JR., in his capacity as Executive Secretary,
HON. SALVADOR ENRIQUEZ, JR., in his capacity as Secretary of the
Department of Budget and Management, HON. CARIDAD VALDEHUESA, in
her capacity as National Treasurer, and THE COMMISSION ON
AUDIT, respondents.
Ramon R. Gonzales for petitioners in G.R. No. 113105.
Eddie Tamondong for petitioners in G.R. Nos. 113766 & 113888.

Roco, Buag, Kapunan, Migallos & Jardeleza for petitioners Raul S. Roco,
Neptali A. Gonzales and Edgardo Angara.
Ceferino Padua Law Office fro intervenor Lawyers Against Monopoly and Poverty
(Lamp).

QUIASON, J.:
Once again this Court is called upon to rule on the conflicting claims of authority
between the Legislative and the Executive in the clash of the powers of the purse
and the sword. Providing the focus for the contest between the President and the
Congress over control of the national budget are the four cases at bench. Judicial
intervention is being sought by a group of concerned taxpayers on the claim that
Congress and the President have impermissibly exceeded their respective
authorities, and by several Senators on the claim that the President has
committed grave abuse of discretion or acted without jurisdiction in the exercise
of his veto power.
I
House Bill No. 10900, the General Appropriation Bill of 1994 (GAB of 1994), was
passed and approved by both houses of Congress on December 17, 1993. As
passed, it imposed conditions and limitations on certain items of appropriations in
the proposed budget previously submitted by the President. It also authorized
members of Congress to propose and identify projects in the "pork barrels"
allotted to them and to realign their respective operating budgets.
Pursuant to the procedure on the passage and enactment of bills as prescribed
by the Constitution, Congress presented the said bill to the President for
consideration and approval.
On December 30, 1993, the President signed the bill into law, and declared the
same to have become Republic Act No. 7663, entitled "AN ACT
APPROPRIATING FUNDS FOR THE OPERATION OF THE GOVERNMENT OF
THE PHILIPPINES FROM JANUARY ONE TO DECEMBER THIRTY ONE,
NINETEEN HUNDRED AND NINETY-FOUR, AND FOR OTHER PURPOSES"
(GAA of 1994). On the same day, the President delivered his Presidential Veto
Message, specifying the provisions of the bill he vetoed and on which he
imposed certain conditions.
No step was taken in either House of Congress to override the vetoes.

In G.R. No. 113105, the Philippine Constitution Association, Exequiel B. Garcia


and Ramon A. Gonzales as taxpayers, prayed for a writ of prohibition to declare
as unconstitutional and void: (a) Article XLI on the Countrywide Development
Fund, the special provision in Article I entitled Realignment of Allocation for
Operational Expenses, and Article XLVIII on the Appropriation for Debt Service
or the amount appropriated under said Article XLVIII in excess of the P37.9
Billion allocated for the Department of Education, Culture and Sports; and (b) the
veto of the President of the Special Provision of
Article XLVIII of the GAA of 1994 (Rollo, pp. 88-90, 104-105)
In G.R. No. 113174, sixteen members of the Senate led by Senate President
Edgardo J. Angara, Senator Neptali A. Gonzales, the Chairman of the Committee
on Finance, and Senator Raul S. Roco, sought the issuance of the writs of
certiorari, prohibition and mandamus against the Executive Secretary, the
Secretary of the Department of Budget and Management, and the National
Treasurer.
Suing as members of the Senate and taxpayers, petitioners question: (1) the
constitutionality of the conditions imposed by the President in the items of the
GAA of 1994: (a) for the Supreme Court, (b) Commission on Audit (COA), (c)
Ombudsman, (d) Commission on Human Rights (CHR), (e) Citizen Armed
Forces Geographical Units (CAFGU'S) and (f) State Universities and Colleges
(SUC's); and (2) the constitutionality of the veto of the special provision in the
appropriation for debt service.
In G.R. No. 113766, Senators Alberto G. Romulo and Wigberto Taada (a copetitioner in G.R. No. 113174), together with the Freedom from Debt Coalition, a
non-stock domestic corporation, sought the issuance of the writs of prohibition
and mandamus against the Executive Secretary, the Secretary of the
Department of Budget and Management, the National Treasurer, and the COA.
Petitioners Taada and Romulo sued as members of the Philippine Senate and
taxpayers, while petitioner Freedom from Debt Coalition sued as a taxpayer.
They challenge the constitutionality of the Presidential veto of the special
provision in the appropriations for debt service and the automatic appropriation of
funds therefor.
In G.R. No. 11388, Senators Taada and Romulo sought the issuance of the
writs of prohibition and mandamus against the same respondents in G.R. No.
113766. In this petition, petitioners contest the constitutionality of: (1) the veto on
four special provision added to items in the GAA of 1994 for the Armed Forces of
the Philippines (AFP) and the Department of Public Works and Highways
(DPWH); and (2) the conditions imposed by the President in the implementation

of certain appropriations for the CAFGU's, the DPWH, and the National Housing
Authority (NHA).
Petitioners also sought the issuance of temporary restraining orders to enjoin
respondents Secretary of Budget and Management, National Treasurer and COA
from enforcing the questioned provisions of the GAA of 1994, but the Court
declined to grant said provisional reliefs on the time- honored principle of
according the presumption of validity to statutes and the presumption of
regularity to official acts.
In view of the importance and novelty of most of the issues raised in the four
petitions, the Court invited former Chief Justice Enrique M. Fernando and former
Associate Justice Irene Cortes to submit their respective memoranda asAmicus
curiae, which they graciously did.
II
Locus Standi
When issues of constitutionality are raised, the Court can exercise its power of
judicial review only if the following requisites are compresent: (1) the existence of
an actual and appropriate case; (2) a personal and substantial interest of the
party raising the constitutional question; (3) the exercise of judicial review is
pleaded at the earliest opportunity; and (4) the constitutional question is the lis
mota of the case (Luz Farms v. Secretary of the Department of Agrarian Reform,
192 SCRA 51 [1990]; Dumlao v. Commission on Elections, 95 SCRA 392 [1980];
People v. Vera, 65 Phil. 56 [1937]).
While the Solicitor General did not question the locus standi of petitioners in G.R.
No. 113105, he claimed that the remedy of the Senators in the other petitions is
political (i.e., to override the vetoes) in effect saying that they do not have the
requisite legal standing to bring the suits.
The legal standing of the Senate, as an institution, was recognized in Gonzales
v. Macaraig, Jr., 191 SCRA 452 (1990). In said case, 23 Senators, comprising
the entire membership of the Upper House of Congress, filed a petition to nullify
the presidential veto of Section 55 of the GAA of 1989. The filing of the suit was
authorized by Senate Resolution No. 381, adopted on February 2, 1989, and
which reads as follows:
Authorizing and Directing the Committee on Finance to Bring in the
Name of the Senate of the Philippines the Proper Suit with the
Supreme Court of the Philippines contesting the Constitutionality of

the Veto by the President of Special and General Provisions,


particularly Section 55, of the General Appropriation Bill of 1989
(H.B. No. 19186) and For Other Purposes.
In the United States, the legal standing of a House of Congress to sue has been
recognized (United States v. American Tel. & Tel. Co., 551 F. 2d 384, 391
[1976]; Notes: Congressional Access To The Federal Courts, 90 Harvard Law
Review 1632 [1977]).
While the petition in G.R. No. 113174 was filed by 16 Senators, including the
Senate President and the Chairman of the Committee on Finance, the suit was
not authorized by the Senate itself. Likewise, the petitions in
G.R. Nos. 113766 and 113888 were filed without an enabling resolution for the
purpose.
Therefore, the question of the legal standing of petitioners in the three cases
becomes a preliminary issue before this Court can inquire into the validity of the
presidential veto and the conditions for the implementation of some items in the
GAA of 1994.
We rule that a member of the Senate, and of the House of Representatives for
that matter, has the legal standing to question the validity of a presidential veto or
a condition imposed on an item in an appropriation bill.
Where the veto is claimed to have been made without or in excess of the
authority vested on the President by the Constitution, the issue of an
impermissible intrusion of the Executive into the domain of the Legislature arises
(Notes: Congressional Standing To Challenge Executive Action, 122 University
of Pennsylvania Law Review 1366 [1974]).
To the extent the power of Congress are impaired, so is the power of each
member thereof, since his office confers a right to participate in the exercise of
the powers of that institution (Coleman v. Miller, 307 U.S. 433 [1939]; Holtzman
v. Schlesinger, 484 F. 2d 1307 [1973]).
An act of the Executive which injures the institution of Congress causes a
derivative but nonetheless substantial injury, which can be questioned by a
member of Congress (Kennedy v. Jones, 412 F. Supp. 353 [1976]). In such a
case, any member of Congress can have a resort to the courts.
Former Chief Justice Enrique M. Fernando, as Amicus Curiae, noted:

This is, then, the clearest case of the Senate as a whole or individual
Senators as such having a substantial interest in the question at
issue. It could likewise be said that there was the requisite injury to
their rights as Senators. It would then be futile to raise any locus
standi issue. Any intrusion into the domain appertaining to the
Senate is to be resisted. Similarly, if the situation were reversed, and
it is the Executive Branch that could allege a transgression, its
officials could likewise file the corresponding action. What cannot be
denied is that a Senator has standing to maintain inviolate the
prerogatives, powers and privileges vested by the Constitution in his
office (Memorandum, p. 14).
It is true that the Constitution provides a mechanism for overriding a veto (Art. VI,
Sec. 27 [1]). Said remedy, however, is available only when the presidential veto
is based on policy or political considerations but not when the veto is claimed to
be ultra vires. In the latter case, it becomes the duty of the Court to draw the
dividing line where the exercise of executive power ends and the bounds of
legislative jurisdiction begin.
III
G.R. No. 113105
1. Countrywide Development Fund
Article XLI of the GAA of 1994 sets up a Countrywide Development Fund of
P2,977,000,000.00 to "be used for infrastructure, purchase of ambulances and
computers and other priority projects and activities and credit facilities to qualified
beneficiaries." Said Article provides:
COUNTRYWIDE DEVELOPMENT FUND
For Fund requirements of countrywide
development projects P 2,977,000,000

New Appropriations, by Purpose


Current Operating Expenditures
A. PURPOSE
Personal Maintenance Capital Total
Services and Other Outlays

Operating
Expenses
1. For Countrywide
Developments Projects P250,000,000 P2,727,000,000
P2,977,000,000
TOTAL NEW
APPROPRIATIONS P250,000,000 P2,727,000,000 P2,977,000,000
Special Provisions
1. Use and Release of Funds. The amount herein appropriated shall
be used for infrastructure, purchase of ambulances and computers
and other priority projects and activities, and credit facilities to
qualified beneficiaries as proposed and identified by officials
concerned according to the following allocations: Representatives,
P12,500,000 each; Senators, P18,000,000 each; Vice-President,
P20,000,000; PROVIDED, That, the said credit facilities shall be
constituted as a revolving fund to be administered by a government
financial institution (GFI) as a trust fund for lending operations. Prior
years releases to local government units and national government
agencies for this purpose shall be turned over to the government
financial institution which shall be the sole administrator of credit
facilities released from this fund.
The fund shall be automatically released quarterly by way of Advice
of Allotments and Notice of Cash Allocation directly to the assigned
implementing agency not later than five (5) days after the beginning
of each quarter upon submission of the list of projects and activities
by the officials concerned.
2. Submission of Quarterly Reports. The Department of Budget and
Management shall submit within thirty (30) days after the end of
each quarter a report to the Senate Committee on Finance and the
House Committee on Appropriations on the releases made from this
Fund. The report shall include the listing of the projects, locations,
implementing agencies and the endorsing officials (GAA of 1994, p.
1245).
Petitioners claim that the power given to the members of Congress to propose
and identify the projects and activities to be funded by the Countrywide
Development Fund is an encroachment by the legislature on executive power,

since said power in an appropriation act in implementation of a law. They argue


that the proposal and identification of the projects do not involve the making of
laws or the repeal and amendment thereof, the only function given to the
Congress by the Constitution (Rollo, pp. 78- 86).
Under the Constitution, the spending power called by James Madison as "the
power of the purse," belongs to Congress, subject only to the veto power of the
President. The President may propose the budget, but still the final say on the
matter of appropriations is lodged in the Congress.
The power of appropriation carries with it the power to specify the project or
activity to be funded under the appropriation law. It can be as detailed and as
broad as Congress wants it to be.
The Countrywide Development Fund is explicit that it shall be used "for
infrastructure, purchase of ambulances and computers and other priority projects
and activities and credit facilities to qualified beneficiaries . . ." It was Congress
itself that determined the purposes for the appropriation.
Executive function under the Countrywide Development Fund involves
implementation of the priority projects specified in the law.
The authority given to the members of Congress is only to propose and identify
projects to be implemented by the President. Under Article XLI of the GAA of
1994, the President must perforce examine whether the proposals submitted by
the members of Congress fall within the specific items of expenditures for which
the Fund was set up, and if qualified, he next determines whether they are in line
with other projects planned for the locality. Thereafter, if the proposed projects
qualify for funding under the Funds, it is the President who shall implement them.
In short, the proposals and identifications made by the members of Congress are
merely recommendatory.
The procedure of proposing and identifying by members of Congress of particular
projects or activities under Article XLI of the GAA of 1994 is imaginative as it is
innovative.
The Constitution is a framework of a workable government and its interpretation
must take into account the complexities, realities and politics attendant to the
operation of the political branches of government. Prior to the GAA of 1991, there
was an uneven allocation of appropriations for the constituents of the members
of Congress, with the members close to the Congressional leadership or who
hold cards for "horse-trading," getting more than their less favored colleagues.
The members of Congress also had to reckon with an unsympathetic President,

who could exercise his veto power to cancel from the appropriation bill a pet
project of a Representative or Senator.
The Countrywide Development Fund attempts to make equal the unequal. It is
also a recognition that individual members of Congress, far more than the
President and their congressional colleagues are likely to be knowledgeable
about the needs of their respective constituents and the priority to be given each
project.
2. Realignment of Operating Expenses
Under the GAA of 1994, the appropriation for the Senate is P472,000,000.00 of
which P464,447,000.00 is appropriated for current operating expenditures, while
the appropriation for the House of Representatives is P1,171,924,000.00 of
which P1,165,297,000.00 is appropriated for current operating expenditures
(GAA of 1994, pp. 2, 4, 9, 12).
The 1994 operating expenditures for the Senate are as follows:
Personal Services
Salaries, Permanent 153,347
Salaries/Wage, Contractual/Emergency 6,870

Total Salaries and Wages 160,217


=======
Other Compensation

Step Increments 1,073


Honoraria and Commutable Allowances 3,731
Compensation Insurance Premiums 1,579
Pag-I.B.I.G. Contributions 1,184
Medicare Premiums 888
Bonus and Cash Gift 14,791
Terminal Leave Benefits 2,000
Personnel Economic Relief Allowance 10,266
Additional Compensation of P500 under A.O. 53 11,130
Others 57,173

Total Other Compensation 103,815


01 Total Personal Services 264,032
=======
Maintenance and Other Operating Expenses
02 Traveling Expenses 32,841
03 Communication Services 7,666
04 Repair and Maintenance of Government Facilities 1,220
05 Repair and Maintenance of Government Vehicles 318
06 Transportation Services 128
07 Supplies and Materials 20,189
08 Rents 24,584
14 Water/Illumination and Power 6,561
15 Social Security Benefits and Other Claims 3,270
17 Training and Seminars Expenses 2,225
18 Extraordinary and Miscellaneous Expenses 9,360
23 Advertising and Publication
24 Fidelity Bonds and Insurance Premiums 1,325
29 Other Services 89,778

Total Maintenance and Other Operating Expenditures 200,415

Total Current Operating Expenditures 464,447


=======
(GAA of 1994, pp. 3-4)
The 1994 operating expenditures for the House of Representatives are as
follows:
Personal Services
Salaries, Permanent 261,557
Salaries/Wages, Contractual/Emergency 143,643

Total Salaries and Wages 405,200


=======
Other Compensation
Step Increments 4,312
Honoraria and Commutable

Allowances 4,764
Compensation Insurance
Premiums 1,159
Pag-I.B.I.G. Contributions 5,231
Medicare Premiums 2,281
Bonus and Cash Gift 35,669
Terminal Leave Benefits 29
Personnel Economic Relief
Allowance 21,150
Additional Compensation of P500 under A.O. 53
Others 106,140

Total Other Compensation 202,863

01 Total Personal Services 608,063


=======
Maintenance and Other Operating Expenses
02 Traveling Expenses 139,611
03 Communication Services 22,514
04 Repair and Maintenance of Government Facilities 5,116
05 Repair and Maintenance of Government Vehicles 1,863
06 Transportation Services 178
07 Supplies and Materials 55,248
10 Grants/Subsidies/Contributions 940
14 Water/Illumination and Power 14,458
15 Social Security Benefits and Other Claims 325
17 Training and Seminars Expenses 7,236
18 Extraordinary and Miscellaneous Expenses 14,474
20 Anti-Insurgency/Contingency Emergency Expenses 9,400
23 Advertising and Publication 242
24 Fidelity Bonds and Insurance Premiums 1,420
29 Other Services 284,209

Total Maintenance and Other Operating Expenditures 557,234

Total Current Operating Expenditures 1,165,297


=======
(GAA of 1994, pp. 11-12)

The Special Provision Applicable to the Congress of the Philippines provides:


4. Realignment of Allocation for Operational Expenses. A member of
Congress may realign his allocation for operational expenses to any
other expenses category provide the total of said allocation is not
exceeded. (GAA of 1994, p. 14).
The appropriation for operating expenditures for each House is further divided
into expenditures for salaries, personal services, other compensation benefits,
maintenance expenses and other operating expenses. In turn, each member of
Congress is allotted for his own operating expenditure a proportionate share of
the appropriation for the House to which he belongs. If he does not spend for one
items of expense, the provision in question allows him to transfer his allocation in
said item to another item of expense.
Petitioners assail the special provision allowing a member of Congress to realign
his allocation for operational expenses to any other expense category (Rollo, pp.
82-92), claiming that this practice is prohibited by Section 25(5), Article VI of the
Constitution. Said section provides:
No law shall be passed authorizing any transfer of appropriations:
however, the President, the President of the Senate, the Speaker of
the House of Representatives, the Chief Justice of the Supreme
Court, and the heads of Constitutional Commissions may, by law, be
authorized to augment any item in the general appropriations law for
their respective offices from savings in other items of their respective
appropriations.
The proviso of said Article of the Constitution grants the President of the Senate
and the Speaker of the House of Representatives the power to augment items in
an appropriation act for their respective offices from savings in other items of
their appropriations, whenever there is a law authorizing such augmentation.
The special provision on realignment of the operating expenses of members of
Congress is authorized by Section 16 of the General Provisions of the GAA of
1994, which provides:
Expenditure Components. Except by act of the Congress of the
Philippines, no change or modification shall be made in the
expenditure items authorized in this Act and other appropriation laws
unless in cases
of augmentations from savings in appropriations as authorized under

Section 25(5) of Article VI of the Constitution (GAA of 1994, p.


1273).
Petitioners argue that the Senate President and the Speaker of the House of
Representatives, but not the individual members of Congress are the ones
authorized to realign the savings as appropriated.
Under the Special Provisions applicable to the Congress of the Philippines, the
members of Congress only determine the necessity of the realignment of the
savings in the allotments for their operating expenses. They are in the best
position to do so because they are the ones who know whether there are savings
available in some items and whether there are deficiencies in other items of their
operating expenses that need augmentation. However, it is the Senate President
and the Speaker of the House of Representatives, as the case may be, who shall
approve the realignment. Before giving their stamp of approval, these two
officials will have to see to it that:
(1) The funds to be realigned or transferred are actually savings in the items of
expenditures from which the same are to be taken; and
(2) The transfer or realignment is for the purposes of augmenting the items of
expenditure to which said transfer or realignment is to be made.
3. Highest Priority for Debt Service
While Congress appropriated P86,323,438,000.00 for debt service (Article XLVII
of the GAA of 1994), it appropriated only P37,780,450,000.00 for the Department
of Education Culture and Sports. Petitioners urged that Congress cannot give
debt service the highest priority in the GAA of 1994 (Rollo, pp. 93-94) because
under the Constitution it should be education that is entitled to the highest
funding. They invoke Section 5(5), Article XIV thereof, which provides:
(5) The State shall assign the highest budgetary priority to education
and ensure that teaching will attract and retain its rightful share of
the best available talents through adequate remuneration and other
means of job satisfaction and fulfillment.
This issue was raised in Guingona, Jr. v. Carague, 196 SCRA 221 (1991), where
this Court held that Section 5(5), Article XIV of the Constitution, is merely
directory, thus:
While it is true that under Section 5(5), Article XIV of the
Constitution, Congress is mandated to "assign the highest budgetary

priority to education" in order to "insure that teaching will attract and


retain its rightful share of the best available talents through adequate
remuneration and other means of job satisfaction and fulfillment," it
does not thereby follow that the hands of Congress are so
hamstrung as to deprive it the power to respond to the imperatives of
the national interest and for the attainment of other state policies or
objectives.
As aptly observed by respondents, since 1985, the budget for
education has tripled to upgrade and improve the facility of the
public school system. The compensation of teachers has been
doubled. The amount of P29,740,611,000.00 set aside for the
Department of Education, Culture and Sports under the General
Appropriations Act (R.A. No. 6381), is the highest budgetary
allocation among all department budgets. This is a clear compliance
with the aforesaid constitutional mandate according highest priority
to education.
Having faithfully complied therewith, Congress is certainly not
without any power, guided only by its good judgment, to provide an
appropriation, that can reasonably service our enormous debt, the
greater portion of which was inherited from the previous
administration. It is not only a matter of honor and to protect the
credit standing of the country. More especially, the very survival of
our economy is at stake. Thus, if in the process Congress
appropriated an amount for debt service bigger than the share
allocated to education, the Court finds and so holds that said
appropriation cannot be thereby assailed as unconstitutional.
G.R. No. 113105
G.R. No. 113174
Veto of Provision on Debt Ceiling
The Congress added a Special Provision to Article XLVIII (Appropriations for
Debt Service) of the GAA of 1994 which provides:
Special Provisions
1. Use of the Fund. The appropriation authorized herein shall be
used for payment of principal and interest of foreign and domestic
indebtedness; PROVIDED, That any payment in excess of the
amount herein appropriated shall be subject to the approval of the

President of the Philippines with the concurrence of the Congress of


the Philippines; PROVIDED, FURTHER, That in no case shall this
fund be used to pay for the liabilities of the Central Bank Board of
Liquidators.
2. Reporting Requirement. The Bangko Sentral ng Pilipinas and the
Department of Finance shall submit a quarterly report of actual
foreign and domestic debt service payments to the House
Committee on Appropriations and Senate Finance Committee within
one (1) month after each quarter (GAA of 1944, pp. 1266).
The President vetoed the first Special Provision, without vetoing the
P86,323,438,000.00 appropriation for debt service in said Article. According to
the President's Veto Message:
IV. APPROPRIATIONS FOR DEBT SERVICE
I would like to emphasize that I concur fully with the desire of
Congress to reduce the debt burden by decreasing the appropriation
for debt service as well as the inclusion of the Special Provision
quoted below. Nevertheless, I believe that this debt reduction
scheme cannot be validly done through the 1994 GAA. This must be
addressed by revising our debt policy by way of innovative and
comprehensive debt reduction programs conceptualized within the
ambit of the Medium-Term Philippine Development Plan.
Appropriations for payment of public debt, whether foreign or
domestic, are automatically appropriated pursuant to the Foreign
Borrowing Act and Section 31 of P.D. No. 1177 as reiterated under
Section 26, Chapter 4, Book VI of E.O. No. 292, the Administrative
Code of 1987. I wish to emphasize that the constitutionality of such
automatic provisions on debt servicing has been upheld by the
Supreme Court in the case of "Teofisto T. Guingona, Jr., and
Aquilino Q. Pimentel, Jr. v. Hon. Guillermo N. Carague, in his
capacity as Secretary of Budget and Management, et al.," G.R. No.
94571, dated April 22, 1991.
I am, therefore vetoing the following special provision for the reason
that the GAA is not the appropriate legislative measure to amend the
provisions of the Foreign Borrowing Act, P.D. No. 1177 and E.O. No.
292:

Use of the Fund. The appropriation authorized herein


shall be used for payment of principal and interest of
foreign and domestic indebtedness: PROVIDED, That
any payment in excess of the amount herein
appropriated shall be subject to the approval of the
President of the Philippines with the concurrence of the
Congress of the
Philippines:PROVIDED, FURTHER, That in no case
shall this fund be used to pay for the liabilities of the
Central Bank Board of Liquidators (GAA of 1994, p.
1290).
Petitioners claim that the President cannot veto the Special Provision on the
appropriation for debt service without vetoing the entire amount of
P86,323,438.00 for said purpose (Rollo, G.R. No. 113105, pp. 93-98; Rollo, G.R.
No. 113174, pp. 16-18). The Solicitor General counterposed that the Special
Provision did not relate to the item of appropriation for debt service and could
therefore be the subject of an item veto (Rollo, G.R. No. 113105, pp. 54-60;Rollo,
G.R. No. 113174, pp. 72-82).
This issue is a mere rehash of the one put to rest in Gonzales v. Macaraig,
Jr., 191 SCRA 452 (1990). In that case, the issue was stated by the Court, thus:
The fundamental issue raised is whether or not the veto by the
President of Section 55 of the 1989 Appropriations Bill (Section 55
FY '89), and subsequently of its counterpart Section 16 of the 1990
Appropriations Bill (Section 16 FY '90), is unconstitutional and
without effect.
The Court re-stated the issue, just so there would not be any misunderstanding
about it, thus:
The focal issue for resolution is whether or not the President
exceeded the item-veto power accorded by the Constitution. Or
differently put, has the President the power to veto "provisions" of an
Appropriations Bill?
The bases of the petition in Gonzales, which are similar to those invoked in the
present case, are stated as follows:
In essence, petitioners' cause is anchored on the following grounds:
(1) the President's line-veto power as regards appropriation bills is
limited to item/s and does not cover provision/s; therefore, she

exceeded her authority when she vetoed Section 55 (FY '89) and
Section 16 (FY '90) which are provisions; (2) when the President
objects to a provision of an appropriation bill, she cannot exercise
the item-veto power but should veto the entire bill; (3) the item-veto
power does not carry with it the power to strike out conditions or
restrictions for that would be legislation, in violation of the doctrine of
separation of powers; and (4) the power of augmentation in Article
VI, Section 25 [5] of the 1987 Constitution, has to be provided for by
law and, therefore, Congress is also vested with the prerogative to
impose restrictions on the exercise of that power.
The restrictive interpretation urged by petitioners that the President
may not veto a provision without vetoing the entire bill not only
disregards the basic principle that a distinct and severable part of a
bill may be the subject of a separate veto but also overlooks the
Constitutional mandate that any provision in the general
appropriations bill shall relate specifically to some particular
appropriation therein and that any such provision shall be limited in
its operation to the appropriation to which it relates (1987
Constitution, Article VI, Section 25 [2]). In other words, in the true
sense of the term, a provision in an Appropriations Bill is limited in its
operation to some particular appropriation to which it relates, and
does not relate to the entire bill.
The Court went one step further and ruled that even assuming arguendo that
"provisions" are beyond the executive power to veto, and Section 55
(FY '89) and Section 16 (FY '90) were not "provisions" in the budgetary sense of
the term, they are "inappropriate provisions" that should be treated as "items" for
the purpose of the President's veto power.
The Court, citing Henry v. Edwards, La., 346 So. 2d 153 (1977), said that
Congress cannot include in a general appropriations bill matters that should be
more properly enacted in separate legislation, and if it does that, the
inappropriate provisions inserted by it must be treated as "item", which can be
vetoed by the President in the exercise of his item-veto power.
It is readily apparent that the Special Provision applicable to the appropriation for
debt service insofar as it refers to funds in excess of the amount appropriated in
the bill, is an "inappropriate" provision referring to funds other than the
P86,323,438,000.00 appropriated in the General Appropriations Act of 1991.
Likewise the vetoed provision is clearly an attempt to repeal Section 31 of P.D.
No. 1177 (Foreign Borrowing Act) and E.O. No. 292, and to reverse the debt

payment policy. As held by the Court in Gonzales, the repeal of these laws
should be done in a separate law, not in the appropriations law.
The Court will indulge every intendment in favor of the constitutionality of a veto,
the same as it will presume the constitutionality of an act of Congress (Texas Co.
v. State, 254 P. 1060; 31 Ariz, 485, 53 A.L.R. 258 [1927]).
The veto power, while exercisable by the President, is actually a part of the
legislative process (Memorandum of Justice Irene Cortes as Amicus Curiae, pp.
3-7). That is why it is found in Article VI on the Legislative Department rather than
in Article VII on the Executive Department in the Constitution. There is, therefore,
sound basis to indulge in the presumption of validity of a veto. The burden shifts
on those questioning the validity thereof to show that its use is a violation of the
Constitution.
Under his general veto power, the President has to veto the entire bill, not merely
parts thereof (1987 Constitution, Art. VI, Sec. 27[1]). The exception to the general
veto power is the power given to the President to veto any particular item or
items in a general appropriations bill (1987 Constitution, Art. VI,
Sec. 27[2]). In so doing, the President must veto the entire item.
A general appropriations bill is a special type of legislation, whose content is
limited to specified sums of money dedicated to a specific purpose or a separate
fiscal unit (Beckman, The Item Veto Power of the Executive,
31 Temple Law Quarterly 27 [1957]).
The item veto was first introduced by the Organic Act of the Philippines passed
by the U.S. Congress on August 29, 1916. The concept was adopted from some
State Constitutions.
Cognizant of the legislative practice of inserting provisions, including conditions,
restrictions and limitations, to items in appropriations bills, the Constitutional
Convention added the following sentence to Section 20(2), Article VI of the 1935
Constitution:
. . . When a provision of an appropriation bill affect one or more
items of the same, the President cannot veto the provision without at
the same time vetoing the particular item or items to which it relates .
...
In short, under the 1935 Constitution, the President was empowered to veto
separately not only items in an appropriations bill but also "provisions".

While the 1987 Constitution did not retain the aforementioned sentence added to
Section 11(2) of Article VI of the 1935 Constitution, it included the following
provision:
No provision or enactment shall be embraced in the general
appropriations bill unless it relates specifically to some particular
appropriation therein. Any such provision or enactment shall be
limited in its operation to the appropriation to which it relates (Art. VI,
Sec. 25[2]).
In Gonzales, we made it clear that the omission of that sentence of Section 16(2)
of the 1935 Constitution in the 1987 Constitution should not be interpreted to
mean the disallowance of the power of the President to veto a "provision".
As the Constitution is explicit that the provision which Congress can include in an
appropriations bill must "relate specifically to some particular appropriation
therein" and "be limited in its operation to the appropriation to which it relates," it
follows that any provision which does not relate to any particular item, or which
extends in its operation beyond an item of appropriation, is considered "an
inappropriate provision" which can be vetoed separately from an item. Also to be
included in the category of "inappropriate provisions" are unconstitutional
provisions and provisions which are intended to amend other laws, because
clearly these kind of laws have no place in an appropriations bill. These are
matters of general legislation more appropriately dealt with in separate
enactments. Former Justice Irene Cortes, as Amicus Curiae, commented that
Congress cannot by law establish conditions for and regulate the exercise of
powers of the President given by the Constitution for that would be an
unconstitutional intrusion into executive prerogative.
The doctrine of "inappropriate provision" was well elucidated in Henry
v. Edwards, supra., thus:
Just as the President may not use his item-veto to usurp
constitutional powers conferred on the legislature, neither can the
legislature deprive the Governor of the constitutional powers
conferred on him as chief executive officer of the state by including
in a general appropriation bill matters more properly enacted in
separate legislation. The Governor's constitutional power to veto bills
of general legislation . . . cannot be abridged by the careful
placement of such measures in a general appropriation bill, thereby
forcing the Governor to choose between approving unacceptable
substantive legislation or vetoing "items" of expenditures essential to
the operation of government. The legislature cannot by location of a

bill give it immunity from executive veto. Nor can it circumvent the
Governor's veto power over substantive legislation by artfully
drafting general law measures so that they appear to be true
conditions or limitations on an item of appropriation. Otherwise, the
legislature would be permitted to impair the constitutional
responsibilities and functions of a co-equal branch of government in
contravention of the separation of powers doctrine . . . We are no
more willing to allow the legislature to use its appropriation power to
infringe on the Governor's constitutional right to veto matters of
substantive legislation than we are to allow the Governor to
encroach on the Constitutional powers of the legislature. In order to
avoid this result, we hold that, when the legislature inserts
inappropriate provisions in a general appropriation bill, such
provisions must be treated as "items" for purposes of the Governor's
item veto power over general appropriation bills.
xxx xxx xxx
. . . Legislative control cannot be exercised in such a manner as to
encumber the general appropriation bill with veto-proof "logrolling
measures", special interest provisions which could not succeed if
separately enacted, or "riders", substantive pieces of legislation
incorporated in a bill to insure passage without veto . . . (Emphasis
supplied).
Petitioners contend that granting arguendo that the veto of the Special Provision
on the ceiling for debt payment is valid, the President cannot automatically
appropriate funds for debt payment without complying with the conditions for
automatic appropriation under the provisions of R.A. No. 4860 as amended by
P.D. No. 81 and the provisions of P.D. No. 1177 as amended by the
Administrative Code of 1987 and P.D. No. 1967 (Rollo, G.R. No. 113766, pp. 915).
Petitioners cannot anticipate that the President will not faithfully execute the laws.
The writ of prohibition will not issue on the fear that official actions will be done in
contravention of the laws.
The President vetoed the entire paragraph one of the Special Provision of the
item on debt service, including the provisions that the appropriation authorized in
said item "shall be used for payment of the principal and interest of foreign and
domestic indebtedness" and that "in no case shall this fund be used to pay for
the liabilities of the Central Bank Board of Liquidators." These provisions are
germane to and have a direct connection with the item on debt service. Inherent

in the power of appropriation is the power to specify how the money shall be
spent (Henry v. Edwards, LA, 346 So., 2d., 153). The said provisos, being
appropriate provisions, cannot be vetoed separately. Hence the item veto of said
provisions is void.
We reiterate, in order to obviate any misunderstanding, that we are sustaining
the veto of the Special Provision of the item on debt service only with respect to
the proviso therein requiring that "any payment in excess of the amount herein,
appropriated shall be subject to the approval of the President of the Philippines
with the concurrence of the Congress of the Philippines . . ."
G.R. NO. 113174
G.R. NO. 113766
G.R. NO. 11388
1. Veto of provisions for revolving funds of SUC's.
In the appropriation for State Universities and Colleges (SUC's), the President
vetoed special provisions which authorize the use of income and the creation,
operation and maintenance of revolving funds. The Special Provisions vetoed are
the following:
(H. 7) West Visayas State University
Equal Sharing of Income. Income earned by the University subject to
Section 13 of the special provisions applicable to all State
Universities and Colleges shall be equally shared by the University
and the University Hospital (GAA of 1994, p. 395).
xxx xxx xxx
(J. 3) Leyte State College
Revolving Fund for the Operation of LSC House and Human
Resources Development Center (HRDC). The income of Leyte State
College derived from the operation of its LSC House and HRDC
shall be constituted into a Revolving Fund to be deposited in an
authorized government depository bank for the operational
expenses of these projects/services. The net income of the
Revolving Fund at the end of the year shall be remitted to the
National Treasury and shall accrue to the General Fund. The
implementing guidelines shall be issued by the Department of
Budget and Management (GAA of 1994, p. 415).

The vetoed Special Provisions applicable to all SUC's are the following:
12. Use of Income from Extension Services. State Universities and
Colleges are authorized to use their income from their extension
services. Subject to the approval of the Board of Regents and the
approval of a special budget pursuant to Sec. 35, Chapter 5, Book
VI of E.O.
No. 292, such income shall be utilized solely for faculty
development, instructional materials and work study program (GAA
of 1994, p. 490).
xxx xxx xxx
13. Income of State Universities and Colleges. The income of State
Universities and Colleges derived from tuition fees and other
sources as may be imposed by governing boards other than those
accruing to revolving funds created under LOI Nos. 872 and 1026
and those authorized to be recorded as trust receipts pursuant to
Section 40, Chapter 5, Book VI of E.O. No. 292 shall be deposited
with the National Treasury and recorded as a Special Account in the
General Fund pursuant to P.D. No. 1234 and P.D. No. 1437 for the
use of the institution, subject to Section 35, Chapter 5, Book VI of
E.O. No. 292L PROVIDED, That disbursements from the Special
Account shall not exceed the amount actually earned and
deposited: PROVIDED, FURTHER, That a cash advance on such
income may be allowed State half of income actually realized during
the preceding year and this cash advance shall be charged against
income actually earned during the budget year: AND PROVIDED,
FINALLY, That in no case shall such funds be used to create
positions, nor for payment of salaries, wages or allowances, except
as may be specifically approved by the Department of Budge and
Management for income-producing activities, or to purchase
equipment or books, without the prior approval of the President of
the Philippines pursuant to Letter of Implementation No. 29.
All collections of the State Universities and Colleges for fees,
charges and receipts intended for private recipient units, including
private foundations affiliated with these institutions shall be duly
acknowledged with official receipts and deposited as a trust receipt
before said income shall be subject to Section 35, Chapter 5, Book
VI of E.O. No. 292
(GAA of 1994, p. 490).

The President gave his reason for the veto thus:


Pursuant to Section 65 of the Government Auditing Code of the
Philippines, Section 44, Chapter 5, Book VI of E.O. No. 292, s. 1987
and Section 22, Article VII of the Constitution, all income earned by
all Government offices and agencies shall accrue to the General
Fund of the Government in line with the One Fund Policy enunciated
by Section 29 (1), Article VI and Section 22, Article VII of the
Constitution. Likewise, the creation and establishment of revolving
funds shall be authorized by substantive law pursuant to Section 66
of the Government Auditing Code of the Philippines and Section 45,
Chapter 5, Book VI of E.O. No. 292.
Notwithstanding the aforementioned provisions of the Constitution
and existing law, I have noted the proliferation of special provisions
authorizing the use of agency income as well as the creation,
operation and maintenance of revolving funds.
I would like to underscore the facts that such income were already
considered as integral part of the revenue and financing sources of
the National Expenditure Program which I previously submitted to
Congress. Hence, the grant of new special provisions authorizing the
use of agency income and the establishment of revolving funds over
and above the agency appropriations authorized in this Act shall
effectively reduce the financing sources of the 1994 GAA and, at the
same time, increase the level of expenditures of some agencies
beyond the well-coordinated, rationalized levels for such agencies.
This corresponding increases the overall deficit of the National
Government (Veto Message, p. 3).
Petitioners claim that the President acted with grave abuse of discretion when he
disallowed by his veto the "use of income" and the creation of "revolving fund" by
the Western Visayas State University and Leyte State Colleges when he allowed
other government offices, like the National Stud Farm, to use their income for
their operating expenses (Rollo, G.R. No. 113174, pp. 15-16).
There was no undue discrimination when the President vetoed said special
provisions while allowing similar provisions in other government agencies. If
some government agencies were allowed to use their income and maintain a
revolving fund for that purpose, it is because these agencies have been enjoying
such privilege before by virtue of the special laws authorizing such practices as
exceptions to the "one-fund policy" (e.g., R.A. No. 4618 for the National Stud

Farm, P.D. No. 902-A for the Securities and Exchange Commission; E.O. No.
359 for the Department of Budget and Management's Procurement Service).
2. Veto of provision on 70% (administrative)/30% (contract) ratio for road
maintenance.
In the appropriation for the Department of Public Works and Highways, the
President vetoed the second paragraph of Special Provision No. 2, specifying the
30% maximum ration of works to be contracted for the maintenance of national
roads and bridges. The said paragraph reads as follows:
2. Release and Use of Road Maintenance Funds. Funds allotted for
the maintenance and repair of roads which are provided in this Act
for the Department of Public Works and Highways shall be released
to the respective Engineering District, subject to such rules and
regulations as may be prescribed by the Department of Budget and
Management. Maintenance funds for roads and bridges shall be
exempt from budgetary reserve.
Of the amount herein appropriated for the maintenance of national
roads and bridges, a maximum of thirty percent (30%) shall be
contracted out in accordance with guidelines to be issued by the
Department of Public Works and Highways. The balance shall be
used for maintenance by force account.
Five percent (5%) of the total road maintenance fund appropriated
herein to be applied across the board to the allocation of each region
shall be set aside for the maintenance of roads which may be
converted to or taken over as national roads during the current year
and the same shall be released to the central office of the said
department for eventual
sub-allotment to the concerned region and district: PROVIDED, That
any balance of the said five percent (5%) shall be restored to the
regions on a pro-rata basis for the maintenance of existing national
roads.
No retention or deduction as reserves or overhead expenses shall
be made, except as authorized by law or upon direction of the
President
(GAA of 1994, pp. 785-786; Emphasis supplied).
The President gave the following reason for the veto:

While I am cognizant of the well-intended desire of Congress to


impose certain restrictions contained in some special provisions, I
am equally aware that many programs, projects and activities of
agencies would require some degree of flexibility to ensure their
successful implementation and therefore risk their completion.
Furthermore, not only could these restrictions and limitations derail
and impede program implementation but they may also result in a
breach of contractual obligations.
D.1.a. A study conducted by the Infrastructure Agencies show that
for practical intent and purposes, maintenance by contract could be
undertaken to an optimum of seventy percent (70%) and the
remaining thirty percent (30%) by force account. Moreover, the
policy of maximizing implementation through contract maintenance
is a covenant of the Road and Road Transport Program Loan from
the Asian Development Bank (ADB Loan No. 1047-PHI-1990) and
Overseas Economic Cooperation Fund (OECF Loan No. PH-C17199). The same is a covenant under the World Bank (IBRD) Loan for
the Highway Management Project (IBRD Loan
No. PH-3430) obtained in 1992.
In the light of the foregoing and considering the policy of the
government to encourage and maximize private sector participation
in the regular repair and maintenance of infrastructure facilities, I am
directly vetoing the underlined second paragraph of Special
Provision No. 2 of the Department of Public Works and Highways
(Veto Message, p. 11).
The second paragraph of Special Provision No. 2 brings to fore the divergence in
policy of Congress and the President. While Congress expressly laid down the
condition that only 30% of the total appropriation for road maintenance should be
contracted out, the President, on the basis of a comprehensive study, believed
that contracting out road maintenance projects at an option of 70% would be
more efficient, economical and practical.
The Special Provision in question is not an inappropriate provision which can be
the subject of a veto. It is not alien to the appropriation for road maintenance, and
on the other hand, it specified how the said item shall be expended 70% by
administrative and 30% by contract.
The 1987 Constitution allows the addition by Congress of special provisions,
conditions to items in an expenditure bill, which cannot be vetoed separately from

the items to which they relate so long as they are "appropriate" in the budgetary
sense (Art. VII, Sec. 25[2]).
The Solicitor General was hard put in justifying the veto of this special provision.
He merely argued that the provision is a complete turnabout from an entrenched
practice of the government to maximize contract maintenance (Rollo, G.R. No.
113888, pp. 85-86). That is not a ground to veto a provision separate from the
item to which it refers.
The veto of the second paragraph of Special Provision No. 2 of the item for the
DPWH is therefore unconstitutional.
3. Veto of provision on purchase of medicines by AFP.
In the appropriation for the Armed Forces of the Philippines (AFP), the President
vetoed the special provision on the purchase by the AFP of medicines in
compliance with the Generics Drugs Law (R.A. No. 6675). The vetoed provision
reads:
12. Purchase of Medicines. The purchase of medicines by all Armed
Forces of the Philippines units, hospitals and clinics shall strictly
comply with the formulary embodied in the National Drug Policy of
the Department of Health (GAA of 1994, p. 748).
According to the President, while it is desirable to subject the purchase of
medicines to a standard formulary, "it is believed more prudent to provide for a
transition period for its adoption and smooth implementation in the Armed Forces
of the Philippines" (Veto Message, p. 12).
The Special Provision which requires that all purchases of medicines by the AFP
should strictly comply with the formulary embodied in the National Drug Policy of
the Department of Health is an "appropriate" provision. it is a mere advertence by
Congress to the fact that there is an existing law, the Generics Act of 1988, that
requires "the extensive use of drugs with generic names through a rational
system of procurement and distribution." The President believes that it is more
prudent to provide for a transition period for the smooth implementation of the
law in the case of purchases by the Armed Forces of the Philippines, as implied
by Section 11 (Education Drive) of the law itself. This belief, however, cannot
justify his veto of the provision on the purchase of medicines by the AFP.
Being directly related to and inseparable from the appropriation item on
purchases of medicines by the AFP, the special provision cannot be vetoed by

the President without also vetoing the said item (Bolinao Electronics Corporation
v. Valencia, 11 SCRA 486 [1964]).
4. Veto of provision on prior approval of Congress for purchase of military
equipment.
In the appropriation for the modernization of the AFP, the President vetoed the
underlined proviso of Special Provision No. 2 on the "Use of Fund," which
requires the prior approval of Congress for the release of the corresponding
modernization funds, as well as the entire Special Provisions
No. 3 on the "Specific Prohibition":
2. Use of the Fund. Of the amount herein appropriated, priority shall
be given for the acquisition of AFP assets necessary for protecting
marine, mineral, forest and other resources within Philippine
territorial borders and its economic zone, detection, prevention or
deterrence of air or surface intrusions and to support diplomatic
moves aimed at preserving national dignity, sovereignty and
patrimony: PROVIDED, That the said modernization fund shall not
be released until a Table of Organization and Equipment for FY
1994-2000 is submitted to and approved by Congress.
3. Specific Prohibition. The said Modernization Fund shall not be
used for payment of six (6) additional S-211 Trainer planes, 18 SF260 Trainer planes and 150 armored personnel carriers (GAA of
1994, p. 747).
As reason for the veto, the President stated that the said condition and
prohibition violate the Constitutional mandate of non-impairment of contractual
obligations, and if allowed, "shall effectively alter the original intent of the AFP
Modernization Fund to cover all military equipment deemed necessary to
modernize the Armed Forces of the Philippines" (Veto Message, p. 12).
Petitioners claim that Special Provision No. 2 on the "Use of Fund" and Special
Provision No. 3 are conditions or limitations related to the item on the AFP
modernization plan.
The requirement in Special Provision No. 2 on the "Use of Fund" for the AFP
modernization program that the President must submit all purchases of military
equipment to Congress for its approval, is an exercise of the "congressional or
legislative veto." By way of definition, a congressional veto is a means whereby
the legislature can block or modify administrative action taken under a statute. It
is a form of legislative control in the implementation of particular executive

actions. The form may be either negative, that is requiring disapproval of the
executive action, or affirmative, requiring approval of the executive action. This
device represents a significant attempt by Congress to move from oversight of
the executive to shared administration (Dixon, The Congressional Veto and
Separation of Powers: The Executive on a Leash,
56 North Carolina Law Review, 423 [1978]).
A congressional veto is subject to serious questions involving the principle of
separation of powers.
However the case at bench is not the proper occasion to resolve the issues of
the validity of the legislative veto as provided in Special Provisions Nos. 2 and 3
because the issues at hand can be disposed of on other grounds. Any provision
blocking an administrative action in implementing a law or requiring legislative
approval of executive acts must be incorporated in a separate and substantive
bill. Therefore, being "inappropriate" provisions, Special Provisions Nos. 2 and 3
were properly vetoed.
As commented by Justice Irene Cortes in her memorandum as Amicus Curiae:
"What Congress cannot do directly by law it cannot do indirectly by attaching
conditions to the exercise of that power (of the President as Commander-inChief) through provisions in the appropriation law."
Furthermore, Special Provision No. 3, prohibiting the use of the Modernization
Funds for payment of the trainer planes and armored personnel carriers, which
have been contracted for by the AFP, is violative of the Constitutional prohibition
on the passage of laws that impair the obligation of contracts (Art. III, Sec. 10),
more so, contracts entered into by the Government itself.
The veto of said special provision is therefore valid.
5. Veto of provision on use of savings to augment AFP pension funds.
In the appropriation for the AFP Pension and Gratuity Fund, the President vetoed
the new provision authorizing the Chief of Staff to use savings in the AFP to
augment pension and gratuity funds. The vetoed provision reads:
2. Use of Savings. The Chief of Staff, AFP, is authorized, subject to
the approval of the Secretary of National Defense, to use savings in
the appropriations provided herein to augment the pension fund
being managed by the AFP Retirement and Separation Benefits
System as provided under Sections 2(a) and 3 of P.D. No. 361 (GAA

of 1994,
p. 746).
According to the President, the grant of retirement and separation benefits
should be covered by direct appropriations specifically approved for the purpose
pursuant to Section 29(1) of Article VI of the Constitution. Moreover, he stated
that the authority to use savings is lodged in the officials enumerated in Section
25(5) of Article VI of the Constitution (Veto Message, pp. 7-8).
Petitioners claim that the Special Provision on AFP Pension and Gratuity Fund is
a condition or limitation which is so intertwined with the item of appropriation that
it could not be separated therefrom.
The Special Provision, which allows the Chief of Staff to use savings to augment
the pension fund for the AFP being managed by the AFP Retirement and
Separation Benefits System is violative of Sections 25(5) and 29(1) of the Article
VI of the Constitution.
Under Section 25(5), no law shall be passed authorizing any transfer of
appropriations, and under Section 29(1), no money shall be paid out of
the Treasury except in pursuance of an appropriation made by law. While
Section 25(5) allows as an exception the realignment of savings to augment
items in the general appropriations law for the executive branch, such right must
and can be exercised only by the President pursuant to a specific law.
6. Condition on the deactivation of the CAFGU's.
Congress appropriated compensation for the CAFGU's, including the payment of
separation benefits but it added the following Special Provision:
1. CAFGU Compensation and Separation Benefit. The appropriation
authorized herein shall be used for the compensation of CAFGU's
including the payment of their separation benefit not exceeding one
(1) year subsistence allowance for the 11,000 members who will be
deactivated in 1994. The Chief of Staff, AFP, shall, subject to the
approval of the Secretary of National Defense, promulgate policies
and procedures for the payment of separation benefit (GAA of 1994,
p. 740).
The President declared in his Veto Message that the implementation of this
Special Provision to the item on the CAFGU's shall be subject to prior
Presidential approval pursuant to P.D. No. 1597 and R.A.. No. 6758. He gave the
following reasons for imposing the condition:

I am well cognizant of the laudable intention of Congress in


proposing the amendment of Special Provision No. 1 of the CAFGU.
However, it is premature at this point in time of our peace process to
earmark and declare through special provision the actual number of
CAFGU members to be deactivated in CY 1994. I understand that
the number to be deactivated would largely depend on the result or
degree of success of the on-going peace initiatives which are not yet
precisely determinable today. I have desisted, therefore, to directly
veto said provisions because this would mean the loss of the entire
special provision to the prejudice of its beneficient provisions. I
therefore declare that the actual implementation of this special
provision shall be subject to prior Presidential approval pursuant to
the provisions of P.D. No. 1597 and
R.A. No. 6758 (Veto Message, p. 13).
Petitioners claim that the Congress has required the deactivation of the CAFGU's
when it appropriated the money for payment of the separation pay of the
members of thereof. The President, however, directed that the deactivation
should be done in accordance to his timetable, taking into consideration the
peace and order situation in the affected localities.
Petitioners complain that the directive of the President was tantamount to an
administrative embargo of the congressional will to implement the Constitution's
command to dissolve the CAFGU's (Rollo, G.R. No. 113174,
p. 14; G.R. No. 113888, pp. 9, 14-16). They argue that the President cannot
impair or withhold expenditures authorized and appropriated by Congress when
neither the Appropriations Act nor other legislation authorize such impounding
(Rollo, G.R. No. 113888, pp. 15-16).
The Solicitor General contends that it is the President, as Commander-in-Chief of
the Armed Forces of the Philippines, who should determine when the services of
the CAFGU's are no longer needed (Rollo, G.R. No. 113888,
pp. 92-95.).
This is the first case before this Court where the power of the President to
impound is put in issue. Impoundment refers to a refusal by the President, for
whatever reason, to spend funds made available by Congress. It is the failure to
spend or obligate budget authority of any type (Notes: Impoundment of
Funds, 86 Harvard Law Review 1505 [1973]).
Those who deny to the President the power to impound argue that once
Congress has set aside the fund for a specific purpose in an appropriations act, it
becomes mandatory on the part of the President to implement the project and to

spend the money appropriated therefor. The President has no discretion on the
matter, for the Constitution imposes on him the duty to faithfully execute the laws.
In refusing or deferring the implementation of an appropriation item, the
President in effect exercises a veto power that is not expressly granted by the
Constitution. As a matter of fact, the Constitution does not say anything about
impounding. The source of the Executive authority must be found elsewhere.
Proponents of impoundment have invoked at least three principal sources of the
authority of the President. Foremost is the authority to impound given to him
either expressly or impliedly by Congress. Second is the executive power drawn
from the President's role as Commander-in-Chief. Third is the Faithful Execution
Clause which ironically is the same provision invoked by petitioners herein.
The proponents insist that a faithful execution of the laws requires that the
President desist from implementing the law if doing so would prejudice public
interest. An example given is when through efficient and prudent management of
a project, substantial savings are made. In such a case, it is sheer folly to expect
the President to spend the entire amount budgeted in the law
(Notes: Presidential Impoundment: Constitutional Theories and Political Realities,
61 Georgetown Law Journal 1295 [1973]; Notes; Protecting the Fisc: Executive
Impoundment and Congressional Power, 82 Yale Law Journal 1686 [1973).
We do not find anything in the language used in the challenged Special Provision
that would imply that Congress intended to deny to the President the right to
defer or reduce the spending, much less to deactivate 11,000 CAFGU members
all at once in 1994. But even if such is the intention, the appropriation law is not
the proper vehicle for such purpose. Such intention must be embodied and
manifested in another law considering that it abrades the powers of the
Commander-in-Chief and there are existing laws on the creation of the CAFGU's
to be amended. Again we state: a provision in an appropriations act cannot
be used to repeal or amend other laws, in this case, P.D. No. 1597 and R.A. No.
6758.
7. Condition on the appropriation for the Supreme Court, etc.
(a) In the appropriations for the Supreme Court, Ombudsman, COA, and CHR,
the Congress added the following provisions:
The Judiciary
xxx xxx xxx

Special Provisions
1. Augmentation of any Item in the Court's Appropriations. Any
savings in the appropriations for the Supreme Court and the Lower
Courts may be utilized by the Chief Justice of the Supreme Court to
augment any item of the Court's appropriations for (a) printing of
decisions and publication of "Philippine Reports"; (b) Commutable
terminal leaves of Justices and other personnel of the Supreme
Court and payment of adjusted pension rates to retired Justices
entitled thereto pursuant to Administrative Matter No. 91-8-225-C.A.;
(c) repair, maintenance, improvement and other operating expenses
of the courts' libraries, including purchase of books and periodicals;
(d) purchase, maintenance and improvement of printing equipment;
(e) necessary expenses for the employment of temporary
employees, contractual and casual employees, for judicial
administration; (f) maintenance and improvement of the Court's
Electronic Data
Processing System; (g) extraordinary expenses of the Chief Justice,
attendance in international conferences and conduct of training
programs; (h) commutable transportation and representation
allowances and fringe benefits for Justices, Clerks of Court, Court
Administrator, Chiefs of Offices and other Court personnel in
accordance with the rates prescribed by law; and (i) compensation of
attorney-de-officio: PROVIDED, That as mandated by LOI No. 489
any increase in salary and allowances shall be subject to the usual
procedures and policies as provided for under
P.D. No. 985 and other pertinent laws (GAA of 1994, p. 1128;
Emphasis supplied).
xxx xxx xxx
Commission on Audit
xxx xxx xxx
5. Use of Savings. The Chairman of the Commission on Audit is
hereby authorized, subject to appropriate accounting and auditing
rules and regulations, to use savings for the payment of fringe
benefits as may be authorized by law for officials and personnel of
the Commission (GAA of 1994, p. 1161; Emphasis supplied).
xxx xxx xxx

Office of the Ombudsman


xxx xxx xxx
6. Augmentation of Items in the appropriation of the Office of the
Ombudsman. The Ombudsman is hereby authorized, subject to
appropriate accounting and auditing rules and regulations to
augment items of appropriation in the Office of the Ombudsman from
savings in other items of appropriation actually released, for: (a)
printing and/or publication of decisions, resolutions, training and
information materials; (b) repair, maintenance and improvement of
OMB Central and Area/Sectoral facilities; (c) purchase of books,
journals, periodicals and equipment;
(d) payment of commutable representation and transportation
allowances of officials and employees who by reason of their
positions are entitled thereto and fringe benefits as may be
authorized specifically by law for officials and personnel of OMB
pursuant to Section 8 of Article IX-B of the Constitution; and (e) for
other official purposes subject to accounting and auditing rules and
regulations (GAA of 1994, p. 1174; Emphasis supplied).
xxx xxx xxx
Commission on Human Rights
xxx xxx xxx
1. Use of Savings. The Chairman of the Commission on Human
Rights (CHR) is hereby authorized, subject to appropriate
accounting and auditing rules and regulations, to augment any item
of appropriation in the office of the CHR from savings in other items
of appropriations actually released, for: (a) printing and/or
publication of decisions, resolutions, training materials and
educational publications; (b) repair, maintenance and improvement
of Commission's central and regional facilities; (c) purchase of
books, journals, periodicals and equipment, (d) payment of
commutable representation and transportation allowances of officials
and employees who by reason of their positions are entitled thereto
and fringe benefits, as may be authorized by law for officials and
personnel of CHR, subject to accounting and auditing rules and
regulations (GAA of 1994, p. 1178; Emphasis supplied).

In his Veto Message, the President expressed his approval of the conditions
included in the GAA of 1994. He noted that:
The said condition is consistent with the Constitutional injunction
prescribed under Section 8, Article IX-B of the Constitution which
states that "no elective or appointive public officer or employee shall
receive additional, double, or indirect compensation unless
specifically authorized by law." I am, therefore, confident that the
heads of the said offices shall maintain fidelity to the law and
faithfully adhere to the well-established principle on compensation
standardization (Veto Message, p. 10).
Petitioners claim that the conditions imposed by the President violated the
independence and fiscal autonomy of the Supreme Court, the Ombudsman, the
COA and the CHR.
In the first place, the conditions questioned by petitioners were placed in the GAB
by Congress itself, not by the President. The Veto Message merely highlighted
the Constitutional mandate that additional or indirect compensation can only be
given pursuant to law.
In the second place, such statements are mere reminders that the disbursements
of appropriations must be made in accordance with law. Such statements may, at
worse, be treated as superfluities.
(b) In the appropriation for the COA, the President imposed the condition that the
implementation of the budget of the COA be subject to "the guidelines to be
issued by the President."
The provisions subject to said condition reads:
xxx xxx xxx
3. Revolving Fund. The income of the Commission on Audit derived
from sources authorized by the Government Auditing Code of the
Philippines (P.D. No. 1445) not exceeding Ten Million Pesos
(P10,000,000) shall be constituted into a revolving fund which shall
be used for maintenance, operating and other incidental expenses to
enhance audit services and audit-related activities. The fund shall be
deposited in an authorized government depository ban, and
withdrawals therefrom shall be made in accordance with the
procedure prescribed by law and implementing rules and
regulations: PROVIDED,That any interests earned on such deposit

shall be remitted at the end of each quarter to the national Treasury


and shall accrue to the General Fund: PROVIDED FURTHER, That
the Commission on Audit shall submit to the Department of Budget
and Management a quarterly report of income and expenditures of
said revolving fund (GAA of 1994, pp. 1160-1161).
The President cited the "imperative need to rationalize" the implementation,
applicability and operation of use of income and revolving funds. The Veto
Message stated:
. . . I have observed that there are old and long existing special
provisions authorizing the use of income and the creation of
revolving funds. As a rule, such authorizations should be
discouraged. However, I take it that these authorizations have
legal/statutory basis aside from being already a vested right to the
agencies concerned which should not be jeopardized through the
Veto Message. There is, however, imperative need to rationalize
their implementation, applicability and operation. Thus, in order to
substantiate the purpose and intention of said provisions, I hereby
declare that the operationalization of the following provisions during
budget implementation shall be subject to theguidelines to be issued
by the President pursuant to Section 35, Chapter 5, Book VI of E.O.
No. 292 and Sections 65 and 66 of P.D. No. 1445 in relation to
Sections 2 and 3 of the General Provisions of this Act (Veto
Message, p. 6; Emphasis Supplied.)
(c) In the appropriation for the DPWH, the President imposed the condition that in
the implementation of DPWH projects, the administrative and engineering
overhead of 5% and 3% "shall be subject to the necessary administrative
guidelines to be formulated by the Executive pursuant to existing laws." The
condition was imposed because the provision "needs further study" according to
the President.
The following provision was made subject to said condition:
9. Engineering and Administrative Overhead. Not more than five
percent (5%) of the amount for infrastructure project released by the
Department of Budget and Management shall be deducted by
DPWH for administrative overhead, detailed engineering and
construction supervision, testing and quality control, and the like,
thus insuring that at least ninety-five percent (95%) of the released
fund is available for direct implementation of the
project. PROVIDED, HOWEVER, That for school buildings, health

centers, day-care centers and barangay halls, the deductible amount


shall not exceed three percent (3%).
Violation of, or non-compliance with, this provision shall subject the
government official or employee concerned to administrative, civil
and/or criminal sanction under Sections 43 and 80, Book VI of E.O.
No. 292 (GAA of 1994, p. 786).
(d) In the appropriation for the National Housing Authority (NHA), the President
imposed the condition that allocations for specific projects shall be released and
disbursed "in accordance with the housing program of the government, subject to
prior Executive approval."
The provision subject to the said condition reads:
3. Allocations for Specified Projects. The following allocations for the
specified projects shall be set aside for corollary works and used
exclusively for the repair, rehabilitation and construction of buildings,
roads, pathwalks, drainage, waterworks systems, facilities and
amenities in the area: PROVIDED, That any road to be constructed
or rehabilitated shall conform with the specifications and standards
set by the Department of Public Works and Highways for such kind
of road: PROVIDED, FURTHER, That savings that may be available
in the future shall be used for road repair, rehabilitation and
construction:
(1) Maharlika Village Road Not less than
P5,000,000
(2) Tenement Housing Project (Taguig)
Not less than P3,000,000
(3) Bagong Lipunan Condominium Project
(Taguig) Not less than P2,000,000
4. Allocation of Funds. Out of the amount appropriated for the
implementation of various projects in resettlement areas, Seven
Million Five Hundred Thousand Pesos (P7,500,000) shall be
allocated to the Dasmarias Bagong Bayan resettlement area,
Eighteen Million Pesos (P18,000,000) to the Carmona Relocation
Center Area (Gen. Mariano Alvarez) and Three Million Pesos
(P3,000,000) to the Bulihan Sites and Services, all of which will be
for the cementing of roads in accordance with DPWH standards.

5. Allocation for Sapang Palay. An allocation of Eight Million Pesos


(P8,000,000) shall be set aside for the asphalting of seven (7)
kilometer main road of Sapang Palay, San Jose Del Monte, Bulacan
(GAA of 1994, p. 1216).
The President imposed the conditions: (a) that the "operationalization" of the
special provision on revolving funds of the COA "shall be subject to guidelines to
be issued by the President pursuant to Section 35, Chapter 5,
Book VI of E.O. 292 and Sections 65 and 66 of P.D. No. 1445 in relation to
Sections 2 and 3 of the General Provisions of this Act" (Rollo, G.R.
No. 113174, pp. 5,7-8); (b) that the implementation of Special Provision No. 9 of
the DPWH on the mandatory retention of 5% and 3% of the amounts released by
said Department "be subject to the necessary administrative guidelines to be
formulated by the Executive pursuant to existing law" (Rollo, G.R. No. 113888;
pp. 10, 14-16); and (c) that the appropriations authorized for the NHA can be
released only "in accordance with the housing program of the government
subject to prior Executive approval" (Rollo, G.R. No. 113888, pp. 10-11;
14-16).
The conditions objected to by petitioners are mere reminders that the
implementation of the items on which the said conditions were imposed, should
be done in accordance with existing laws, regulations or policies. They did not
add anything to what was already in place at the time of the approval of the GAA
of 1994.
There is less basis to complain when the President said that the expenditures
shall be subject to guidelines he will issue. Until the guidelines are issued, it
cannot be determined whether they are proper or inappropriate. The issuance of
administrative guidelines on the use of public funds authorized by Congress is
simply an exercise by the President of his constitutional duty to see that the laws
are faithfully executed (1987 Constitution, Art. VII, Sec. 17; Planas v. Gil 67 Phil.
62 [1939]). Under the Faithful Execution Clause, the President has the power to
take "necessary and proper steps" to carry into execution the law (Schwartz, On
Constitutional Law, p. 147 [1977]). These steps are the ones to be embodied in
the guidelines.
IV
Petitioners chose to avail of the special civil actions but those remedies can be
used only when respondents have acted "without or in excess" of jurisdiction, or
"with grave abuse of discretion," (Revised Rules of Court,
Rule 65, Section 2). How can we begrudge the President for vetoing the Special
Provision on the appropriation for debt payment when he merely followed our

decision in Gonzales? How can we say that Congress has abused its discretion
when it appropriated a bigger sum for debt payment than the amount
appropriated for education, when it merely followed our dictum in Guingona?
Article 8 of the Civil Code of Philippines, provides:
Judicial decisions applying or interpreting the laws or the constitution
shall from a part of the legal system of the Philippines.
The Court's interpretation of the law is part of that law as of the date of its
enactment since the court's interpretation merely establishes the contemporary
legislative intent that the construed law purports to carry into effect (People v.
Licera, 65 SCRA 270 [1975]). Decisions of the Supreme Court assume the same
authority as statutes (Floresca v. Philex Mining Corporation, 136 SCRA 141
[1985]).
Even if Guingona and Gonzales are considered hard cases that make bad laws
and should be reversed, such reversal cannot nullify prior acts done in reliance
thereof.
WHEREFORE, the petitions are DISMISSED, except with respect to
(1) G.R. Nos. 113105 and 113766 only insofar as they pray for the annulment of
the veto of the special provision on debt service specifying that the fund therein
appropriated "shall be used for payment of the principal and interest of foreign
and domestic indebtedness" prohibiting the use of the said funds "to pay for the
liabilities of the Central Bank Board of Liquidators", and (2) G.R. No. 113888 only
insofar as it prays for the annulment of the veto of: (a) the second paragraph of
Special Provision No. 2 of the item of appropriation for the Department of Public
Works and Highways (GAA of 1994, pp. 785-786); and (b) Special Provision No.
12 on the purchase of medicines by the Armed Forces of the Philippines (GAA of
1994, p. 748), which is GRANTED.
SO ORDERED.
G.R.

No.

113105

August

19

1994

[Article

VI

Section

25

Appropriations]

FACTS:
Petitioners assailed the validity of RA 7663 or General Appropriations Act of 1994.
GAA contains a special provision that allows any members of the Congress the REalignment of Allocation
for Operational Expenses, provided that the total of said allocation is not exceeded.
Philconsa claims that only the Senate President and the Speaker of the House of Representatives are the
ones authorized under the Constitution to realign savings, not the individual members of Congress
themselves.
President signed the law, but Vetoes certain provisions of the law and imposed certain provisional
conditions: that the AFP Chief of Staff is authorized to use savings to augment the pension funds under
the
Retirement
and
Separation
Benefits
of
the
AFP.

ISSUE:
Whether or not RA 7663 is violative of Article VI, Section 25 (5) of 1987 Constitution.
RULING:
Yes. Only the Senate President and the Speaker of the House are allowed to approve the realignment.
Furthermore, two conditions must be met: 1) the funds to be realigned are actually savings, and 2) the
transfer is for the purpose of augmenting the items of expenditures to which said transfer to be made.
As to the certain condition given to the AFP Chief of Staff, it is violative of of Sections 25(5) and 29(1) of
the Article VI of the Constitution. The list of those who may be authorized to transfer funds is exclusive.
the AFP Chief of Staff may not be given authority.

Republic of the Philippines


SUPREME COURT
Manila
EN BANC
G.R. No. L-23326

December 18, 1965

PHILIPPINE CONSTITUTION ASSOCIATION, INC., JOSE E. ROMERO,


SALVADOR ARANETA, GUILLERMO B. GUEVARA, PIO PEDROSA,
CONRADO BENITEZ, JOSE M. ARUEGO, SOTERO H. LAUREL,
FELIXBERTO M. SERRANO, and ROMAN OZAETA, petitioners,
vs.
PEDRO M. GIMENEZ, JOSE VELASCO, ELADIO SALITA and JOSE
AVILES, respondents.
Roman Ozaeta, Guillermo B. Guevara, Jose M. Aruego, Sotero H. Laurel and
Felixberto M. Serrano for themselves and for other petitioners.
Office of the Solicitor General for respondents.
REGALA, J.:
We are called upon in this case to decide the grave and fundamental problem of
the constitutionality of Republic Act No. 3836 "insofar as the same allows
retirement gratuity and commutation of vacation and sick leave to Senators and
Representatives, and to the elective officials of both houses (of Congress)." The
suit was instituted by the Philippine Constitution Association, Inc. (Philconsa, for
short), a non-profit civic organization, duly incorporated under Philippine laws, by
way of a petition for prohibition with preliminary injunction to restrain the Auditor
General of the Philippines and the disbursing officers of both Houses of
Congress from "passing in audit the vouchers, and from countersigning the
checks or treasury warrants for the payment to any former Senator or former

Member of the House of Representatives of retirement and vacation gratuities


pursuant to Republic Act No. 3836; and likewise restraining the respondent
disbursing officers of the House and Senate, respectively, and their successors
in office from paying the said retirement and vacation gratuities."
It is argued that the above-numbered Republic Act, at least to the end that it
provided for the retirement of the members of Congress in the manner and terms
that it did, is unconstitutional and void. The challenge to the constitutionality of
the law is centered on the following propositions:
1. The provision for the retirement of the members and certain officers of
Congress is not expressed in the title of the bill, in violation of section 21
(1) of Article VI of the Constitution.
2. The provision on retirement gratuity is an attempt to circumvent the
Constitutional ban on increase of salaries of the members of Congress
during their term of office, contrary to the provisions of Article VI, Section
14 of the Constitution.
3. The same provision constitutes "selfish class legislation" because it
allows members and officers of Congress to retire after twelve (12) years
of service and gives them a gratuity equivalent to one year salary for every
four years of service, which is not refundable in case of reinstatement or
re-election of the retiree, while all other officers and employees of the
government can retire only after at least twenty (20) years of service and
are given a gratuity which is only equivalent to one month salary for every
year of service, which, in any case, cannot exceed 24 months.
4. The provision on vacation and sick leave, commutable at the highest
rate received, insofar as members of Congress are concerned, is another
attempt of the legislators to further increase their compensation in violation
of the Constitution.
The text of Republic Act No. 3836
The text of Republic Act No. 3836 reads:
AN ACT AMENDING SUBSECTION (c), SECTION TWELVE OF
COMMONWEALTH ACT NUMBERED ONE HUNDRED EIGHTY-SIX, AS
AMENDED BY REPUBLIC ACT NUMBERED THIRTY HUNDRED
NINETY-SIX:

Be it enacted by the Senate and House of Representatives of the


Philippines in Congress assembled:
SECTION 1. Subsection (c), Section twelve of Commonwealth Act
Numbered One Hundred eighty-six, as amended by Republic Act
Numbered Thirty hundred ninety-six, is further amended to read as follows:
"(c) Retirement is likewise allowed to a member, regardless of age, who
has rendered at least twenty years of service. The benefit shall, in addition
to the return of his personal contributions plus interest and the payment of
the corresponding employer's premiums described in subsection (a) of
Section five hereof, without interest, be only a gratuity equivalent to one
month's salary for every year of service, based on the highest rate
received, but not to exceed twenty-four months: Provided, That the retiring
officer or employee has been in the service of the said employer or office
for at least four years immediately preceding his retirement.
"Retirement is also allowed to a senator or a member of the House of
Representatives and to an elective officer of either House of the Congress,
regardless of age, provided that in the case of a Senator or Member, he
must have served at least twelve years as a Senator and/or as a member
of the House of Representatives, and, in the case of an elective officer of
either House, he must have served the government for at least twelve
years, not less than four years of which must have been rendered as such
elective officer:Provided, That the gratuity payable to a retiring senator,
member of the House of Representatives, or elective officer, of either
House, shall be equivalent to one year's salary for every four years of
service in the government and the same shall be exempt from any tax
whatsoever and shall be neither liable to attachment or execution nor
refundable in case of reinstatement or re-election of the retiree.
"This gratuity is payable by the employer or office concerned which is
hereby authorized to provide the necessary appropriation or pay the same
from any unexpended items of appropriations or savings in its
appropriations or saving in its appropriations.
"Elective or appointive officials and employees paid gratuity under this
subsection shall be entitled to the commutation of the unused vacation and
sick leave, based on the highest rate received, which they may have to
their credit at the time of retirement."
SECTION 2. This Act shall take effect upon its approval.

Approved, June 22, 1963.


The Solicitor General's Office, in representation of the respondent, filed its
answer on September 8, 1964, and contends, by way of special and affirmative
defenses that:
1. The grant of retirement or pension benefits under Republic Act No. 3836
to the officers objected to by the petitioner does not constitute "forbidden
compensation" within the meaning of Section 14 of Article VI of the
Philippine Constitution.
2. The title of the law in question sufficiently complies with the provisions of
Section 21, Article VI, of the Constitution that "no bill which may be
enacted into law shall embrace more than one subject which shall be
expressed in the title of the bill.
3. The law in question does not constitute legislation.
4. Certain indispensable parties, specifically the elected officers of
Congress who are authorized to approve vouchers for payments for funds
under the law in question, and the claimants to the vouchers to be
presented for payment under said items, were not included in the petition.
5. The petitioner has no standing to institute this suit.
6. The payment of commutable vacation and sick leave benefits under the
said Act is merely "in the nature of a basis for computing the gratuity due
each retiring member" and, therefore, is not an indirect scheme to increase
their salary.
A brief historical background of Republic Act No. 3836
Republic Act No. 3836 was originally House Bill No. 6051, which was introduced
by Congressmen Marcial R. Pimentel of Camarines Norte and Marcelino R.
Veloso of the Third District of Leyte, on May 6, 1963. On the same date, it was
referred to the Committee on Civil Service. which on the following May 8,
submitted its REPORT No. 3129, recommending approval of the bill with
amendments, among others, that the word "TWENTY" in the bill as filed
representing the number of years that a senator or member must serve in
Congress to entitle him to retirement under the bill must be reduced to
"TWELVE" years, and that the following words were inserted, namely, "AND THE
SAME (referring to gratuity) SHALL BE EXEMPT FROM ANY TAX
WHATSOEVER AND SHALL NOT BE LIABLE FROM ATTACHMENT OR

EXECUTION NOR REFUNDABLE IN CASE OF REINSTATEMENT OR


REELECTION OF THE RETIREE." On May 8, 1963, the bill with the proposed
amendments was approved on second reading. It was passed on third reading
on May 13, 1963, and on the same day was sent to the Senate, which, in turn, on
May 23, 1963, passed it without amendment. The bill was finally approved on
June 22, 1963. As explained in the EXPLANATORY NOTE attached to the bill,
among others
The inclusion of members of Congress in subsection (c), Section 12 of
C.A. 186, as amended, will enable them to retire voluntarily, regardless of
age, after serving a minimum of twenty years as a Member of Congress.
This gratuity will insure the security of the family of the retiring member of
Congress with the latter engaging in other activities which may detract
from his exalted position and usefulness as lawmaker. It is expected that
with this assurance of security for his loved ones, deserving and wellintentioned but poor men will be attracted to serve their people in
Congress.
As finally approved, the law (Subsection [c], paragraph 2, Section 1, R.A. 3836)
allows a Senator or a Member of the House of Representatives and an elective
officer of either House of Congress to retire regardless of age. To be eligible for
retirement, he must have served for at least twelve years as such Senator and/or
as member of the House of Representatives. For an elective officer of either
House, he must have served the government for at least twelve years, of which
not less than four years must have been rendered as such elective officer. The
gratuity payable by the employer or office concerned is equivalent to one year's
salary for every four years of service in the government. Said gratuity is exempt
from taxation, not liable to attachment or execution, and not refundable in case of
reinstatement or re-election of the retiree.
First legal point personality of the Petitioner to bring suit.
The first point to be considered is whether petitioner Philconsa has a standing to
institute this action. This Court has not hesitated to examine past decisions
involving this matter. This Court has repeatedly held that when the petitioner, like
in this case, is composed of substantial taxpayers, and the outcome will affect
their vital interests, they are allowed to bring this suit. (Pascual v. Secretary, G.R.
No. L-10405, December 29, 1960; and Gonzales v. Hechanova, 60 Off. Gaz. 802
[1963]).
The petitioner, Philconsa, is precisely a non-profit, civic organization composed
of several leaders from all walks of life whose main objective is to uphold the
principles of the Constitution.

In rejecting the motion to dismiss in the case of Pascual v. Secretary, supra, this
Court stated, among other things, that "there are many decisions nullifying, at the
instance of the taxpayers, laws providing the disbursement of public funds, upon
the theory that the expenditure of public funds by an officer of the State for the
purpose of administering an unconstitutional act constitutes a misappropriation of
such funds, which may be enjoined at the request of the taxpayers."1 This
legislation (Republic Act 3836) involves the disbursement of public funds.
We are not, however, unmindful of the ruling laid down by the Supreme Court of
the United States in the case ofMassachusetts v. Mellon, 262 U.S. 447, holding
that:
... the relation of a taxpayer of the United States to the Federal
Government is very different. His interest in the moneys of the Treasury
partly realized from taxation and partly from other sources is shared
with millions of others; is comparatively minute and indeterminable; and
the effect upon future taxation of any payment out of the funds, so remote,
fluctuating and uncertain, that no basis is afforded for an appeal to the
preventive powers of equity.
The general view in the United States, which is followed here, is stated in the
American Jurisprudence, thus
In the determination of the degree of interest essential to give the requisite
standing to attack the constitutionality of a statute the general rule is that
not only persons individually affected, but also taxpayers have sufficient
interest in preventing the illegal expenditure of moneys raised by taxation
and may therefore question the constitutionality of statutes requiring
expenditure of public moneys. (11 Am. Jur. 761; emphasis supplied.)
As far as the first point is concerned, We hold, therefore, that the contention of
the Solicitor General is untenable.
Second legal point Whether or not Republic Act No. 3836 falls within the
prohibition embodied in Art. VI, section 14 of the Constitution.
The first constitutional question is whether Republic Act 3836 violates Section 14,
Article VI, of the Constitution, which reads as follows:
The senators and the Members of the House of Representatives shall,
unless otherwise provided by law, receive an annual compensation of
seven thousand two hundred pesos each, including per diems and other
emoluments or allowances, and exclusive only of travelling expenses to

and from their respective districts in the case of Members of the House of
Representative and to and from their places of residence in the case of
Senators, when attending sessions of the Congress. No increase in said
compensation shall take effect until after the expiration of the full term of all
the Members of the Senate and of the House of Representatives
approving such increase. Until otherwise provided by law, the President of
the Senate and the Speaker of the House of Representatives shall each
receive an annual compensation of sixteen thousand pesos (emphasis
supplied)
Before discussing this point, it is worthy to note that the Constitution embodies
some limitations and prohibitions upon the members of Congress, to wit:
1. They may not hold any other office or employment in the Government
without forfeiting their respective seats;
2. They shall not be appointed, during the time for which they are elected,
to any civil office which may have been created or the emoluments
whereof shall have been increased while they were members of Congress;
(Section 16, Article VI, Constitution)
3. They cannot be financially interested in any franchise;
4. They cannot appear in any civil case wherein the Government is an
adverse party;
5. They cannot appear as counsel before any Electoral Tribunal; and
6. They cannot appear as counsel in any criminal case where an officer or
employee of the Government is accused. (Section 17, Article VI,
Constitution)
In addition to the above prohibitions, the Anti-Graft Law (Republic Act 3019) also
prohibits members of Congress to have any special interest in any specific
business which will directly or indirectly be favored by any law or resolution
authored by them during their term of office.
It is thus clear that the Constitutional Convention wisely surrounded the
Constitution with these limitations and prohibitions upon Members of Congress.
This is a practical demonstration or application of the principle of the and
balances which is one of the peculiar characteristics of our Constitution. In the
light of this background, can We conclude that Congress can validly enact
Republic Act 3836, providing retirement benefits to its members, without violating

the provisions in the aforementioned Article VI, Section 14, of the Constitution,
regarding increase of the compensation act including other emoluments?
It is worthy to note that the original salary for the members of the National
Assembly (unicameral body) was fixed at P5,000.00 per annum each. This was
raised to P7,200 per annum by the enactment of the 1940 Constitutional
amendment, when the unicameral body, the National Assembly, was changed to
Congress, composed of two bodies, the Senate and the House of
Representatives. Again, in 1964, by the enactment of Republic Act 4143, the
salary for the Members of Congress was raised to P32,000.00 per annum for
each of them; and for the President of the Senate and the Speaker of the House
of Representatives, to P40,000.00 per annum each.
Likewise, it is significant that, as stated above, when the Constitutional
Convention first determined the compensation for the Members of Congress, the
amount fixed by it was only P5,000.00 per annum, but it embodies a special
proviso which reads as follows: "No increase in said compensation shall take
effect until after the expiration of the full term of all the members of the National
Assembly elected subsequent to approval of such increase." In other words,
under the original constitutional provision regarding the power of the National
Assembly to increase the salaries of its members, no increase would take effect
until after the expiration of the full term of the members of the Assembly elected
subsequent to the approval of such increase. (See Aruego, The Framing of the
Constitution, Vol. 1, pp. 296-300; Sinco, Philippine Government and Political
Law, 4th ed., p. 187)
This goes to show how zealous were the members of the Constitutional
Convention in guarding against the temptation for members of Congress to
increase their salaries. However, the original strict prohibition was modified by
the subsequent provision when the Constitutional amendments were approved in
19402
The Constitutional provision in the aforementioned Section 14, Article VI,
includes in the term compensation "other emoluments." This is the pivotal point
on this fundamental question as to whether the retirement benefits as provided
for in Republic Act 3836 fall within the purview of the term "other emoluments."
Most of the authorities and decided cases have regarded "emolument" as "the
profit arising from office or employment; that which is received as compensation
for services or which is annexed to the possession of an office, as salary, fees
and perquisites.3

In another set of cases, "emolument" has been defined as "the profit arising from
office or employment; that which is received as compensation for services, or
which is annexed to the possession of office, as salary, fees and perquisites;
advantage, gain, public or private." The gain, profit or advantage which is
contemplated in the definition or significance of the word "emolument" as applied
to public officers, clearly comprehends, We think, a gain, profit, or advantage
which is pecuniary in character. (citing Taxpayers' League of Cargon County v.
McPherson, 54 P. 2d. 897, 90l.: 49 Wy. 26; 106 A.L.R. 767)
In Schieffelin v. Berry, 216 N.Y.S. (citing Wright v. Craig, 202 App. Div. 684, 195
N.Y.S. 391, affirmed 234 N.Y. 548, 138 N.E. 441), it has been established that
pensions and retirement allowances are part of compensation of public officials;
otherwise their payment would be unconstitutional.
In another case, State v. Schmahl, 145 N.W. 795, 125 Minn. 104, it is stated that
"as used in Article 4, section 9, of the Constitution of Minnesota, providing that no
Senator or Representative shall hold any office, the emoluments of which have
been increased during the session of the Legislature of which he was a member,
until after the expiration of his term of office in the Legislature, the word
"emoluments" does not refer to the fixed salary alone, but includes fees and
compensation as the incumbent of the office is by law entitled to receive because
he holds such office and performed some service required of the occupant
thereof."
From the decisions of these cases, it is evident that retirement benefit is a form
or another species of emolument, because it is a part of compensation for
services of one possessing any office.
Republic Act No. 3836 provides for an increase in the emoluments of Senators
and Members of the House of Representatives, to take effect upon the approval
of said Act, which was on June 22, 1963. Retirement benefits were immediately
available thereunder, without awaiting the expiration of the full term of all the
Members of the Senate and the House of Representatives approving such
increase. Such provision clearly runs counter to the prohibition in Article VI,
Section 14 of the Constitution.
Third Legal Point Whether or not the law in question violates the equal
protection clause of the Constitution.
Another reason in support of the conclusion reached herein is that the features of
said Republic Act 3836 are patently discriminatory, and therefore violate the
equal protection clause of the Constitution. (Art. III, Sec. 1, part. 1.)

In the first place, while the said law grants retirement benefits to Senators and
Members of the House of Representatives who are elective officials, it does not
include other elective officials such as the governors of provinces and the
members of the provincial boards, and the elective officials of the municipalities
and chartered cities.
The principle of equal protection of law embodied in our Constitution has been
fully explained by Us in the case ofPeople v. Vera, 65 Phil. 56, 126, where We
stated that the classification to be reasonable must be based upon substantial
distinctions which make real differences and must be germane to the purposes of
the law.
As well stated by Willoughby on the Constitution of the United States (second
edition), p. 1937, the principle of the requirement of equal protection of law
applies to all persons similarly situated. Why limit the application of the benefits
of Republic Act 3836 to the elected members of Congress? We feel that the
classification here is not reasonable. (See also Sinco, Philippine Political Law,
11th ed. [1962]; Selected Essays on Constitutional Law [1938-62], p. 789; The
Equal Protection of the Laws, 37 Cal. Law Rev. 341.)
Secondly, all members of Congress under Republic Act 3836 are given
retirement benefits after serving twelve years, not necessarily continuous,
whereas, most government officers and employees are given retirement benefits
after serving for at least twenty years. In fact, the original bill of Act 3836
provided for twenty years of service.
In the third place, all government officers and employees are given only one
retirement benefit irrespective of their length of service in the government,
whereas, under Republic Act 3836, because of no age limitation, a Senator or
Member of the House of Representatives upon being elected for 24 years will be
entitled to two retirement benefits or equivalent to six years' salary.
Also, while the payment of retirement benefits (annuity) to an employee who had
been retired and reappointed is suspended during his new employment (under
Commonwealth Act 186, as amended), this is not so under Republic Act 3836.
Lastly, it is peculiar that Republic Act 3836 grants retirement benefits to officials
who are not members of the Government Service Insurance System. Most
grantees of retirement benefits under the various retirement laws have to be
members or must at least contribute a portion of their monthly salaries to the
System.4

The arguments advanced against the discriminatory features of Republic Act


3836, as far as Members of Congress are concerned, apply with equal force to
the elected officers of each House, such as the Secretaries and the Sergeantsat-arms. Under Republic Act 3836, the Secretaries and Sergeants-at-arms of
each House are given the benefits of retirement without having served for twenty
years as required with other officers and employees of the Government.
Fourth Legal Point Whether or not the title of Republic Act No. 3836 is
germane to the subject matter expressed in the act.
Another Constitutional point to determine is whether the title of Republic Act 3836
complies with the requirement of paragraph 1, section 21, Article VI of the
Constitution, which reads as follows:
No bill which may be enacted into law shall embrace more than one
subject which shall be expressed in the title of the bill.
We are not unmindful of the fact that there has been a general disposition in all
courts to construe the constitutional provision with reference to the subject and
title of the Act, liberally.
It is the contention of petitioner that the said title of Republic Act 3836 gives no
inkling or notice whatsoever to the public regarding the retirement gratuities and
commutable vacation and sick leave privileges to members of Congress. It is
claimed that petitioner learned of this law for the first time only when Jose
Velasco, disbursing officer of the House, testified on January 30, 1964, before
Justice Labrador, in connection with the hearing of the case, and he revealed
that in 1963, Congress enacted the retirement law for its members. In fact the
Appropriation Act for the fiscal year 1964-65, Republic Act No. 4164, provides:
13. For payment of retirement gratuities of members of the Senate
pursuant to the provisions of Republic Act No. 3836: PROVIDED, That no
portion of this Appropriation shall be transferred to any other item until all
approved claims shall have been paid P210,000.00.
In the appropriations for the House of Representatives the following items
appear:
7. For government share of premiums on life insurance and retirement of
Members and employees of the House of Representatives, as provided for
under Republic Act No. 1616 P300,000.00

8. For payment of the cash commutation of the accumulated vacation and


sick leaves as provided for under Republic Act No. 611, and retirement
gratuities of Members and employees of the House of Representatives
under Republic Act No. 1616 P1,300,000.00.
In the Appropriations Act of 1965-1966 (Republic Act No. 4642), the following
item appears in the appropriations for the Senate:
13. For payment of retirement gratuities of Senate personnel pursuant to
the provisions of Republic Act No. 1616: PROVIDED, That no portion of
this appropriation shall be transferred to any other item until all approved
claims shall have been paid P100,000.00.
It is thus clear that in the Appropriations Act for 1965-1966, the item in the
Senate for P210,000.00 to implement Republic Act 3836 was eliminated.
In the appropriations for the House (1965-1966), the following items appear:
7. For government share of premiums on life insurance and retirement of
Members and employees of the House Of Representatives as provided for
under Republic Act No. 1616 P1,200,000.00.
8. For payment of the cash commutation of the accumulated vacation and
sick leaves as provided for under Republic Act No. 611, and retirement
gratuities of Members and employees of the House of Representatives
under Republic Act No. 1616 P1,700,000.00.
It is to be observed that under Republic Act 3836, amending the first paragraph
of section 12, subsection (c) of Commonwealth Act 186, as amended by
Republic Acts Nos. 660 and. 3096, the retirement benefits are granted to
members of the Government Service Insurance System, who have rendered at
least twenty years of service regardless of age. This paragraph is related and
germane to the subject of Commonwealth Act No. 186.
On the other hand, the succeeding paragraph of Republic Act 3836 refers to
members of Congress and to elective officers thereof who are not members of
the Government Service Insurance System. To provide retirement benefits,
therefore, for these officials, would relate to subject matter which is not germane
to Commonwealth Act No. 186. In other words, this portion of the amendment (re
retirement benefits for Members of Congress and elected officers, such as the
Secretary and Sergeants-at-arms for each House) is not related in any manner to
the subject of Commonwealth Act 186 establishing the Government Service

Insurance System and which provides for both retirement and insurance benefits
to its members.
Parenthetically, it may be added that the purpose of the requirement that the
subject of an Act should be expressed in its title is fully explained by Cooley,
thus: (1) to prevent surprise or fraud upon the Legislature; and (2) to fairly
apprise the people, through such publication of legislation that are being
considered, in order that they may have the opportunity of being heard thereon
by petition or otherwise, if they shall so desire (Cooley, Constitutional Limitations,
8th ed., Vol. 1, p. 162; See also Martin, Political Law Reviewer, Book One [1965],
p. 119)
With respect to sufficiency of title this Court has ruled in two cases:
The Constitutional requirement with respect to titles of statutes as sufficient
to reflect their contents is satisfied if all parts of a law relate to the subject
expressed in its title, and it is not necessary that the title be a complete
index of the content. (People v. Carlos, 78 Phil. 535)
The Constitutional requirement that the subject of an act shall be
expressed in its title should be reasonably construed so as not to interfere
unduly with the enactment of necessary legislation. It should be given a
practical, rather than technical, construction. It should be a sufficient
compliance with such requirement if the title expresses the general subject
and all the provisions of the statute are germane to that general subject.
(Sumulong v. The Commission on Elections, 73 Phil. 288, 291)
The requirement that the subject of an act shall be expressed in its title is wholly
illustrated and explained in Central Capiz v. Ramirez, 40 Phil. 883. In this case,
the question raised was whether Commonwealth Act 2784, known as the Public
Land Act, was limited in its application to lands of the public domain or whether
its provisions also extended to agricultural lands held in private ownership. The
Court held that the act was limited to lands of the public domain as indicated in
its title, and did not include private agricultural lands. The Court further stated
that this provision of the Constitution expressing the subject matter of an Act in
its title is not a mere rule of legislative procedure, directory to Congress, but it
is mandatory. It is the duty of the Court to declare void any statute not
conforming to this constitutional provision. (See Walker v. State, 49 Alabama
329; Cooley, Constitutional Limitations, pp. 162-164;5 See also Agcaoili v.
Suguitan, 48 Phil. 676; Sutherland on Statutory Construction, Sec. 111.)
In the light of the history and analysis of Republic Act 3836, We conclude that the
title of said Republic Act 3836 is void as it is not germane to the subject matter

and is a violation of the aforementioned paragraph 1, section 21, Article VI of the


Constitution.
In short, Republic Act 3836 violates three constitutional provisions, namely: first,
the prohibition regarding increase in the salaries of Members of Congress;
second, the equal protection clause; and third, the prohibition that the title of a bill
shall not embrace more than one subject.
IN VIEW OF THE FOREGOING CONSIDERATIONS, Republic Act No. 3836 is
hereby declared null and void, in so far as it refers to the retirement of Members
of Congress and the elected officials thereof, as being unconstitutional. The
restraining order issued in our resolution on December 6, 1965 is hereby made
permanent. No costs.
PHILCONSA v. Gimenez
G.R. No. 113105 August 19, 1994
Regala, J.
Facts:
Philippine Constitution Association, Inc (PHILCONSA) assails the validity of RA 3836
insofar as the same allows retirement gratuity and commutation of vacation and sick leave to
Senators and Representatives, and to the elective officials of both Houses (of Congress). The
provision on retirement gratuity is an attempt to circumvent the Constitutional ban on increase of
salaries of the members of Congress during their term of office, contrary to the provisions of Article
VI, Section 14 of the Constitution. The same provision constitutes selfish class legislation because
it allows members and officers of Congress to retire after twelve (12) years of service and gives
them a gratuity equivalent to one year salary for every four years of service, which is not refundable
in case of reinstatement or re election of the retiree, while all other officers and employees of the
government can retire only after at least twenty (20) years of service and are given a gratuity which
is only equivalent to one month salary for every year of service, which, in any case, cannot exceed
24 months. The provision on vacation and sick leave, commutable at the highest rate received,
insofar as members of Congress are concerned, is another attempt of the legislator to further
increase their compensation in violation of the Constitution.
The Solicitor General counter-argued alleging that the grant of retirement or pension benefits
under Republic Act No. 3836 to the officers objected to by the petitioner does not constitute
forbidden compensation within the meaning of Section 14 of Article VI of the Philippine
Constitution. The law in question does not constitute class legislation. The payment of commutable

vacation and sick leave benefits under the said Act is merely in the nature of a basis for computing
the gratuity due each retiring member and, therefore, is not an indirect scheme to increase their
salary.
Issue:
whether Republic Act 3836 violates Section 14, Article VI, of the Constitution which reads
as follows:
The senators and the Members of the House of Representatives shall, unless otherwise provided by
law, receive an annual compensation of seven thousand two hundred pesos each, including per
diems and other emoluments or allowances, and exclusive only of travelling expenses to and from
their respective districts in the case of Members of the House of Representative and to and from
their places of residence in the case of Senators, when attending sessions of the Congress. No
increase in said compensation shall take effect until after the expiration of the full term of all the
Members of the Senate and of the House of Representatives approving such increase. Until
otherwise provided by law, the President of the Senate and the Speaker of the House of
Representatives shall each receive an annual compensation of sixteen thousand pesos.
Held:
Yes. When the Constitutional Convention first determined the compensation for the
Members of Congress, the amount fixed by it was only P5,000.00 per annum but it embodies a
special proviso which reads as follows: No increase in said compensation shall take effect until after
the expiration of the full term of all the members of the National Assembly elected subsequent to
approval of such increase. In other words, under the original constitutional provision regarding the
power of the National Assembly to increase the salaries of its members, no increase would take
effect until after the expiration of the full term of the members of the Assembly elected subsequent to
the approval of such increase.
The Constitutional provision in the aforementioned Section 14, Article VI, includes in the term
compensation other emoluments. This is the pivotal point on this fundamental question as to
whether the retirement benefit as provided for in Republic Act 3836 fall within the purview of the term
other emoluments.
Emolument is defined as the profit arising from office or employment; that which is received
as compensation for services or which is annexed to the possession of an office, as salary, fees and
perquisites.

It is evident that retirement benefit is a form or another species of emolument, because it is a


part of compensation for services of one possessing any office.
Republic Act 3836 provides for an increase in the emoluments of Senators and Members of
the House of Representatives, to take effect upon the approval of said Act, which was on June 22,
1963. Retirement benefits were immediately available thereunder, without awaiting the expiration of
the full term of all the Members of the Senate and the House of Representatives approving such
increase. Such provision clearly runs counter to the prohibition in Article VI, Section 14 of the
Constitution. RA 3836 is therefore unconstitutional.

Republic of the Philippines


SUPREME COURT
Manila
EN BANC
G.R. No. L-75697 June 18, 1987
VALENTIN TIO doing business under the name and style of OMI
ENTERPRISES, petitioner,
vs.
VIDEOGRAM REGULATORY BOARD, MINISTER OF FINANCE, METRO
MANILA COMMISSION, CITY MAYOR and CITY TREASURER OF
MANILA, respondents.
Nelson Y. Ng for petitioner.
The City Legal Officer for respondents City Mayor and City Treasurer.

MELENCIO-HERRERA, J.:
This petition was filed on September 1, 1986 by petitioner on his own behalf and
purportedly on behalf of other videogram operators adversely affected. It assails
the constitutionality of Presidential Decree No. 1987 entitled "An Act Creating the
Videogram Regulatory Board" with broad powers to regulate and supervise the
videogram industry (hereinafter briefly referred to as the BOARD). The Decree
was promulgated on October 5, 1985 and took effect on April 10, 1986, fifteen
(15) days after completion of its publication in the Official Gazette.

On November 5, 1985, a month after the promulgation of the abovementioned


decree, Presidential Decree No. 1994 amended the National Internal Revenue
Code providing, inter alia:
SEC. 134. Video Tapes. There shall be collected on each
processed video-tape cassette, ready for playback, regardless of
length, an annual tax of five pesos; Provided, That locally
manufactured or imported blank video tapes shall be subject to sales
tax.
On October 23, 1986, the Greater Manila Theaters Association, Integrated Movie
Producers, Importers and Distributors Association of the Philippines, and
Philippine Motion Pictures Producers Association, hereinafter collectively referred
to as the Intervenors, were permitted by the Court to intervene in the case, over
petitioner's opposition, upon the allegations that intervention was necessary for
the complete protection of their rights and that their "survival and very existence
is threatened by the unregulated proliferation of film piracy." The Intervenors
were thereafter allowed to file their Comment in Intervention.
The rationale behind the enactment of the DECREE, is set out in its preambular
clauses as follows:
1. WHEREAS, the proliferation and unregulated circulation of
videograms including, among others, videotapes, discs, cassettes or
any technical improvement or variation thereof, have greatly
prejudiced the operations of moviehouses and theaters, and have
caused a sharp decline in theatrical attendance by at least forty
percent (40%) and a tremendous drop in the collection of sales,
contractor's specific, amusement and other taxes, thereby resulting
in substantial losses estimated at P450 Million annually in
government revenues;
2. WHEREAS, videogram(s) establishments collectively earn around
P600 Million per annum from rentals, sales and disposition of
videograms, and such earnings have not been subjected to tax,
thereby depriving the Government of approximately P180 Million in
taxes each year;
3. WHEREAS, the unregulated activities of videogram
establishments have also affected the viability of the movie industry,
particularly the more than 1,200 movie houses and theaters
throughout the country, and occasioned industry-wide displacement

and unemployment due to the shutdown of numerous moviehouses


and theaters;
4. "WHEREAS, in order to ensure national economic recovery, it is
imperative for the Government to create an environment conducive
to growth and development of all business industries, including the
movie industry which has an accumulated investment of about P3
Billion;
5. WHEREAS, proper taxation of the activities of videogram
establishments will not only alleviate the dire financial condition of
the movie industry upon which more than 75,000 families and
500,000 workers depend for their livelihood, but also provide an
additional source of revenue for the Government, and at the same
time rationalize the heretofore uncontrolled distribution of
videograms;
6. WHEREAS, the rampant and unregulated showing of obscene
videogram features constitutes a clear and present danger to the
moral and spiritual well-being of the youth, and impairs the mandate
of the Constitution for the State to support the rearing of the youth
for civic efficiency and the development of moral character and
promote their physical, intellectual, and social well-being;
7. WHEREAS, civic-minded citizens and groups have called for
remedial measures to curb these blatant malpractices which have
flaunted our censorship and copyright laws;
8. WHEREAS, in the face of these grave emergencies corroding the
moral values of the people and betraying the national economic
recovery program, bold emergency measures must be adopted with
dispatch; ... (Numbering of paragraphs supplied).
Petitioner's attack on the constitutionality of the DECREE rests on the following
grounds:
1. Section 10 thereof, which imposes a tax of 30% on the gross
receipts payable to the local government is a RIDER and the same
is not germane to the subject matter thereof;
2. The tax imposed is harsh, confiscatory, oppressive and/or in
unlawful restraint of trade in violation of the due process clause of
the Constitution;

3. There is no factual nor legal basis for the exercise by the


President of the vast powers conferred upon him by Amendment No.
6;
4. There is undue delegation of power and authority;
5. The Decree is an ex-post facto law; and
6. There is over regulation of the video industry as if it were a
nuisance, which it is not.
We shall consider the foregoing objections in seriatim.
1. The Constitutional requirement that "every bill shall embrace only one subject
which shall be expressed in the title thereof" 1 is sufficiently complied with if the title be comprehensive
enough to include the general purpose which a statute seeks to achieve. It is not necessary that the title express each and every end that the
statute wishes to accomplish. The requirement is satisfied if all the parts of the statute are related, and are germane to the subject matter
2

An act having a single


general subject, indicated in the title, may contain any number of provisions, no matter how diverse they
may be, so long as they are not inconsistent with or foreign to the general subject, and may be
considered in furtherance of such subject by providing for the method and means of carrying out the
3
general object." The rule also is that the constitutional requirement as to the title of a bill should not be
4
so narrowly construed as to cripple or impede the power of legislation. It should be given practical rather
5
than technical construction.
expressed in the title, or as long as they are not inconsistent with or foreign to the general subject and title.

Tested by the foregoing criteria, petitioner's contention that the tax provision of
the DECREE is a rider is without merit. That section reads, inter alia:
Section 10. Tax on Sale, Lease or Disposition of Videograms.
Notwithstanding any provision of law to the contrary, the province
shall collect a tax of thirty percent (30%) of the purchase price or
rental rate, as the case may be, for every sale, lease or disposition
of a videogram containing a reproduction of any motion picture or
audiovisual program. Fifty percent (50%) of the proceeds of the tax
collected shall accrue to the province, and the other fifty percent
(50%) shall acrrue to the municipality where the tax is collected;
PROVIDED, That in Metropolitan Manila, the tax shall be shared
equally by the City/Municipality and the Metropolitan Manila
Commission.
xxx xxx xxx
The foregoing provision is allied and germane to, and is reasonably necessary
for the accomplishment of, the general object of the DECREE, which is the
regulation of the video industry through the Videogram Regulatory Board as
expressed in its title. The tax provision is not inconsistent with, nor foreign to that

general subject and title. As a tool for regulation 6 it is simply one of the regulatory and
control mechanisms scattered throughout the DECREE. The express purpose of the DECREE to include
taxation of the video industry in order to regulate and rationalize the heretofore uncontrolled distribution of
videograms is evident from Preambles 2 and 5, supra. Those preambles explain the motives of the
lawmaker in presenting the measure. The title of the DECREE, which is the creation of the Videogram
Regulatory Board, is comprehensive enough to include the purposes expressed in its Preamble and
reasonably covers all its provisions. It is unnecessary to express all those objectives in the title or that the
7
latter be an index to the body of the DECREE.

2. Petitioner also submits that the thirty percent (30%) tax imposed is harsh and
oppressive, confiscatory, and in restraint of trade. However, it is beyond serious
question that a tax does not cease to be valid merely because it regulates,
discourages, or even definitely deters the activities taxed. 8 The power to impose taxes
is one so unlimited in force and so searching in extent, that the courts scarcely venture to declare that it is
subject to any restrictions whatever, except such as rest in the discretion of the authority which exercises
9
it. In imposing a tax, the legislature acts upon its constituents. This is, in general, a sufficient security
against erroneous and oppressive taxation. 10

The tax imposed by the DECREE is not only a regulatory but also a revenue
measure prompted by the realization that earnings of videogram establishments
of around P600 million per annum have not been subjected to tax, thereby
depriving the Government of an additional source of revenue. It is an end-user
tax, imposed on retailers for every videogram they make available for public
viewing. It is similar to the 30% amusement tax imposed or borne by the movie
industry which the theater-owners pay to the government, but which is passed on
to the entire cost of the admission ticket, thus shifting the tax burden on the
buying or the viewing public. It is a tax that is imposed uniformly on all videogram
operators.
The levy of the 30% tax is for a public purpose. It was imposed primarily to
answer the need for regulating the video industry, particularly because of the
rampant film piracy, the flagrant violation of intellectual property rights, and the
proliferation of pornographic video tapes. And while it was also an objective of
the DECREE to protect the movie industry, the tax remains a valid imposition.
The public purpose of a tax may legally exist even if the motive
which impelled the legislature to impose the tax was to favor one
industry over another. 11
It is inherent in the power to tax that a state be free to select the
subjects of taxation, and it has been repeatedly held that "inequities
which result from a singling out of one particular class for taxation or
exemption infringe no constitutional limitation". 12 Taxation has been made the
implement of the state's police power. 13

At bottom, the rate of tax is a matter better addressed to the taxing legislature.

3. Petitioner argues that there was no legal nor factual basis for the promulgation
of the DECREE by the former President under Amendment No. 6 of the 1973
Constitution providing that "whenever in the judgment of the President ... , there
exists a grave emergency or a threat or imminence thereof, or whenever the
interim Batasang Pambansa or the regular National Assembly fails or is unable to
act adequately on any matter for any reason that in his judgment requires
immediate action, he may, in order to meet the exigency, issue the necessary
decrees, orders, or letters of instructions, which shall form part of the law of the
land."
In refutation, the Intervenors and the Solicitor General's Office aver that the 8th
"whereas" clause sufficiently summarizes the justification in that grave
emergencies corroding the moral values of the people and betraying the national
economic recovery program necessitated bold emergency measures to be
adopted with dispatch. Whatever the reasons "in the judgment" of the then
President, considering that the issue of the validity of the exercise of legislative
power under the said Amendment still pends resolution in several other cases,
we reserve resolution of the question raised at the proper time.
4. Neither can it be successfully argued that the DECREE contains an undue
delegation of legislative power. The grant in Section 11 of the DECREE of
authority to the BOARD to "solicit the direct assistance of other agencies and
units of the government and deputize, for a fixed and limited period, the heads or
personnel of such agencies and units to perform enforcement functions for the
Board" is not a delegation of the power to legislate but merely a conferment of
authority or discretion as to its execution, enforcement, and implementation. "The
true distinction is between the delegation of power to make the law, which
necessarily involves a discretion as to what it shall be, and conferring authority or
discretion as to its execution to be exercised under and in pursuance of the law.
The first cannot be done; to the latter, no valid objection can be made." 14 Besides, in
the very language of the decree, the authority of the BOARD to solicit such assistance is for a "fixed and limited period" with the deputized
agencies concerned being "subject to the direction and control of the BOARD." That the grant of such authority might be the source of graft
and corruption would not stigmatize the DECREE as unconstitutional. Should the eventuality occur, the aggrieved parties will not be without
adequate remedy in law.

5. The DECREE is not violative of the ex post facto principle. An ex post


facto law is, among other categories, one which "alters the legal rules of
evidence, and authorizes conviction upon less or different testimony than the law
required at the time of the commission of the offense." It is petitioner's position
that Section 15 of the DECREE in providing that:
All videogram establishments in the Philippines are hereby given a
period of forty-five (45) days after the effectivity of this Decree within
which to register with and secure a permit from the BOARD to
engage in the videogram business and to register with the BOARD

all their inventories of videograms, including videotapes, discs,


cassettes or other technical improvements or variations thereof,
before they could be sold, leased, or otherwise disposed of.
Thereafter any videogram found in the possession of any person
engaged in the videogram business without the required proof of
registration by the BOARD, shall be prima facie evidence of violation
of the Decree, whether the possession of such videogram be for
private showing and/or public exhibition.
raises immediately a prima facie evidence of violation of the DECREE when the
required proof of registration of any videogram cannot be presented and thus
partakes of the nature of an ex post facto law.
The argument is untenable. As this Court held in the recent case of Vallarta vs.
Court of Appeals, et al. 15
... it is now well settled that "there is no constitutional objection to the
passage of a law providing that the presumption of innocence may
be overcome by a contrary presumption founded upon the
experience of human conduct, and enacting what evidence shall be
sufficient to overcome such presumption of innocence" (People vs.
Mingoa 92 Phil. 856 [1953] at 858-59, citing 1 COOLEY, A
TREATISE ON THE CONSTITUTIONAL LIMITATIONS, 639-641).
And the "legislature may enact that when certain facts have been
proved that they shall be prima facie evidence of the existence of the
guilt of the accused and shift the burden of proof provided there be a
rational connection between the facts proved and the ultimate facts
presumed so that the inference of the one from proof of the others is
not unreasonable and arbitrary because of lack of connection
between the two in common experience".16
Applied to the challenged provision, there is no question that there is a rational
connection between the fact proved, which is non-registration, and the ultimate
fact presumed which is violation of the DECREE, besides the fact that theprima
facie presumption of violation of the DECREE attaches only after a forty-five-day
period counted from its effectivity and is, therefore, neither retrospective in
character.
6. We do not share petitioner's fears that the video industry is being overregulated and being eased out of existence as if it were a nuisance. Being a
relatively new industry, the need for its regulation was apparent. While the
underlying objective of the DECREE is to protect the moribund movie industry,
there is no question that public welfare is at bottom of its enactment, considering

"the unfair competition posed by rampant film piracy; the erosion of the moral
fiber of the viewing public brought about by the availability of unclassified and
unreviewed video tapes containing pornographic films and films with brutally
violent sequences; and losses in government revenues due to the drop in
theatrical attendance, not to mention the fact that the activities of video
establishments are virtually untaxed since mere payment of Mayor's permit and
municipal license fees are required to engage in business. 17
The enactment of the Decree since April 10, 1986 has not brought about the
"demise" of the video industry. On the contrary, video establishments are seen to
have proliferated in many places notwithstanding the 30% tax imposed.
In the last analysis, what petitioner basically questions is the necessity, wisdom
and expediency of the DECREE. These considerations, however, are primarily
and exclusively a matter of legislative concern.
Only congressional power or competence, not the wisdom of the
action taken, may be the basis for declaring a statute invalid. This is
as it ought to be. The principle of separation of powers has in the
main wisely allocated the respective authority of each department
and confined its jurisdiction to such a sphere. There would then be
intrusion not allowable under the Constitution if on a matter left to
the discretion of a coordinate branch, the judiciary would substitute
its own. If there be adherence to the rule of law, as there ought to
be, the last offender should be courts of justice, to which rightly
litigants submit their controversy precisely to maintain unimpaired
the supremacy of legal norms and prescriptions. The attack on the
validity of the challenged provision likewise insofar as there may be
objections, even if valid and cogent on its wisdom cannot be
sustained. 18
In fine, petitioner has not overcome the presumption of validity which attaches to
a challenged statute. We find no clear violation of the Constitution which would
justify us in pronouncing Presidential Decree No. 1987 as unconstitutional and
void.
WHEREFORE, the instant Petition is hereby dismissed.
No costs.
SO ORDERED.
151 SCRA 208 Political Law The Embrace of Only One Subject by a Bill

Delegation of Power Delegation to Administrative Bodies


In 1985, Presidential Dedree No. 1987 entitled An Act Creating the Videogram
Regulatory Board was enacted which gave broad powers to the VRB to regulate
and supervise the videogram industry. The said law sought to minimize the
economic effects of piracy. There was a need to regulate the sale of videograms
as it has adverse effects to the movie industry. The proliferation of videograms
has significantly lessened the revenue being acquired from the movie industry,
and that such loss may be recovered if videograms are to be taxed. Section 10 of
the PD imposes a 30% tax on the gross receipts payable to the LGUs.
In 1986, Valentin Tio assailed the said PD as he averred that it is unconstitutional
on the following grounds:
1. Section 10 thereof, which imposed the 30% tax on gross receipts, is a rider
and is not germane to the subject matter of the law.
2. There is also undue delegation of legislative power to the VRB, an
administrative body, because the law allowed the VRB to deputize, upon its
discretion, other government agencies to assist the VRB in enforcing the said
PD.
ISSUE: Whether or not the Valentin Tios arguments are correct.
HELD: No.
1. The Constitutional requirement that every bill shall embrace only one subject
which shall be expressed in the title thereof is sufficiently complied with if the
title be comprehensive enough to include the general purpose which a statute
seeks to achieve. In the case at bar, the questioned provision is allied and
germane to, and is reasonably necessary for the accomplishment of, the general
object of the PD, which is the regulation of the video industry through the VRB as
expressed in its title. The tax provision is not inconsistent with, nor foreign to that
general subject and title. As a tool for regulation it is simply one of the regulatory
and control mechanisms scattered throughout the PD.
2. There is no undue delegation of legislative powers to the VRB. VRB is not
being tasked to legislate. What was conferred to the VRB was the authority or
discretion to seek assistance in the execution, enforcement, and implementation
of the law. Besides, in the very language of the decree, the authority of the
BOARD to solicit such assistance is for a fixed and limited period with the
deputized agencies concerned being subject to the direction and control of the
[VRB].

Republic of the Philippines


SUPREME COURT
Manila
EN BANC

G.R. No. 105371 November 11, 1993


THE PHILIPPINE JUDGES ASSOCIATION, duly rep. by its President,
BERNARDO P. ABESAMIS, Vice-President for Legal Affairs, MARIANO M.
UMALI, Director for Pasig, Makati, and Pasay, Metro Manila, ALFREDO C.
FLORES, and Chairman of the Committee on Legal Aid, JESUS G.
BERSAMIRA, Presiding Judges of the Regional Trial Court, Branch 85,
Quezon City and Branches 160, 167 and 166, Pasig, Metro Manila,
respectively: the NATIONAL CONFEDERATION OF THE JUDGES
ASSOCIATION OF THE PHILIPPINES, composed of the METROPOLITAN
TRIAL COURT JUDGES ASSOCIATION rep. by its President. REINATO
QUILALA of the MUNICIPAL TRIAL CIRCUIT COURT, Manila; THE
MUNICIPAL JUDGES LEAGUE OF THE PHILIPPINES rep. by its President,
TOMAS G. TALAVERA; by themselves and in behalf of all the Judges of the
Regional Trial and Shari'a Courts, Metropolitan Trial Courts and Municipal
Courts throughout the Country, petitioners,
vs.
HON. PETE PRADO, in his capacity as Secretary of the Department of
Transportation and Communications, JORGE V. SARMIENTO, in his
capacity as Postmaster General, and the PHILIPPINE POSTAL
CORP.,respondents.

CRUZ, J.:
The basic issue raised in this petition is the independence of the Judiciary. It is
asserted by the petitioners that this hallmark of republicanism is impaired by the
statute and circular they are here challenging. The Supreme Court is itself
affected by these measures and is thus an interested party that should ordinarily
not also be a judge at the same time. Under our system of government, however,
it cannot inhibit itself and must rule upon the challenge, because no other office
has the authority to do so. We shall therefore act upon this matter not with
officiousness but in the discharge of an unavoidable duty and, as always, with
detachment and fairness.

The main target of this petition is Section 35 of R.A. No. 7354 as implemented by
the Philippine Postal Corporation through its Circular No.
92-28. These measures withdraw the franking privilege from the Supreme Court,
the Court of Appeals, the Regional Trial Courts, the Metropolitan Trial Courts, the
Municipal Trial Courts, and the Land Registration Commission and its Registers
of Deeds, along with certain other government offices.
The petitioners are members of the lower courts who feel that their official
functions as judges will be prejudiced by the above-named measures. The
National Land Registration Authority has taken common cause with them insofar
as its own activities, such as sending of requisite notices in registration cases,
affect judicial proceedings. On its motion, it has been allowed to intervene.
The petition assails the constitutionality of R.A. No. 7354 on the grounds that: (1)
its title embraces more than one subject and does not express its purposes; (2) it
did not pass the required readings in both Houses of Congress and printed
copies of the bill in its final form were not distributed among the members before
its passage; and (3) it is discriminatory and encroaches on the independence of
the Judiciary.
We approach these issues with one important principle in mind, to wit, the
presumption of the constitutionality of statutes. The theory is that as the joint act
of the Legislature and the Executive, every statute is supposed to have first been
carefully studied and determined to be constitutional before it was finally enacted.
Hence, unless it is clearly shown that it is constitutionally flawed, the attack
against its validity must be rejected and the law itself upheld. To doubt is to
sustain.
I
We consider first the objection based on Article VI, Sec. 26(l), of the Constitution
providing that "Every bill passed by the Congress shall embrace only one subject
which shall be expressed in the title thereof."
The purposes of this rule are: (1) to prevent hodge-podge or "log-rolling"
legislation; (2) to prevent surprise or fraud upon the legislature by means of
provisions in bills of which the title gives no intimation, and which might therefore
be overlooked and carelessly and unintentionally adopted; and (3) to fairly
apprise the people, through such publication of legislative proceedings as is
usually made, of the subject of legislation that is being considered, in order that
they may have opportunity of being heard thereon, by petition or otherwise, if
they shall so desire. 1

It is the submission of the petitioners that Section 35 of R.A. No. 7354 which
withdrew the franking privilege from the Judiciary is not expressed in the title of
the law, nor does it reflect its purposes.
R.A. No. 7354 is entitled "An Act Creating the Philippine Postal Corporation,
Defining its Powers, Functions and Responsibilities, Providing for Regulation of
the Industry and for Other Purposes Connected Therewith."
The objectives of the law are enumerated in Section 3, which provides:
The State shall pursue the following objectives of a nationwide
postal system:
a) to enable the economical and speedy transfer of mail and other
postal matters, from sender to addressee, with full recognition of
their privacy or confidentiality;
b) to promote international interchange, cooperation and
understanding through the unhampered flow or exchange of postal
matters between nations;
c) to cause or effect a wide range of postal services to cater to
different users and changing needs, including but not limited to,
philately, transfer of monies and valuables, and the like;
d) to ensure that sufficient revenues are generated by and within the
industry to finance the overall cost of providing the varied range of
postal delivery and messengerial services as well as the expansion
and continuous upgrading of service standards by the same.
Sec. 35 of R.A. No. 7354, which is the principal target of the petition, reads as
follows:
Sec. 35. Repealing Clause. All acts, decrees, orders, executive
orders, instructions, rules and regulations or parts thereof
inconsistent with the provisions of this Act are repealed or modified
accordingly.
All franking privileges authorized by law are hereby repealed, except
those provided for under Commonwealth Act No. 265, Republic Acts
Numbered 69, 180, 1414, 2087 and 5059. The Corporation may
continue the franking privilege under Circular No. 35 dated October

24, 1977 and that of the Vice President, under such arrangements
and conditions as may obviate abuse or unauthorized use thereof.
The petitioners' contention is untenable. We do not agree that the title of the
challenged act violates the Constitution.
The title of the bill is not required to be an index to the body of the act, or to be as
comprehensive as to cover every single detail of the measure. It has been held
that if the title fairly indicates the general subject, and reasonably covers all the
provisions of the act, and is not calculated to mislead the legislature or the
people, there is sufficient compliance with the constitutional requirement. 2
To require every end and means necessary for the accomplishment of the
general objectives of the statute to be expressed in its title would not only be
unreasonable but would actually render legislation impossible. 3 As has been correctly
explained:

The details of a legislative act need not be specifically stated in its


title, but matter germane to the subject as expressed in the title, and
adopted to the accomplishment of the object in view, may properly
be included in the act. Thus, it is proper to create in the same act the
machinery by which the act is to be enforced, to prescribe the
penalties for its infraction, and to remove obstacles in the way of its
execution. If such matters are properly connected with the subject as
expressed in the title, it is unnecessary that they should also have
special mention in the title (Southern Pac. Co. v. Bartine, 170 Fed.
725).
This is particularly true of the repealing clause, on which Cooley writes: "The
repeal of a statute on a given subject is properly connected with the subject
matter of a new statute on the same subject; and therefore a repealing section in
the new statute is valid, notwithstanding that the title is silent on the subject. It
would be difficult to conceive of a matter more germane to an act and to the
object to be accomplished thereby than the repeal of previous legislations
connected therewith." 4
The reason is that where a statute repeals a former law, such repeal is the effect
and not the subject of the statute; and it is the subject, not the effect of a law,
which is required to be briefly expressed in its title. 5 As observed in one case, 6 if the title
of an act embraces only one subject, we apprehend it was never claimed that every other act which
repeals it or alters by implication must be mentioned in the title of the new act. Any such rule would be
neither within the reason of the Constitution, nor practicable.

We are convinced that the withdrawal of the franking privilege from some
agencies is germane to the accomplishment of the principal objective of R.A. No.
7354, which is the creation of a more efficient and effective postal service
system. Our ruling is that, by virtue of its nature as a repealing clause, Section 35
did not have to be expressly included in the title of the said law.
II
The petitioners maintain that the second paragraph of Sec. 35 covering the
repeal of the franking privilege from the petitioners and this Court under E.O.
207, PD 1882 and PD 26 was not included in the original version of Senate Bill
No. 720 or House Bill No. 4200. As this paragraph appeared only in the
Conference Committee Report, its addition, violates Article VI, Sec. 26(2) of the
Constitution, reading as follows:
(2) No bill passed by either House shall become a law unless it has
passed three readings on separate days, and printed copies thereof
in its final form have been distributed to its Members three days
before its passage, except when the President certifies to the
necessity of its immediate enactment to meet a public calamity or
emergency. Upon the last reading of a bill, no amendment thereto
shall be allowed, and the vote thereon shall be taken immediately
thereafter, and the yeas and nays entered in the Journal.
The petitioners also invoke Sec. 74 of the Rules of the House of
Representatives, requiring that amendment to any bill when the House and the
Senate shall have differences thereon may be settled by a conference committee
of both chambers. They stress that Sec. 35 was never a subject of any
disagreement between both Houses and so the second paragraph could not
have been validly added as an amendment.
These argument are unacceptable.
While it is true that a conference committee is the mechanism for compromising
differences between the Senate and the House, it is not limited in its jurisdiction
to this question. Its broader function is described thus:
A conference committee may, deal generally with the subject matter
or it may be limited to resolving the precise differences between the
two houses. Even where the conference committee is not by rule
limited in its jurisdiction, legislative custom severely limits the
freedom with which new subject matter can be inserted into the
conference bill. But occasionally a conference committee produces

unexpected results, results beyond its mandate, These excursions


occur even where the rules impose strict limitations on conference
committee jurisdiction. This is symptomatic of the authoritarian
power of conference committee (Davies, Legislative Law and
Process: In a Nutshell, 1986 Ed., p.81).
It is a matter of record that the conference Committee Report on the bill in
question was returned to and duly approved by both the Senate and the House
of Representatives. Thereafter, the bill was enrolled with its certification by
Senate President Neptali A. Gonzales and Speaker Ramon V. Mitra of the House
of Representatives as having been duly passed by both Houses of Congress. It
was then presented to and approved by President Corazon C. Aquino on April 3,
1992.
Under the doctrine of separation powers, the Court may not inquire beyond the
certification of the approval of a bill from the presiding officers of
Congress. Casco Philippine Chemical Co. v. Gimenez 7 laid down the rule that the
enrolled bill, is conclusive upon the Judiciary (except in matters that have to be entered in the journals like
the yeas and nayson the final reading of the
8
bill). The journals are themselves also binding on the Supreme Court, as we held in the old (but still
9
valid) case of U.S. vs. Pons, where we explained the reason thus:

To inquire into the veracity of the journals of the Philippine


legislature when they are, as we have said, clear and explicit, would
be to violate both the, letter and spirit of the organic laws by which
the Philippine Government was brought into existence, to invade a
coordinate and independent department of the Government, and to
interfere with the legitimate powers and functions, of the Legislature.
Applying these principles, we shall decline to look into the petitioners' charges
that an amendment was made upon the last reading of the bill that eventually
became R.A. No. 7354 and that copies thereof in its final form were not
distributed among the members of each House. Both the enrolled bill and the
legislative journals certify that the measure was duly enacted i.e., in accordance
with Article VI, Sec. 26(2) of the Constitution. We are bound by such official
assurances from a coordinate department of the government, to which we owe,
at the very least, a becoming courtesy.
III
The third and most serious challenge of the petitioners is based on the equal
protection clause.

It is alleged that R.A. No. 7354 is discriminatory because while withdrawing the
franking privilege from the Judiciary, it retains the same for the President of the
Philippines, the Vice President of the Philippines; Senators and Members of the
House of Representatives, the Commission on Elections; former Presidents of
the Philippines; the National Census and Statistics Office; and the general public
in the filing of complaints against public offices and officers. 10
The respondents counter that there is no discrimination because the law is based
on a valid classification in accordance with the equal protection clause. In fact,
the franking privilege has been withdrawn not only from the Judiciary but also the
Office of Adult Education, the Institute of National Language; the
Telecommunications Office; the Philippine Deposit Insurance Corporation; the
National Historical Commission; the Armed Forces of the Philippines; the Armed
Forces of the Philippines Ladies Steering Committee; the City and Provincial
Prosecutors; the Tanodbayan (Office of Special Prosecutor); the Kabataang
Barangay; the Commission on the Filipino Language; the Provincial and City
Assessors; and the National Council for the Welfare of Disabled Persons. 11
The equal protection of the laws is embraced in the concept of due process, as
every unfair discrimination offends the requirements of justice and fair play. It has
nonetheless been embodied in a separate clause in Article III Sec. 1., of the
Constitution to provide for a more, specific guaranty against any form of undue
favoritism or hostility from the government. Arbitrariness in general may be
challenged on the basis of the due process clause. But if the particular act
assailed partakes of an unwarranted partiality or prejudice, the sharper weapon
to cut it down is the equal protection clause.
According to a long line of decisions, equal protection simply requires that all
persons or things similarly situated should be treated alike, both as to rights
conferred and responsibilities imposed, 12 Similar subjects, in other words, should not be
treated differently, so as to give undue favor to some and unjustly discriminate against others.

The equal protection clause does not require the universal application of the laws
on all persons or things without distinction. This might in fact sometimes result in
unequal protection, as where, for example, a law prohibiting mature books to all
persons, regardless of age, would benefit the morals of the youth but violate the
liberty of adults. What the clause requires is equality among equals as
determined according to a valid classification. By classification is meant the
grouping of persons or things similar to each other in certain particulars and
different from all others in these same particulars. 13
What is the reason for the grant of the franking privilege in the first place? Is the
franking privilege extended to the President of the Philippines or the Commission

on Elections or to former Presidents of the Philippines purely as acourtesy from


the lawmaking body? Is it offered because of the importance or status of the
grantee or because of itsneed for the privilege? Or have the grantees been
chosen pell-mell, as it were, without any basis at all for the selection?
We reject outright the last conjecture as there is no doubt that the statute as a
whole was carefully deliberated upon, by the political departments before it was
finally enacted. There is reason to suspect, however, that not enough care or
attention was given to its repealing clause, resulting in the unwitting withdrawal of
the franking privilege from the Judiciary.
We also do not believe that the basis of the classification was mere courtesy, for
it is unimaginable that the political departments would have intended this serious
slight to the Judiciary as the third of the major and equal departments the
government. The same observations are made if the importance or status of the
grantee was the criterion used for the extension of the franking privilege, which is
enjoyed by the National Census and Statistics Office and even some private
individuals but not the courts of justice.
In our view, the only acceptable reason for the grant of the franking privilege was
the perceived need of the grantee for the accommodation, which would justify a
waiver of substantial revenue by the Corporation in the interest of providing for a
smoother flow of communication between the government and the people.
Assuming that basis, we cannot understand why, of all the departments of the
government, it is the Judiciary, that has been denied the franking privilege. There
is no question that if there is any major branch of the government that needs the
privilege, it is the Judicial Department, as the respondents themselves point out.
Curiously, the respondents would justify the distinction on the basis precisely of
this need and, on this basis, deny the Judiciary the franking privilege while
extending it to others less deserving.
In their Comment, the respondents point out that available data from the Postal
Service Office show that from January 1988 to June 1992, the total volume of
frank mails amounted to P90,424,175.00. Of this amount, frank mails from the
Judiciary and other agencies whose functions include the service of judicial
processes, such as the intervenor, the Department of Justice and the Office of
the Ombudsman, amounted to P86,481,759. Frank mails coming fromthe
Judiciary amounted to P73,574,864.00, and those coming from the petitioners
reached the total amount of P60,991,431.00. The respondents' conclusion is that
because of this considerable volume of mail from the Judiciary, the franking
privilege must be withdrawn from it.

The argument is self-defeating. The respondents are in effect saying that the
franking privilege should be extended only to those who do not need it very
much, if at all, (like the widows of former Presidents) but not to those who need it
badly (especially the courts of justice). It is like saying that a person may be
allowed cosmetic surgery although it is not really necessary but not an operation
that can save his life.
If the problem of the respondents is the loss of revenues from the franking
privilege, the remedy, it seems to us, is to withdraw it altogether from all agencies
of government, including those who do not need it. The problem is not solved by
retaining it for some and withdrawing it from others, especially where there is no
substantial distinction between those favored, which may or may not need it at
all, and the Judiciary, which definitely needs it. The problem is not solved by
violating the Constitution.
In lumping the Judiciary with the other offices from which the franking privilege
has been withdrawn, Section 35 has placed the courts of justice in a category to
which it does not belong. If it recognizes the need of the President of the
Philippines and the members of Congress for the franking privilege, there is no
reason why it should not recognize a similar and in fact greater need on the part
of the Judiciary for such privilege. While we may appreciate the withdrawal of the
franking privilege from the Armed Forces of the Philippines Ladies Steering
Committee, we fail to understand why the Supreme Court should be similarly
treated as that Committee. And while we may concede the need of the National
Census and Statistics Office for the franking privilege, we are intrigued that a
similar if not greater need is not recognized in the courts of justice.
(On second thought, there does not seem to be any justifiable need for
withdrawing the privilege from the Armed Forces of the Philippines Ladies
Steering Committee, which, like former Presidents of the Philippines or their
widows, does not send as much frank mail as the Judiciary.)
It is worth observing that the Philippine Postal Corporation, as a governmentcontrolled corporation, was created and is expected to operate for the purpose of
promoting the public service. While it may have been established primarily for
private gain, it cannot excuse itself from performing certain functions for the
benefit of the public in exchange for the franchise extended to it by the
government and the many advantages it enjoys under its charter. 14Among the
services it should be prepared to extend is free carriage of mail for certain offices of the government that
need the franking privilege in the discharge of their own public functions.

We also note that under Section 9 of the law, the Corporation is capitalized at
P10 billion pesos, 55% of which is supplied by the Government, and that it
derives substantial revenues from the sources enumerated in Section 10, on top

of the exemptions it enjoys. It is not likely that the retention of the franking
privilege of the Judiciary will cripple the Corporation.
At this time when the Judiciary is being faulted for the delay in the administration
of justice, the withdrawal from it of the franking privilege can only further deepen
this serious problem. The volume of judicial mail, as emphasized by the
respondents themselves, should stress the dependence of the courts of justice
on the postal service for communicating with lawyers and litigants as part of the
judicial process. The Judiciary has the lowest appropriation in the national budget
compared to the Legislative and Executive Departments; of the P309 billion
budgeted for 1993, only .84%, or less than 1%, is alloted for the judiciary. It
should not be hard to imagine the increased difficulties of our courts if they have
to affix a purchased stamp to every process they send in the discharge of their
judicial functions.
We are unable to agree with the respondents that Section 35 of R.A. No. 7354
represents a valid exercise of discretion by the Legislature under the police
power. On the contrary, we find its repealing clause to be a discriminatory
provision that denies the Judiciary the equal protection of the laws guaranteed for
all persons or things similarly situated. The distinction made by the law is
superficial. It is not based on substantial distinctions that make real differences
between the Judiciary and the grantees of the franking privilege.
This is not a question of wisdom or power into which the Judiciary may not
intrude. It is a matter of arbitrariness that this Court has the duty and power to
correct.
IV
In sum, we sustain R.A. No. 7354 against the attack that its subject is not
expressed in its title and that it was not passed in accordance with the prescribed
procedure. However, we annul Section 35 of the law as violative of Article 3, Sec.
1, of the Constitution providing that no person shall "be deprived of the equal
protection of laws."
We arrive at these conclusions with a full awareness of the criticism it is certain
to provoke. While ruling against the discrimination in this case, we may ourselves
be accused of similar discrimination through the exercise of our ultimate power in
our own favor. This is inevitable. Criticism of judicial conduct, however
undeserved, is a fact of life in the political system that we are prepared to
accept.. As judges, we cannot debate with our detractors. We can only decide
the cases before us as law imposes on us the duty to be fair and our own
conscience gives us the light to be right.

ACCORDINGLY, the petition is partially GRANTED and Section 35 of R.A. No.


7354 is declared UNCONSTITUTIONAL. Circular No. 92-28 is SET ASIDE
insofar as it withdraws the franking privilege from the Supreme Court, the Court
of Appeals, the Regional trail Courts, the Municipal trial Courts, and the National
Land Registration Authority and its Register of Deeds to all of which offices the
said privilege shall be RESTORED. The temporary restraining order dated June
2, 1992, is made permanent.
SO ORDERED.
G.R. No. 105371 November 11, 1993
THE PHILIPPINE JUDGES ASSOCIATION, duly rep. by its President, BERNARDO P. ABESAMIS,
Vice-President for Legal Affairs, MARIANO M. UMALI, Director for Pasig, Makati, and Pasay,
Metro Manila, ALFREDO C. FLORES, and Chairman of the Committee on Legal Aid, JESUS G.
BERSAMIRA, Presiding Judges of the Regional Trial Court, Branch 85, Quezon City and
Branches 160, 167 and 166, Pasig, Metro Manila, respectively: the NATIONAL
CONFEDERATION OF THE JUDGES ASSOCIATION OF THE PHILIPPINES, composed of the
METROPOLITAN TRIAL COURT JUDGES ASSOCIATION rep. by its President. REINATO QUILALA
of the MUNICIPAL TRIAL CIRCUIT COURT, Manila; THE MUNICIPAL JUDGES LEAGUE OF THE
PHILIPPINES rep. by its President, TOMAS G. TALAVERA; by themselves and in behalf of all the
Judges of the Regional Trial and Shari'a Courts, Metropolitan Trial Courts and Municipal
Courts throughout the Country, petitioners,
vs.
HON. PETE PRADO, in his capacity as Secretary of the Department of Transportation and
Communications, JORGE V. SARMIENTO, in his capacity as Postmaster General, and the
PHILIPPINE POSTAL CORP., respondents.

Facts: Petitioners, members of the lower courts, are assailing the constitutionality of Sec 35 of
RA 7354 due to, inter alia, its being discriminatory because of withdrawing the franking
privilege from the Judiciary but retaining said privilege for the President, the VP, members of
Congress, the Comelec, former Presidents, and the National Census and Statistics Office.
Respondents counter that there is no discrimination as the franking privilege has also been
withdrawn from the Office of Adult Education, the Institute of National Language, the
Telecommunications Office, the Philippine Deposit Insurance Corporation, the National
Historical Commission, the AFP, the AFP Ladies Steering Committee, the City and Provincial
Prosecutors, the Tanodbayan (Office of the Special Prosecutor), the Kabataang Baranggay, the
Commission on the Filipino Language, the Provincial and City Assessors, and the National
Council for the Welfare of Disabled Persons.

Issue: Constitutionality of Sec. 35of RA 7354

Held: Hereby declared unconstitutional.


The EPC is embraced in the concept of due process, as every unfair discrimination offends the
requirements of justice and fair play. According to a long line of decisions, equal protection
simply requires that all persons or things similarly situated should be treated alike, both as to
rights conferred and responsibilities imposed, 12 Similar subjects, in other words, should not
be treated differently, so as to give undue favor to some and unjustly discriminate against
others. The equal protection clause does not require the universal application of the laws on all
persons or things without distinction. In lumping the Judiciary with the other offices from which
the franking privilege has been withdrawn, Section 35 has placed the courts of justice in a
category to which it does not belong. If it recognizes the need of the President of the
Philippines and the members of Congress for the franking privilege, there is no reason why it
should not recognize a similar and in fact greater need on the part of the Judiciary for such
privilege. While we may appreciate the withdrawal of the franking privilege from the Armed
Forces of the Philippines Ladies Steering Committee, we fail to understand why the Supreme
Court should be similarly treated as that Committee.

In the SCs view, the only acceptable reason for the grant of the franking privilege was the
perceived need of the grantee for the accommodation, which would justify a waiver of
substantial revenue by the Corporation in the interest of providing for a smoother flow of
communication between the government and the people. If the problem of the respondents is
the loss of revenues from the franking privilege, the remedy, it seems to us, is to withdraw it
altogether from all agencies of government, including those who do not need it. The problem is
not solved by retaining it for some and withdrawing it from others, especially where there is no
substantial distinction between those favored, which may or may not need it at all, and the
Judiciary, which definitely needs it. The problem is not solved by violating the Constitution.

Republic of the Philippines


SUPREME COURT
Manila
EN BANC

G.R. No. L-114783 December 8, 1994


ROBERT V. TOBIAS, RAMON M. GUZMAN, TERRY T. LIM, GREGORIO D. GABRIEL, and
ROBERTO R. TOBIAS, JR. petitioners,
vs.
HON. CITY MAYOR BENJAMIN S. ABALOS, CITY TREASURER WILLIAM MARCELINO, and
THE SANGGUNIANG PANLUNGSOD, all of the City of Mandaluyong, Metro
Manila, respondents.
Estrella, Bautista & Associates for petitioners.

BIDIN, J.:
Invoking their rights as taxpayers and as residents of Mandaluyong, herein petitioners assail the
constitutionality of Republic Act No. 7675, otherwise known as "An Act Converting the Municipality of
Mandaluyong into a Highly Urbanized City to be known as the City of Mandaluyong."
Prior to the enactment of the assailed statute, the municipalities of Mandaluyong and San Juan
belonged to only one legislative district. Hon. Ronaldo Zamora, the incumbent congressional
representative of this legislative district, sponsored the bill which eventually became R.A. No. 7675.
President Ramos signed R.A. No. 7675 into law on February 9, 1994.
Pursuant to the Local Government Code of 1991, a plebiscite was held on April 10, 1994. The
people of Mandaluyong were asked whether they approved of the conversion of the Municipality of
Mandaluyong into a highly urbanized city as provided under R.A. No. 7675. The turnout at the
plebiscite was only 14.41% of the voting population. Nevertheless, 18,621 voted "yes" whereas
7,911 voted "no." By virtue of these results, R.A. No. 7675 was deemed ratified and in effect.
Petitioners now come before this Court, contending that R.A. No. 7675, specifically Article VIII,
Section 49 thereof, is unconstitutional for being violative of three specific provisions of the
Constitution.
Article VIII, Section 49 of R.A. No. 7675 provides:
As a highly-urbanized city, the City of Mandaluyong shall have its own legislative
district with the first representative to be elected in the next national elections after
the passage of this Act. The remainder of the former legislative district of San
Juan/Mandaluyong shall become the new legislative district of San Juan with its first
representative to be elected at the same election.
Petitioner's first objection to the aforequoted provision of R.A. No. 7675 is that it contravenes the
"one subject-one bill" rule, as enunciated in Article VI, Section 26(1) of the Constitution, to wit:
Sec. 26(1). Every bill passed by the Congress shall embrace only one subject which
shall be expressed in the title thereof.
Petitioners allege that the inclusion of the assailed Section 49 in the subject law resulted in the latter
embracing two principal subjects, namely: (1) the conversion of Mandaluyong into a highly urbanized
city; and (2) the division of the congressional district of San Juan/Mandaluyong into two separate
districts.
Petitioners contend that the second aforestated subject is not germane to the subject matter of R.A.
No. 7675 since the said law treats of the conversion of Mandaluyong into a highly urbanized city, as
expressed in the title of the law. Therefore, since Section 49 treats of a subject distinct from that
stated in the title of the law, the "one subject-one bill" rule has not been complied with.
Petitioners' second and third objections involve Article VI, Sections 5(1) and (4) of the Constitution,
which provide, to wit:

Sec. 5(1). The House of Representatives shall be composed of not more than two
hundred and fifty members, unless otherwise fixed by law, who shall be elected from
legislative districts apportioned among the provinces, cities, and the Metropolitan
Manila area in accordance with the number of their respective inhabitants, and on the
basis of a uniform and progressive ratio, and those who, as provided by law, shall be
elected through a party list system of registered national, regional and sectoral
parties or organizations.
Sec. 5(4). Within three years following the return of every census, the Congress shall
make a reapportionment of legislative districts based on the standard provided in this
section.
Petitioners argue that the division of San Juan and Mandaluyong into separate congressional
districts under Section 49 of the assailed law has resulted in an increase in the composition of the
House of Representatives beyond that provided in Article VI, Sec. 5(1) of the Constitution.
Furthermore, petitioners contend that said division was not made pursuant to any census showing
that the subject municipalities have attained the minimum population requirements. And finally,
petitioners assert that Section 49 has the effect of preempting the right of Congress to reapportion
legislative districts pursuant to Sec. 5(4) as aforecited.
The contentions are devoid of merit.
Anent the first issue, we agree with the observation of the Solicitor General that the statutory
conversion of Mandaluyong into a highly urbanized city with a population of not less than two
hundred fifty thousand indubitably ordains compliance with the "one city-one representative"
proviso in the Constitution:
. . . Each city with a population of at least two hundred fifty thousand, or each
province, shall have at least one representative" (Article VI, Section 5(3),
Constitution).
Hence, it is in compliance with the aforestated constitutional mandate that the creation of a separate
congressional district for the City of Mandaluyong is decreed under Article VIII, Section 49 of R.A.
No. 7675.
Contrary to petitioners' assertion, the creation of a separate congressional district for Mandaluyong
is not a subject separate and distinct from the subject of its conversion into a highly urbanized city
but is a natural and logical consequence of its conversion into a highly urbanized city. Verily, the title
of R.A. No. 7675, "An Act Converting the Municipality of Mandaluyong Into a Highly Urbanized City
of Mandaluyong" necessarily includes and contemplates the subject treated under Section 49
regarding the creation of a separate congressional district for Mandaluyong.
Moreover, a liberal construction of the "one title-one subject" rule has been invariably adopted by this
court so as not to cripple or impede legislation. Thus, in Sumulong v. Comelec (73 Phil. 288 [1941]),
we ruled that the constitutional requirement as now expressed in Article VI, Section 26(1) "should be
given a practical rather than a technical construction. It should be sufficient compliance with such
requirement if the title expresses the general subject and all the provisions are germane to that
general subject."
The liberal construction of the "one title-one subject" rule had been further elucidated in Lidasan v.
Comelec (21 SCRA 496 [1967]), to wit:

Of course, the Constitution does not require Congress to employ in the title of an
enactment, language of such precision as to mirror, fully index or catalogue all the
contents and the minute details therein. It suffices if the title should serve the
purpose of the constitutional demand that it inform the legislators, the persons
interested in the subject of the bill and the public, of the nature, scope
and consequencesof the proposed law and its operation" (emphasis supplied).
Proceeding now to the other constitutional issues raised by petitioners to the effect that there is no
mention in the assailed law of any census to show that Mandaluyong and San Juan had each
attained the minimum requirement of 250,000 inhabitants to justify their separation into two
legislative districts, the same does not suffice to strike down the validity of R.A. No. 7675. The said
Act enjoys the presumption of having passed through the regular congressional processes, including
due consideration by the members of Congress of the minimum requirements for the establishment
of separate legislative districts. At any rate, it is not required that all laws emanating from the
legislature must contain all relevant data considered by Congress in the enactment of said laws.
As to the contention that the assailed law violates the present limit on the number of representatives
as set forth in the Constitution, a reading of the applicable provision, Article VI, Section 5(1), as
aforequoted, shows that the present limit of 250 members is not absolute. The Constitution clearly
provides that the House of Representatives shall be composed of not more than 250 members,
"unless otherwise provided by law." The inescapable import of the latter clause is that the present
composition of Congress may be increased, if Congress itself so mandates through a legislative
enactment. Therefore, the increase in congressional representation mandated by R.A. No. 7675 is
not unconstitutional.
Thus, in the absence of proof that Mandaluyong and San Juan do not qualify to have separate
legislative districts, the assailed Section 49 of R.A.
No. 7675 must be allowed to stand.
As to the contention that Section 49 of R.A. No. 7675 in effect preempts the right of Congress to
reapportion legislative districts, the said argument borders on the absurd since petitioners overlook
the glaring fact that it was Congress itself which drafted, deliberated upon and enacted the assailed
law, including Section 49 thereof. Congress cannot possibly preempt itself on a right which pertains
to itself.
Aside from the constitutional objections to R.A. No. 7675, petitioners present further arguments
against the validity thereof.
Petitioners contend that the people of San Juan should have been made to participate in the
plebiscite on R.A. No. 7675 as the same involved a change in their legislative district. The contention
is bereft of merit since the principal subject involved in the plebiscite was the conversion of
Mandaluyong into a highly urbanized city. The matter of separate district representation was only
ancillary thereto. Thus, the inhabitants of San Juan were properly excluded from the said plebiscite
as they had nothing to do with the change of status of neighboring Mandaluyong.
Similarly, petitioners' additional argument that the subject law has resulted in "gerrymandering,"
which is the practice of creating legislative districts to favor a particular candidate or party, is not
worthy of credence. As correctly observed by the Solicitor General, it should be noted that Rep.
Ronaldo Zamora, the author of the assailed law, is the incumbent representative of the former San
Juan/Mandaluyong district, having consistently won in both localities. By dividing San
Juan/Mandaluyong, Rep. Zamora's constituency has in fact been diminished, which development
could hardly be considered as favorable to him.

WHEREFORE, the petition is hereby DISMISSED for lack of merit.


SO ORDERED.

G.R.No. L-114785

08 December 1994

PONENTE: BIDIN, J.

FACTS:

Prior to Republic Act No., 7675 also known as An Act Converting the Municipality of
Mandaluyong into a Highly Urbanized City to be known as the City of Mandaluyong,
Mandaluyong and San Juan belonged to only one legislative district. A plebiscite was held for
the people of Mandaluyong whether or not they approved of the said conversion. The
plebiscite was only 14.41% of the said conversion. Nevertheless, 18,621 voted yes whereas
7, 911 voted no.

ISSUE:

Whether or not the ratification of RA7675 was unconstitutional citing Article VI, Sections 5(1), 4
and 26(1)

HELD/RULING:

For the purposes of discussion, lets breakdown all of the claimed violations to the 1987
Constitution.

Section 26(1). Every bill passed by the Congress shall embrace only one subject which shall be
expressed in the title thereof.

The creation of a separate congressional district for Mandaluyong is not a subject separate and
distinct from the subject of its conversion. Moreover, a liberal construction of the one-title-

one-subject rule has been liberally adopted by the court as to not impede legislation (Lidasan
v. Comelec).

Sec. 5(1). The House of Representatives shall be composed of not more than two hundred and
fifty members, unless otherwise fixed by law, who shall be elected from legislative districts
apportioned among the provinces, cities, and the Metropolitan Manila area in accordance with
the number of their respective inhabitants, and on the basis of a uniform and progressive ratio,
and those who, as provided by law, shall be elected through a party list system of registered
national, regional and sectoral parties or organizations.

The Constitution clearly provides that the House of Representatives shall be composed of not
more than 250 members,unless otherwise provided by law. The emphasis on the latter
clause indicates that the number of the House of Representatives may be increased, if
mandated via a legislative enactment. Therefore, the increase in congressional representation
is not unconstitutional.

Sec. 5(4). Within three years following the return of every census, the Congress shall make a
reapportionment of legislative districts based on the standard provided in this section.

The argument on the violation of the above provision is absurd since it was the Congress itself
which drafted, deliberated upon and enacted the assailed law.

The petition is thereby DISMISSED for lack of merit. SO ORDERED.


EN BANC
[G.R. No. 87636. November 19, 1990.]
NEPTALI A. GONZALES, ERNESTO M. MACEDA, ALBERTO G. ROMULO,
HEHERSON T. ALVAREZ, EDGARDO J. ANGARA, AGAPITO A. AQUINO, TEOFISTO
T. GUINGONA, JR., ERNESTO F. HERRERA, JOSE D. LINA, JR., JOHN OSMEA,
VICENTE T. PATERNO, RENE A. SAGUISAG, LETICIA RAMOS-SHAHANI,
MAMINTAL ABDUL J. TAMANO, WIGBERTO E. TAADA, JOVITO R. SALONGA,
ORLANDO S. MERCADO, JUAN PONCE ENRILE, JOSEPH ESTRADA, SOTERO
LAUREL, AQUILINO PIMENTEL, JR., SANTANINA RASUL, VICTOR
ZIGA, Petitioners, v. HON. CATALINO MACARAIG, JR., HON. VICENTE JAYME,

HON. CARLOS DOMINGUEZ, HON. FULGENCIO FACTORAN, HON. FIORELLO


ESTUAR, HON. LOURDES QUISUMBING, HON. RAUL MANGLAPUS, HON.
ALFREDO BENGSON, HON. JOSE CONCEPCION, HON. LUIS SANTOS, HON. MITA
PARDO DE TAVERA, HON. RAINERIO REYES, HON. GUILLERMO CARAGUE, HON.
ROSALINA CAJUCOM and HON. EUFEMIO C. DOMINGO, Respondents.
Gonzales, Batiller, Bilog & Associates for petitioners.

DECISION

MELENCIO-HERRERA, J.:

This constitutional controversy between the legislative and executive departments of


government stemmed from Senate Resolution No. 381, adopted on 2 February 1989,
"Authorizing and Directing the Committee on Finance to Bring in the Name of the
Senate of the Philippines the Proper Suit with the Supreme Court of the Philippines
contesting the Constitutionality of the Veto by the President of Special and General
Provisions, particularly Section 55, of the General Appropriation Bill of 1989 (H.B. No.
19186) and For Other Purposes."
cralaw virt ua1aw li bra ry

Petitioners are thus before us as members and ex-officio members of the Committee on
Finance of the Senate and as "substantial taxpayers whose vital interests may be
affected by this case."
cralaw virt ua1aw lib rary

Respondents are members of the Cabinet tasked with the implementation of the
General Appropriations Act of 1989 and 1990, some of them incumbents, while others
have already been replaced, and include the National Treasurer and the Commission on
Audit Chairman, all of whom are being sued in their official capacities.
chanroble s.com:c ralaw:re d

The Background Facts


On 16 December 1988, Congress passed House Bill No. 19186, or the General
Appropriations Bill for the Fiscal Year 1989. As passed, it eliminated or decreased
certain items included in the proposed budget submitted by the President.
Pursuant to the constitutional provision on the passage of bills, Congress presented the
said Bill to the President for consideration and approval.
On 29 December 1988, the President signed the Bill into law, and declared the same to
have become Rep. Act No. 6688. In the process, seven (7) Special Provisions and
Section 55, a "General Provision," were vetoed.
On 2 February 1989, the Senate, in the same Resolution No. 381 mentioned at the
outset, further expressed:
jgc:chanrob les.com. ph

"WHEREAS, Be it Resolved, as it is hereby Resolved, That the Senate express its sense

that the veto by the President of Section 55 of the GENERAL PROVISIONS of the
General Appropriation Bill of 1989 (H.B. No. 19186) is unconstitutional and, therefore,
void and without any force and effect; hence, the aforesaid Section 55 remains;
"x

x"

Thus it is that, on 11 April 1989, this Petition for Prohibition/ Mandamus was filed, with
a prayer for the issuance of a Writ of Preliminary Injunction and Restraining Order,
assailing mainly the constitutionality or legality of the Presidential veto of Section 55,
and seeking to enjoin respondents from implementing Rep. Act No. 6688. No
Restraining Order was issued by the Court.
The Comment, submitted by the Solicitor General on 25 August 1989 (after several
extensions granted), was considered as the Answer to the Petition and, on 7 September
1989, the Court Resolved to give due course to the Petition and to require the parties to
submit their respective Memoranda. Petitioners filed their Memorandum on 12
December 1989. But, on 19 January 1990, they filed a Motion for Leave to File and to
Admit Supplemental Petition, which was granted, basically raising the same issue as in
the original Petition, this time questioning the Presidents veto of certain provisions,
particularly Section 16, of House Bill 26934, or the General Appropriations Bill for Fiscal
Year 1990, which the President declared to have become Rep. Act No. 6831.
chanrobles v irt ualawli bra ry chan roble s.com:c han robles. com.ph

The Solicitor Generals Comment on the Supplemental Petition, on behalf of respondent


public officials, was submitted on 24 April 1990. On 15 May 1990, the Court required
the parties to file simultaneously their consolidated memoranda, to include the
Supplemental Petition, within an inextendible period of thirty (30) days from notice.
However, because the original Resolution of 15 May 1990 merely required the filing of a
memorandum on the Supplemental Petition, a revised Resolution requiring consolidated
memoranda, within thirty (30) days from notice, was released on 28 June 1990.
The Consolidated Memoranda were respectively filed on 26 June 1990 by petitioners,
and on 1 August 1990 by respondents. On 14 August 1990, both Memoranda were
Noted and the case was deemed submitted for deliberation.
On 11 September 1990, the Court heard the case on oral argument and required the
submittal of supplemental Memoranda, the last of which was filed on 26 September
1990.
The Vetoed Provisions and Reasons Therefor
Section 55 of the Appropriations Act of 1989 (Section 55 [FY 89] hereinafter), which
was vetoed by the President, reads:
jgc:chanrob les.co m.ph

"SEC. 55. Prohibition Against the Restoration or Increase of Recommended


Appropriations Disapproved and/or Reduced by Congress: No item of appropriation
recommended by the President in the Budget submitted to Congress pursuant to Article
VII, Section 22 of the Constitution which has been disapproved or reduced in this Act
shall be restored or increased by the use of appropriations authorized for other
purposes by augmentation. An item of appropriation for any purpose recommended by
the President in the Budget shall be deemed to have been disapproved by Congress if

no corresponding appropriation for the specific purpose is provided in this Act."

cralaw virt ua1aw lib rary

We quote below the reason for the Presidential veto:

jgc:chanrob les.co m.ph

"The provision violates Section 25 (5) of Article VI of the Constitution. If allowed, this
Section would nullify not only the constitutional and statutory authority of the
President, but also that of the President of the Senate, the Speaker of the House of
Representatives, the Chief Justice of the Supreme Court, and Heads of Constitutional
Commissions, to augment any item in the general appropriations law for their
respective offices from savings in other items of their respective appropriations. A
careful review of the legislative action on the budget as submitted shows that in almost
all cases, the budgets of agencies as recommended by the President, as well as those
of the Senate, the House of Representatives, and the Constitutional Commissions, have
been reduced. An unwanted consequence of this provision is the inability of the
President, the President of the Senate, Speaker of the House of Representatives, the
Chief Justice of the Supreme Court, and the heads of Constitutional Commissions to
augment any item of appropriation of their respective offices from savings in other
items of their respective appropriations even in cases of calamity or in the event of
urgent need to accelerate the implementation of essential public services and
infrastructure projects.
"Furthermore, this provision is inconsistent with Section 12 and other similar provisions
of this General Appropriations Act."
cralaw virtua 1aw lib rary

A substantially similar provision as the vetoed Section 55 appears in the Appropriations


Act of 1990, this time crafted as follows:
jgc:chanrobles .com.p h

"B. GENERAL PROVISIONS


"Sec. 16. Use of Savings. The President of the Philippines, the President of the
Senate, the Speaker of the House of Representatives, the Chief Justice of the Supreme
Court, the Heads of Constitutional Commissions under Article IX of the Constitution and
the Ombudsman are hereby authorized to augment any item in this Act for their
respective offices from savings in other items of their appropriations: PROVIDED, THAT
NO ITEM OF APPROPRIATION RECOMMENDED BY THE PRESIDENT IN THE BUDGET
SUBMITTED TO CONGRESS PURSUANT TO ARTICLE VII, SECTION 22 OF THE
CONSTITUTION WHICH HAS BEEN DISAPPROVED OR REDUCED BY CONGRESS SHALL
BE RESTORED OR INCREASED BY THE USE OF APPROPRIATIONS AUTHORIZED FOR
OTHER PURPOSES IN THIS ACT BY AUGMENTATION. AN ITEM OF APPROPRIATION FOR
ANY PURPOSE RECOMMENDED BY THE PRESIDENT IN THE BUDGET SHALL BE DEEMED
TO HAVE BEEN DISAPPROVED BY CONGRESS IF NO CORRESPONDING APPROPRIATION
FOR THE SPECIFIC PURPOSE IS PROVIDED IN THIS ACT."
cralaw virtua 1aw lib rary

It should be noted that in the 1989 Appropriations Act, the "Use of Savings" appears in
Section 12, separate and apart from Section 55; whereas in the 1990 Appropriations
Act, the "Use of Savings" and the vetoed provision have been commingled in Section 16
only, with the vetoed provision made to appear as a condition or restriction.
Essentially the same reason was given for the veto of Section 16 (FY 90), thus:

jgc:chanroble s.com. ph

"I am vetoing this provision for the reason that it violates Section 25 (5) of Article VI of
the Constitution in relation to Sections 44 and 45 of P.D. No. 1177 as amended by R.A.
No. 6670 which authorizes the President to use savings to augment any item of
appropriations in the Executive Branch of the Government.
"Parenthetically, there is a case pending in the Supreme Court relative to the validity of
the Presidents veto on Section 55 of the General Provisions of Republic Act No. 6688
upon which the amendment on this Section was based. Inclusion, therefore, of the
proviso in the last sentence of this section might prejudice the Executive Branchs
position in the case.
"Moreover, if allowed, this Section would nullify not only the constitutional and
statutory authority of the President, but also that of the officials enumerated under
Section 25 (5) of Article VI of the Constitution, to augment any item in the general
appropriations law for their respective appropriations.
"An unwanted consequence of this provision would be the inability of the President, the
President of the Senate, Speaker of the House of Representatives, the Chief Justice of
the Supreme Court, and heads of Constitutional Commissions to augment any item of
appropriation of their respective offices from savings in other items of their respective
appropriations even in cases of national emergency or in the event of urgent need to
accelerate the implementation of essential public services and infrastructure projects."

cralaw virt ua1aw

libra ry

The fundamental issue raised is whether or not the veto by the President of Section 55
of the 1989 Appropriations Bill (Section 55 FY 89), and subsequently of its counterpart
Section 16 of the 1990 Appropriations Bill (Section 16 FY 90), is unconstitutional and
without effect.
chanrobles. com:cra law:red

The Contending Views


In essence, petitioners cause is anchored on the following grounds: (1) the Presidents
line-veto power as regards appropriation bills is limited to item/s and does not cover
provision/s; therefore, she exceeded her authority when she vetoed Section 55 (FY 89)
and Section 16 (FY 90) which are provisions; (2) when the President objects to a
provision of an appropriation bill, she cannot exercise the item-veto power but should
veto the entire bill; (3) the item-veto power does not carry with it the power to strike
out conditions or restrictions for that would be legislation, in violation of the doctrine of
separation of powers; and (4) the power of augmentation in Article VI, Section 25 [5]
of the 1987 Constitution, has to be provided for by law and, therefore, Congress is also
vested with the prerogative to impose restrictions on the exercise of that power.
The Solicitor General, as counsel for public respondents, counters that the issue at bar
is a political question beyond the power of this Court to determine; that petitioners had
a political remedy, which was to override the veto; that Section 55 is a "rider" because
it is extraneous to the Appropriations Act and, therefore, merits the Presidents veto;
that the power of the President to augment items in the appropriations for the
executive branches had already been provided for in the Budget Law, specifically
Sections 44 and 45 of Pres. Decree No. 1177, as amended by Rep. Act No. 6670 (4
August 1988); and that the President is empowered by the Constitution to veto
provisions or other "distinct and severable parts" of an Appropriations Bill.

Judicial Determination
With the Senate maintaining that the Presidents veto is unconstitutional, and that
charge being controverted, there is an actual case or justiciable controversy between
the Upper House of Congress and the executive department that may be taken
cognizance of by this Court.
"Indeed, where the legislature or the executive branch is acting within the limits of its
authority, the judiciary cannot and ought not to interfere with the former. But where
the legislature or the executive acts beyond the scope of its constitutional powers, it
becomes the duty of the judiciary to declare what the other branches of the
government had assumed to do as void. This is the essence of judicial power conferred
by the Constitution in one Supreme Court and in such lower courts as may be
established by law [Art. VIII, Section 1 of the 1935 Constitution; Art. X, Section 1 of
the 1973 Constitution and which was adopted as part of the Freedom Constitution, and
Art. VIII, Section 1 of the 1987 Constitution] and which power this Court has exercised
in many instances" (Demetria v. Alba, G.R. No. 71977, 27 February 1987, 148 SCRA
209).
We take note as well of what petitioners stress as the "imperative need for a definitive
ruling by this Court as to the exact parameters of the exercise of the item-veto power
of the President as regards appropriation bills . . . in order to obviate the recurrence of
a similar problem whenever a general appropriations bill is passed by Congress."
Indeed, the contextual reiteration of Section 55 (FY 89) in Section 16 (FY 90) and
again, its veto by the President, underscore the need for judicial arbitrament. The Court
does not thereby assert its superiority over or exhibit lack of respect due the other coordinate departments but discharges a solemn and sacred duty to determine essentially
the scope of intersecting powers in regard which the Executive and the Senate are in
dispute.
chanroble s.com : vi rtua l law lib rary

Petitioners have also brought this suit as taxpayers. As ruled in Sanidad v. COMELEC
(No. L-44640, 12 October 1976, 73 SCRA 333), this Court enjoys the open discretion to
entertain taxpayers suits or not. In Tolentino v. COMELEC (No. L-34150, 16 October
1961, 41 SCRA 702), it was also held that a member of the Senate has the requisite
personality to bring a suit where a constitutional issue is raised.
cralawnad

The political question doctrine neither interposes an obstacle to judicial determination of


the rival claims. The jurisdiction to delimit constitutional boundaries has been given to
this Court. It cannot abdicate that obligation mandated by the 1987 Constitution,
although said provision by no means does away with the applicability of the principle in
appropriate cases.
"SECTION 1. The judicial power shall be vested in one Supreme Court and in such lower
courts as may be established by law.
Judicial power includes the duty of the courts of justice to settle actual controversies
involving rights which are legally demandable and enforceable, and to determine
whether or not there has been a grave abuse of discretion amounting to lack or excess
of jurisdiction on the part of any branch or instrumentality of the Government."
cralaw virtua1aw l ib rary

Nor is this the first time that the constitutionality of a Presidential veto is raised to the
Court. The two oft-cited cases are Bengson v. Secretary of Justice (62 Phil. 912
[1936]), penned by Justice George A. Malcolm, which upheld the veto questioned
before it, but which decision was reversed by the U.S. Supreme Court in the same
entitled case in 292 U.S. 410, infra, essentially on the ground that an Appropriations
Bill was not involved. The second case is Bolinao Electronics v. Valencia (G.R. No. L20740, 30 June 1964, 11 SCRA 486), infra, which rejected the Presidents veto of a
condition or restriction in an Appropriations Bill.
The Extent of the Presidents Item-veto Power
The focal issue for resolution is whether or not the President exceeded the item-veto
power accorded by the Constitution. Or differently put, has the President the power to
veto "provisions" of an Appropriations Bill?
Petitioners contend that Section 55 (FY 89) and Section 16 (FY 90) are provisions and
not items and are, therefore, outside the scope of the item-veto power of the
President.
chanrobles lawlib rary : rednad

The veto power of the President is expressed in Article VI, Section 27 of the 1987
Constitution reading, in full, as follows:
jgc:chanrobles. com.ph

"Sec. 27. (1) Every bill passed by the Congress shall, before it becomes a law, be
presented to the President. If he approves the same, he shall sign it; otherwise, he
shall veto it and return the same with his objections to the House where it originated,
which shall enter the objections at large in its Journal and proceed to reconsider it. If,
after such reconsideration, two-thirds of all the Members of such House shall agree to
pass the bill, it shall be sent, together with the objections, to the other House by which
it shall likewise be reconsidered, and if approved by two-thirds of all the Members of
that House, it shall become a law. In all such cases, the votes of each House shall be
determined by yeas or nays, and the names of the Members voting for or against shall
be entered in its Journal. The President shall communicate his veto of any bill to the
House where it originated within thirty days after the date of receipt thereof; otherwise,
it shall become a law as if he had signed it.
"(2) The President shall have the power to veto any particular item or items in an
appropriation, revenue, or tariff bill, but the veto shall not affect the item or items to
which he does not object."
cralaw virtua 1aw lib rary

Paragraph (1) refers to the general veto power of the President and if exercised would
result in the veto of the entire bill, as a general rule. Paragraph (2) is what is referred
to as the item-veto power or the line-veto power. It allows the exercise of the veto over
a particular item or items in an appropriation, revenue, or tariff bill. As specified, the
President may not veto less than all of an item of an Appropriations Bill. In other words,
the power given the executive to disapprove any item or items in an Appropriations Bill
does not grant the authority to veto a part of an item and to approve the remaining
portion of the same item.
Originally, item veto exclusively referred to veto of items of appropriation bills and first

came into being in the former Organic Act, the Act of Congress of 29 August 1916. This
was followed by the 1935 Constitution, which contained a similar provision in its Section
11(2), Article VI, except that the veto power was made more expansive by the inclusion
of this sentence:
jgc:chanrob les.com. ph

". . . When a provision of an appropriation bill affects one or more items of the same,
the President can not veto the provision without at the same time vetoing the particular
item or items to which it relates . . ."
cralaw virt ua1aw li bra ry

The 1935 Constitution further broadened the Presidents veto power to include the veto
of item or items of revenue and tariff bills.
With the advent of the 1973 Constitution, the section took a more simple and compact
form, thus:
jgc:chanrob les.co m.ph

"Section 20 (2). The Prime Minister shall have the power to veto any particular item or
items in an appropriation, revenue, or tariff bill, but the veto shall not affect the item or
items to which he does not object."
cralaw virt ua1aw li bra ry

It is to be noted that the counterpart provision in the 1987 Constitution (Article VI,
Section 27 [2], supra), is a verbatim reproduction except for the public official
concerned. In other words, also eliminated has been any reference to the veto of a
provision. The vital question is: should this exclusion be interpreted to mean as a
disallowance of the power to veto a provision, as petitioners urge?
The terms item and provision in budgetary legislation and practice are concededly
different. An item in a bill refers to the particulars, the details, the distinct and
severable parts . . . of the bill (Bengzon, supra, at 916). It is an indivisible sum of
money dedicated to a stated purpose (Commonwealth v. Dodson, 11 S.E., 2d 120, 124,
125, etc., 176 Va. 281). The United States Supreme Court, in the case of Bengzon v.
Secretary of Justice (299 U.S. 410, 414, 57 S.Ct 252, 81 L. Ed., 312) declared "that an
item of an appropriation bill obviously means an item which in itself is a specific
appropriation of money, not some general provision of law, which happens to be put
into an appropriation bill."
cralaw vi rtua 1aw lib rary

It is our considered opinion that, notwithstanding the elimination in Article VI, Section
27 (2) of the 1987 Constitution of any reference to the veto of a provision, the extent
of the Presidents veto power as previously defined by the 1935 Constitution has not
changed. This is because the eliminated proviso merely pronounces the basic principle
that a distinct and severable part of a bill may be the subject of a separate veto
(Bengzon v. Secretary of Justice, 62 Phil., 912, 916 (1926); 2 BERNAS, Joaquin, S.J.,
The Constitution of the Republic of the Philippines, 1st ed., 154-155, [1988]).
The restrictive interpretation urged by petitioners that the President may not veto a
provision without vetoing the entire bill not only disregards the basic principle that a
distinct and severable part of a bill may be the subject of a separate veto but also
overlooks the Constitutional mandate that any provision in the general appropriations
bill shall relate specifically to some particular appropriation therein and that any such
provision shall be limited in its operation to the appropriation to which it relates (1987
Constitution, Article VI, Section 25 [2]). In other words, in the true sense of the term, a

provision in an Appropriations Bill is limited in its operation to some particular


appropriation to which it relates, and does not relate to the entire bill.
chanro bles law l ibra ry

Petitioners further submission that, since the exercise of the veto power by the
President partakes of the nature of legislative powers it should be strictly construed, is
negative by the following dictum in Bengzon, supra, reading:
jgc:chanroble s.com.p h

"The Constitution is a limitation upon the power of the legislative department of the
government, but in this respect it is a grant of power to the executive department. The
Legislature has the affirmative power to enact laws; the Chief Executive has the
negative power by the constitutional exercise of which he may defeat the will of the
Legislature. It follows that the Chief Executive must find his authority in the
Constitution. But in exercising that authority he may not be confined to rules of strict
construction or hampered by the unwise interference of the judiciary. The courts will
indulge every intendment in favor of the constitutionality of a veto the same as they
will presume the constitutionality of an act as originally passed by the Legislature"
(Commonwealth v. Barnett [1901], 199 Pa., 161; 55 L.R.A., 882; People v. Board of
Councilmen [1892], 20 N.Y.S., 52; Fulmore v. Lane [1911], 104 Tex., 499; Texas Co.
v. State [1927], 53 A.L.R., 258 [at 917]).
Inappropriateness of the so-called "Provisions"
But even assuming arguendo that provisions are beyond the executive power to veto,
we are of the opinion that Section 55 (FY 89) and Section 16 (FY 90) are not
provisions in the budgetary sense of the term. Article VI, Section 25 (2) of the 1987
Constitution provides:
jgc:chanro bles.com. ph

"Sec. 25 (2) No provision or enactment shall be embraced in the general appropriations


bill unless it relates specifically to some particular appropriation therein. Any such
provision or enactment shall be limited in its operation to the appropriation to which it
relates."
cralaw virt ua1aw lib rary

Explicit is the requirement that a provision in the Appropriations Bill should relate
specifically to some" particular appropriation" therein. The challenged "provisions" fall
short of this requirement. Firstly, the vetoed "provisions" do not relate to any particular
or distinctive appropriation. They apply generally to all items disapproved or reduced by
Congress in the Appropriations Bill. Secondly, the disapproved or reduced items are
nowhere to be found on the face of the Bill. To discover them, resort will have to be
made to the original recommendations made by the President and to the source
indicated by petitioners themselves, i.e., the "Legislative Budget Research and
Monitoring Office" (Annex B-1 and B-2, Petition). Thirdly, the vetoed Sections are more
of an expression of Congressional policy in respect of augmentation from savings rather
than a budgetary appropriation. Consequently, Section 55 (FY 89) and Section 16 (FY
90) although labelled as "provisions," are actually inappropriate provisions that should
be treated as items for the purpose of the Presidents veto power. (Henry v. Edwards
[1977] 346 S Rep. 2d, 157-158)
"Just as the President may not use his item-veto to usurp constitutional powers
conferred on the legislature, neither can the legislature deprive the Governor of the
constitutional powers conferred on him as chief executive officer of the state by

including in a general appropriation bill matters more properly enacted in separate


legislation. The Governors constitutional power to veto bills of general legislation . . .
cannot be abridged by the careful placement of such measures in a general
appropriation bill, thereby forcing the Governor to choose between approving
unacceptable substantive legislation or vetoing items of expenditure essential to the
operation of government. The legislature cannot by location of a bill give it immunity
from executive veto. Nor can it circumvent the Governors veto power over substantive
legislation by artfully drafting general law measures so that they appear to be true
conditions or limitations on an item of appropriation. Otherwise, the legislature would
be permitted to impair the constitutional responsibilities and functions of a co-equal
branch of government in contravention of the separation of powers doctrine . . . We are
no more willing to allow the legislature to use its appropriation power to infringe on the
Governors constitutional right to veto matters of substantive legislation than we are to
allow the Governor to encroach on the constitutional powers of the legislature. In order
to avoid this result, we hold that, when the legislature inserts inappropriate provisions
in a general appropriation bill, such provisions must be treated as items for purposes
of the Governors item veto power over general appropriation bills.
x

". . . Legislative control cannot be exercised in such a manner as to encumber the


general appropriation bill with veto-proof logrolling measure, special interest
provisions which could not succeed if separately enacted, or riders, substantive pieces
of legislation incorporated in a bill to insure passage without veto. . . ." (Emphasis
supplied)
Inappropriateness of the so-called "Conditions/Restrictions"
Petitioners maintain, however, that Congress is free to impose conditions in an
Appropriations Bill and where conditions are attached, the veto power does not carry
with it the power to strike them out, citing Commonwealth v. Dodson (11 SE, 2d 130,
supra) and Bolinao Electronics Corporation v. Valencia (No. L-20740, June 30, 1964, 11
SCRA 486). In other words, their theory is that Section 55 (FY 89) and Section 16 (FY
90) are such conditions/restrictions and thus beyond the veto power.
chanro bles vi rtua l lawlib ra ry

There can be no denying that inherent in the power of appropriation is the power to
specify how money shall be spent; and that in addition to distinct "items" of
appropriation, the Legislature may include in Appropriation Bills qualifications,
conditions, limitations or restrictions on expenditure of funds. Settled also is the rule
that the Executive is not allowed to veto a condition or proviso of an appropriation while
allowing the appropriation itself to stand (Fairfield v. Foster, supra, at 320). That was
also the ruling in Bolinao, supra, which held that the veto of a condition in an
Appropriations Bill which did not include a veto of the items to which the condition
related was deemed invalid and without effect whatsoever.
However, for the rule to apply, restrictions should be such in the real sense of the term,
not some matters which are more properly dealt with in a separate legislation (Henry v.
Edwards, La, 346, So 2d 153). Restrictions or conditions in an Appropriations Bill must
exhibit a connection with money items in a budgetary sense in the schedule of

expenditures. Again, the test is appropriateness.


"It is not enough that a provision be related to the institution or agency to which funds
are appropriated. Conditions and limitations properly included in an appropriation bill
must exhibit such a connexity with money items of appropriation that they logically
belong in a schedule of expenditures . . . the ultimate test is one of appropriateness"
(Henry v. Edwards, supra, at 158).
Tested by these criteria, Section 55 (FY 89) and Section 16 (FY 90) must also be held
to be inappropriate "conditions." While they, particularly, Section 16 (FY 90), have
been "artfully drafted" to appear as true conditions or limitations, they are actually
general law measures more appropriate for substantive and, therefore, separate
legislation.
Further, neither of them shows the necessary connection with a schedule of
expenditures. The reason, as explained earlier, is that items reduced or disapproved by
Congress would not appear on the face of the enrolled bill or Appropriations Act itself.
They can only be detected when compared with the original budgetary submittals of the
President. In fact, Sections 55 (FY 89) and 16 (FY 90) themselves provide that an item
"shall be deemed to have been disapproved by Congress if no corresponding
appropriation for the specific purpose is provided in this Act."
cralaw virtua 1aw lib rary

Considering that the vetoed provisions are not, in the budgetary sense of the term,
conditions or restrictions, the case of Bolinao Electronics Corporation v. Valencia
(supra), invoked by petitioners, becomes inapplicable. In that case, a public works bill
contained an item appropriating a certain sum for assistance to television stations,
subject to the condition that the amount would not be available to places where there
were commercial television stations. Then President Macapagal approved the
appropriation but vetoed the condition. When challenged before this Court, it was held
that the veto was ineffectual and that the approval of the item carried with it the
approval of the condition attached to it. In contrast with the case at bar, there is no
condition, in the budgetary sense of the term, attached to an appropriation or item in
the appropriation bill which was struck out. For obviously, Sections 55 (FY 89) and 16
(FY 90) partake more of a curtailment on the power to augment from savings; in other
words, "a general provision of law, which happens to be put in an appropriation bill"
(Bengzon v. Secretary of Justice, supra).
The Power of Augmentation and The Validity of the Veto
The President promptly vetoed Section 55 (FY 89) and Section 16 (FY 90) because
they nullify the authority of the Chief Executive and heads of different branches of
government to augment any item in the General Appropriations Law for their respective
offices from savings in other items of their respective appropriations, as guaranteed by
Article VI, Section 25 (5) of the Constitution. Said provision reads:
jgc:chanroble s.com.p h

"Sec. 25. (5) No law shall be passed authorizing any transfer of appropriations;
however, the President, the President of the Senate, the Speaker of the House of
Representatives, the Chief Justice of the Supreme Court, and the heads of
Constitutional Commissions may, by law, be authorized to augment any item in the
general appropriations law for their respective offices from savings in other items of

their respective appropriations" (Emphasis ours).


Noteworthy is the fact that the power to augment from savings lies dormant until
authorized by law.
This Court upheld the validity of the power of augmentation from savings in Demetria v.
Alba, which ruled:
jgc:chanrob les.com. ph

". . . to afford the heads of the different branches of the government and those of the
constitutional commissions considerable flexibility in the use of public funds and
resources, the constitution allowed the enactment of a law authorizing the transfer of
funds for the purpose of augmenting an item from savings in another item in the
appropriation of the government branch or constitutional body concerned. The leeway
granted was thus limited. The purpose and conditions for which funds may be
transferred were specified, i.e., transfer may be allowed for the purpose of augmenting
an item and such transfer may be made only if there are savings from another item in
the appropriation of the government branch or constitutional body" (G.R. No. 71977, 27
February 1987, 148 SCRA 214).
The 1973 Constitution contained an identical authority to augment from savings in its
Article VIII, Section 16 (5), except for mention of the Prime Minister among the officials
vested with that power. 1
In 1977, the statutory authority of the President to augment any appropriation of the
executive department in the General Appropriations Act from savings was specifically
provided for in Section 44 of Presidential Decree No. 1177, as amended (RA 6670, 4
August 1988), otherwise known as the "Budget Reform Decree of 1977." It reads:
jgc:chanrob les.c om.ph

"Sec. 44. . . .
"The President shall, likewise, have the authority to augment any appropriation of the
Executive Department in the General Appropriations Act, from savings in the
appropriations of another department, bureau, office or agency within the Executive
Branch, pursuant to the provisions of Art. VIII, Sec. 16 (5) of the Constitution (now
Sec. 25 (5), Art. VI)" (Emphasis ours), (N.B.: The first paragraph declared void in
Demetria v. Alba, supra, has been deleted).
Similarly, the use by the President of savings to cover deficits is specifically authorized
in the same Decree. Thus:
jgc:chanroble s.com.p h

"Sec. 45. Authority to Use Savings in Appropriations to Cover Deficits. Except as


otherwise provided in the General Appropriations Act, any savings in the regular
appropriations authorized in the General Appropriations Act for programs and projects
of any department, office or agency, may, with the approval of the President be used to
cover a deficit in any other item of the regular appropriations: ". . .
A more recent grant is found in Section 12 of the General Appropriations Act of 1989,
the text of which is repeated in the first paragraph of Section 16 (FY 90). Section 12
reads:
chanrobles vi rtua l lawli bra ry

"Sec. 12. Use of Savings. The President, the President of the Senate, the Speaker of
the House of Representatives, the Chief Justice of the Supreme Court, the heads of the
Constitutional Commissions, and the Ombudsman are hereby authorized to augment
any item in this Act for their respective offices from savings in other items of their
respective appropriations."
cralaw virtua 1aw lib rary

There should be no question, therefore, that statutory authority has, in fact, been
granted. And once given, the heads of the different branches of the Government and
those of the Constitutional Commissions are afforded considerable flexibility in the use
of public funds and resources (Demetria v. Alba, supra). The doctrine of separation of
powers is in no way endangered because the transfer is made within a department (or
branch of government) and not from one department (branch) to another (CRUZ,
Isagani A., Philippine Political Law [1989] p. 155).
When Sections 55 (FY 89) and 16 (FY 90), therefore, prohibit the restoration or
increase by augmentation of appropriations disapproved or reduced by Congress, they
impair the constitutional and statutory authority of the President and other key officials
to augment any item or any appropriation from savings in the interest of expediency
and efficiency. The exercise of such authority in respect of disapproved or reduced
items by no means vests in the Executive the power to rewrite the entire budget, as
petitioners contend, the leeway granted being delimited to transfers within the
department or branch concerned, the sourcing to come only from savings.
More importantly, it strikes us, too, that for such a special power as that of
augmentation from savings, the same is merely incorporated in the General
Appropriations Bill. An Appropriations Bill is "one the primary and specific aim of which
is to make appropriation of money from the public treasury" (Bengzon v. Secretary of
Justice, 292 U.S., 410, 57 S.Ct. 252). It is a legislative authorization of receipts and
expenditures. The power of augmentation from savings, on the other hand, can by no
means be considered a specific appropriation of money. It is a non-appropriation item
inserted in an appropriation measure.
chanroble s law lib ra ry : red

The same thing must be said of Section 55 (FY 89), taken in conjunction with Section
12, and Section 16 (FY 90), which prohibit the restoration or increase by augmentation
of appropriations disapproved and/or reduced by Congress. They are non-appropriation
items, an appropriation being a setting apart by law of a certain sum from the public
revenue for a specific purpose (Bengzon v. Secretary of Justice, 62 Phil. 912, 916
[1936]). It bears repeating that they are more of a substantive expression of a
legislative objective to restrict the power of augmentation granted to the President and
other key officials. They are actually matters of general law and more properly the
subject of a separate legislation that will embody, define and delimit the scope of the
special power of augmentation from savings instead of being inappropriately
incorporated annually in the Appropriation Act. To sanction this practice would be to
give the Legislature the freedom to grant or withhold the power from the Executive and
other officials, and thus put in yearly jeopardy the exercise of that power.
If, indeed, by the later enactments of Section 55 (FY 89) and Section 16 (FY 90),
Congress, as petitioners argue, intended to amend or repeal Pres. Decree No. 1177,
with all the more reason should it have so provided in a separate enactment, it being
basic that implied repeals are not favored. For the same reason, we cannot subscribe to

petitioners allegation that Pres. Decree No. 1177 has been revoked by the 1987
Constitution. The 1987 Constitution itself provides for the continuance of laws, decrees,
executive orders, proclamations, letters of instructions, and other executive issuances
not inconsistent with the Constitution until amended, repealed, or revoked (1987
Constitution, Article XVIII, Section 3).
If, indeed, the legislature believed that the exercise of the veto powers by the executive
were unconstitutional, the remedy laid down by the Constitution is crystal clear. A
Presidential veto may be overriden by the votes of two-thirds of members of Congress
(1987 Constitution, Article VI, Section 27[1], supra). But Congress made no attempt to
override the Presidential veto. Petitioners argument that the veto is ineffectual so that
there is "nothing to override" (citing Bolinao) has lost force and effect with the
executive veto having been herein upheld.
As we see it, there need be no future conflict if the legislative and executive branches
of government adhere to the spirit of the Constitution, each exercising its respective
powers with due deference to the constitutional responsibilities and functions of the
other. Thereby, the delicate equilibrium of governmental powers remains on even keel.
WHEREFORE, the constitutionality of the assailed Presidential veto is UPHELD and this
Petition is hereby DISMISSED.
No costs.
Gonzales v. Macaraig, Jr. 1990

GR 87636 -EN BANC


Facts:
December 16, 1988 Congress passed House Bill No. 19186 (GAB of Fiscal Year 1989) which eliminated
or decreased certain items included in the proposed budget submitted by the president
December 29, 1988 President signed bill into law (RA 6688) but vetoed 7 special provisions and Sec
55, a general provision.
February 2, 1989 Senate passed Res. No. 381 Senate as an institution decided to contest the
constitutionality of the veto of the president of SEC 55 only.
April 11, 1989 this petition was filed
January 19, 1990 filed motion for leave to file and to admit supplemental petition same issues but
included SEC 16 of House Bill 26934 (Gab for FY 1990 or RA 6831)
SEC. 55 disallows the president and heads of several department to augment any item in the GAB
thereby violation CONSTI ART VI SEC 25 (5) (page 459)
SEC 16 of the GAB of 1990 provides for the same and the reason for veto remains the same with the
additional legal basis of violation of PD 1177 SEC 44 and 45 as amended by RA 6670 that authorizes the
president and the heads of depts. To use saving to augment any item of appropriations in the exec
branch of government (page 460)

ISSUE:
Whether or not the veto by the President of SEC 55 of GAB for FY 1989 and SEC 16 of GAB for FY 1990
is unconstitutional.
HELD:
The veto is CONSTITUTIONAL. Although the petitioners contend that the veto exceeded the mandate of
the line-veto power of the president because SEC 55 and SEC 16 are provisions the court held that
inappropriate provisions can be treated as items (Henry v. Edwards) and therefore can be vetoed validly
by the president. Furthermore inappropriate provisions must be struck down because they contravene the
constitution because it limits the power of the executive to augment appropriations (ART VI SEC 25 PAR
5.)
The provisions are inappropriate because
o They do not relate to particular or distinctive appropriations
o Disapproved or reduces items are nowhere to be found on the face of the bill
o It is more of an expression of policy than an appropriation
Court also said that to make the GAB veto-proof would be logrolling on the part of the legislative the
subject matter of the provisions should be dealt with in separate and complete legislation but because
they are aware that it would be NOT passed in that manner they attempt hide it in the GAB
If the legislature really believes that the exercise of veto is really invalid then congress SHOULD resort to
their constitutionally vested power to override the veto. (ART VI SEC 21 PAR 1)
DECISION: Veto UPHELD. Petition DISMISSED.

Republic of the Philippines


SUPREME COURT
Manila
EN BANC

G.R. No. 103524 April 15, 1992


CESAR BENGZON, QUERUBE MAKALINTAL, LINO M. PATAJO, JOSE
LEUTERIO, ET AL., petitioners,
vs.
HON. FRANKLIN N. DRILON, in his capacity as Executive Secretary, HON.
GUILLERMO CARAGUE, in his capacity as Secretary of Department of
Budget and Management, and HON. ROSALINA CAJUCOM, in her capacity
as National Treasurer, respondents.

A.M. No. 91-8-225-CA April 15, 1992


REQUEST OF RETIRED JUSTICES MANUEL P. BARCELONA, JUAN P.
ENRIQUEZ, JUAN O. REYES, JR. and GUARDSON R. LOOD FOR
READJUSTMENT OF THEIR MONTHLY PENSION.

GUTIERREZ, JR., J.:


The issue in this petition is the constitutionality of the veto by the President of
certain provisions in the General Appropriations Act for the Fiscal Year 1992
relating to the payment of the adjusted pensions of retired Justices of the
Supreme Court and the Court of Appeals.
The petitioners are retired Justices of the Supreme Court and Court of Appeals
who are currently receiving monthly pensions under Republic Act No. 910 as
amended by Republic Act No. 1797. They filed the instant petition on their own
behalf and in representation of all other retired Justices of the Supreme Court
and the Court of Appeals similarly situated.
Named respondents are Hon. Franklin Drilon the Executive Secretary, Hon.
Guillermo Carague as Secretary of the Department of Budget and Management,
and Hon. Rosalinda Cajucom, the Treasurer of the Philippines. The respondents
are sued in their official capacities, being officials of the Executive Department
involved in the implementation of the release of funds appropriated in the Annual
Appropriations Law.
We treat the Comments of the Office of the Solicitor General (OSG) as an
Answer and decide the petition on its merits.
The factual backdrop of this case is as follows:
On June 20, 1953, Republic Act No, 910 was enacted to provide the retirement
pensions of Justices of the Supreme Court and of the Court of Appeals who have
rendered at least twenty (20) years service either in the Judiciary or in any other
branch of the Government or in both, having attained the age of seventy (70)
years or who resign by reason of incapacity to discharge the duties of the office.
The retired Justice shall receive during the residue of his natural life the salary
which he was receiving at the time of his retirement or resignation.
Republic Act No. 910 was amended by Republic Act No. 1797 (approved on
June 21, 1957) which provided that:

Sec. 3-A. In case the salary of Justices of the Supreme Court or of


the Court of Appeals is increased or decreased, such increased or
decreased salary shall, for purposes of this Act, be deemed to be
the salary or the retirement pension which a Justice who as of June
twelve, nineteen hundred fifty-four had ceased to be such to accept
another position in the Government or who retired was receiving at
the time of his cessation in office. Provided, that any benefits that
have already accrued prior to such increase or decrease shall not be
affected thereby.
Identical retirement benefits were also given to the members of the Constitutional
Commissions under Republic Act No. 1568, as amended by Republic Act No.
3595. On November 12, 1974, on the occasion of the Armed Forces Loyalty Day,
President Marcos signed Presidential Decree 578 which extended similar
retirement benefits to the members of the Armed Forces giving them also the
automatic readjustment features of Republic Act No. 1797 and Republic Act No.
3595.
Two months later, however, President Marcos issued Presidential Decree 644 on
January 25, 1975 repealing Section 3-A of Republic Act No. 1797 and Republic
Act No. 3595 (amending Republic Act No. 1568 and Presidential Decree No.
578) which authorized the adjustment of the pension of the retired Justices of the
Supreme Court, Court of Appeals, Chairman and members of the Constitutional
Commissions and the officers and enlisted members of the Armed Forces to the
prevailing rates of salaries.
Significantly, under Presidential Decree 1638 the automatic readjustment of the
retirement pension of officers and enlisted men was subsequently restored by
President Marcos. A later decree Presidential Decree 1909 was also issued
providing for the automatic readjustment of the pensions of members of the
Armed Forces who have retired prior to September 10, 1979.
While the adjustment of the retirement pensions for members of the Armed
Forces who number in the tens of thousands was restored, that of the retired
Justices of the Supreme Court and Court of Appeals who are only a handful and
fairly advanced in years, was not.
Realizing the unfairness of the discrimination against the members of the
Judiciary and the Constitutional Commissions, Congress approved in 1990 a bill
for the reenactment of the repealed provisions of Republic Act No. 1797 and
Republic Act No. 3595. Congress was under the impression that Presidential
Decree 644 became law after it was published in the Official Gazette on April 7,
1977. In the explanatory note of House Bill No. 16297 and Senate Bill No. 740,

the legislature saw the need to reenact Republic Act Nos. 1797 and 3595 to
restore said retirement pensions and privileges of the retired Justices and
members of the Constitutional Commissions, in order to assure those serving in
the Supreme Court, Court of Appeals and Constitutional Commissions adequate
old age pensions even during the time when the purchasing power of the peso
has been diminished substantially by worldwide recession or inflation. This is
underscored by the fact that the petitioner retired Chief Justice, a retired
Associate Justice of the Supreme Court and the retired Presiding Justice are
presently receiving monthly pensions of P3,333.33, P2,666.66 and P2,333.33
respectively.
President Aquino, however vetoed House Bill No. 16297 on July 11, 1990 on the
ground that according to her "it would erode the very foundation of the
Government's collective effort to adhere faithfully to and enforce strictly the policy
on standardization of compensation as articulated in Republic Act No. 6758
known as Compensation and Position Classification Act of 1989." She further
said that "the Government should not grant distinct privileges to select group of
officials whose retirement benefits under existing laws already enjoy preferential
treatment over those of the vast majority of our civil service servants."
Prior to the instant petition, however, Retired Court of Appeals Justices Manuel
P. Barcelona, Juan P. Enriquez, Juan O. Reyes, Jr. and Guardson R. Lood filed
a letter/petition dated April 22, 1991 which we treated as Administrative Matter
No. 91-8-225-CA. The petitioners asked this Court far a readjustment of their
monthly pensions in accordance with Republic Act No. 1797. They reasoned out
that Presidential Decree 644 repealing Republic Act No. 1797 did not become
law as there was no valid publication pursuant to Taada v. Tuvera, (136 SCRA
27 [1985]) and 146 SCRA 446 [1986]). Presidential Decree 644 promulgated on
January 24, 1975 appeared for the first time only in the supplemental issue of the
Official Gazette, (Vol. 74, No. 14) purportedly dated April 4, 1977 but published
only on September 5, 1983. Since Presidential Decree 644 has no binding force
and effect of law, it therefore did not repeal Republic Act No. 1797.
In a Resolution dated November 28, 1991 the Court acted favorably on the
request. The dispositive portion reads as follows:
WHEREFORE, the requests of retired Justices Manuel P.
Barcelona, Juan P. Enriquez, Juan O. Reyes and Guardson Lood
are GRANTED. It is hereby AUTHORIZED that their monthly
pensions be adjusted and paid on the basis of RA 1797 effective
January 1, 1991 without prejudice to the payment on their pension
differentials corresponding to the previous years upon the availability
of funds for the purpose.

Pursuant to the above resolution, Congress included in the General


Appropriations Bill for Fiscal Year 1992 certain appropriations for the Judiciary
intended for the payment of the adjusted pension rates due the retired Justices of
the Supreme Court and Court of Appeals.
The pertinent provisions in House Bill No. 34925 are as follows:
XXVIII. THE JUDICIARY
A. Supreme Court of the Philippines and the Lower Courts.
For general administration, administration of personnel benefits,
supervision of courts, adjudication of constitutional questions
appealed and other cases, operation and maintenance of the
Judicial and Bar Council in the Supreme Court, and the adjudication
of regional court cases, metropolitan court cases, municipal trial
court cases in Cities, municipal circuit court cases, municipal, court
cases, Shari'a district court cases and Shari'a circuit court cases as
indicated hereunder P2,095,651,000
xxx xxx xxx
Special Provisions.
1. Augmentation of any Item in the Court's Appropriations. Any
savings in the appropriation for the Supreme Court and the Lower
Courts may be utilized by the Chief Justice of the Supreme Court to
augment any item of the Court's appropriations for: (a) printing of
decisions and publications of Philippine Reports; b) commutable
terminal leaves of Justices and other personnel of the Supreme
Court and any payment of adjusted pension rates to retired Justices
entitled thereto pursuant to Administrative Matter No. 91-8-225CA; (c) repair, maintenance, improvement, and other operating
expenses of the courts' books and periodicals; (d) purchase,
maintenance and improvement of printing equipment; e) necessary
expenses for the employment of temporary employees, contractual
and casual employees, for judicial administration; f) maintenance
and improvement of the Court's Electronic Data Processing; (g)
extraordinary expenses of the Chief Justice, attendance in
international conferences and conduct of training programs; (h)
commutable transportation and representation allowances and fringe
benefits for Justices, Clerks of Court, Court Administrator, Chief of
Offices and other Court personnel in accordance with the rates

prescribed by law; and (i) compensation of attorneys-de-oficio;


PROVIDED, that as mandated by LOI No. 489 any increases in
salary and allowances shall be subject to the usual procedures and
policies as provided for under P.D. No. 985 and other pertinent laws.
(page 1071, General Appropriations Act, FY 1992; Emphasis
supplied)
xxx xxx xxx
4. Payment of Adjusted Pension Rates to Retired Justices. The
amount herein appropriated for payment of pensions to retired
judges and justices shall include the payment of pensions at the
adjusted rates to retired justices of the Supreme Court entitled
thereto pursuant to the ruling of the Court in Administrative Matter
No. 91-8-225-C.A. (page 1071, General Appropriations Act, FY
1992).
xxx xxx xxx
Activities and Purposes
1. General Administration and Support Services.
a. General administrative Services P 43,515,000
b. Payment of retirement gratuity
of national goverment officials
and employees P 206,717,000
c. Payment of terminal leave benefits to
officials and employees antitled thereto P 55,316,000
d. Payment of pension totired jude
and justice entitled thereto P 22,500,000
(page 1071, General Appropriations Act, FY 1992)
C. COURT OF APPEALS
For general administration, administration
of personnel benefit, benefits and the
adjudication of appealed and other cases
as indicated hereunder P114,615,000
Special Provisions.

1. Authority to Use Savings. Subject to the approval of the Chief


Justice of the Supreme Court in accordance with Section 25(5),
Article VI of the Constitution of the Republic of the Philippines, the
Presiding Justice may be authorized to use any savings in any item
of the appropriation for the Court of Appeals for purposes of: (1)
improving its compound and facilities; and (2) for augmenting any
deficiency in any item of its appropriation including its extraordinary
expenses and payment of adjusted pension rates to retired justices
entitled thereto pursuant to Administrative Matter No. 91-8-225C.A.(page 1079, General Appropriations Act, FY 1992; Emphasis
supplied)
2. Payment of adjustment Pension Rates to Retired Justices. The
amount herein appropriated for payment of pensions to retired
judges and justices shall include the payment of pensions at the
adjusted rates to retired justices of the Court of Appeals entitled
thereto pursuant to the Ruling of the Supreme Court in
Administrative Matter No. 91-6-225-C.A. (page 1079 General
Appropriations Act, FY 1992).
XL. GENERAL FUND ADJUSTMENT
For general fund adjustment for
operational and special requirements
as indicated hereunder P500,000,000
xxx xxx xxx
Special Provisions
1. Use of the Fund. This fund shall be used for:
xxx xxx xxx
1.3. Authorized overdrafts and/or valid unbooked
obligations, including the payment of back salaries and
related personnel benefits arising from decision of
competent authorityincluding the Supreme Court
decision in Administrative Matter No. 91-8-225-C.A. and
COA decision in No. 1704." (page 11649 Gen.
Appropriations Act, FY 1992; Emphasis supplied)

On January 15, 1992, the President vetoed the underlined portions of Section 1
and the entire Section 4 the Special Provisions for the Supreme Court of the
Philippines and the Lower Courts (General Appropriations Act, FY 1992, page
1071) and the underlined portions of Section 1 and the entire Section 2, of the
Special Provisions for the Court of Appeals (page 1079) and the underlined
portions of Section 1.3 of Article XLV of the Special Provisions of the General
Fund Adjustments (page 1164, General Appropriations Act, FY 1992).
The reason given for the veto of said provisions is that "the resolution of this
Honorable Court in Administrative Matter No. 91-8-225-CA pursuant to which the
foregoing appropriations for the payment of the retired Justices of the Supreme
Court and the Court of Appeals have been enacted effectively nullified the veto of
the President on House Bill No. 16297, the bill which provided for the automatic
increase in the retirement pensions of the Justices of the Supreme Court and the
Court of Appeals and chairmen of the Constitutional Commissions by re-enacting
Republic Act No. 1797 and Republic Act No. 3595. The President's veto of the
aforesaid provisions was further justified by reiterating the earlier reasons for
vetoing House Bill No. 16297: "they would erode the very foundation of our
collective effort to adhere faithfully to and enforce strictly the policy and
standardization of compensation. We should not permit the grant of distinct
privileges to select group of officials whose retirement pensions under existing
laws already enjoy preferential treatment over those of the vast majority of our
civil servants."
Hence, the instant petition filed by the petitioners with the assertions that:
1) The subject veto is not an item veto;
2) The veto by the Executive is violative of the doctrine of separation
of powers;
3) The veto deprives the retired Justices of their rights to the
pensions due them;
4) The questioned veto impairs the Fiscal Autonomy guaranteed by
the Constitution.
Raising similar grounds, the petitioners in AM-91-8-225-CA, brought to the
attention of this Court that the veto constitutes no legal obstacle to the continued
payment of the adjusted pensions pursuant to the Court's resolution.
On February 14, 1992, the Court resolved to consolidate Administrative Matter
No. 91-8-225-CA with G.R. No. 103524.

The petitioners' contentions are well-taken.


I
It cannot be overstressed that in a constitutional government such as ours, the
rule of law must prevail. The Constitution is the basic and paramount law to
which all other laws must conform and to which all persons including the highest
official of this land must defer. From this cardinal postulate, it follows that the
three branches of government must discharge their respective functions within
the limits of authority conferred by the Constitution. Under the principle of
separation of powers, neither Congress, the President nor the Judiciary may
encroach on fields allocated to the other branches of government. The legislature
is generally limited to the enactment of laws, the executive to the enforcement of
laws and the judiciary to their interpretation and application to cases and
controversies.
The Constitution expressly confers or the judiciary the power to maintain inviolate
what it decrees. As the guardian of the Constitution we cannot shirk the duty of
seeing to it that the officers in each branch of government do not go beyond their
constitutionally allocated boundaries and that the entire Government itself or any
of its branches does not violate the basic liberties of the people. The essence of
this judicial duty was emphatically explained by Justice Laurel in the leading case
of Angara v. Electoral Commission, (63 Phil. 139 [1936]) to wit:
The Constitution is a definition of the powers of government. Who is
to determine the nature, scope and extent of such powers? The
Constitution itself has provided for the instrumentality of the judiciary
as the rational way. And when the judiciary mediates to allocate
constitutional boundaries it does not assert any superiority over the
other department, it does not in reality nullify or invalidate an act of
the legislature, but only asserts the solemn and sacred obligation
assigned to it by the Constitution to determine conflicting claims of
authority under the Constitution and to establish for the parties in an
actual controversy the rights which that instrument secures and
guarantees to them. (Emphasis supplied)
The act of the Executive in vetoing the particular provisions is an exercise of a
constitutionally vested power. But even as the Constitution grants the power, it
also provides limitations to its exercise. The veto power is not absolute.
The pertinent provision of the Constitution reads:

The President shall have the power to veto any particular item or
items in an appropriation, revenue or tariff bill but the veto shall not
affect the item or items to which he does not object. (Section 27(2),
Article VI, Constitution)
The OSG is correct when it states that the Executive must veto a bill in its
entirety or not at all. He or she cannot act like an editor crossing out specific
lines, provisions, or paragraphs in a bill that he or she dislikes. In the exercise of
the veto power, it is generally all or nothing. However, when it comes to
appropriation, revenue or tariff bills, the Administration needs the money to run
the machinery of government and it can not veto the entire bill even if it may
contain objectionable features. The President is, therefore, compelled to approve
into law the entire bill, including its undesirable parts. It is for this reason that the
Constitution has wisely provided the "item veto power" to avoid inexpedient riders
being attached to an indispensable appropriation or revenue measure.
The Constitution provides that only a particular item or items may be vetoed. The
power to disapprove any item or items in an appropriate bill does not grant the
authority to veto a part of an item and to approve the remaining portion of the
same item. (Gonzales v. Macaraig, Jr., 191 SCRA 452, 464 [1990])
We distinguish an item from a provision in the following manner:
The terms item and provision in budgetary legislation and practice
are concededly different. An item in a bill refers to the particulars, the
details, the distinct and severable parts . . . of the bill
(Bengzon,supra, at 916.) It is an indivisible sum of money dedicated
to a stated purpose (Commonwealth v. Dodson, 11 S.E. 2d 120,
124, 125, etc., 176 Va. 281) The United States Supreme Court, in
the case ofBengzon v. Secretary of Justice (299 U.S. 410, 414, 57
Ct. 252, 81 L. Ed, 312) declared "that an "tem"of an appropriation bill
obviously means an item which in itself is a specific appropriation of
money, not some general provision of law, which happens to be put
into an appropriation bill." (id. at page 465)
We regret having to state that misimpressions or unfortunately wrong advice
must have been the basis of the disputed veto.
The general fund adjustment is an item which appropriates P500,000,000.00 to
enable the Government to meet certain unavoidable obligations which may have
been inadequately funded by the specific items for the different branches,
departments, bureaus, agencies, and offices of the government.

The President did not veto this item. What were vetoed were methods or systems
placed by Congress to insure that permanent and continuing obligations to
certain officials would be paid when they fell due.
An examination of the entire sections and the underlined portions of the law
which were vetoed will readily show that portions of the item have been chopped
up into vetoed and unvetoed parts. Less than all of an item has been vetoed.
Moreover, the vetoed portions are not items. They are provisions.
Thus, the augmentation of specific appropriations found inadequate to pay
retirement payments, by transferring savings from other items of appropriation is
a provision and not an item. It gives power to the Chief Justice to transfer funds
from one item to another. There is no specific appropriation of money involved.
In the same manner, the provision which states that in compliance with decisions
of the Supreme Court and the Commission on Audit, funds still undetermined in
amount may be drawn from the general fund adjustment is not an item. It is the
"general fund adjustment" itself which is the item. This was not touched. It was
not vetoed.
More ironic is the fact that misinformation led the Executive to believe that the
items in the 1992 Appropriations Act were being vetoed when, in fact, the veto
struck something else.
What were really vetoed are:
(1) Republic Act No. 1797 enacted as early as June 21, 1957; and
(2) The Resolution of the Supreme Court dated November 28, 1991 in
Administrative Matter No. 91-8-225-CA.
We need no lengthy justifications or citations of authorities to declare that no
President may veto the provisions of a law enacted thirty-five (35) years before
his or her term of office. Neither may the President set aside or reverse a final
and executory judgment of this Court through the exercise of the veto power.
A few background facts may be reiterated to fully explain the unhappy situation.
Republic Act No. 1797 provided for the adjustment of pensions of retired Justices
which privilege was extended to retired members of Constitutional Commissions
by Republic Act No. 3595.
On January 25, 1975, President Marcos issued Presidential Decree No. 644
which repealed Republic Acts 1797 and 3595. Subsequently, automatic

readjustment of pensions for retired Armed Forces officers and men was
surreptitiously restored through Presidential Decree Nos. 1638 and 1909.
It was the impression that Presidential Decree No. 644 had reduced the pensions
of Justices and Constitutional Commissioners which led Congress to restore the
repealed provisions through House Bill No. 16297 in 1990. When her finance and
budget advisers gave the wrong information that the questioned provisions in the
1992 General Appropriations Act were simply an attempt to overcome her earlier
1990 veto, she issued the veto now challenged in this petition.
It turns out, however, that P.D. No. 644 never became valid law. If P.D. No. 644
was not law, it follows that Rep. Act No. 1797 was not repealed and continues to
be effective up to the present. In the same way that it was enforced from 1951 to
1975, so should it be enforced today.
House Bill No. 16297 was superfluous as it tried to restore benefits which were
never taken away validly. The veto of House Bill No. 16297 in 1991 did not also
produce any effect. Both were based on erroneous and non-existent premises.
From the foregoing discussion, it can be seen that when the President vetoed
certain provisions of the 1992 General Appropriations Act, she was actually
vetoing Republic Act No. 1797 which, of course, is beyond her power to
accomplish.
Presidential Decree No. 644 which purportedly repealed Republic Act No. 1717
never achieved that purpose because it was not properly published. It never
became a law.
The case of Tada v. Tuvera (134 SCRA 27 [1985]and 146 SCRA 446 [1986])
specifically requires that "all laws shall immediately upon their approval or as
soon thereafter as possible, be published in full in the Official Gazette, to become
effective only after fifteen days from their publication, or on another date
specified by the legislature, in accordance with Article 2 of the Civil Code." This
was the Court's answer to the petition of Senator Lorenzo Taada and other
opposition leaders who challenged the validity of Marcos' decrees which, while
never published, were being enforced. Secret decrees are anathema in a free
society.
In support of their request, the petitioners in Administrative Matter No. 91-9-225CA secured certification from Director Lucita C. Sanchez of the National Printing
Office that the April 4, 1977 Supplement to the Official Gazette was published
only on September 5, 1983 and officially released on September 29, 1983.

On the issue of whether or not Presidential Decree 644 became law, the Court
has already categorically spoken in a definitive ruling on the matter, to wit:
xxx xxx xxx
PD 644 was promulgated by President Marcos on January 24, 1975,
but was not immediately or soon thereafter published although
preceding and subsequent decrees were duly published in the
Official Gazette. It now appears that it was intended as a secret
decree "NOT FOR PUBLICATION" as the notation on the face of the
original copy thereof plainly indicates (Annex B). It is also clear that
the decree was published in the back-dated Supplement only after it
was challenged in the Taada case as among the presidential
decrees that had not become effective for lack of the required
publication. The petition was filed on May 7, 1983, four months
before the actual publication of the decree.
It took more than eight years to publish the decree after its
promulgation in 1975. Moreover, the publication was made in bad
faith insofar as it purported to show that it was done in 1977 when
the now demonstrated fact is that the April 4, 1977 supplement was
actually published and released only in September 1983. The
belated publication was obviously intended to refute the petitioner's
claim in theTaada case and to support the Solicitor General's
submission that the petition had become moot and academic.
xxx xxx xxx
We agree that PD 644 never became a law because it was not
validly published and that, consequently, it did not have the effect of
repealing RA 1797. The requesting Justices (including Justice Lood,
whose request for the upgrading of his pension was denied on
January 15, 1991) are therefore entitled to be paid their monthly
pensions on the basis of the latter measure, which remains
unchanged to date.
The Supreme Court has spoken and it has done so with finality, logically and
rightly so as to assure stability in legal relations, and avoid confusion. (see Ver v.
Quetullo, 163 SCRA 80 [1988]) Like other decisions of this Court, the ruling and
principles set out in the Court resolution constitute binding precedent. (BuligBulig Kita Kamaganak Association, et al. v. Sulpicio Lines, Inc., Regional Trial
Court, etc., G.R. 847500 16 May 1989, En Banc, Minute Resolution)

The challenged veto has far-reaching implications which the Court can not
countenance as they undermine the principle of separation of powers. The
Executive has no authority to set aside and overrule a decision of the Supreme
Court.
We must emphasize that the Supreme Court did not enact Rep. Act No. 1797. It
is not within its powers to pass laws in the first place. Its duty is confined to
interpreting or defining what the law is and whether or not it violates a provision
of the Constitution.
As early as 1953, Congress passed a law providing for retirement pensions to
retired Justices of the Supreme Court and the Court of Appeals. This law was
amended by Republic Act 1797 in 1957. Funds necessary to pay the retirement
pensions under these statutes are deemed automatically appropriated every
year.
Thus, Congress included in the General Appropriations Act of 1992, provisions
identifying funds and savings which may be used to pay the adjusted pensions
pursuant to the Supreme Court Resolution. As long as retirement laws remain in
the statute book, there is an existing obligation on the part of the government to
pay the adjusted pension rate pursuant to RA 1797 and AM-91-8-225-CA.
Neither may the veto power of the President be exercised as a means of
repealing RA 1797. This is arrogating unto the Presidency legislative powers
which are beyond its authority. The President has no power to enact or amend
statutes promulgated by her predecessors much less to repeal existing laws. The
President's power is merely to execute the laws as passed by Congress.
II
There is a matter of greater consequence arising from this petition. The attempt
to use the veto power to set aside a Resolution of this Court and to deprive
retirees of benefits given them by Rep. Act No. 1797 trenches upon the
constitutional grant of fiscal autonomy to the Judiciary.
Sec. 3, Art. VIII mandates that:
Sec. 3 The Judiciary shall enjoy fiscal autonomy. Appropriations for
the Judiciary may not be reduced by the legislature below the
amount appropriated for the previous year and, after approval, shall
be automatically and regularly released.

We can not overstress the importance of and the need for an independent
judiciary. The Court has on various past occasions explained the significance of
judicial independence. In the case of De la Llana v. Alba (112 SCRA 294 [1982]),
it ruled:
It is a cardinal rule of faith of our constitutional regime that it is the
people who are endowed with rights, to secure which a government
is instituted. Acting as it does through public officials, it has to grant
them either expressly or implicitly certain powers. These they
exercise not for their own benefit but for the body politic. . . .
A public office is a public trust. That is more than a moral adjuration.
It is a legal imperative. The law may vest in a public official certain
rights. It does so to enable them to perform his functions and fulfill
his responsibilities more efficiently. . . . It is an added guarantee that
justices and judges can administer justice undeterred by any fear of
reprisal or untoward consequence. Their judgments then are even
more likely to be inspired solely by their knowledge of the law and
the dictates of their conscience, free from the corrupting influence of
base or unworthy motives. The independence of which they are
assured is impressed with a significance transcending that of a
purely personal right. (At pp. 338-339)
The exercise of the veto power in this case may be traced back to the efforts of
the Department of Budget and Management (DBM) to ignore or overlook the
plain mandate of the Constitution on fiscal autonomy. The OSG Comment
reflects the same truncated view of the provision.
We have repeatedly in the past few years called the attention of DBM that not
only does it allocate less than one percent (1%) of the national budget annually
for the 22,769 Justices, Judges, and court personnel all over the country but it
also examines with a fine-toothed come how we spend the funds appropriated by
Congress based on DBM recommendations.
The gist of our position papers and arguments before Congress is as follows:
The DBM requires the Supreme Court, with Constitutional
Commissions, and the Ombudsman to submit budget proposals in
accordance with parameters it establishes. DBM evaluates the
proposals, asks each agency to defend its proposals during DBM
budget hearings, submits its own version of the proposals to
Congress without informing the agency of major alterations and

mutilations inflicted on their proposals, and expects each agency to


defend in Congress proposals not of the agency's making.
After the general appropriations bill is passed by Congress and
signed into law by the President, the tight and officious control by
DBM continues. For the release of appropriated funds, the Judiciary,
Constitutional Commissions, and Ombudsman are instructed
through "guidelines", how to prepare Work and Financial Plans and
requests for monthly allotments. The DBM evaluates and approves
these plans and requests and on the basis of its approval authorizes
the release of allotments with corresponding notices of cash
allocation. These notices specify the maximum withdrawals each
month which the Supreme Court, the Commissions and the
Ombudsman may make from the servicing government bank. The
above agencies are also required to submit to DBM monthly,
quarterly and year-end budget accountability reports to indicate their
performance, physical and financial operations and income,
The DBM reserves to itself the power to review the accountability
reports and when importuned for needed funds, to release additional
allotments to the agency. Since DBM always prunes the budget
proposals to below subsistence levels and since emergency
situations usually occur during the fiscal year, the Chief Justices,
Chairmen of the Commissions, and Ombudsman are compelled to
make pilgrimages to DBM for additional funds to tide their respective
agencies over the emergency.
What is fiscal autonomy?
As envisioned in the Constitution, the fiscal autonomy enjoyed by the Judiciary,
the Civil Service Commission, the Commission on Audit, the Commission on
Elections, and the Office of the Ombudsman contemplates a guarantee on full
flexibility to allocate and utilize their resources with the wisdom and dispatch that
their needs require. It recognizes the power and authority to levy, assess and
collect fees, fix rates of compensation not exceeding the highest rates authorized
by law for compensation and pay plans of the government and allocate and
disburse such sums as may be provided by law or prescribed by them in the
course of the discharge of their functions.
Fiscal autonomy means freedom from outside control. If the Supreme Court says
it needs 100 typewriters but DBM rules we need only 10 typewriters and sends
its recommendations to Congress without even informing us, the autonomy given
by the Constitution becomes an empty and illusory platitude.

The Judiciary, the Constitutional Commissions, and the Ombudsman must have
the independence end flexibility needed in the discharge of their constitutional
duties. The imposition of restrictions and constraints on the manner the
independent constitutional offices allocate and utilize the funds appropriated for
their operations is anathema to fiscal autonomy and violative not only of the
express mandate of the Constitution but especially as regards the Supreme
Court, of the independence and separation of powers upon which the entire
fabric of our constitutional system is based. In the interest of comity and
cooperation, the Supreme Court, Constitutional Commissions, and the
Ombudsman have so far limited their objections to constant reminders. We now
agree with the petitioners that this grant of autonomy should cease to be a
meaningless provision.
In the case at bar, the veto of these specific provisions in the General
Appropriations Act is tantamount to dictating to the Judiciary how its funds should
be utilized, which is clearly repugnant to fiscal autonomy. The freedom of the
Chief Justice to make adjustments in the utilization of the funds appropriated for
the expenditures of the judiciary, including the use of any savings from any
particular item to cover deficits or shortages in other items of the Judiciary is
withheld. Pursuant to the Constitutional mandate, the Judiciary must enjoy
freedom in the disposition of the funds allocated to it in the appropriations law. It
knows its priorities just as it is aware of the fiscal restraints. The Chief Justice
must be given a free hand on how to augment appropriations where
augmentation is needed.
Furthermore, in the case of Gonzales v. Macaraig (191 SCRA 452 [1990]), the
Court upheld the authority of the President and other key officials to augment any
item or any appropriation from savings in the interest of expediency and
efficiency. The Court stated that:
There should be no question, therefore, that statutory authority has,
in fact, been granted. And once given, the heads of the different
branches of the Government and those of the Constitutional
Commissions are afforded considerable flexibility in the use of public
funds and resources (Demetria v. Alba, supra). The doctrine of
separation of powers is in no way endangered because the transfer
is made within a department (or branch of government) and not from
one department (branch) to another.
The Constitution, particularly Article VI, Section 25(5) also provides:
Sec. 25. (5) No law shall be passed authorizing any transfer of
appropriations; however, the President, the President of the Senate,

the Speaker of the House of Representatives, the Chief Justice of


the Supreme Court, and the heads of Constitutional Commissions
may, by law, be authorized to augment any item in the general
appropriations law for their respective offices from savings in other
items of their respective appropriations.
In the instant case, the vetoed provisions which relate to the use of savings for
augmenting items for the payment of the pension differentials, among others, are
clearly in consonance with the abovestated pronouncements of the Court. The
veto impairs the power of the Chief Justice to augment other items in the
Judiciary's appropriation, in contravention of the constitutional provision on "fiscal
autonomy."
III
Finally, it can not be denied that the retired Justices have a vested right to the
accrued pensions due them pursuant to RA 1797.
The right to a public pension is of statutory origin and statutes dealing with
pensions have been enacted by practically all the states in the United States
(State ex rel. Murray v, Riley, 44 Del 505, 62 A2d 236), and presumably in most
countries of the world. Statutory provisions for the support of Judges or Justices
on retirement are founded on services rendered to the state. Where a judge has
complied with the statutory prerequisite for retirement with pay, his right to retire
and draw salary becomes vested and may not, thereafter, be revoked or
impaired. (Gay v. Whitehurst, 44 So ad 430)
Thus, in the Philippines, a number of retirement laws have been enacted, the
purpose of which is to entice competent men and women to enter the
government service and to permit them to retire therefrom with relative security,
not only those who have retained their vigor but, more so, those who have been
incapacitated by illness or accident. (In re: Amount of the Monthly Pension of
Judges and Justices Starting From the Sixth Year of their Retirement and After
the Expiration of the Initial Five-year Period of Retirement, (190 SCRA 315
[1990]).
As early as 1953, Rep. Act No. 910 was enacted to grant pensions to retired
Justices of the Supreme Court and Court of Appeals.
This was amended by RA 1797 which provided for an automatic adjustment of
the pension rates. Through the years, laws were enacted and jurisprudence
expounded to afford retirees better benefits.

P.D. No. 1438, for one, was promulgated on June 10, 1978 amending RA 910
providing that the lump sum of 5 years gratuity to which the retired Justices of the
Supreme Court and Court of Appeals were entitled was to be computed on the
basis of the highest monthly aggregate of transportation, living and
representation allowances each Justice was receiving on the date of his
resignation. The Supreme Court in a resolution dated October 4, 1990, stated
that this law on gratuities covers the monthly pensions of retired Judges and
Justices which should include the highest monthly aggregate of transportation,
living and representation allowances the retiree was receiving on the date of
retirement. (In Re: Amount of the Monthly Pension of Judges and
Justices, supra)
The rationale behind the veto which implies that Justices and Constitutional
officers are unduly favored is, again, a misimpression.
Immediately, we can state that retired Armed Forces officers and enlisted
men number in the tens of thousands while retired Justices are so few they can
be immediately identified. Justices retire at age 70 while military men retire at a
much younger age some retired Generals left the military at age 50 or earlier.
Yet the benefits in Rep. Act No. 1797 are made to apply equally to both groups.
Any ideas arising from an alleged violation of the equal protection clause should
first be directed to retirees in the military or civil service where the reason for the
retirement provision is not based on indubitable and constitutionally sanctioned
grounds, not to a handful of retired Justices whose retirement pensions are
founded on constitutional reasons.
The provisions regarding retirement pensions of justices arise from the package
of protections given by the Constitution to guarantee and preserve the
independence of the Judiciary.
The Constitution expressly vests the power of judicial review in this Court. Any
institution given the power to declare, in proper cases, that act of both the
President and Congress are unconstitutional needs a high degree of
independence in the exercise of its functions. Our jurisdiction may not be
reduced by Congress. Neither may it be increased without our advice and
concurrence. Justices may not be removed until they reach age 70 except
through impeachment. All courts and court personnel are under the
administrative supervision of the Supreme Court. The President may not appoint
any Judge or Justice unless he or she has been nominated by the Judicial and
Bar Council which, in turn, is under the Supreme Court's supervision. Our
salaries may not be decreased during our continuance in office. We cannot be
designated to any agency performing administrative or quasi-judicial functions.
We are specifically given fiscal autonomy. The Judiciary is not only independent

of, but also co-equal and coordinate with the Executive and Legislative
Departments. (Article VIII and section 30, Article VI, Constitution)
Any argument which seeks to remove special privileges given by law to former
Justices of this Court and the ground that there should be no "grant of distinct
privileges" or "preferential treatment" to retired Justices ignores these provisions
of the Constitution and, in effect, asks that these Constitutional provisions on
special protections for the Judiciary be repealed. The integrity of our entire
constitutional system is premised to a large extent on the independence of the
Judiciary. All these provisions are intended to preserve that independence. So
are the laws on retirement benefits of Justices.
One last point.
The Office of the Solicitor General argues that:
. . . Moreover, by granting these benefits to retired Justices implies
that public funds, raised from taxes on other citizens, will be paid off
to select individuals who are already leading private lives and have
ceased performing public service. Said the United States Supreme
Court, speaking through Mr. Justice Miller: "To lay with one hand the
power of the government on the property of the citizen, and with the
other to bestow upon favored individuals . . . is nonetheless a
robbery because it is done under the forms of law . . ." (Law
Association V. Topeka, 20 Wall. 655) (Comment, p. 16)
The above arguments are not only specious, impolite and offensive; they
certainly are unbecoming of an office whose top officials are supposed to be,
under their charter, learned in the law.
Chief Justice Cesar Bengzon and Chief Justice Querube Makalintal, Justices
J.B.L. Reyes, Cecilia Muoz Palma, Efren Plana, Vicente Abad Santos, and, in
fact, all retired Justices of the Supreme Court and the Court of Appeals may no
longer be in the active service. Still, the Solicitor General and all lawyers under
him who represent the government before the two courts and whose
predecessors themselves appeared before these retirees, should show some
continuing esteem and good manners toward these Justices who are now in the
evening of their years.
All that the retirees ask is to be given the benefits granted by law. To
characterize them as engaging in "robbery" is intemperate, abrasive, and
disrespectful more so because the argument is unfounded.

If the Comment is characteristic of OSG pleadings today, then we are sorry to


state that the then quality of research in that institution has severely deteriorated.
In the first place, the citation of the case is, wrong. The title is not LAW
Association v. Topeka but Citizen's Savings and Loan Association of Cleveland,
Ohio v. Topeka City (20 Wall. 655; 87 U.S. 729; 22 Law. Ed. 455 [1874]. Second,
the case involved the validity of a statute authorizing cities and counties to issue
bonds for the purpose of building bridges, waterpower, and other public works to
aid private railroads improve their services. The law was declared void on the
ground that the right of a municipality to impose a tax cannot be used for private
interests.
The case was decided in 1874. The world has turned over more than 40,000
times since that ancient period. Public use is now equated with public interest.
Public money may now be used for slum clearance, low-cost housing, squatter
resettlement, urban and agrarian reform where only private persons are the
immediate beneficiaries. What was "robbery" in 1874 is now called "social
justice." There is nothing about retirement benefits in the cited case. Obviously,
the OSG lawyers cited from an old textbook or encyclopedia which could not
even spell "loan" correctly. Good lawyers are expected to go to primary sources
and to use only relevant citations.
The Court has been deluged with letters and petitions by former colleagues in the
Judiciary requesting adjustments in their pensions just so they would be able to
cope with the everyday living expenses not to mention the high cost of medical
bills that old age entails. As Justice Cruz aptly stated in Teodoro J. Santiago v.
COA, (G.R. No. 92284, July 12, 1991);
Retirement laws should be interpreted liberally in favor of the retiree
because their intention is to provide for his sustenance, and
hopefully even comfort, when he no longer has the stamina to
continue earning his livelihood. After devoting the best years of his
life to the public service, he deserves the appreciation of a grateful
government as best concretely expressed in a generous retirement
gratuity commensurate with the value and length of his services.
That generosity is the least he should expect now that his work is
done and his youth is gone. Even as he feels the weariness in his
bones and glimpses the approach of the lengthening shadows, he
should be able to luxuriate in the thought that he did his task well,
and was rewarded for it.

For as long as these retired Justices are entitled under laws which continue to be
effective, the government can not deprive them of their vested right to the
payment of their pensions.
WHEREFORE, the petition is hereby GRANTED. The questioned veto is SET
ASIDE as illegal and unconstitutional. The vetoed provisions of the 1992
Appropriations Act are declared valid and subsisting. The respondents are
ordered to automatically and regularly release pursuant to the grant of fiscal
autonomy the funds appropriated for the subject pensions as well as the other
appropriations for the Judiciary. The resolution in Administrative Matter No. 91-8225-CA dated November 28, 1991 is likewise ordered to be implemented as
promulgated.
SO ORDERED.

BENGZON VS. DRILON G.R. 103524 April


15, 1992 208 SCRA 133
BENGZON VS. DRILON
G.R. 103524 April 15, 1992 208 SCRA 133
Gutierrez, J.:
FACTS:
Petitioners are retired justices of the Supreme Court and Court of Appeals who are
currently receiving pensions under RA 910 as amended by RA 1797. President Marcos
issued a decree repealing section 3-A of RA 1797 which authorized the adjustment of
the pension of retired justices and officers and enlisted members of the AFP. PD 1638
was eventually issued by Marcos which provided for the automatic readjustment of the
pension of officers and enlisted men was restored, while that of the retired justices was
not. RA 1797 was restored through HB 16297 in 1990. When her advisers gave the
wrong information that the questioned provisions in 1992 GAA were an attempt to
overcome her earlier veto in 1990, President Aquino issued the veto now challenged in
this petition.
It turns out that PD 644 which repealed RA 1797 never became a valid law absent its
publication, thus there was no law. It follows that RA 1797 was still in effect and HB
16297 was superfluous because it tried to restore benefits which were never taken
away validly. The veto of HB 16297 did not also produce any effect.
ISSUE:
Whether or not the veto of the President of certain provisions in the GAA of FY 1992
relating to the payment of the adjusted pensions of retired Justices is constitutional or
valid.
HELD:
The veto of these specific provisions in the GAA is tantamount to dictating to the
Judiciary ot its funds should be utilized, which is clearly repugnant to fiscal autonomy.

Pursuant to constitutional mandate, the Judiciary must enjoy freedom in the disposition
of the funds allocated to it in the appropriations law.
Any argument which seeks to remove special privileges given by law to former Justices
on the ground that there should be no grant of distinct privileges or preferential
treatment to retired Justices ignores these provisions of the Constitution and in effect
asks that these Constitutional provisions on special protections for the Judiciary be
repealed.
The petition is granted and the questioned veto is illegal and the provisions of 1992
GAA are declared valid and subsisting.
PHILCONSA VS ENRIQUEZ

235 SCRA 506 Political Law Veto Power Part of the Legislative Process
Constitutionality of the Pork Barrel Countrywide Development Fund
This is a consolidation of cases which sought to question the veto authority of the
president involving theGeneral Appropriations Bill of 1994 as well as the
constitutionality
of
the
pork
barrel.
The
Philippine
ConstitutionAssociation (PHILCONSA)
questions
the
countrywide development fund. PHILCONSA said that Congress can only
allocate funds but they cannot specify the items as to which those funds would
be applied for since that is already the function of the executive.
In G.R. No. 113766, after the vetoing by the president of some provisions of the
GAB of 1994, neither house of congress took steps to override the veto. Instead,
Senators Wigberto Taada and Alberto Romulo sought the issuance of the writs
of prohibition and mandamus against Executive Secretary Teofisto Guingona et
al. Taada et al contest the constitutionality of: (1) the veto on four special
provisions added to items in the GAB of 1994 for the Armed Forces of the
Philippines (AFP) and the Department of Public Works and Highways (DPWH);
and (2) the conditions imposed by the President in the implementation of certain
appropriations for the CAFGUs, the DPWH, and the National Housing Authority
(NHA).
ISSUE: Whether or not the Presidents veto is valid.
HELD: In the PHILCONSA petition, the SC ruled that Congress acted within its
power and that the CDF is constitutional. In the Taada petitions the SC
dismissed the other petitions and granted the others.
Veto on special provisions
The president did his veto with certain conditions and compliant to the ruling
in Gonzales vs Macaraig. The president particularly vetoed the debt reduction
scheme in the GAA of 1994 commenting that the scheme is already taken cared
of by other legislation and may be more properly addressed by revising the debt
policy. He, however did not delete the P86,323,438,000.00 appropriation

therefor. Taada et al averred that the president cannot validly veto that provision
w/o vetoing the amount allotted therefor. The veto of the president herein is
sustained for the vetoed provision is considered inappropriate; in fact the Sc
found that such provision if not vetoed would in effect repeal the Foreign
Borrowing Act making the legislation as a log-rolling legislation.
Veto of provisions for revolving funds of SUCs
The appropriation for State Universities and Colleges (SUCs), the President
vetoed special provisions which authorize the use of income and the creation,
operation and maintenance of revolving funds was likewise vetoed. The reason
for the veto is that there were already funds allotted for the same in the National
expenditure Program. Taada et al claimed this as unconstitutional. The SC
ruled that the veto is valid for it is in compliant to the One Fund Policy it
avoided double funding and redundancy.
Veto of provision on 70% (administrative)/30% (contract) ratio for road
maintenance
The President vetoed this provision on the basis that it may result to a breach of
contractual obligations. The funds if allotted may result to abandonment of some
existing contracts. The SC ruled that this Special Provision in question is not an
inappropriate provision which can be the subject of a veto. It is not alien to the
appropriation for road maintenance, and on the other hand, it specifies how the
said item shall be expended 70% by administrative and 30% by contract. The
1987 Constitution allows the addition by Congress of special provisions,
conditions to items in an expenditure bill, which cannot be vetoed separately from
the items to which they relate so long as they are appropriate in the budgetary
sense. The veto herein is then not valid.
Veto of provision on prior approval of Congress for purchase of military
equipment
As reason for the veto, the President stated that the said condition and
prohibition violate the Constitutional mandate of non-impairment of contractual
obligations, and if allowed, shall effectively alter the original intent of the AFP
Modernization Fund to cover all military equipment deemed necessary to
modernize the AFP. The SC affirmed the veto. Any provision blocking an
administrative action in implementing a law or requiring legislative approval of
executive acts must be incorporated in a separate and substantive bill.
Therefore, being inappropriate provisions.
Veto of provision on use of savings to augment AFP pension funds
According to the President, the grant of retirement and separation benefits
should be covered by direct appropriations specifically approved for the purpose

pursuant to Section 29(1) of Article VI of the Constitution. Moreover, he stated


that the authority to use savings is lodged in the officials enumerated in Section
25(5) of Article VI of the Constitution. The SC retained the veto per reasons
provided by the president.
Condition on the deactivation of the CAFGUs
Congress appropriated compensation for the CAFGUs including the payment of
separation benefits. The President declared in his Veto Message that the
implementation of this Special Provision to the item on the CAFGUs shall be
subject to prior Presidential approval pursuant to P.D. No. 1597 and R.A. No.
6758. The SC ruled to retain the veto per reasons provided by the president.
Further, if this provision is allowed the it would only lead to the repeal of said
existing laws.
Conditions on the appropriation for the Supreme Court, etc
In his veto message: The said condition is consistent with the Constitutional
injunction prescribed under Section 8, Article IX-B of the Constitutional which
states that no elective or appointive public officer or employee shall receive
additional, double, or indirect compensation unless specifically authorized by
law. I am, therefore, confident that the heads of the said offices shall maintain
fidelity to the law and faithfully adhere to the well-established principle on
compensation standardization. Taada et al claim that the conditions imposed by
the President violated the independence and fiscal autonomy of the Supreme
court, the Ombudsman, the COA and the CHR. The SC sustained the veto: In
the first place, the conditions questioned by petitioners were placed in the GAB
by Congress itself, not by the President. The Veto Message merely highlighted
the Constitutional mandate that additional or indirect compensation can only be
given pursuant to law. In the second place, such statements are mere reminders
that the disbursements of appropriations must be made in accordance with law.
Such statements may, at worse, be treated as superfluities.
Pork Barrel Constitutional
The pork barrel makes the unequal equal. The Congressmen, being
representatives of their local districts know more about the problems in their
constituents areas than the national government or the president for that matter.
Hence, with that knowledge, the Congressmen are in a better position to
recommend as to where funds should be allocated.

Republic of the Philippines


SUPREME COURT
Manila
EN BANC
G.R. No. 81311 June 30, 1988
KAPATIRAN NG MGA NAGLILINGKOD SA PAMAHALAAN NG PILIPINAS,
INC., HERMINIGILDO C. DUMLAO, GERONIMO Q. QUADRA, and MARIO
C. VILLANUEVA, petitioners,
vs.
HON. BIENVENIDO TAN, as Commissioner of Internal
Revenue, respondent.
G.R. No. 81820 June 30, 1988
KILUSANG MAYO UNO LABOR CENTER (KMU), its officers and affiliated
labor federations and alliances, petitioners,
vs.
THE EXECUTIVE SECRETARY, SECRETARY OF FINANCE, THE
COMMISSIONER OF INTERNAL REVENUE, and SECRETARY OF
BUDGET, respondents.
G.R. No. 81921 June 30, 1988
INTEGRATED CUSTOMS BROKERS ASSOCIATION OF THE
PHILIPPINES and JESUS B. BANAL, petitioners,
vs.
The HON. COMMISSIONER, BUREAU OF INTERNAL
REVENUE, respondent.
G.R. No. 82152 June 30, 1988
RICARDO C. VALMONTE, petitioner,
vs.
THE EXECUTIVE SECRETARY, SECRETARY OF FINANCE,
COMMISSIONER OF INTERNAL REVENUE and SECRETARY OF
BUDGET, respondent.
Franklin S. Farolan for petitioner Kapatiran in G.R. No. 81311.

Jaime C. Opinion for individual petitioners in G.R. No. 81311.


Banzuela, Flores, Miralles, Raeses, Sy, Taquio and Associates for
petitioners in G.R. No 81820.
Union of Lawyers and Advocates for Peoples Right collaborating counsel for
petitioners in G.R. No 81820.
Jose C. Leabres and Joselito R. Enriquez for petitioners in G.R. No. 81921.

PADILLA, J.:
These four (4) petitions, which have been consolidated because of the similarity of the main issues involved therein, seek to nullify Executive
Order No. 273 (EO 273, for short), issued by the President of the Philippines on 25 July 1987, to take effect on 1 January 1988, and which
amended certain sections of the National Internal Revenue Code and adopted the value-added tax (VAT, for short), for being unconstitutional
in that its enactment is not alledgedly within the powers of the President; that the VAT is oppressive, discriminatory, regressive, and violates
the due process and equal protection clauses and other provisions of the 1987 Constitution.
The Solicitor General prays for the dismissal of the petitions on the ground that the petitioners have failed to show justification for the
exercise of its judicial powers, viz. (1) the existence of an appropriate case; (2) an interest, personal and substantial, of the party raising the
constitutional questions; (3) the constitutional question should be raised at the earliest opportunity; and (4) the question of constitutionality is
directly and necessarily involved in a justiciable controversy and its resolution is essential to the protection of the rights of the parties.
According to the Solicitor General, only the third requisite that the constitutional question should be raised at the earliest opportunity
has been complied with. He also questions the legal standing of the petitioners who, he contends, are merely asking for an advisory opinion
from the Court, there being no justiciable controversy for resolution.
Objections to taxpayers' suit for lack of sufficient personality standing, or interest are, however, in the main procedural matters. Considering
the importance to the public of the cases at bar, and in keeping with the Court's duty, under the 1987 Constitution, to determine wether or not
the other branches of government have kept themselves within the limits of the Constitution and the laws and that they have not abused the
discretion given to them, the Court has brushed aside technicalities of procedure and has taken cognizance of these petitions.
But, before resolving the issues raised, a brief look into the tax law in question is in order.
The VAT is a tax levied on a wide range of goods and services. It is a tax on the value, added by every seller, with aggregate gross annual
sales of articles and/or services, exceeding P200,00.00, to his purchase of goods and services, unless exempt. VAT is computed at the rate
of 0% or 10% of the gross selling price of goods or gross receipts realized from the sale of services.
The VAT is said to have eliminated privilege taxes, multiple rated sales tax on manufacturers and producers, advance sales tax, and
compensating tax on importations. The framers of EO 273 that it is principally aimed to rationalize the system of taxing goods and services;
simplify tax administration; and make the tax system more equitable, to enable the country to attain economic recovery.
The VAT is not entirely new. It was already in force, in a modified form, before EO 273 was issued. As pointed out by the Solicitor General,
the Philippine sales tax system, prior to the issuance of EO 273, was essentially a single stage value added tax system computed under the
"cost subtraction method" or "cost deduction method" and was imposed only on original sale, barter or exchange of articles by
manufacturers, producers, or importers. Subsequent sales of such articles were not subject to sales tax. However, with the issuance of PD
1991 on 31 October 1985, a 3% tax was imposed on a second sale, which was reduced to 1.5% upon the issuance of PD 2006 on 31
December 1985, to take effect 1 January 1986. Reduced sales taxes were imposed not only on the second sale, but on everysubsequent
sale, as well. EO 273 merely increased the VAT on every sale to 10%, unless zero-rated or exempt.
Petitioners first contend that EO 273 is unconstitutional on the Ground that the President had no authority to issue EO 273 on 25 July 1987.
The contention is without merit.
It should be recalled that under Proclamation No. 3, which decreed a Provisional Constitution, sole legislative authority was vested upon the
President. Art. II, sec. 1 of the Provisional Constitution states:

Sec. 1. Until a legislature is elected and convened under a new Constitution, the President shall continue to exercise
legislative powers.
On 15 October 1986, the Constitutional Commission of 1986 adopted a new Constitution for the Republic of the Philippines which was
ratified in a plebiscite conducted on 2 February 1987. Article XVIII, sec. 6 of said Constitution, hereafter referred to as the 1987 Constitution,
provides:
Sec. 6. The incumbent President shall continue to exercise legislative powers until the first Congress is convened.
It should be noted that, under both the Provisional and the 1987 Constitutions, the President is vested with legislative powers until a
legislature under a new Constitution is convened. The first Congress, created and elected under the 1987 Constitution, was convened on 27
July 1987. Hence, the enactment of EO 273 on 25 July 1987, two (2) days before Congress convened on 27 July 1987, was within the
President's constitutional power and authority to legislate.
Petitioner Valmonte claims, additionally, that Congress was really convened on 30 June 1987 (not 27 July 1987). He contends that the word
"convene" is synonymous with "the date when the elected members of Congress assumed office."
The contention is without merit. The word "convene" which has been interpreted to mean "to call together, cause to assemble, or
1
convoke," is clearly different from assumption of office by the individual members of Congress or their

taking the oath of office. As an example, we call to mind the interim National Assembly created under the
1973 Constitution, which had not been "convened" but some members of the body, more particularly the
delegates to the 1971 Constitutional Convention who had opted to serve therein by voting affirmatively for
the approval of said Constitution, had taken their oath of office.
To uphold the submission of petitioner Valmonte would stretch the definition of the word "convene" a bit
too far. It would also defeat the purpose of the framers of the 1987 Constitutional and render meaningless
some other provisions of said Constitution. For example, the provisions of Art. VI, sec. 15, requiring
Congress to convene once every year on the fourth Monday of July for its regular session would be a
contrariety, since Congress would already be deemed to be in session after the individual members have
taken their oath of office. A portion of the provisions of Art. VII, sec. 10, requiring Congress toconvene for
the purpose of enacting a law calling for a special election to elect a President and Vice-President in case
a vacancy occurs in said offices, would also be a surplusage. The portion of Art. VII, sec. 11, third
paragraph, requiring Congress to convene, if not in session, to decide a conflict between the President
and the Cabinet as to whether or not the President and the Cabinet as to whether or not the President
can re-assume the powers and duties of his office, would also be redundant. The same is true with the
portion of Art. VII, sec. 18, which requires Congress to convene within twenty-four (24) hours following the
declaration of martial law or the suspension of the privilage of the writ of habeas corpus.
The 1987 Constitution mentions a specific date when the President loses her power to legislate. If the
framers of said Constitution had intended to terminate the exercise of legislative powers by the President
at the beginning of the term of office of the members of Congress, they should have so stated (but did
not) in clear and unequivocal terms. The Court has not power to re-write the Constitution and give it a
meaning different from that intended.
The Court also finds no merit in the petitioners' claim that EO 273 was issued by the President in grave
abuse of discretion amounting to lack or excess of jurisdiction. "Grave abuse of discretion" has been
defined, as follows:
Grave abuse of discretion" implies such capricious and whimsical exercise of judgment
as is equivalent to lack of jurisdiction (Abad Santos vs. Province of Tarlac, 38 Off. Gaz.
834), or, in other words, where the power is exercised in an arbitrary or despotic manner
by reason of passion or personal hostility, and it must be so patent and gross as to
amount to an evasion of positive duty or to a virtual refusal to perform the duty enjoined
2
or to act at all in contemplation of law. (Tavera-Luna, Inc. vs. Nable, 38 Off. Gaz. 62).
Petitioners have failed to show that EO 273 was issued capriciously and whimsically or in an arbitrary or
despotic manner by reason of passion or personal hostility. It appears that a comprehensive study of the

VAT had been extensively discussed by this framers and other government agencies involved in its
implementation, even under the past administration. As the Solicitor General correctly sated. "The signing
of E.O. 273 was merely the last stage in the exercise of her legislative powers. The legislative process
started long before the signing when the data were gathered, proposals were weighed and the final
wordings of the measure were drafted, revised and finalized. Certainly, it cannot be said that the
3
President made a jump, so to speak, on the Congress, two days before it convened."
Next, the petitioners claim that EO 273 is oppressive, discriminatory, unjust and regressive, in violation of
the provisions of Art. VI, sec. 28(1) of the 1987 Constitution, which states:
Sec. 28 (1) The rule of taxation shall be uniform and equitable. The Congress shall
evolve a progressive system of taxation.
The petitioners" assertions in this regard are not supported by facts and circumstances to warrant their
conclusions. They have failed to adequately show that the VAT is oppressive, discriminatory or unjust.
Petitioners merely rely upon newspaper articles which are actually hearsay and have evidentiary value.
To justify the nullification of a law. there must be a clear and unequivocal breach of the Constitution, not a
4
doubtful and argumentative implication.
As the Court sees it, EO 273 satisfies all the requirements of a valid tax. It is uniform. The court, in City of
5
Baguio vs. De Leon, said:
... In Philippine Trust Company v. Yatco (69 Phil. 420), Justice Laurel, speaking for the
Court, stated: "A tax is considered uniform when it operates with the same force and
effect in every place where the subject may be found."
There was no occasion in that case to consider the possible effect on such a
constitutional requirement where there is a classification. The opportunity came in
Eastern Theatrical Co. v. Alfonso (83 Phil. 852, 862). Thus: "Equality and uniformity in
taxation means that all taxable articles or kinds of property of the same class shall be
taxed at the same rate. The taxing power has the authority to make reasonable and
natural classifications for purposes of taxation; . . ." About two years later, Justice
Tuason, speaking for this Court in Manila Race Horses Trainers Assn. v. de la Fuente (88
Phil. 60, 65) incorporated the above excerpt in his opinion and continued; "Taking
everything into account, the differentiation against which the plaintiffs complain conforms
to the practical dictates of justice and equity and is not discriminatory within the meaning
of the Constitution."
To satisfy this requirement then, all that is needed as held in another case decided two
years later, (Uy Matias v. City of Cebu, 93 Phil. 300) is that the statute or ordinance in
question "applies equally to all persons, firms and corporations placed in similar
situation." This Court is on record as accepting the view in a leading American case
(Carmichael v. Southern Coal and Coke Co., 301 US 495) that "inequalities which result
from a singling out of one particular class for taxation or exemption infringe no
constitutional limitation." (Lutz v. Araneta, 98 Phil. 148, 153).
The sales tax adopted in EO 273 is applied similarly on all goods and services sold to the public, which
are not exempt, at the constant rate of 0% or 10%.
The disputed sales tax is also equitable. It is imposed only on sales of goods or services by persons
engage in business with an aggregate gross annual sales exceeding P200,000.00. Small corner sarisari stores are consequently exempt from its application. Likewise exempt from the tax are sales of farm
and marine products, spared as they are from the incidence of the VAT, are expected to be relatively
6
lower and within the reach of the general public.

The Court likewise finds no merit in the contention of the petitioner Integrated Customs Brokers
Association of the Philippines that EO 273, more particularly the new Sec. 103 (r) of the National Internal
Revenue Code, unduly discriminates against customs brokers. The contested provision states:
Sec. 103. Exempt transactions. The following shall be exempt from the value-added
tax:
xxx xxx xxx
(r) Service performed in the exercise of profession or calling (except customs brokers)
subject to the occupation tax under the Local Tax Code, and professional services
performed by registered general professional partnerships;
The phrase "except customs brokers" is not meant to discriminate against customs brokers. It was
inserted in Sec. 103(r) to complement the provisions of Sec. 102 of the Code, which makes the services
of customs brokers subject to the payment of the VAT and to distinguish customs brokers from other
professionals who are subject to the payment of an occupation tax under the Local Tax Code. Pertinent
provisions of Sec. 102 read:
Sec. 102. Value-added tax on sale of services. There shall be levied, assessed and
collected, a value-added tax equivalent to 10% percent of gross receipts derived by any
person engaged in the sale of services. The phrase sale of services" means the
performance of all kinds of services for others for a fee, remuneration or consideration,
including those performed or rendered by construction and service contractors; stock,
real estate, commercial, customs and immigration brokers; lessors of personal property;
lessors or distributors of cinematographic films; persons engaged in milling, processing,
manufacturing or repacking goods for others; and similar services regardless of whether
or not the performance thereof call for the exercise or use of the physical or mental
faculties: ...
With the insertion of the clarificatory phrase "except customs brokers" in Sec. 103(r), a potential conflict
between the two sections, (Secs. 102 and 103), insofar as customs brokers are concerned, is averted.
At any rate, the distinction of the customs brokers from the other professionals who are subject to
occupation tax under the Local Tax Code is based upon material differences, in that the activities of
customs brokers (like those of stock, real estate and immigration brokers) partake more of a business,
rather than a profession and were thus subjected to the percentage tax under Sec. 174 of the National
Internal Revenue Code prior to its amendment by EO 273. EO 273 abolished the percentage tax and
replaced it with the VAT. If the petitioner Association did not protest the classification of customs brokers
then, the Court sees no reason why it should protest now.
The Court takes note that EO 273 has been in effect for more than five (5) months now, so that the fears
expressed by the petitioners that the adoption of the VAT will trigger skyrocketing of prices of basic
commodities and services, as well as mass actions and demonstrations against the VAT should by now
be evident. The fact that nothing of the sort has happened shows that the fears and apprehensions of the
petitioners appear to be more imagined than real. It would seem that the VAT is not as bad as we are
made to believe.
In any event, if petitioners seriously believe that the adoption and continued application of the VAT are
prejudicial to the general welfare or the interests of the majority of the people, they should seek recourse
and relief from the political branches of the government. The Court, following the time-honored doctrine of
separation of powers, cannot substitute its judgment for that of the President as to the wisdom, justice
and advisability of the adoption of the VAT. The Court can only look into and determine whether or not
EO 273 was enacted and made effective as law, in the manner required by, and consistent with, the

Constitution, and to make sure that it was not issued in grave abuse of discretion amounting to lack or
excess of jurisdiction; and, in this regard, the Court finds no reason to impede its application or continued
implementation.
WHEREFORE, the petitions are DISMISSED. Without pronouncement as to costs.
SO ORDERED.

Republic of the Philippines


SUPREME COURT
Manila
SECOND DIVISION
G.R. No. L-49336 August 31, 1981
THE PROVINCE OF ABRA, represented by LADISLAO ANCHETA, Provincial
Assessor, petitioner,
vs.
HONORABLE HAROLD M. HERNANDO, in his capacity as Presiding Judge
of Branch I, Court of First Instance Abra; THE ROMAN CATHOLIC BISHOP
OF BANGUED, INC., represented by Bishop Odilo etspueler and Reverend
Felipe Flores, respondents.

FERNANDO, C.J.:
On the face of this certiorari and mandamus petition filed by the Province of
Abra, 1 it clearly appears that the actuation of respondent Judge Harold M. Hernando of the Court of
2

First Instance of Abra left much to be desired. First, there was a denial of a motion to dismiss an action
for declaratory relief by private respondent Roman Catholic Bishop of Bangued desirous of being
3
exempted from a real estate tax followed by a summary judgment granting such exemption, without
even hearing the side of petitioner. In the rather vigorous language of the Acting Provincial Fiscal, as
counsel for petitioner, respondent Judge "virtually ignored the pertinent provisions of the Rules of Court;
... wantonly violated the rights of petitioner to due process, by giving due course to the petition of private
respondent for declaratory relief, and thereafter without allowing petitioner to answer and without any
hearing, adjudged the case; all in total disregard of basic laws of procedure and basic provisions of due
process in the constitution, thereby indicating a failure to grasp and understand the law, which goes into
4
the competence of the Honorable Presiding Judge."

It was the submission of counsel that an action for declaratory relief would be
proper only before a breach or violation of any statute, executive order or
regulation. 5 Moreover, there being a tax assessment made by the Provincial Assessor on the
properties of respondent Roman Catholic Bishop, petitioner failed to exhaust the administrative remedies
available under Presidential Decree No. 464 before filing such court action. Further, it was pointed out to
respondent Judge that he failed to abide by the pertinent provision of such Presidential Decree which
provides as follows: "No court shall entertain any suit assailing the validity of a tax assessed under this

Code until the taxpayer, shall have paid, under protest, the tax assessed against him nor shall any court
declare any tax invalid by reason of irregularities or informalities in the proceedings of the officers
charged with the assessment or collection of taxes, or of failure to perform their duties within this time
herein specified for their performance unless such irregularities, informalities or failure shall have impaired
the substantial rights of the taxpayer; nor shall any court declare any portion of the tax assessed under
the provisions of this Code invalid except upon condition that the taxpayer shall pay the just amount of the
6
tax, as determined by the court in the pending proceeding."

When asked to comment, respondent Judge began with the allegation that there
"is no question that the real properties sought to be taxed by the Province of Abra
are properties of the respondent Roman Catholic Bishop of Bangued, Inc." 7 The
very next sentence assumed the very point it asked when he categorically stated: "Likewise, there is no
dispute that the properties including their procedure are actually, directly and exclusively used by the
8
Roman Catholic Bishop of Bangued, Inc. for religious or charitable purposes." For him then: "The proper
9
remedy of the petitioner is appeal and not this special civil action." A more exhaustive comment was
submitted by private respondent Roman Catholic Bishop of Bangued, Inc. It was, however, unable to
lessen the force of the objection raised by petitioner Province of Abra, especially the due process aspect.
it is to be admitted that his opposition to the petition, pressed with vigor, ostensibly finds a semblance of
support from the authorities cited. It is thus impressed with a scholarly aspect. It suffers, however, from
the grave infirmity of stating that only a pure question of law is presented when a claim for exemption is
made.

The petition must be granted.


1. Respondent Judge would not have erred so grievously had he merely
compared the provisions of the present Constitution with that appearing in the
1935 Charter on the tax exemption of "lands, buildings, and improvements."
There is a marked difference. Under the 1935 Constitution: "Cemeteries,
churches, and parsonages or convents appurtenant thereto, and all lands,
buildings, and improvements used exclusively for religious, charitable, or
educational purposes shall be exempt from taxation." 10 The present Constitution added
"charitable institutions, mosques, and non-profit cemeteries" and required that for the exemption of
":lands, buildings, and improvements," they should not only be "exclusively" but also "actually and
11
"directly" used for religious or charitable purposes. The Constitution is worded differently. The change
should not be ignored. It must be duly taken into consideration. Reliance on past decisions would have
sufficed were the words "actually" as well as "directly" not added. There must be proof therefore of
the actual anddirect use of the lands, buildings, and improvements for religious or charitable purposes to
12
be exempt from taxation. According to Commissioner of Internal Revenue v. Guerrero: "From 1906,
in Catholic Church v. Hastings to 1966, in Esso Standard Eastern, Inc. v. Acting Commissioner of
Customs, it has been the constant and uniform holding that exemption from taxation is not favored and is
never presumed, so that if granted it must be strictly construed against the taxpayer. Affirmatively put, the
law frowns on exemption from taxation, hence, an exempting provision should be construed strictissimi
13
14
juris." In Manila Electric Company v. Vera, a 1975 decision, such principle was reiterated, reference
15
being made toRepublic Flour Mills, Inc. v. Commissioner of Internal Revenue; Commissioner of
16
Customs v. Philippine Acetylene Co. & CTA; and Davao Light and Power Co., Inc. v. Commissioner of
17
Customs.

2. Petitioner Province of Abra is therefore fully justified in invoking the protection


of procedural due process. If there is any case where proof is necessary to
demonstrate that there is compliance with the constitutional provision that allows

an exemption, this is it. Instead, respondent Judge accepted at its face the
allegation of private respondent. All that was alleged in the petition for
declaratory relief filed by private respondents, after mentioning certain parcels of
land owned by it, are that they are used "actually, directly and exclusively" as
sources of support of the parish priest and his helpers and also of private
respondent Bishop. 18 In the motion to dismiss filed on behalf of petitioner Province of Abra, the
objection was based primarily on the lack of jurisdiction, as the validity of a tax assessment may be
questioned before the Local Board of Assessment Appeals and not with a court. There was also mention
of a lack of a cause of action, but only because, in its view, declaratory relief is not proper, as there had
been breach or violation of the right of government to assess and collect taxes on such property. It clearly
appears, therefore, that in failing to accord a hearing to petitioner Province of Abra and deciding the case
immediately in favor of private respondent, respondent Judge failed to abide by the constitutional
command of procedural due process.

WHEREFORE, the petition is granted and the resolution of June 19, 1978 is set
aside. Respondent Judge, or who ever is acting on his behalf, is ordered to hear
the case on the merit. No costs.
Facts: The provincial assessor made a tax assessment on the properties of the
Roman Catholic Bishop of Bangued. The bishop claims tax exemption from real
estate tax, through an action for declaratory relief. A summary judgment was
made granting the exemption without hearing the side of the Province of Abra.
Issue: Whether the properties of the Bishop of Bangued are tax-exempt.
Held: The 1935 and the 1973 Constitutions differ in language as to the
exemption of religious property from taxes as tehy should not only be
exclusively but also actually and directly used for religious purposes. Herein,
the judge accepted at its face the allegation of the Bishop instead of
demonstrating that there is compliance with the constitutional provision that
allows an exemption. There was an allegation of lack of jurisdiction and of lack of
cause of action, which should have compelled the judge to accord a hearing to
the province rather than deciding the case immediately in favor of the Bishop.
Exemption from taxation is not favored and is never presumed, so that if granted,
it must be strictly construed against the taxpayer. There must be proof of the
actual and direct use of the lands, buildings, and improvements for religious (or
charitable) purposes to be exempted from taxation.

The case was remanded to the lower court for a trial on merits.

Republic of the Philippines


SUPREME COURT
Manila
SECOND DIVISION
G.R. No. L-39086 June 15, 1988
ABRA VALLEY COLLEGE, INC., represented by PEDRO V.
BORGONIA, petitioner,
vs.
HON. JUAN P. AQUINO, Judge, Court of First Instance, Abra; ARMIN M.
CARIAGA, Provincial Treasurer, Abra; GASPAR V. BOSQUE, Municipal
Treasurer, Bangued, Abra; HEIRS OF PATERNO MILLARE,respondents.

PARAS, J.:
This is a petition for review on certiorari of the decision * of the defunct Court of First Instance of
Abra, Branch I, dated June 14, 1974, rendered in Civil Case No. 656, entitled "Abra Valley Junior College, Inc., represented by Pedro V.
Borgonia, plaintiff vs. Armin M. Cariaga as Provincial Treasurer of Abra, Gaspar V. Bosque as Municipal Treasurer of Bangued, Abra and
Paterno Millare, defendants," the decretal portion of which reads:

IN VIEW OF ALL THE FOREGOING, the Court hereby declares:


That the distraint seizure and sale by the Municipal Treasurer of
Bangued, Abra, the Provincial Treasurer of said province against the
lot and building of the Abra Valley Junior College, Inc., represented
by Director Pedro Borgonia located at Bangued, Abra, is valid;
That since the school is not exempt from paying taxes, it should
therefore pay all back taxes in the amount of P5,140.31 and back
taxes and penalties from the promulgation of this decision;
That the amount deposited by the plaintaff him the sum of
P60,000.00 before the trial, be confiscated to apply for the payment
of the back taxes and for the redemption of the property in question,
if the amount is less than P6,000.00, the remainder must be

returned to the Director of Pedro Borgonia, who represents the


plaintiff herein;
That the deposit of the Municipal Treasurer in the amount of
P6,000.00 also before the trial must be returned to said Municipal
Treasurer of Bangued, Abra;
And finally the case is hereby ordered dismissed with costs against
the plaintiff.
SO ORDERED. (Rollo, pp. 22-23)
Petitioner, an educational corporation and institution of higher learning duly
incorporated with the Securities and Exchange Commission in 1948, filed a
complaint (Annex "1" of Answer by the respondents Heirs of Paterno Millare;
Rollo, pp. 95-97) on July 10, 1972 in the court a quo to annul and declare void
the "Notice of Seizure' and the "Notice of Sale" of its lot and building located at
Bangued, Abra, for non-payment of real estate taxes and penalties amounting to
P5,140.31. Said "Notice of Seizure" of the college lot and building covered by
Original Certificate of Title No. Q-83 duly registered in the name of petitioner,
plaintiff below, on July 6, 1972, by respondents Municipal Treasurer and
Provincial Treasurer, defendants below, was issued for the satisfaction of the
said taxes thereon. The "Notice of Sale" was caused to be served upon the
petitioner by the respondent treasurers on July 8, 1972 for the sale at public
auction of said college lot and building, which sale was held on the same date.
Dr. Paterno Millare, then Municipal Mayor of Bangued, Abra, offered the highest
bid of P6,000.00 which was duly accepted. The certificate of sale was
correspondingly issued to him.
On August 10, 1972, the respondent Paterno Millare (now deceased) filed
through counstel a motion to dismiss the complaint.
On August 23, 1972, the respondent Provincial Treasurer and Municipal
Treasurer, through then Provincial Fiscal Loreto C. Roldan, filed their answer
(Annex "2" of Answer by the respondents Heirs of Patemo Millare; Rollo, pp. 98100) to the complaint. This was followed by an amended answer (Annex "3," ibid,
Rollo, pp. 101-103) on August 31, 1972.
On September 1, 1972 the respondent Paterno Millare filed his answer (Annex
"5," ibid; Rollo, pp. 106-108).
On October 12, 1972, with the aforesaid sale of the school premises at public
auction, the respondent Judge, Hon. Juan P. Aquino of the Court of First

Instance of Abra, Branch I, ordered (Annex "6," ibid; Rollo, pp. 109-110) the
respondents provincial and municipal treasurers to deliver to the Clerk of Court
the proceeds of the auction sale. Hence, on December 14, 1972, petitioner,
through Director Borgonia, deposited with the trial court the sum of P6,000.00
evidenced by PNB Check No. 904369.
On April 12, 1973, the parties entered into a stipulation of facts adopted and
embodied by the trial court in its questioned decision. Said Stipulations reads:
STIPULATION OF FACTS
COME NOW the parties, assisted by counsels, and to this
Honorable Court respectfully enter into the following agreed
stipulation of facts:
1. That the personal circumstances of the parties as stated in
paragraph 1 of the complaint is admitted; but the particular person of
Mr. Armin M. Cariaga is to be substituted, however, by anyone who
is actually holding the position of Provincial Treasurer of the
Province of Abra;
2. That the plaintiff Abra Valley Junior College, Inc. is the owner of
the lot and buildings thereon located in Bangued, Abra under
Original Certificate of Title No. 0-83;
3. That the defendant Gaspar V. Bosque, as Municipal treasurer of
Bangued, Abra caused to be served upon the Abra Valley Junior
College, Inc. a Notice of Seizure on the property of said school
under Original Certificate of Title No. 0-83 for the satisfaction of real
property taxes thereon, amounting to P5,140.31; the Notice of
Seizure being the one attached to the complaint as Exhibit A;
4. That on June 8, 1972 the above properties of the Abra Valley
Junior College, Inc. was sold at public auction for the satisfaction of
the unpaid real property taxes thereon and the same was sold to
defendant Paterno Millare who offered the highest bid of P6,000.00
and a Certificate of Sale in his favor was issued by the defendant
Municipal Treasurer.
5. That all other matters not particularly and specially covered by this
stipulation of facts will be the subject of evidence by the parties.

WHEREFORE, it is respectfully prayed of the Honorable Court to


consider and admit this stipulation of facts on the point agreed upon
by the parties.
Bangued, Abra, April 12, 1973.
Sgd.
Agripi
no
Brilla
ntes
Typ
AGRI
PINO
BRIL
LANT
ES
Attorn
ey for
Plainti
ff
Sgd.
Loret
o
Rolda
n
Typ
LORE
TO
ROL
DAN
Provi
ncial
Fiscal
Coun
sel for
Defen
dants
Provi
ncial
Treas

urer
of
Abra
and
the
Munic
ipal
Treas
urer
of
Bang
ued,
Abra
Sgd.
Deme
trio V.
Pre
Typ.
DEM
ETRI
O V.
PRE
Attorn
ey for
Defen
dant
Pater
no
Millar
e
(Rollo
, pp.
1718)
Aside from the Stipulation of Facts, the trial court among others, found the
following: (a) that the school is recognized by the government and is offering
Primary, High School and College Courses, and has a school population of more
than one thousand students all in all; (b) that it is located right in the heart of the
town of Bangued, a few meters from the plaza and about 120 meters from the
Court of First Instance building; (c) that the elementary pupils are housed in a
two-storey building across the street; (d) that the high school and college

students are housed in the main building; (e) that the Director with his family is in
the second floor of the main building; and (f) that the annual gross income of the
school reaches more than one hundred thousand pesos.
From all the foregoing, the only issue left for the Court to determine and as
agreed by the parties, is whether or not the lot and building in question are used
exclusively for educational purposes. (Rollo, p. 20)
The succeeding Provincial Fiscal, Hon. Jose A. Solomon and his Assistant, Hon.
Eustaquio Z. Montero, filed a Memorandum for the Government on March 25,
1974, and a Supplemental Memorandum on May 7, 1974, wherein they opined
"that based on the evidence, the laws applicable, court decisions and
jurisprudence, the school building and school lot used for educational purposes
of the Abra Valley College, Inc., are exempted from the payment of taxes."
(Annexes "B," "B-1" of Petition; Rollo, pp. 24-49; 44 and 49).
Nonetheless, the trial court disagreed because of the use of the second floor by
the Director of petitioner school for residential purposes. He thus ruled for the
government and rendered the assailed decision.
After having been granted by the trial court ten (10) days from August 6, 1974
within which to perfect its appeal (Per Order dated August 6, 1974; Annex "G" of
Petition; Rollo, p. 57) petitioner instead availed of the instant petition for review
on certiorari with prayer for preliminary injunction before this Court, which petition
was filed on August 17, 1974 (Rollo, p.2).
In the resolution dated August 16, 1974, this Court resolved to give DUE
COURSE to the petition (Rollo, p. 58). Respondents were required to answer
said petition (Rollo, p. 74).
Petitioner raised the following assignments of error:
I
THE COURT A QUO ERRED IN SUSTAINING AS VALID THE SEIZURE AND
SALE OF THE COLLEGE LOT AND BUILDING USED FOR EDUCATIONAL
PURPOSES OF THE PETITIONER.
II
THE COURT A QUO ERRED IN DECLARING THAT THE COLLEGE LOT AND
BUILDING OF THE PETITIONER ARE NOT USED EXCLUSIVELY FOR

EDUCATIONAL PURPOSES MERELY BECAUSE THE COLLEGE PRESIDENT


RESIDES IN ONE ROOM OF THE COLLEGE BUILDING.
III
THE COURT A QUO ERRED IN DECLARING THAT THE COLLEGE LOT AND
BUILDING OF THE PETITIONER ARE NOT EXEMPT FROM PROPERTY
TAXES AND IN ORDERING PETITIONER TO PAY P5,140.31 AS REALTY
TAXES.
IV
THE COURT A QUO ERRED IN ORDERING THE CONFISCATION OF THE
P6,000.00 DEPOSIT MADE IN THE COURT BY PETITIONER AS PAYMENT
OF THE P5,140.31 REALTY TAXES. (See Brief for the Petitioner, pp. 1-2)
The main issue in this case is the proper interpretation of the phrase "used
exclusively for educational purposes."
Petitioner contends that the primary use of the lot and building for educational
purposes, and not the incidental use thereof, determines and exemption from
property taxes under Section 22 (3), Article VI of the 1935 Constitution. Hence,
the seizure and sale of subject college lot and building, which are contrary
thereto as well as to the provision of Commonwealth Act No. 470, otherwise
known as the Assessment Law, are without legal basis and therefore void.
On the other hand, private respondents maintain that the college lot and building
in question which were subjected to seizure and sale to answer for the unpaid tax
are used: (1) for the educational purposes of the college; (2) as the permanent
residence of the President and Director thereof, Mr. Pedro V. Borgonia, and his
family including the in-laws and grandchildren; and (3) for commercial purposes
because the ground floor of the college building is being used and rented by a
commercial establishment, the Northern Marketing Corporation (See photograph
attached as Annex "8" (Comment; Rollo, p. 90]).
Due to its time frame, the constitutional provision which finds application in the
case at bar is Section 22, paragraph 3, Article VI, of the then 1935 Philippine
Constitution, which expressly grants exemption from realty taxes for "Cemeteries,
churches and parsonages or convents appurtenant thereto, and all lands,
buildings, and improvements used exclusively for religious, charitable or
educational purposes ...

Relative thereto, Section 54, paragraph c, Commonwealth Act No. 470 as


amended by Republic Act No. 409, otherwise known as the Assessment Law,
provides:
The following are exempted from real property tax under the
Assessment Law:
xxx xxx xxx
(c) churches and parsonages or convents appurtenant thereto, and
all lands, buildings, and improvements used exclusively for religious,
charitable, scientific or educational purposes.
xxx xxx xxx
In this regard petitioner argues that the primary use of the school lot and building
is the basic and controlling guide, norm and standard to determine tax
exemption, and not the mere incidental use thereof.
As early as 1916 in YMCA of Manila vs. Collector of lnternal Revenue, 33 Phil.
217 [1916], this Court ruled that while it may be true that the YMCA keeps a
lodging and a boarding house and maintains a restaurant for its members, still
these do not constitute business in the ordinary acceptance of the word, but an
institution used exclusively for religious, charitable and educational purposes,
and as such, it is entitled to be exempted from taxation.
In the case of Bishop of Nueva Segovia v. Provincial Board of Ilocos Norte, 51
Phil. 352 [1972], this Court included in the exemption a vegetable garden in an
adjacent lot and another lot formerly used as a cemetery. It was clarified that the
term "used exclusively" considers incidental use also. Thus, the exemption from
payment of land tax in favor of the convent includes, not only the land actually
occupied by the building but also the adjacent garden devoted to the incidental
use of the parish priest. The lot which is not used for commercial purposes but
serves solely as a sort of lodging place, also qualifies for exemption because this
constitutes incidental use in religious functions.
The phrase "exclusively used for educational purposes" was further clarified by
this Court in the cases of Herrera vs. Quezon City Board of assessment Appeals,
3 SCRA 186 [1961] and Commissioner of Internal Revenue vs. Bishop of the
Missionary District, 14 SCRA 991 [1965], thus
Moreover, the exemption in favor of property used exclusively for
charitable or educational purposes is 'not limited to property actually

indispensable' therefor (Cooley on Taxation, Vol. 2, p. 1430), but


extends to facilities which are incidental to and reasonably
necessary for the accomplishment of said purposes, such as in the
case of hospitals, "a school for training nurses, a nurses' home,
property use to provide housing facilities for interns, resident
doctors, superintendents, and other members of the hospital staff,
and recreational facilities for student nurses, interns, and residents'
(84 CJS 6621), such as "Athletic fields" including "a firm used for the
inmates of the institution. (Cooley on Taxation, Vol. 2, p. 1430).
The test of exemption from taxation is the use of the property for purposes
mentioned in the Constitution (Apostolic Prefect v. City Treasurer of Baguio, 71
Phil, 547 [1941]).
It must be stressed however, that while this Court allows a more liberal and nonrestrictive interpretation of the phrase "exclusively used for educational
purposes" as provided for in Article VI, Section 22, paragraph 3 of the 1935
Philippine Constitution, reasonable emphasis has always been made that
exemption extends to facilities which are incidental to and reasonably necessary
for the accomplishment of the main purposes. Otherwise stated, the use of the
school building or lot for commercial purposes is neither contemplated by law,
nor by jurisprudence. Thus, while the use of the second floor of the main building
in the case at bar for residential purposes of the Director and his family, may find
justification under the concept of incidental use, which is complimentary to the
main or primary purposeeducational, the lease of the first floor thereof to the
Northern Marketing Corporation cannot by any stretch of the imagination be
considered incidental to the purpose of education.
It will be noted however that the aforementioned lease appears to have been
raised for the first time in this Court. That the matter was not taken up in the to
court is really apparent in the decision of respondent Judge. No mention thereof
was made in the stipulation of facts, not even in the description of the school
building by the trial judge, both embodied in the decision nor as one of the issues
to resolve in order to determine whether or not said properly may be exempted
from payment of real estate taxes (Rollo, pp. 17-23). On the other hand, it is
noteworthy that such fact was not disputed even after it was raised in this Court.
Indeed, it is axiomatic that facts not raised in the lower court cannot be taken up
for the first time on appeal. Nonetheless, as an exception to the rule, this Court
has held that although a factual issue is not squarely raised below, still in the
interest of substantial justice, this Court is not prevented from considering a
pivotal factual matter. "The Supreme Court is clothed with ample authority to
review palpable errors not assigned as such if it finds that their consideration is

necessary in arriving at a just decision." (Perez vs. Court of Appeals, 127 SCRA
645 [1984]).
Under the 1935 Constitution, the trial court correctly arrived at the conclusion that
the school building as well as the lot where it is built, should be taxed, not
because the second floor of the same is being used by the Director and his
family for residential purposes, but because the first floor thereof is being used
for commercial purposes. However, since only a portion is used for purposes of
commerce, it is only fair that half of the assessed tax be returned to the school
involved.
PREMISES CONSIDERED, the decision of the Court of First Instance of Abra,
Branch I, is hereby AFFIRMED subject to the modification that half of the
assessed tax be returned to the petitioner.
SO ORDERED.
Abra Valley College v. Aquino
G.R. No. L-39086 June 15, 1988
Paras, J.
Facts:
Petitioner, an educational corporation and institution of higher learning duly incorporated
with the Securities and Exchange Commission in 1948, filed a complaint to annul and declare void
the Notice of Seizure and the Notice of Sale of its lot and building located at Bangued, Abra, for
non-payment of real estate taxes and penalties amounting to P5,140.31. Said Notice of Seizure by
respondents Municipal Treasurer and Provincial Treasurer, defendants below, was issued for the
satisfaction of the said taxes thereon.
The parties entered into a stipulation of facts adopted and embodied by the trial court in its
questioned decision. The trial court ruled for the government, holding that the second floor of the
building is being used by the director for residential purposes and that the ground floor used and
rented by Northern Marketing Corporation, a commercial establishment, and thus the property is not
being used exclusively for educational purposes. Instead of perfecting an appeal, petitioner availed
of the instant petition for review on certiorari with prayer for preliminary injunction before the
Supreme Court, by filing said petition on 17 August 1974.
Issue:

whether or not the lot and building are used exclusively for educational purposes
Held:
Section 22, paragraph 3, Article VI, of the then 1935 Philippine Constitution, expressly
grants exemption from realty taxes for cemeteries, churches and parsonages or convents
appurtenant thereto, and all lands, buildings, and improvements used exclusively for religious,
charitable or educational purposes. Reasonable emphasis has always been made that the
exemption extends to facilities which are incidental to and reasonably necessary for the
accomplishment of the main purposes. The use of the school building or lot for commercial purposes
is neither contemplated by law, nor by jurisprudence. In the case at bar, the lease of the first floor of
the building to the Northern Marketing Corporation cannot by any stretch of the imagination be
considered incidental to the purpose of education. The test of exemption from taxation is the use of
the property for purposes mentioned in the Constitution.
The decision of the CFI Abra (Branch I) is affirmed subject to the modification that half of
the assessed tax be returned to the petitioner. The modification is derived from the fact that the
ground floor is being used for commercial purposes (leased) and the second floor being used as
incidental to education (residence of the director).

Republic of the Philippines


SUPREME COURT
Manila
EN BANC
G.R. No. L-10405

December 29, 1960

WENCESLAO PASCUAL, in his official capacity as Provincial Governor of


Rizal, petitioner-appellant,
vs.
THE SECRETARY OF PUBLIC WORKS AND COMMUNICATIONS, ET
AL., respondents-appellees.
Asst. Fiscal Noli M. Cortes and Jose P. Santos for appellant.
Office of the Asst. Solicitor General Jose G. Bautista and Solicitor A. A. Torres
for appellee.

CONCEPCION, J.:
Appeal, by petitioner Wenceslao Pascual, from a decision of the Court of First
Instance of Rizal, dismissing the above entitled case and dissolving the writ of
preliminary injunction therein issued, without costs.
On August 31, 1954, petitioner Wenceslao Pascual, as Provincial Governor of
Rizal, instituted this action for declaratory relief, with injunction, upon the ground
that Republic Act No. 920, entitled "An Act Appropriating Funds for Public
Works", approved on June 20, 1953, contained, in section 1-C (a) thereof, an
item (43[h]) of P85,000.00 "for the construction, reconstruction, repair, extension
and improvement" of Pasig feeder road terminals (Gen. Roxas Gen. Araneta
Gen. Lucban Gen. Capinpin Gen. Segundo Gen. Delgado Gen.
Malvar Gen. Lim)"; that, at the time of the passage and approval of said Act,
the aforementioned feeder roads were "nothing but projected and planned
subdivision roads, not yet constructed, . . . within the Antonio Subdivision . . .
situated at . . . Pasig, Rizal" (according to the tracings attached to the petition as
Annexes A and B, near Shaw Boulevard, not far away from the intersection
between the latter and Highway 54), which projected feeder roads "do not
connect any government property or any important premises to the main
highway"; that the aforementioned Antonio Subdivision (as well as the lands on
which said feeder roads were to be construed) were private properties of
respondent Jose C. Zulueta, who, at the time of the passage and approval of
said Act, was a member of the Senate of the Philippines; that on May, 1953,
respondent Zulueta, addressed a letter to the Municipal Council of Pasig, Rizal,
offering to donate said projected feeder roads to the municipality of Pasig, Rizal;
that, on June 13, 1953, the offer was accepted by the council, subject to the
condition "that the donor would submit a plan of the said roads and agree to
change the names of two of them"; that no deed of donation in favor of the
municipality of Pasig was, however, executed; that on July 10, 1953, respondent
Zulueta wrote another letter to said council, calling attention to the approval of
Republic Act. No. 920, and the sum of P85,000.00 appropriated therein for the
construction of the projected feeder roads in question; that the municipal council
of Pasig endorsed said letter of respondent Zulueta to the District Engineer of
Rizal, who, up to the present "has not made any endorsement thereon" that
inasmuch as the projected feeder roads in question were private property at the
time of the passage and approval of Republic Act No. 920, the appropriation of
P85,000.00 therein made, for the construction, reconstruction, repair, extension
and improvement of said projected feeder roads, was illegal and, therefore,
void ab initio"; that said appropriation of P85,000.00 was made by Congress
because its members were made to believe that the projected feeder roads in
question were "public roads and not private streets of a private subdivision"'; that,
"in order to give a semblance of legality, when there is absolutely none, to the

aforementioned appropriation", respondents Zulueta executed on December 12,


1953, while he was a member of the Senate of the Philippines, an alleged deed
of donation copy of which is annexed to the petition of the four (4) parcels
of land constituting said projected feeder roads, in favor of the Government of the
Republic of the Philippines; that said alleged deed of donation was, on the same
date, accepted by the then Executive Secretary; that being subject to an onerous
condition, said donation partook of the nature of a contract; that, such, said
donation violated the provision of our fundamental law prohibiting members of
Congress from being directly or indirectly financially interested in any contract
with the Government, and, hence, is unconstitutional, as well as null and void ab
initio, for the construction of the projected feeder roads in question with public
funds would greatly enhance or increase the value of the aforementioned
subdivision of respondent Zulueta, "aside from relieving him from the burden of
constructing his subdivision streets or roads at his own expense"; that the
construction of said projected feeder roads was then being undertaken by the
Bureau of Public Highways; and that, unless restrained by the court, the
respondents would continue to execute, comply with, follow and implement the
aforementioned illegal provision of law, "to the irreparable damage, detriment and
prejudice not only to the petitioner but to the Filipino nation."
Petitioner prayed, therefore, that the contested item of Republic Act No. 920 be
declared null and void; that the alleged deed of donation of the feeder roads in
question be "declared unconstitutional and, therefor, illegal"; that a writ of
injunction be issued enjoining the Secretary of Public Works and
Communications, the Director of the Bureau of Public Works and Highways and
Jose C. Zulueta from ordering or allowing the continuance of the abovementioned feeder roads project, and from making and securing any new and
further releases on the aforementioned item of Republic Act No. 920, and the
disbursing officers of the Department of Public Works and Highways from making
any further payments out of said funds provided for in Republic Act No. 920; and
that pending final hearing on the merits, a writ of preliminary injunction be issued
enjoining the aforementioned parties respondent from making and securing any
new and further releases on the aforesaid item of Republic Act No. 920 and from
making any further payments out of said illegally appropriated funds.
Respondents moved to dismiss the petition upon the ground that petitioner had
"no legal capacity to sue", and that the petition did "not state a cause of action".
In support to this motion, respondent Zulueta alleged that the Provincial Fiscal of
Rizal, not its provincial governor, should represent the Province of Rizal,
pursuant to section 1683 of the Revised Administrative Code; that said
respondent is " not aware of any law which makes illegal the appropriation of
public funds for the improvements of . . . private property"; and that, the
constitutional provision invoked by petitioner is inapplicable to the donation in

question, the same being a pure act of liberality, not a contract. The other
respondents, in turn, maintained that petitioner could not assail the appropriation
in question because "there is no actual bona fide case . . . in which the validity of
Republic Act No. 920 is necessarily involved" and petitioner "has not shown that
he has a personal and substantial interest" in said Act "and that its enforcement
has caused or will cause him a direct injury."
Acting upon said motions to dismiss, the lower court rendered the
aforementioned decision, dated October 29, 1953, holding that, since public
interest is involved in this case, the Provincial Governor of Rizal and the
provincial fiscal thereof who represents him therein, "have the requisite
personalities" to question the constitutionality of the disputed item of Republic Act
No. 920; that "the legislature is without power appropriate public revenues for
anything but a public purpose", that the instructions and improvement of the
feeder roads in question, if such roads where private property, would not be a
public purpose; that, being subject to the following condition:
The within donation is hereby made upon the condition that the
Government of the Republic of the Philippines will use the parcels of land
hereby donated for street purposes only and for no other purposes
whatsoever; it being expressly understood that should the Government of
the Republic of the Philippines violate the condition hereby imposed upon
it, the title to the land hereby donated shall, upon such violation, ipso facto
revert to the DONOR, JOSE C. ZULUETA. (Emphasis supplied.)
which is onerous, the donation in question is a contract; that said donation or
contract is "absolutely forbidden by the Constitution" and consequently "illegal",
for Article 1409 of the Civil Code of the Philippines, declares in existence and
void from the very beginning contracts "whose cause, objector purpose is
contrary to law, morals . . . or public policy"; that the legality of said donation may
not be contested, however, by petitioner herein, because his "interest are not
directly affected" thereby; and that, accordingly, the appropriation in question
"should be upheld" and the case dismissed.
At the outset, it should be noted that we are concerned with a decision granting
the aforementioned motions to dismiss, which as much, are deemed to have
admitted hypothetically the allegations of fact made in the petition of appellant
herein. According to said petition, respondent Zulueta is the owner of several
parcels of residential land situated in Pasig, Rizal, and known as the Antonio
Subdivision, certain portions of which had been reserved for the projected feeder
roads aforementioned, which, admittedly, were private property of said
respondent when Republic Act No. 920, appropriating P85,000.00 for the
"construction, reconstruction, repair, extension and improvement" of said roads,

was passed by Congress, as well as when it was approved by the President on


June 20, 1953. The petition further alleges that the construction of said roads, to
be undertaken with the aforementioned appropriation of P85,000.00, would have
the effect of relieving respondent Zulueta of the burden of constructing his
subdivision streets or roads at his own expenses, 1and would "greatly enhance or
increase the value of the subdivision" of said respondent. The lower court held
that under these circumstances, the appropriation in question was "clearly for a
private, not a public purpose."
Respondents do not deny the accuracy of this conclusion, which is selfevident. 2However, respondent Zulueta contended, in his motion to dismiss that:
A law passed by Congress and approved by the President can never be
illegal because Congress is the source of all laws . . . Aside from the fact
that movant is not aware of any law which makes illegal the appropriation
of public funds for the improvement of what we, in the meantime, may
assume as private property . . . (Record on Appeal, p. 33.)
The first proposition must be rejected most emphatically, it being inconsistent
with the nature of the Government established under the Constitution of the
Republic of the Philippines and the system of checks and balances underlying
our political structure. Moreover, it is refuted by the decisions of this Court
invalidating legislative enactments deemed violative of the Constitution or organic
laws. 3
As regards the legal feasibility of appropriating public funds for a public purpose,
the principle according to Ruling Case Law, is this:
It is a general rule that the legislature is without power to appropriate public
revenue for anything but a public purpose. . . . It is the essential character
of the direct object of the expenditure which must determine its validity as
justifying a tax, and not the magnitude of the interest to be affected nor the
degree to which the general advantage of the community, and thus the
public welfare, may be ultimately benefited by their promotion. Incidental to
the public or to the state, which results from the promotion of private
interest and the prosperity of private enterprises or business, does not
justify their aid by the use public money. (25 R.L.C. pp. 398-400; Emphasis
supplied.)
The rule is set forth in Corpus Juris Secundum in the following language:
In accordance with the rule that the taxing power must be exercised for
public purposes only, discussed suprasec. 14, money raised by taxation

can be expended only for public purposes and not for the advantage of
private individuals. (85 C.J.S. pp. 645-646; emphasis supplied.)
Explaining the reason underlying said rule, Corpus Juris Secundum states:
Generally, under the express or implied provisions of the
constitution, public funds may be used only for public purpose. The right of
the legislature to appropriate funds is correlative with its right to tax, and,
under constitutional provisions against taxation except for public purposes
and prohibiting the collection of a tax for one purpose and the devotion
thereof to another purpose, no appropriation of state funds can be made
for other than for a public purpose.
xxx

xxx

xxx

The test of the constitutionality of a statute requiring the use of public funds
is whether the statute is designed to promote the public interest, as
opposed to the furtherance of the advantage of individuals, although each
advantage to individuals might incidentally serve the public. (81 C.J.S. pp.
1147; emphasis supplied.)
Needless to say, this Court is fully in accord with the foregoing views which, apart
from being patently sound, are a necessary corollary to our democratic system of
government, which, as such, exists primarily for the promotion of the general
welfare. Besides, reflecting as they do, the established jurisprudence in the
United States, after whose constitutional system ours has been patterned, said
views and jurisprudence are, likewise, part and parcel of our own constitutional
law.
law phil.net

This notwithstanding, the lower court felt constrained to uphold the appropriation
in question, upon the ground that petitioner may not contest the legality of the
donation above referred to because the same does not affect him directly. This
conclusion is, presumably, based upon the following premises, namely: (1) that, if
valid, said donation cured the constitutional infirmity of the aforementioned
appropriation; (2) that the latter may not be annulled without a previous
declaration of unconstitutionality of the said donation; and (3) that the rule set
forth in Article 1421 of the Civil Code is absolute, and admits of no exception. We
do not agree with these premises.
The validity of a statute depends upon the powers of Congress at the time of its
passage or approval, not upon events occurring, or acts performed, subsequently
thereto, unless the latter consists of an amendment of the organic law, removing,
with retrospective operation, the constitutional limitation infringed by said statute.

Referring to the P85,000.00 appropriation for the projected feeder roads in


question, the legality thereof depended upon whether said roads were public or
private property when the bill, which, latter on, became Republic Act 920, was
passed by Congress, or, when said bill was approved by the President and the
disbursement of said sum became effective, or on June 20, 1953 (see section 13
of said Act). Inasmuch as the land on which the projected feeder roads were to
be constructed belonged then to respondent Zulueta, the result is that said
appropriation sought a private purpose, and hence, was null and void. 4 The
donation to the Government, over five (5) months after the approval and
effectivity of said Act, made, according to the petition, for the purpose of giving a
"semblance of legality", or legalizing, the appropriation in question, did not cure
its aforementioned basic defect. Consequently, a judicial nullification of said
donation need not precede the declaration of unconstitutionality of said
appropriation.
Again, Article 1421 of our Civil Code, like many other statutory enactments, is
subject to exceptions. For instance, the creditors of a party to an illegal contract
may, under the conditions set forth in Article 1177 of said Code, exercise the
rights and actions of the latter, except only those which are inherent in his
person, including therefore, his right to the annulment of said contract, even
though such creditors are not affected by the same, except indirectly, in the
manner indicated in said legal provision.
Again, it is well-stated that the validity of a statute may be contested only by one
who will sustain a direct injury in consequence of its enforcement. Yet, there are
many decisions nullifying, at the instance of taxpayers, laws providing for the
disbursement of public funds, 5upon the theory that "the expenditure of public
funds by an officer of the State for the purpose of administering
an unconstitutional act constitutes a misapplication of such funds," which may be
enjoined at the request of a taxpayer. 6Although there are some decisions to the
contrary, 7the prevailing view in the United States is stated in the American
Jurisprudence as follows:
In the determination of the degree of interest essential to give the requisite
standing to attack the constitutionality of a statute, the general rule is that
not only persons individually affected, but also taxpayers, have sufficient
interest in preventing the illegal expenditure of moneys raised by taxation
and may therefore question the constitutionality of statutes requiring
expenditure of public moneys. (11 Am. Jur. 761; emphasis supplied.)
However, this view was not favored by the Supreme Court of the U.S. in
Frothingham vs. Mellon (262 U.S. 447), insofar as federal laws are concerned,
upon the ground that the relationship of a taxpayer of the U.S. to its Federal

Government is different from that of a taxpayer of a municipal corporation to its


government. Indeed, under thecomposite system of government existing in the
U.S., the states of the Union are integral part of the Federation from
an international viewpoint, but, each state enjoys internally a substantial measure
of sovereignty, subject to the limitations imposed by the Federal Constitution. In
fact, the same was made by representatives of each state of the Union, not of the
people of the U.S., except insofar as the former represented the people of the
respective States, and the people of each State has, independently of that of the
others, ratified said Constitution. In other words, the Federal Constitution and the
Federal statutes have become binding upon the people of the U.S. in
consequence of an act of, and, in this sense, through the respective states of the
Union of which they are citizens. The peculiar nature of the relation between said
people and the Federal Government of the U.S. is reflected in the election of its
President, who is chosen directly, not by the people of the U.S., but by electors
chosen by each State, in such manner as the legislature thereof may direct
(Article II, section 2, of the Federal Constitution).
l awphi1.net

The relation between the people of the Philippines and its taxpayers, on the other
hand, and the Republic of the Philippines, on the other, is not identical to that
obtaining between the people and taxpayers of the U.S. and its Federal
Government. It is closer, from a domestic viewpoint, to that existing between the
people and taxpayers of each state and the government thereof, except that the
authority of the Republic of the Philippines over the people of the Philippines
is more fully direct than that of the states of the Union, insofar as
the simple and unitary type of our national government is not subject to
limitations analogous to those imposed by the Federal Constitution upon the
states of the Union, and those imposed upon the Federal Government in the
interest of the Union. For this reason, the rule recognizing the right of taxpayers
to assail the constitutionality of a legislation appropriating local or state public
funds which has been upheld by the Federal Supreme Court
(Crampton vs. Zabriskie, 101 U.S. 601) has greater application in the
Philippines than that adopted with respect to acts of Congress of the United
States appropriating federal funds.
Indeed, in the Province of Tayabas vs. Perez (56 Phil., 257), involving the
expropriation of a land by the Province of Tayabas, two (2) taxpayers thereof
were allowed to intervene for the purpose of contesting the price being paid to
the owner thereof, as unduly exorbitant. It is true that in Custodio vs. President of
the Senate (42 Off. Gaz., 1243), a taxpayer and employee of the Government
was not permitted to question the constitutionality of an appropriation for backpay
of members of Congress. However, in Rodriguez vs. Treasurer of the Philippines
and Barredo vs.Commission on Elections (84 Phil., 368; 45 Off. Gaz., 4411), we
entertained the action of taxpayers impugning the validity of certain

appropriations of public funds, and invalidated the same. Moreover, the reason
that impelled this Court to take such position in said two (2) cases the
importance of the issues therein raised is present in the case at bar. Again,
like the petitioners in the Rodriguez and Barredo cases, petitioner herein is not
merely a taxpayer. The Province of Rizal, which he represents officially as its
Provincial Governor, is our most populated political subdivision, 8and, the
taxpayers therein bear a substantial portion of the burden of taxation, in the
Philippines.
Hence, it is our considered opinion that the circumstances surrounding this case
sufficiently justify petitioners action in contesting the appropriation and donation
in question; that this action should not have been dismissed by the lower court;
and that the writ of preliminary injunction should have been maintained.
Wherefore, the decision appealed from is hereby reversed, and the records are
remanded to the lower court for further proceedings not inconsistent with this
decision, with the costs of this instance against respondent Jose C. Zulueta. It is
so ordered.

Pascual v. Secretary of Public Works Digest


G.R. No. L-10405 December 29, 1960
Ponente: Concepcion, J.
Legal Standing
Facts:
1. Petitioner was the governor of Rizal, filed a petition assailing the validity of
R.A. 920 which contains an item providing for an appropriation of P85,000.00 for
the construction and repair of a feeder road in Pasig. The said law was passed in
Congress and approved by the President.
2. The property over which the feeder road will be constructed is however owned
by Sen. Zulueta. The property was to be donated to the local government, though
the donation was made a few months after the appropriation was included in RA
920. The petition alleged that the said planned feeder road would relieve Zulueta
the responsibility of improving the road which is inside a private subdivision.
3. The lower court (RTC) ruled that the petitioner has standing to assail the
validity of RA 920, due to the public interest involved in the appropriation.
However, he does not have a standing with respect to the donation since he
does not have an interest that will be injured by said donation, hence it dismissed
the petition.

Issue: Whether or not the petitioner has the standing to file the petition
YES.
1. Petitioner has standing. He is not merely a taxpayer but the governor of the
province of Rizal which is considered one of the most populated biggest
provinces during that time, its taxpayers bear a substantial portion of the burden
of taxation in the country.
2. Public funds can only be appropriated for a public purpose. The test of the
constitutionality of a statute requiring the use of public funds is whether it is used
to promote public interest. Moreover, the validity of a stature depends on the
powers of the Congress at the time of its passage or approval, not upon events
occurring, or acts performed subsequent thereto, unless it is an amendment of
the organic law.

Republic of the Philippines


SUPREME COURT
Manila
EN BANC

G.R. No. 94571 April 22, 1991


TEOFISTO T. GUINGONA, JR. and AQUILINO Q. PIMENTEL, JR., petitioners,
vs.
HON. GUILLERMO CARAGUE, in his capacity as Secretary, Budget &
Management, HON. ROZALINA S. CAJUCOM in her capacity as National
Treasurer and COMMISSION ON AUDIT, respondents.
Ramon A. Gonzales for petitioners.

GANCAYCO, J.:p
This is a case of first impression whereby petitioners question the
constitutionality of the automatic appropriation for debt service in the 1990
budget.

As alleged in the petition, the facts are as follows:


The 1990 budget consists of P98.4 Billion in automatic appropriation (with P86.8
Billion for debt service) and P155.3 Billion appropriated under Republic Act No.
6831, otherwise known as the General Appropriations Act, or a total of P233.5
Billion, 1 while the appropriations for the Department of Education, Culture and Sports amount to
P27,017,813,000.00.

The said automatic appropriation for debt service is authorized by P.D. No. 81,
entitled "Amending Certain Provisions of Republic Act Numbered Four Thousand
Eight Hundred Sixty, as Amended (Re: Foreign Borrowing Act)," by P.D. No.
1177, entitled "Revising the Budget Process in Order to Institutionalize the
Budgetary Innovations of the New Society," and by P.D. No. 1967, entitled "An
Act Strenghthening the Guarantee and Payment Positions of the Republic of the
Philippines on Its Contingent Liabilities Arising out of Relent and Guaranteed
Loan by Appropriating Funds For The Purpose.
There can be no question that petitioners as Senators of the Republic of the
Philippines may bring this suit where a constitutional issue is raised. 3 Indeed, even a
taxpayer has personality to restrain unlawful expenditure of public funds.

The petitioner seek the declaration of the unconstitutionality of P.D. No. 81,
Sections 31 of P.D. 1177, and P.D. No. 1967. The petition also seeks to restrain
the disbursement for debt service under the 1990 budget pursuant to said
decrees.
Respondents contend that the petition involves a pure political question which is
the repeal or amendment of said laws addressed to the judgment, wisdom and
patriotism of the legislative body and not this Court.
In Gonzales, 5 the main issue was the unconstitutionality of the presidential veto of certain provision
particularly Section 16 of the General Appropriations Act of 1990, R.A. No. 6831. This Court, in disposing
of the issue, stated

The political question doctrine neither interposes an obstacle to


judicial determination of the rival claims. The jurisdiction to delimit
constitutional boundaries has been given to this Court. It cannot
abdicate that obligation mandated by the 1987 Constitution,
although said provision by no means does away with the
applicability of the principle in appropriate cases.
Sec. 1. The judicial power shad be vested in one
Supreme Court and in such lower courts as may be
established by law.

Judicial power includes the duty of the courts of justice


to settle actual controversies involving rights which are
legally demandable and enforceable, and to determine
whether or not there has been a grave abuse of
discretion amounting to lack or excess of jurisdiction on
the part of any branch or instrumentality of the
Government.
With the Senate maintaining that the President's veto is
unconstitutional and that charge being controverted, there is
an actual case or justiciable controversy between the Upper House
of Congress and the executive department that may be taken
cognizance of by this Court.
The questions raised in the instant petition are
I. IS THE APPROPRIATION OF P86 BILLION IN THE P233
BILLION 1990 BUDGET VIOLATIVE OF SECTION 5, ARTICLE XIV
OF THE CONSTITUTION?
II. ARE PD No. 81, PD No. 1177 AND PD No. 1967 STILL
OPERATIVE UNDER THE CONSTITUTION?
III. ARE THEY VIOLATIVE OF SECTION 29(l), ARTICLE VI OF THE
CONSTITUTION? 6
There is thus a justiciable controversy raised in the petition which this Court may
properly take cognizance of On the first issue, the petitioners aver
According to Sec. 5, Art. XIV of the Constitution:
(5) The State shall assign the highest budgetary priority
to education and ensure that teaching will attract and
retain its rightful share of the best available talents
through adequate remuneration and other means of job
satisfaction and fulfillment.
The reason behind the said provision is stated, thus:
In explaining his proposed amendment, Mr. Ople stated
that all the great and sincere piety professed by every
President and every Congress of the Philippines since
the end of World War II for the economic welfare of the

public schoolteachers always ended up in failure and


this failure, he stated, had caused mass defection of the
best and brightest teachers to other careers,
including menial jobs in overseas employment and
concerted actions by them to project their
grievances, mainly over low pay and abject working
conditions.
He pointed to the high expectations generated by the
February Revolution, especially keen among public
schoolteachers, which at present exacerbate these long
frustrated hopes.
Mr. Ople stated that despite the sincerity of all
administrations that tried vainly to respond to the needs
of the teachers, the central problem that always
defeated their pious intentions was really the one
budgetary priority in the sense that any proposed
increase for public schoolteachers had to be multiplied
many times by the number of government employees in
general and their equitable claims to any pay
standardization such that the pay rate of teachers is
hopelessly pegged to the rate of government workers in
general. This, he stated, foredoomed the prospect of a
significant pay increase for teachers.
Mr. Ople pointed out that the recognition by the
Constitution of the highest priority for public
schoolteachers, and by implication, for all teachers,
would ensure that the President and Congress would be
strongly urged by a constitutional mandate to grant to
them such a level of remuneration and other incentives
that would make teaching competitive again and
attractive to the best available talents in the nation.
Finally, Mr. Ople recalled that before World War II,
teaching competed most successfully against all other
career choices for the best and the brightest of the
younger generation. It is for this reason, he stated, that
his proposed amendment if approved, would ensure
that teaching would be restored to its lost glory as the
career of choice for the most talented and most publicspirited of the younger generation in the sense that it

would become the countervailing measure against the


continued decline of teaching and the wholesale
desertion of this noble profession presently taking place.
He further stated that this would ensure that the future
and the quality of the population would be asserted as a
top priority against many clamorous and importunate
but less important claims of the present. (Journal of the
Constitutional Commission, Vol. II, p. 1172)
However, as against this constitutional intention, P86 Billion is
appropriated for debt service while only P27 Billion is appropriated
for the Department of Education in the 1990 budget. It plain,
therefore, that the said appropriation for debt services is inconsistent
with the Constitution, hence, viod (Art. 7, New Civil Code). 7
While it is true that under Section 5(5), Article XIV of the Constitution Congress is
mandated to "assign the highest budgetary priority to education" in order to
"insure that teaching will attract and retain its rightful share of the best available
talents through adequate remuneration and other means of job satisfaction and
fulfillment," it does not thereby follow that the hands of Congress are so
hamstrung as to deprive it the power to respond to the imperatives of the national
interest and for the attainment of other state policies or objectives.
As aptly observed by respondents, since 1985, the budget for education has
tripled to upgrade and improve the facility of the public school system. The
compensation of teachers has been doubled. The amount of
P29,740,611,000.00 8 set aside for the Department of Education, Culture and Sports under the
General Appropriations Act (R.A. No. 6831), is the highest budgetary allocation among all department
budgets. This is a clear compliance with the aforesaid constitutional mandate according highest priority to
education.

Having faithfully complied therewith, Congress is certainly not without any power,
guided only by its good judgment, to provide an appropriation, that can
reasonably service our enormous debt, the greater portion of which was inherited
from the previous administration. It is not only a matter of honor and to protect
the credit standing of the country. More especially, the very survival of our
economy is at stake. Thus, if in the process Congress appropriated an amount
for debt service bigger than the share allocated to education, the Court finds and
so holds that said appropriation cannot be thereby assailed as unconstitutional.
Now to the second issue. The petitioners made the following observations:
To begin with, Rep. Act 4860 entitled "AN ACT AUTHORIZING THE
PRESIDENT OF THE PHILIPPINES TO OBTAIN SUCH

FOREIGN LOANS AND CREDITS, OR TO INCUR SUCH FOREIGN


INDEBTEDNESS, AS MAY BE NECESSARY TO FINANCE
APPROVED ECONOMIC DEVELOPMENT PURPOSES OR
PROJECTS, AND TO GUARANTEE, IN BEHALF OF THE
REPUBLIC OF THE PHILIPPINES, FOREIGN LOANS OBTAINED
OR BONDS ISSUED BY CORPORATIONS OWNED OR
CONTROLLED BY THE GOVERNMENT OF THE PHILIPPINES
FOR ECONOMIC DEVELOPMENT PURPOSES INCLUDING
THOSE INCURRED FOR PURPOSES OF RELENDING TO THE
PRIVATE SECTOR, APPROPRIATING THE NECESSARY FUNDS
THEREFOR, AND FOR OTHER PURPOSES, provides:
Sec. 2. The total amount of loans, credits and
indebtedness, excluding interests, which the President
of the Philippines is authorized to incur under this
Act shall not exceed one billion United States dollars or
its equivalent in other foreign currencies at the
exchange rate prevailing at the time the loans, credits
and indebtedness are incurred: Provided, however, That
the total loans, credits and indebtedness incurred under
this Act shall not exceed two hundred fifty million in the
fiscal year of the approval of this Act, and two hundred
fifty million every fiscal year thereafter, all in United
States dollars or its equivalent in other currencies.
Sec. 5. It shall be the duty of the President, within thirty
days after the opening of every regular session,
to report to the Congress the amount of loans, credits
and indebtedness contracted, as well as the guarantees
extended, and the purposes and projects for which the
loans, credits and indebtedness were incurred, and the
guarantees extended, as well as such loans which may
be reloaned to Filipino owned or controlled corporations
and similar purposes.
Sec. 6. The Congress shall appropriate the necessary
amount out of any funds in the National Treasury not
otherwise appropriated, to cover the payment of the
principal and interest on such loans, credits or
indebtedness as and when they shall become due.
However, after the declaration of martial law, President Marcos
issued PD 81 amending Section 6, thus:

Sec. 7. Section six of the same Act is hereby further


amended to read as follows:
Sec. 6. Any provision of law to the contrary
notwithstanding, and in order to enable the
Republic of the Philippines to pay the
principal, interest, taxes and other normal
banking charges on the loans, credits or
indebtedness, or on the bonds, debentures,
securities or other evidences of
indebtedness sold in international markets
incurred under the authority of this Act, the
proceeds of which are deemed
appropriated for the projects, all the
revenue realized from the projects financed
by such loans, credits or indebtedness, or
on the bonds, debentures, securities or
other evidences of indebtedness, shall be
turned over in full, after deducting actual
and necessary expenses for the operation
and maintenance of said projects, to the
National Treasury by the government
office, agency or instrumentality, or
government-owned or controlled
corporation concerned, which is hereby
appropriated for the purpose as and when
they shall become due. In case the revenue
realized is insufficient to cover the principal,
interest and other charges, such portion of
the budgetary savings as may be
necessary to cover the balance or
deficiency shall be set aside exclusively for
the purpose by the government office,
agency or instrumentality, or governmentowned or controlled corporation
concerned: Provided, That, if there still
remains a deficiency, such amount
necessary to cover the payment of the
principal and interest on such loans, credit
or indebtedness as and when they shall
become due is hereby appropriated out of
any funds in the national treasury not
otherwise appropriated: . . .

President Marcos also issued PD 1177, which provides:


Sec. 31. Automatic appropriations. All expenditures
for (a) personnel retirement premiums, government
service insurance, and other similar fixed expenditures,
(b)principal and interest on public debt, (c)
national government guarantees of obligations which
are drawn upon, are automatically
appropriated; Provided, that no obligations shall be
incurred or payments made from funds thus
automatically appropriated except as issued in the form
of regular budgetary allotments.
and PD 1967, which provides:
Sec. 1. There is hereby appropriated, out of any funds in the
National Treasury not otherwise appropriated, such amounts as may
be necessary to effect payments on foreign or domestic loans, or
foreign or domestic loans whereon creditors make a call on the
direct and indirect guarantee of the Republic of the Philippines,
obtained by:
a. The Republic of the Philippines the proceeds of which
were relent to government-owned or controlled
corporations and/or government financial institutions;
b. government-owned or controlled corporations and/or
government financial institutions the proceeds of which
were relent to public or private institutions;
c. government-owned or controlled corporations and/or
financial institutions and guaranteed by the Republic of
the Philippines;
d. other public or private institutions and guaranteed by
government-owned or controlled corporations and/or
government financial institutions.
Sec. 2. All repayments made by borrower institutions on the loans
for whose account advances were made by the National Treasury
will revert to the General Fund.

Sec. 3. In the event that any borrower institution is unable to settle


the advances made out of the appropriation provided therein, the
Treasurer of the Philippines shall make the proper recommendation
to the Minister of Finance on whether such advances shall be
treated as equity or subsidy of the National Government to the
institution concerned, which shall be considered in the budgetary
program of the Government.
In the "Budget of Expenditures and Sources of Financing Fiscal Year
1990," which accompanied her budget message to Congress, the
President of the Philippines, Corazon C. Aquino, stated:
Sources Appropriation
The P233.5 billion budget proposed for fiscal year 1990
will require P132.1 billion of new programmed
appropriations out of a total P155.3 billion in new
legislative authorization from Congress. The rest of the
budget, totalling P101.4 billion, will be sourced from
existing appropriations: P98.4 billion from Automatic
Appropriations and P3.0 billion from Continuing
Appropriations (Fig. 4).
And according to Figure 4, . . ., P86.8 billion out of the P98.4 Billion
are programmed for debt service. In other words, the President had,
on her own, determined and set aside the said amount of P98.4
Billion with the rest of the appropriations of P155.3 Billion to be
determined and fixed by Congress, which is now Rep. Act 6831. 9
Petitioners argue that the said automatic appropriations under the aforesaid
decrees of then President Marcos became functus oficio when he was ousted in
February, 1986; that upon the expiration of the one-man legislature in the person
of President Marcos, the legislative power was restored to Congress on February
2, 1987 when the Constitution was ratified by the people; that there is a need for
a new legislation by Congress providing for automatic appropriation, but
Congress, up to the present, has not approved any such law; and thus the said
P86.8 Billion automatic appropriation in the 1990 budget is an administrative act
that rests on no law, and thus, it cannot be enforced.
Moreover, petitioners contend that assuming arguendo that P.D. No. 81, P.D. No.
1177 and P.D. No. 1967 did not expire with the ouster of President Marcos, after
the adoption of the 1987 Constitution, the said decrees are inoperative under
Section 3, Article XVIII which provides

Sec. 3. All existing laws, decrees, executive orders, proclamations,


letters of instructions, and other executive issuances not inconsistent
with this Constitution shall remain operative until amended,
repealed, or revoked." (Emphasis supplied.)
They then point out that since the said decrees are inconsistent with
Section 24, Article VI of the Constitution,i.e.,
Sec. 24. All appropriation, revenue or tariff bills, bills authorizing
increase of the public debt, bills of local application, and private bills
shall originate exclusively in the House of Representatives, but the
Senate may propose or concur with amendments. (Emphasis
supplied.)
whereby bills have to be approved by the President, 10 then a law must be passed
by Congress to authorize said automatic appropriation. Further, petitioners state said decrees
violate Section 29(l) of Article VI of the Constitution which provides as follows

Sec. 29(l). No money shall be paid out of the Treasury except in


pursuance of an appropriation made by law.
They assert that there must be definiteness, certainty and exactness in an
appropriation, 11 otherwise it is an undue delegation of legislative power to the President who
determines in advance the amount appropriated for the debt service.

12

The Court is not persuaded.


Section 3, Article XVIII of the Constitution recognizes that
"All existing laws, decrees, executive orders, proclamations, letters of instructions
and other executive issuances not inconsistent with the Constitution shall remain
operative until amended, repealed or revoked."
This transitory provision of the Constitution has precisely been adopted by its
framers to preserve the social order so that legislation by the then President
Marcos may be recognized. Such laws are to remain in force and effect unless
they are inconsistent with the Constitution or, are otherwise amended, repealed
or revoked.
An examination of the aforecited presidential decrees show the clear intent that
the amounts needed to cover the payment of the principal and interest on all
foreign loans, including those guaranteed by the national government, should be
made available when they shall become due precisely without the necessity of
periodic enactments of separate laws appropriating funds therefor, since both the
periods and necessities are incapable of determination in advance.

The automatic appropriation provides the flexibility for the effective execution of
debt management policies. Its political wisdom has been convincingly discussed
by the Solicitor General as he argues
. . . First, for example, it enables the Government to take advantage
of a favorable turn of market conditions by redeeming high-interest
securities and borrowing at lower rates, or to shift from short-term to
long-term instruments, or to enter into arrangements that could
lighten our outstanding debt burden debt-to-equity, debt to asset,
debt-to-debt or other such schemes. Second, the automatic
appropriation obviates the serious difficulties in debt servicing arising
from any deviation from what has been previously programmed. The
annual debt service estimates, which are usually made one year in
advance, are based on a mathematical set or matrix or, in layman's
parlance, "basket" of foreign exchange and interest
rate assumptions which may significantly differ from actual rates not
even in proportion to changes on the basis of the assumptions.
Absent an automatic appropriation clause, the Philippine
Government has to await and depend upon Congressional action,
which by the time this comes, may no longer be responsive to the
intended conditions which in the meantime may have already
drastically changed. In the meantime, also, delayed payments and
arrearages may have supervened, only to worsen our debt serviceto-total expenditure ratio in the budget due to penalties and/or
demand for immediate payment even before due dates.
Clearly, the claim that payment of the loans and indebtedness is
conditioned upon the continuance of the person of President Marcos
and his legislative power goes against the intent and purpose of the
law. The purpose is foreseen to subsist with or without the person of
Marcos. 13
The argument of petitioners that the said presidential decrees did not meet the
requirement and are therefore inconsistent with Sections 24 and 27 of Article VI
of the Constitution which requires, among others, that "all appropriations, . . . bills
authorizing increase of public debt" must be passed by Congress and approved
by the President is untenable. Certainly, the framers of the Constitution did not
contemplate that existing laws in the statute books including existing presidential
decrees appropriating public money are reduced to mere "bills" that must again
go through the legislative million The only reasonable interpretation of said
provisions of the Constitution which refer to "bills" is that they mean appropriation
measures still to be passed by Congress. If the intention of the framers thereof

were otherwise they should have expressed their decision in a more direct or
express manner.
Well-known is the rule that repeal or amendment by implication is frowned upon.
Equally fundamental is the principle that construction of the Constitution and law
is generally applied prospectively and not retrospectively unless it is so clearly
stated.
On the third issue that there is undue delegation of legislative power, in Edu
vs. Ericta, 14 this Court had this to say
What cannot be delegated is the authority under the Constitution to
make laws and to alter and repeal them; the test is the completeness
of the statute in all its terms and provisions when it leaves the hands
of the legislature. To determine whether or not there is an undue
delegation of legislative power, the inequity must be directed to the
scope and definiteness of the measure enacted. The legislature
does not abdicate its function when it describes what job must be
done, who is to do it, and what is the scope of his authority. For a
complex economy, that may indeed be the only way in which
legislative process can go forward . . .
To avoid the taint of unlawful delegation there must be a standard,
which implies at the very least that the legislature itself determines
matters of principle and lays down fundamental policy . . .
The standard may be either express or implied . . . from the policy
and purpose of the act considered as whole . . .
In People vs. Vera, 15 this Court said "the true distinction is between the delegation of power to
make the law, which necessarily involves discretion as to what the law shall be, and conferring authority
or discretion as to its execution, to be exercised under and in pursuance of the law. The first cannot be
done; to the latter no valid objection can be made."

Ideally, the law must be complete in all its essential terms and conditions when it
leaves the legislature so that there will be nothing left for the delegate to do when
it reaches him except enforce it. If there are gaps in the law that will prevent its
enforcement unless they are first filled, the delegate will then have been given
the opportunity to step in the shoes of the legislature and exercise a discretion
essentially legislative in order to repair the omissions. This is invalid delegation. 16
The Court finds that in this case the questioned laws are complete in all their
essential terms and conditions and sufficient standards are indicated therein.

The legislative intention in R.A. No. 4860, as amended, Section 31 of P.D. No.
1177 and P.D. No. 1967 is that the amount needed should be automatically set
aside in order to enable the Republic of the Philippines to pay the principal,
interest, taxes and other normal banking charges on the loans, credits or
indebtedness incurred as guaranteed by it when they shall become due without
the need to enact a separate law appropriating funds therefor as the need arises.
The purpose of these laws is to enable the government to make prompt payment
and/or advances for all loans to protect and maintain the credit standing of the
country.
Although the subject presidential decrees do not state specific amounts to be
paid, necessitated by the very nature of the problem being addressed, the
amounts nevertheless are made certain by the legislative parameters provided in
the decrees. The Executive is not of unlimited discretion as to the amounts to be
disbursed for debt servicing. The mandate is to pay only the principal, interest,
taxes and other normal banking charges on the loans, credits or indebtedness, or
on the bonds, debentures or security or other evidences of indebtedness sold in
international markets incurred by virtue of the law, as and when they shall
become due. No uncertainty arises in executive implementation as the limit will
be the exact amounts as shown by the books of the Treasury.
The Government budgetary process has been graphically described to consist of
four major phases as aptly discussed by the Solicitor General:
The Government budgeting process consists of four major phases:
1. Budget preparation. The first step is essentially tasked upon the
Executive Branch and covers the estimation of government
revenues, the determination of budgetary priorities and activities
within the constraints imposed by available revenues and
by borrowing limits, and the translation of desired priorities and
activities into expenditure levels.
Budget preparation starts with the budget call issued by the
Department of Budget and Management. Each agency is required to
submit agency budget estimates in line with the requirements
consistent with the general ceilings set by the Development Budget
Coordinating Council (DBCC).
With regard to debt servicing, the DBCC staff, based on the macroeconomic projections of interest rates (e.g. LIBOR rate) and
estimated sources of domestic and foreign financing, estimates debt
service levels. Upon issuance of budget call, the Bureau of Treasury

computes for the interest and principal payments for the year for all
direct national government borrowings and other liabilities assumed
by the same.
2. Legislative authorization. At this stage, Congress enters the
picture and deliberates or acts on the budget proposals of the
President, and Congress in the exercise of its own judgment and
wisdomformulates an appropriation act precisely following the
process established by the Constitution, which specifies that no
money may be paid from the Treasury except in accordance with an
appropriation made by law.
Debt service is not included in the General Appropriation Act, since
authorization therefor already exists under RA No. 4860 and 245, as
amended and PD 1967. Precisely in the fight of this subsisting
authorization as embodied in said Republic Acts and PD for debt
service, Congress does not concern itself with details for
implementation by the Executive, but largely with annual levels and
approval thereof upon due deliberations as part of the whole
obligation program for the year. Upon such approval, Congress has
spoken and cannot be said to have delegated its wisdom to the
Executive, on whose part lies the implementation or execution of the
legislative wisdom.
3. Budget Execution. Tasked on the Executive, the third phase of the
budget process covers the various operational aspects of budgeting.
The establishment of obligation authority ceilings, the evaluation of
work and financial plans for individual activities, the continuing
review of government fiscal position, the regulation of funds
releases, the implementation of cash payment schedules, and other
related activities comprise this phase of the budget cycle.
Release from the debt service fired is triggered by a request of the
Bureau of the Treasury for allotments from the Department of
Budget and Management, one quarter in advance of payment
schedule, to ensure prompt payments. The Bureau of Treasury,
upon receiving official billings from the creditors, remits payments to
creditors through the Central Bank or to the Sinking Fund
established for government security issues (Annex F).
4. Budget accountability. The fourth phase refers to the evaluation of
actual performance and initially approved work targets, obligations

incurred, personnel hired and work accomplished are compared with


the targets set at the time the agency budgets were approved.
There being no undue delegation of legislative power as clearly
above shown, petitioners insist nevertheless that subject presidential
decrees constitute undue delegation of legislative power to the
executive on the alleged ground that the appropriations therein are
not exact, certain or definite,invoking in support therefor the
Constitution of Nebraska, the constitution under which the case of
State v. Moore, 69 NW 974, cited by petitioners, was decided. Unlike
the Constitution of Nebraska, however, our Constitution does not
require a definite, certain, exact or "specific appropriation made by
law." Section 29, Article VI of our 1987 Constitution omits any of
these words and simply states:
Section 29(l). No money shall be paid out of the
treasury except in pursuance of an appropriation made
by law.
More significantly, there is no provision in our Constitution that
provides or prescribes any particular form of words or religious
recitals in which an authorization or appropriation by Congress shall
be made, except that it be "made by law," such as precisely the
authorization or appropriation under the questioned presidential
decrees. In other words, in terms of time horizons, an appropriation
may be made impliedly (as by past but subsisting legislations) as
well as expressly for the current fiscal year (as by enactment of laws
by the present Congress), just as said appropriation may be made in
general as well as in specific terms. The Congressional authorization
may be embodied in annual laws, such as a general appropriations
act or in special provisions of laws of general or special application
which appropriate public funds for specific public purposes, such as
the questioned decrees. An appropriation measure is sufficient if the
legislative intention clearly and certainly appears from the language
employed (In re Continuing Appropriations, 32 P. 272), whether in
the past or in the present. 17
Thus, in accordance with Section 22, Article VII of the 1987 Constitution,
President Corazon C. Aquino submitted to Congress the Budget of Expenditures
and Sources of Financing for the Fiscal Year 1990. The proposed 1990
expenditure program covering the estimated obligation that will be incurred by
the national government during the fiscal year amounts to P233.5 Billion. Of the
proposed budget, P86.8 is set aside for debt servicing as follows:

National Government Debt


Service Expenditures, 1990
(in million pesos)
Domestic Foreign Total
RA 245, as RA 4860
amended as amended,
PD 1967
Interest
Payments P36,861 P18,570 P55,431
Principal
Amortization 16,310 15,077 31,387
Total P53,171 P33,647 P86,818 18
as authorized under P.D. 1967 and R.A. 4860 and 245, as amended.
The Court, therefor, finds that R.A. No. 4860, as amended by P.D. No. 81,
Section 31 of P.D. 1177 and P.D. No. 1967 constitute lawful authorizations or
appropriations, unless they are repealed or otherwise amended by Congress.
The Executive was thus merely complying with the duty to implement the same.
There can be no question as to the patriotism and good motive of petitioners in
filing this petition. Unfortunately, the petition must fail on the constitutional and
legal issues raised. As to whether or not the country should honor its
international debt, more especially the enormous amount that had been incurred
by the past administration, which appears to be the ultimate objective of the
petition, is not an issue that is presented or proposed to be addressed by the
Court. Indeed, it is more of a political decision for Congress and the Executive to
determine in the exercise of their wisdom and sound discretion.
WHEREFORE, the petition is DISMISSED, without pronouncement as to costs.
SO ORDERED.

Guingona v. Carague
G.R. No. 94571 April 22, 1991
Gancayco, J.
Facts:
The 1990 budget consists of P98.4 Billion in automatic appropriation (with P86.8 Billion for
debt service) and P155.3 Billion appropriated under Republic Act No. 6831, otherwise known as the
General Appropriations Act, or a total of P233.5 Billion, while the appropriations for the Department
of Education, Culture and Sports amount to P27,017,813,000.00.
The said automatic appropriation for debt service is authorized by P.D. No. 81, entitled
Amending Certain Provisions of Republic Act Numbered Four Thousand Eight Hundred Sixty, as
Amended (Re: Foreign Borrowing Act), by P.D. No. 1177, entitled Revising the Budget Process in
Order to Institutionalize the Budgetary Innovations of the New Society, and by P.D. No. 1967,
entitled An Act Strengthening the Guarantee and Payment Positions of the Republic of the
Philippines on Its Contingent Liabilities Arising out of Relent and Guaranteed Loan by Appropriating
Funds For The Purpose.
The petitioner seek the declaration of the unconstitutionality of P.D. No. 81, Sections 31 of
P.D. 1177, and P.D. No. 1967. The petition also seeks to restrain the disbursement for debt service
under the 1990 budget pursuant to said decrees.
Issue:
Is the appropriation of P86 billion in the P233 billion 1990 budget violative of Section 29(1),
Article VI of the Constitution?
Held:
No. There is no provision in our Constitution that provides or prescribes any particular form
of words or religious recitals in which an authorization or appropriation by Congress shall be made,
except that it be made by law, such as precisely the authorization or appropriation under the
questioned presidential decrees. In other words, in terms of time horizons, an appropriation may be
made impliedly (as by past but subsisting legislations) as well as expressly for the current fiscal year
(as by enactment of laws by the present Congress), just as said appropriation may be made in
general as well as in specific terms. The Congressional authorization may be embodied in annual
laws, such as a general appropriations act or in special provisions of laws of general or special

application which appropriate public funds for specific public purposes, such as the questioned
decrees. An appropriation measure is sufficient if the legislative intention clearly and certainly
appears from the language employed (In re Continuing Appropriations, 32 P. 272), whether in the
past or in the present.

FIRST DIVISION
G.R. No. L-109698 December 5, 1994
ANTONIO DIAZ AND KOSUMO DABAW, Petitioners, v. COURT
OF APPEALS, ENERGY REGULATORY BOARD AND DAVAO
LIGHT AND POWER CO., INC., Respondents.
RESOLUTION
BELLOSILLO, J.:
On 23 January 1991, Davao Light and Power Company, Inc. (DLPC)
filed with the Energy Regulatory Board (ERB) an application for the
approval of the sound value appraisal of its property in service.
chanro blesvi rtua lawlib rary chan roble s virtual law lib rary

The Asian Appraisal Company valued the property and equipment of


DLPC as of 12 March 1990 at One Billion One Hundred Forty One
Million Seven Hundred Seventy Four Thousand Pesos
(P1,141,774,000.00).
chanroblesv irt ualawli bra rycha nrob les vi rtual law lib rary

On 6 December 1992, ERB approved the application of DLPC after


deducting Fourteen Million Eight Hundred Thousand Pesos
(P14,800,000.00) worth of property and equipment which were not
used by DLPC in its operation.
chanroblesv irt ualawli bra rycha nrob les vi rtua l law lib rary

On 6 July 1992, petitioners filed a petition for review


on certiorari before this Court assailing the decision of ERB on the
ground of lack of jurisdiction and/or grave abuse of discretion
amounting to lack of jurisdiction.
chanroblesvi rtua lawlib raryc han robles v irt ual law lib rary

In our resolution of 8 September 1992, we referred the case for


proper disposition to the Court of Appeals which subsequently
dismissed the petition on the ground that (1) the filing of the

petition for review with the Supreme Court was a wrong mode of
appeal, and (2) the petition did not comply with the provisions of
Supreme Court Circular 1-88 in that (a) it did not state the date
when the petitioners received notice of the ERB decision, (b) it did
not state the date when the petitioners filed a motion for
reconsideration, and (c) it inconsistently alleged different dates
when petitioners supposedly received the denial of their motion by
ERB.
chanroble svirtualawl ibraryc hanrobles vi rt ual law li bra ry

On 18 December 1992, petitioners filed a motion for reconsideration


contending that our resolution of 8 September 1992 was a directive
for the Court of Appeals to disregard the above circular.
chanrob lesvi rtualaw lib raryc han robles v irt ual law li bra ry

In its resolution of 24 March 1993, the Court of Appeals denied the


motion for reconsideration for lack of merit. Hence, the instant
recourse.
chanroblesvi rtua lawlib rary chan roble s virtual law l ibra ry

We deny the petition. The predecessor of the Energy Regulatory


Board was the Board of Energy created under P.D. No. 1206.
Thereunder, appeals from the decisions of the Board of Energy were
appealable to the Office of the President. However, under the
Interim Rules Implementing the Judiciary Reorganization Act of
1980, final decisions, orders, awards or resolutions of the Board of
Energy were made appealable to the Intermediate Appellate Court
(Sec. 9).
chanroblesvi rtua lawlib rary chan roble s virtual law l ibra ry

On 2 February 1987, the New Constitution took effect. Sec. 30, Art.
VI, thereof provides: "No law shall be passed increasing the
appellate jurisdiction of the Supreme Court as provided in this
Constitution without its advice and concurrence."
chanrobles vi rtua l law li bra ry

On 8 May 1987, the President promulgated E.O. No. 172 creating


the Energy Regulatory Board to replace the Board of Energy. Under
Sec. 10 thereof, "[a] party adversely affected by a decision, order
or ruling of the Board . . . may file a petition to be known as petition
for review with the Supreme Court."
chanrob les vi rtual law lib rary

On 27 February 1991, the Supreme Court promulgated Circular No.


1-91, par. (1) of which specifically provides that the proper mode of

appeal from any quasi-judicial agency, including ERB, is by way of a


petition for review with the Court of Appeals.
chanroble svi rtualaw lib raryc han robles v irt ual law li bra ry

It is very patent that since Sec. 10 of E.O. No. 172 was enacted
without the advice and concurrence of this Court, this provision
never became effective, with the result that it cannot be deemed to
have amended the Judiciary Reorganization Act of 1980.
Consequently, the authority of the Court of Appeals to decide cases
from the Board of Energy, now ERB, remains (Cf. First Lepanto
Ceramics, Inc. v. Court of Appeals, G.R. No. 110571, 7 October
1994).
chanroblesv irt ualawli bra rycha nrob les vi rtua l law lib rary

If the appeal is brought to either Court (Supreme Court or Court of


Appeals) by the wrong procedure, the only course of action open to
it is to dismiss the appeal. There is no longer any justification for
allowing transfers of erroneous appeals from one court to another
(Quesada v. Court of Appeals, G.R. No. 93869, 12 November
1990).
chanroblesv irt ualawli bra rycha nrob les vi rtua l law lib rary

Prior to Circular No. 1-91, the Supreme Court promulgated Circular


No. 2-90 dated 9 March 1990, Item No. 4 of which states that "[a]n
appeal taken to either the Supreme Court or the Court of Appeals
by the wrong or inappropriate mode shall be dismissed".
chanrob lesvi rtua lawlib rary chan roble s virtual law l ibra ry

Paragraph (d) of said Circular No. 2-90 also provides that "[n]o
transfer of appeals erroneously taken to the Supreme Court or to
the Court of Appeals to whichever of these Tribunals has
appropriate appellate jurisdiction will be allowed; continued
ignorance or willful disregard of the law on appeals will not be
tolerated."
chanrobles v irt ual law l ibra ry

Consequently, the Court of Appeals was correct when it held Contrary to petitioners' stand, the Supreme Court's Resolution
dated September 8, 1992, referring "this case to the Court of
Appeals for further disposition" was not a directive for this court to
disregard the above circulars and precedents. Rather the said SC
resolution could mean only that this court should dispose of the
subject petition in conformity with, and not in violation of, those
circulars and precedents (Rollo, p. 26).

Both Circulars Nos. 1-88 and 2-90 were duly published in


newspapers of general circulation in the Philippines. Hence, lawyers
are expected to keep themselves abreast with the decisions of this
Court and with its Circulars and other issuances relating to
procedure or affecting their duties and responsibilities as officers of
the court (Teehankee, Jr. v. Hon. Madayag, G.R. No. 102717, 12
December 1992).
chanroblesvi rtualaw lib raryc han robles vi rt ual law li bra ry

SC Circular No. 1-88, which took effect on 1 January 1989, was not
adopted and approved by this Court for childish, flimsy or petty
reasons, nor for pure love of technicalities, but to compel the strict
observance of the Revised Rules of Court in order that proceedings
before this Court may not be needlessly delayed (Gallardo v.
Quintus, A.M. No. RTJ-90-577, 18 April 1991).
chanroble svi rtualawl ib raryc hanrobles vi rt ual law li bra ry

WHEREFORE, the instant petition is DISMISSED.


[Syllabus]

EN BANC

[G.R. No. 125416. September 26, 1996]

SUBIC
BAY
METROPOLITAN
AUTHORITY, petitioner,
vs. COMMISSION ON ELECTIONS, ENRIQUE T. GARCIA and
CATALINO A. CALIMBAS,respondents.
DECISION
PANGANIBAN, J.:

The 1987 Constitution is unique in many ways. For one thing, it


institutionalized people power in law-making. Learning from the bitter lesson
of completely surrendering to Congress the sole authority to make, amend or
repeal laws, the present Constitution concurrently vested such prerogatives in
the electorate by expressly recognizing their residual and sovereign authority
to ordain legislation directly through the concepts and processes of initiative
and of referendum.

In this Decision, this Court distinguishes referendum from initiative and


discusses the practical and legal implications of such differences. It also sets
down some guidelines in the conduct and implementation of these two novel
and vital features of popular democracy, as well as settles some relevant
questions on jurisdiction -- all with the purpose of nurturing, protecting and
promoting the people's exercise of direct democracy.
In this action for certiorari and prohibition, petitioner seeks to nullify the
respondent Commission on Elections' Ruling dated April 17, 1996 and
Resolution No. 2848 promulgated on June 27, 1996[1] denying petitioner's plea
to stop the holding of a local initiative and referendum on the proposition to
recall Pambayang Kapasyahan Blg. 10, Serye 1993, of the Sangguniang
Bayan of Morong, Bataan.
The Facts
On March 13, 1992, Congress enacted Republic Act No. 7227 (The Bases
Conversion and Development Act of 1992), which among others, provided for
the creation of the Subic Special Economic Zone, thus:
"Sec. 12. Subic Special Economic Zone. - Subject to the concurrence by resolution of
the Sangguniang Panlungsod of the City of Olongapo and the Sangguniang Bayan of
the Municipalities of Subic, Morong and Hermosa, there is hereby created a Special
Economic and Free-port Zone consisting of the City of Olongapo and the
Municipality of Subic, Province of Zambales, the lands occupied by the Subic Naval
Base and its contiguous extensions as embraced, covered and defined by the 1947
Military Bases Agreement between the Philippines and the United States of America
as amended, and within the territorial jurisdiction of the Municipalities of Morong and
Hermosa, Province of Bataan, hereinafter referred to as the Subic Special Economic
Zone whose metes and bounds shall be delineated in a proclamation to be issued by
the President of the Philippines. Within thirty (30) days after the approval of this Act,
each local government unit shall submit its resolution of concurrence to join the Subic
Special Economic Zone to the Office of the President. Thereafter, the President of
the Philippines shall issue a proclamation defining the metes and bounds of the zone
as provided herein." (Underscoring supplied)
RA 7227 likewise created petitioner to implement the declared national
policy of converting the Subic military reservation into alternative productive
uses.[2] Petitioner was organized with an authorized capital stock of P20 billion
which was fully subscribed and fully paid up by the Republic of the Philippines
with, among other assets, "(a)ll lands embraced, covered and defined in

Section 12 hereof, as well as permanent improvements and fixtures upon


proper inventory not otherwise alienated, conveyed, or transferred to another
government agency.[3]
On November 24, 1992, the American navy turned over the Subic military
reservation to the Philippine government. Immediately, petitioner commenced
the implementation of its task, particularly the preservation of the seaports,
airports, buildings, houses and other installations left by the American navy.
In April 1993, the Sangguniang Bayan of Morong, Bataan passed
a Pambayang Kapasyahan Bilang 10, Serye 1993, expressing therein its
absolute concurrence, as required by said Sec. 12 of RA 7227, to join the
Subic Special Economic Zone. On September 5, 1993, the Sangguniang
Bayan of Morong submitted Pambayang Kapasyahan Bilang 10, Serye
1993 to the Office of the President.
On May 24, 1993, respondents Garcia, Calimbas and their companions
filed a petition with the Sangguniang Bayan of Morong to annul Pambayang
Kapasyahan Blg. 10, Serye 1993. The petition prayed for the following:
"I. Bawiin, nulipikahin at pawalang-bisa ang Pambayang Kapasyahan Blg. 10
Serye 1993 ng Sangguniang Bayan para sa pag-anib ng Morong sa SSEFZ na walang
kundisyon.
II.
Palitan ito ng isang Pambayang kapasiyahan na aanib lamang ang Morong sa
SSEFZ kung ang mga sumusunod na kondisyones ay ipagkakaloob, ipatutupad at
isasagawa para sa kapakanan at interes ng Morong at Bataan:
(A) Ibalik sa Bataan ang 'Virgin Forests' -- isang bundok na hindi nagagalaw at
punong-puno ng malalaking punong-kahoy at iba't-ibang halaman.
(B) Ihiwalay ang Grande Island sa SSEFZ at ibalik ito sa Bataan.
(K) Isama ang mga lupain ng Bataan na nakapaloob sa SBMA sa pagkukuenta ng
salaping ipinagkaloob ng pamahalaang national o 'Internal Revenue Allotment' (IRA)
sa Morong, Hermosa at sa Lalawigan.
(D) Payagang magtatag rin ng sariling 'special economic zones' ang bawat bayan ng
Morong, Hermosa at Dinalupihan.
(E) Ibase sa laki ng kanya-kanyang lupa ang pamamahagi ng kikitain ng SBMA.
(G) Ibase rin ang alokasyon ng pagbibigay ng trabaho sa laki ng nasabing mga lupa.

(H) Pabayaang bukas ang pinto ng SBMA na nasa Morong ng 24 na oras at bukod
dito sa magbukas pa ng pinto sa hangganan naman ng Morong at Hermosa upang
magkaroon ng pagkakataong umunlad rin ang mga nasabing bayan, pati na rin ng iba
pang bayan ng Bataan.
(I) Tapusin ang pagkokonkreto ng mga daang Morong-Tala-Orani at Morong-TasigDinalupihan para sa kabutihan ng mga taga-Bataan at tuloy makatulong sa
pangangalaga ng mga kabundukan.
(J) Magkakaroon ng sapat na representasyon sa pamunuan ng SBMA ang Morong,
Hermosa at Bataan."
The Sangguniang Bayan of Morong acted upon the petition of
respondents Garcia, Calimbas, et al. by promulgating Pambayang
Kapasyahan Blg. 18, Serye 1993, requesting Congress of the Philippines to
amend certain provisions of R.A. No. 7227, particularly those concerning the
matters cited in items (A), (B), (K), (E) and (G) of private respondents'
petition. TheSangguniang Bayan of Morong also informed respondents that
items (D) and (H) had already been referred to and favorably acted upon by
the government agencies concerned, such as the Bases Conversion
Development Authority and the Office of the President.
Not satisfied, and within 30 days from submission of their petition, herein
respondents resorted to their power of initiative under the Local Government
Code of 1991,[4] Sec. 122 paragraph (b) of which provides as follows:
"Sec. 122. Procedure in Local Initiative. xxx

xxx

xxx

(b) If no favorable action thereon is taken by the sanggunian concerned, the


proponents, through their duly authorized and registered representatives, may invoke
their power of initiative, giving notice thereof to the sanggunian concerned.
xxx

xxx

x x x."

On July 6, 1993, respondent Commission En Banc in Comelec Resolution


No. 93-1623 denied the petition for local initiative by herein private
respondents on the ground that the subject thereof was merely a
resolution (pambayang kapasyahan) and not an ordinance. On July 13, 1993,
public respondent Comelec En Banc (thru Comelec Resolution no. 93-1676)
further directed its Provincial Election Supervisor to hold action on the
authentication of signatures being solicited by private respondents.

On August 15, 1993, private respondents instituted a petition


for certiorari and mandamus[5] before this Court against the Commission on
Elections and the Sangguniang Bayan of Morong, Bataan, to set aside
Comelec Resolution No. 93-1623 insofar as it disallowed the conduct of a
local initiative to annul Pambayang Kapasyahan Bilang 10, Serye 1993, and
Comelec Resolution No. 93-1676 insofar as it prevented the Provincial
Election Supervisor of Bataan from proceeding with the authentication of the
required number of signatures in support of the initiative and the gathering of
signatures.
On February 1, 1995, pursuant to Sec. 12 of RA 7227, the President of
the Philippines issued proclamation No. 532 defining the metes and bounds of
the SSEZ. Said proclamation included in the SSEZ all the lands within the
former Subic Naval Base, including Grande Island and that portion of the
former
naval
base
within
the
territorial
jurisdiction
of
the Municipality ofMorong.
On June 18, 1996, respondent Comelec issued Resolution No. 2845,
adopting therein a "Calendar of Activities for local referendum on certain
municipal ordinance passed by the Sangguniang Bayan of Morong, Bataan",
and which indicated, among others, the scheduled referendum Day (July 27,
1996, Saturday). On June 27, 1996, the Comelec promulgated the assailed
Resolution No. 2848 providing for "the rules and guidelines to govern the
conduct of the referendum proposing to annul or repeal Kapasyahan Blg. 10,
Serye 1993 of theSangguniang Bayan of Morong, Bataan".
On July 10, 1996, petitioner instituted the present petition for certiorari and
prohibition contesting the validity of Resolution No. 2848 and alleging, inter
alia, that public respondent "is intent on proceeding with a local initiative that
proposes an amendment of a national law. x x x"
The Issues
The petition[6] presents the following "argument":
"Respondent Commission on Elections committed grave abuse of discretion
amounting to lack of jurisdiction in scheduling a local initiative which seeks the
amendment of a national law."
In his Comment, private respondent Garcia claims that (1) petitioner has
failed to show the existence of an actual case or controversy; (2) x x x
petitioner seeks to overturn a decision/judgment which has long become final

and executory; (3) x x x public respondent has not abused its discretion and
has in fact acted within its jurisdiction; (and) (4) x x x the concurrence of local
government units is required for the establishment of the Subic Special
Economic Zone."
Private respondent Calimbas, now the incumbent Mayor of Morong, in his
Reply (should be Comment) joined petitioner's cause because "(a)fter several
meetings with petitioner's Chairman and staff and after consultation with legal
counsel, respondent Calimbas discovered that the demands in the petition for
a local initiative/referendum were not legally feasible."[7]
The Solicitor General, as counsel for public respondent, identified two
issues, as follows:
"1. Whether or not the Comelec can be enjoined from scheduling/conducting the local
intiative proposing to annul Pambayang Kapasyahan Blg. 10, Serye 1993 of the
Sangguniang Bayan of Morong, Bataan.
2. Whether or not the Comelec committed grave abuse of discretion in denying the
request of petitioner SBMA to stop the local initiative."
On July 23, 1996, the Court heard oral argument by the parties, after
which, it issued the following resolution:
"The Court Resolved to (1) GRANT the Motion to Admit the Attached Comment
filed by counsel for private respondent Enrique T. Garcia, dated July 22, 1996 and (2)
NOTE the: (a) Reply (should be comment) to the petition for certiorari and
prohibition with prayer for temporary restraining order and/or writ of preliminary
injunctiom, filed by counsel for respondent Catalino Calimbas, dated July 22, 1996;
(b) Separate Comments on the petition, filed by: (b-1) the Solicitor General for
respondent Commission on Elections dated July 19, 1996 and (b-2) counsel for
private respondent Enrique T. Garcia, dated July 22, 1996 and (c) Manifestation filed
by counsel for petitioner dated July 22, 1996.
At the hearing of this case this morning, Atty. Rodolfo O. Reyes appeared and argued
for petitioner Subic Bay Metropolitan Authority (SBMA) while Atty. Sixto Brillantes
for private respondent Enrique T. Garcia, and Atty. Oscar L. Karaan for respondent
Catalino Calimbas. Solicitor General Raul Goco, Assistant Solicitor General Cecilio
O. Estoesta and Solicitor Zenaida Hernandez-Perez appeared for respondent
Commission on Elections with Solicitor General Goco arguing.
Before the Court adjourned, the Court directed the counsel for both parties to
INFORM this Court by Friday, July 26, 1996, whether or not Commission on

Elections would push through with the initiative/referendum this Saturday, July 27,
1996.
Thereafter, the case shall be considered SUBMITTED for resolution.
At 2:50 p.m. July 23, 1996, the Court received by facsimile transmission an Order
dated also on July 23, 1996 from the respondent Commission on Elections En
Banc inter alia 'to hold in abeyance the scheduled referendum (initiative) on July 27,
1996 pending resolution of G.R. No. 125416.' In view of this Order, the petitioner's
application for a temporary restraining order and/or writ of preliminary injunction has
become moot and academic and will thus not be passed upon by this Court at this
time. Puno, J., no part due to relationship. Bellosillo, J., is on leave."
After careful study of and judicious deliberation on the submissions and
arguments of the parties, the Court believes that the issues may be restated
as follows:
(1) Whether this petition "seeks to overturn a decision/judgment which has long
become final and executory"; namely G.R. No. 111230, Enrique Garcia, et al. vs.
Commission on Elections, et al.;
(2) Whether the respondent Comelec committed grave abuse of discretion in
promulgating and implementing its Resolution No. 2848 which "govern(s) the
conduct of the referendum proposing to annul or repeal Pambayang Kapasyahan Blg.
10, Serye 1993 of the Sangguniang Bayan of Morong, Bataan;" and
(3) Whether the questioned local initiative covers a subject within the powers of the
people of Morong to enact; i.e., whether such initiative "seeks the amendment of a
national law."
First Issue: Bar by Final Judgment
Respondent Garcia contends that this Court had already ruled with finality
in Enrique T. Garcia, et al. vs. Commission on Elections, et. al.[8] on "the very
issue raised in (the) petition: whether or not there can be an initiative by the
people of Morong, Bataan on the subject proposition -- the very same
proposition, it bears emphasizing, the submission of which to the people of
Morong, Bataan is now sought to be enjoined by petitioner x x x".
We disagree. The only issue resolved in the earlier Garcia case is
whether a municipal resolution as contra-distinguished from an ordinance may

be the proper subject of an initiative and/or referendum. We quote from our


said Decision:[9]
"In light of this legal backdrop, the essential issue to be resolved in the case at bench
is whether Pambayang Kapasyahan Blg. 10, serye 1993 of the Sangguniang Bayan of
Morong, Bataan is the proper subject of an initiative. Respondents take the negative
stance as they contend that under the Local Government Code of 1991 only an
ordinance can be the subject of initiative. They rely on Section 120, Chapter 2, Title
XI, Book I of the Local Government Code of 1991 which provides: 'Local Initiative
Defined. -- Local initiative is the legal process whereby the registered voters of a local
government unit may directly propose, enact, or amend any ordinance.'
We reject respondent's narrow and literal reading of the above provision for it will
collide with the Constitution and will subvert the intent of the lawmakers in enacting
the provisions of the Local Government of 1991 on initiative and referendum.
The Constitution clearly includes not only ordinances but resolutions as appropriate
subjects of a local initiative. Section 32 of Article VI provides in luminous
language: 'The Congress shall, as early as possible, provide for a system of initiative
and referendum, and the exceptions therefrom, whereby the people can directly
propose and enact laws or approve or reject any act or law or part thereof passed by
the Congress, or local legislative body x x x'. An act includes a resolution. Black
defines an acts 'an expression of will or purpose . . . it may denote something done . . .
as a legislature, including not merely physical acts, but also decrees, edicts, laws,
judgement, resolves, awards and determination x x x.' It is basic that a law should be
construed in harmony with and not in violation of the Constitution. In line with this
postulates, we held in In Re Guarina that if there is doubt or uncertainly as to the
meaning of the legislative, if the words or provisions are obscure, or if the enactment
is fairly susceptible of two or more construction, that interpretations will be adopted
which will avoid the effect of unconstitutionality, even though it may be necessary,
for this purpose, to disregard the more usual or apparent import of the language used.'
"
Moreover, we reviewed our rollo in said G.R. No. 111230 and we found
that the sole issue presented by the pleadings was the question of "whether or
not a Sangguniang Bayan Resolution can be the subject of a valid initiative or
referendum".[10]
In the present case, petitioner is not contesting the propriety of municipal
resolution as the form by which these two new constitutional prerogatives of
the people may validly exercised. What is at issue here is
whether Pambayang Kapasyahan Blg. 10, Serye 1993, as worded, is

sufficient in form and substance for submission to the people for their
approval; in fine, whether the Comelec acted properly and juridically in
promulgating and implementing Resolution No. 2848.
Second Issue: Sufficiency of Comelec Resolution No. 2848
The main issue in this case may be re-started thus: Did respondent
Comelec commit grave abuse of discretion in promulgating and implementing
Resolution No. 2848?
We answer the question in the affirmative.
To begin with, the process started by private respondents was an
INITIATIVE but respondent Comelec made preparations for a
REFERENDUM only. In fact, in the body of the Resolution[11] as reproduced
in the footnote below the word "referendum" is repeated at least 27 times, but
"initiative" is not mentioned at all. The Comelec labeled the exercise as a
"Referendum"; the counting of votes was entrusted to a "Referendum
Committee"; the documents were called "referendum returns"; the
canvassers, "Referendum Board of Canvassers" and the ballots themselves
bore the description "referendum". To repeat, not once was the word
"initiative" used in said body of Resolution No. 2848. And yet, this exercise is
unquestionably an INITIATIVE.
There are statutory and conceptual demarcations between a referendum
and an initiative. In enacting the "Initiative and Referendum Act,[12] Congress
differentiated one term from the other, thus:
(a) "Initiative" is the power of the people to propose amendments to the Constitution
or to propose and enact legislations through an election called for the purpose.
There are three (3) systems of initiative, namely:
a.1. Initiative on the Constitution which refers to a petition proposing amendments to
the Constitution;
a.2. Initiative on statutes which refers to a petition proposing to enact a national
legislation; and
a.3. Initiative on local legislation which refers to a petition proposing to enact a
regional, provincial, city, municipal, or barangay law, resolution or ordinance.

(b) "Indirect initiative" is exercise of initiative by the people through a proposition


sent to Congress or the local legislative body for action.

(c) "Referendum" is the power of the electorate to approve or reject a legislation


through an election called for the purpose. It may be of two classes, namely:
c.1. Referendum on statutes which refers to a petition to approve or reject an act or
law, or part thereof, passed by Congress; and
c.2. Referendum on local law which refers to a petition to approve or reject a law,
resolution or ordinance enacted by regional assemblies and local legislative bodies.

Along these statutory definitions, Justice Isagani A. Cruz[13] defines


initiative as the "power of the people to propose bills and laws, and to enact or
reject them at the polls independent of the legislative assembly." On the other
hand, he explains that referendum "is the right reserved to the people to adopt
or reject any act or measure which has been passed by a legislative body and
which in most cases would without action on the part of electors become a
law." The foregoing definitions, which are based on Black's[14] and other
leading American authorities, are echoed in the Local Government Code (RA
7160) substantially as follows:
"SEC. 120. Local Initiative Defined. -- Local Initiative is the legal process whereby
the registered voters of a local government unit may directly propose, enact, or amend
any ordinance.
"SEC. 126. Local Referendum Defined. -- Local referendum is the legal process
whereby the registered voters of the local government units may approve, amend or
reject any ordinance enacted by the sanggunian.
The local referendum shall be held under the control and direction of the Comelec
within sixty (60) days in case of provinces and cities, forty-five (45) days in case of
municipalities and thirty (30) days in case of barangays.
The Comelec shall certify and proclaim the results of the said referendum."
Prescinding from these definitions, we gather that initiative is resorted to
(or initiated) by the people directly either because the law-making body fails or
refuses to enact the law, ordinance, resolution or act that they desire or
because they want to amend or modify one already existing. Under Sec. 13
of R.A. 6735, the local legislative body is given the opportunity to enact the
proposal. If its refuses/neglects to do so within thirty (30) days from its
presentation, the proponents through their duly-authorized and registered
representatives may invoke their power of initiative, giving notice thereof to
the local legislative body concerned. Should the proponents be able to collect
the number of signed conformities within the period granted by said statute,
the Commission on Elections "shall then set a date for the initiative (not

referendum) at which the proposition shall be submitted to the registered


voters in the local government unit concerned x x x".
On the other hand, in a local referendum, the law-making body submits to
the registered voters of its territorial jurisdiction, for approval or rejection, any
ordinance or resolution which is duly enacted or approved by such law-making
authority. Said referendum shall be conducted also under the control and
direction of the Commission on Elections.[15]
In other words, while initiative is entirely the work of the electorate,
referendum is begun and consented to by the law-making body. Initiative is a
process of law-making by the people themselves without the participation and
against the wishes of their elected representatives, while referendum consists
merely of the electorate approving or rejecting what has been drawn up or
enacted by a legislative body. Hence, the process and the voting in an
initiative are understandably more complex than in a referendum where
expectedly the voters will simply write either "Yes" or "No" in the ballot.
[Note: While the above quoted laws variously refer to initiative and
referendum as "powers" or "legal processes", these can also be "rights", as
Justice Cruz terms them, or "concepts", or "the proposal" itself (in the case of
initiative) being referred to in this Decision.]
From the above differentiation, it follows that there is need for the Comelec
to supervise an initiative more closely, its authority thereon extending not only
to the counting and canvassing of votes but also to seeing to it that the matter
or act submitted to the people is in the proper form and language so it may be
easily understood and voted upon by the electorate. This is especially true
where the proposed legislation is lengthy and complicated, and should thus be
broken down into several autonomous parts, each such part to be voted upon
separately. Care must also be exercised that "(n)o petition embracing more
than one subject shall be submitted to the electorate,"[16] although "two or
more propositions may be submitted in an initiative".[17]
It should be noted that under Sec. 13 (c) of RA 6735, the "Secretary of
Local Government or his designated representative shall extend assistance in
the formulation of the proposition."
In initiative and referendum, the Comelec exercises administration and
supervision of the process itself, akin to its powers over the conduct of
elections. These law-making powers belong to the people, hence the
respondent Commission cannot control or change the substance or the
content of legislation. In the exercise of its authority, it may (in fact it should

have done so already) issue relevant and adequate guidelines and rules for
the orderly exercise of these "people-power" features of our Constitution.
Third Issue: Withdrawal of Adherence and Imposition of Conditionalities
-- Ultra Vires?
Petitioner maintains that the proposition sought to be submitted in the
plebiscite, namely, Pambayang Kapasyahan Blg. 10, Serye 1993, is ultra
vires or beyond the powers of the Sangguniang Bayan to enact,[18] stressing
that under Sec. 124 (b) of RA 7160 (the Local Government Code), "local
initiative shall cover only such subjects or matters as are within the legal
powers of the sanggunians to enact." Elsewise stated, a local initiative may
enact only such ordinances or resolutions as the municipal council itself could,
if it decided to so enact.[19]After the Sangguniang Bayan of Morong and the
other municipalities concerned (Olongapo, Subic and Hermosa) gave their
resolutions of concurrence, and by reason of which the SSEZ had been
created, whose metes and bounds had already been delineated by
Proclamation No. 532 issued on February 1, 1995 in accordance with Section
12 of R.A. No. 7227, the power to withdraw such concurrence and/or to
substitute therefor a conditional concurrence is no longer within the authority
and competence of the Municipal Council of Morong to legislate. Furthermore,
petitioner adds, the specific conditionalities included in the questioned
municipal resolution are beyond the powers of the Council to impose. Hence,
such withdrawal can no longer be enacted or conditionalities imposed by
initiative. In other words, petitioner insists, the creation of SSEZ is now a fait
accompli for the benefit of the entire nation. Thus, Morong cannot unilaterally
withdraw its concurrence or impose new conditions for such concurrence as
this would effectively render nugatory the creation by (national) law of the
SSEZ and would deprive the entire nation of the benefits to be derived
therefrom. Once created, SSEZ has ceased to be a local concern. It has
become a national project.
On the other hand, private respondent Garcia counters that such
argument is premature and conjectural because at this point, the resolution is
just a proposal. If the people should reject it during the referendum, then
there is nothing to declare as illegal.
Deliberating on this issue, the Court agrees with private respondent Garcia
that indeed, the municipal resolution is still in the proposal stage. It is not yet
an approved law. Should the people reject it, then there would be nothing to
contest and to adjudicate. It is only when the people have voted for it and it

has become an approved ordinance or resolution that rights and obligations


can be enforced or implemented thereunder. At this point, it is merely a
proposal and the writ of prohibition cannot issue upon a mere conjecture or
possibility. Constitutionally speaking, courts may decide only actual
controversies, not hypothetical questions or cases.[20]
We also note that the Initiative and Referendum Act itself provides [21] that
"(n)othing in this Act shall prevent or preclude the proper courts from declaring
null and void any propositionapproved pursuant to this Act x x x."
So too, the Supreme Court is basically a review court.[22] It passes upon
errors of law (and sometimes of fact, as in the case of mandatory appeals of
capital offenses) of lower courts as well as determines whether there had
been grave abuse of discretion amounting to lack or excess of jurisdiction on
the part of any "branch or instrumentality" of government. In the present case,
it is quite clear that the Court has authority to review Comelec Resolution No.
2848 to determine the commission of grave abuse of discretion. However, it
does not have the same authority in regard to the proposed initiative since it
has not been promulgated or approved, or passed upon by any "branch or
instrumentality" or lower court, for that matter. The Commission on Elections
itself has made no reviewable pronouncements about the issues brought by
the pleadings. The Comelec simply included verbatim the proposal in its
questioned Resolution No. 2848. Hence, there is really no decision or action
made by a branch, instrumentality or court which this Court could take
cognizance of and acquire jurisdiction over, in the exercise of its review
powers.
Having said that, we are in no wise suggesting that the Comelec itself has
no power to pass upon proposed resolutions in an initiative. Quite the
contrary, we are ruling that these matters are in fact within the initiatory
jurisdiction of the Commission -- to which then the herein basic questions
ought to have been addressed, and by which the same should have been
decided in the first instance. In other words, while regular courts may take
jurisdiction over "approved propositions" per said Sec. 18 of R.A. 6735, the
Comelec in the exercise of its quasi-judicial and administrative powers may
adjudicate and pass upon such proposals insofar as their form and language
are concerned, as discussed earlier; and it may be added, even as to content,
where the proposals or parts thereof are patently and clearly outside the
"capacity of the local legislative body to enact."[23] Accordingly, the question of
whether the subject of this initiative is within the capacity of the Municipal
Council of Morong to enact may be ruled upon by the Comelec upon remand
and after hearing the parties thereon.

While on the subject of capacity of the local lawmaking body, it would be


fruitful for the parties and the Comelec to plead and adjudicate, respectively,
the question of whether Grande Island and the "virgin forests" mentioned in
the proposed initiative belong to the national government and thus cannot be
segregated from the Zone and "returned to Bataan" by the simple expedient of
passing a municipal resolution. We note that Sec. 13 (e) of R.A. 7227 speaks
of the full subscription and payment of the P20 billion authorized capital stock
of the Subic Authority by the Republic, with, aside from cash and other assets,
the "... lands, embraced, covered and defined in Section 12 hereof, ..." which
includes said island and forests. The ownership of said lands is a question of
fact that may be taken up in the proper forum -- the Commission on Elections.
Another question which the parties may wish to submit to the Comelec
upon remand of the initiative is whether the proposal, assuming it is within the
capacity of the Municipal Council to enact, may be divided into several parts
for purposes of voting. Item "I" is a proposal to recall, nullify and render
without effect (bawiin, nulipikahin at pawalangbisa) Municipal Resolution No.
10, Series of 1993. On the other hand, Item "II" proposes to change or
replace (palitan) said resolution with another municipal resolution of
concurrence provided certain conditions enumerated thereunder would be
granted, obeyed and implemented (ipagkakaloob, ipatutupad at isasagawa)
for the benefit and interest of Morong and Bataan. A voter may favor Item I -i.e., he may want a total dismemberment of Morong from the Authority -- but
may not agree with any of the conditions set forth in Item II. Should the
proposal then be divided and be voted upon separately and independently?
All told, we shall not pass upon the third issue of ultra vires on the ground
of prematurity.
Epilogue
In sum, we hold that (i) our decision in the earlier Garcia case is not a bar
to the present controversy as the issue raised and decided therein is different
from the questions involved here; (ii) the respondent Commission should be
given an opportunity to review and correct its errors in promulgating its
Resolution No. 2848 and in preparing -- if necessary -- for the plebiscite; and
(iii) that the said Commission has administrative and initiatory quasi-judicial
jurisdiction to pass upon the question of whether the proposal is sufficient in
form and language and whether such proposal or part or parts thereof
are clearly and patently outside the powers of the municipal council of
Morong to enact, and therefore violative of law.

In deciding this case, the Court realizes that initiative and referendum, as
concepts and processes, are new in our country. We are remanding the
matter to the Comelec so that proper corrective measures, as above
discussed, may be undertaken, with a view to helping fulfill our people's
aspirations for the actualization of effective direct sovereignty. Indeed we
recognize that "(p)rovisions for initiative and referendum are liberally
construed to effectuate their purposes, to facilitate and not to hamper the
exercise by the voters of the rights granted thereby."[24]In his authoritative
treatise on the Constitution, Fr. Joaquin G. Bernas, S.J. treasures these
"instruments which can be used should the legislature show itself indifferent to
the needs of the people."[25] Impelled by a sense of urgency, Congress
enacted Republic Act No. 6735 to give life and form to the constitutional
mandate. Congress also interphased initiative and referendum into the
workings of local governments by including a chapter on this subject in the
local Government Code of 1991.[26] And the Commission on Elections can do
no less by seasonably and judiciously promulgating guidelines and rules, for
both national and local use, in implementation of these laws. For its part, this
Court early on expressly recognized the revolutionary import of reserving
people power in the process of law-making.[27]
Like elections, initiative and referendum are powerful and valuable modes
of expressing popular sovereignty. And this Court as a matter of policy and
doctrine will exert every effort to nurture, protect and promote their legitimate
exercise. For it is but sound public policy to enable the electorate to express
their free and untrammeled will, not only in the election of their anointed
lawmakers and executives, but also in the formulation of the very rules and
laws by which our society shall be governed and managed.
WHEREFORE the petition is GRANTED. Resolution No. 2848
is ANNULLED and SET ASIDE. The initiative on Pambayang Kapasyahan
Blg. 10, Serye 1993 is REMANDED to the Commission on Elections for
further proceedings consistent with the foregoing discussion. No costs.
IT IS SO ORDERED.

Vous aimerez peut-être aussi